You are on page 1of 320

Version 3.

Version 3.0

Chief Producer
YU Minhong

Executive Producer
ZHANG Hongwei

Production Manager
SHI Yu; YANG Binghui

Director
GE Xu, HE Fanfan

Author
A Ercha

Co-Authors
GONG Chuanlong, LI Xin, WEI Chen, XU Man, YANG Fan, YUAN
Meng

Board of Censors
Anna & Richard

[] Version 3.0

Table of Contents

................................................................................................................................................ 7

iBT ................................................................................................................. 8
1.1 Key Features ......................................................................................................... 8
1.2 In-Depth Analysis ................................................................................................ 11
1.2.1 .............................................................................................................................. 11
1.2.2 Classification of the Questions ............................................................ 11
1.2.3 Subject Classification ........................................................................... 12
1.2.4 Sources of Contents ............................................................................ 13
1.3 Additional Remarks ............................................................................................. 14
1.4 Test Yourself................................................................................................ 15
1.5 Defect Analysis ............................................................................................ 21
Vocabulary Workshop ...................................................................................... 23
2.1 Basic Recognition ........................................................................................ 23
2.2 Word Memorization and Tricks .................................................................. 24
2.2.1 Word-Associative Memorization ............................................................ 24
2.2.2 Root and Affix Memorization .................................................................. 24
2.2.3 Mechanical Memorization ................................................................................................... 25
2.3 Vocabulary Questions ................................................................................ 26
2.3.1 Root and Affix Clues ............................................................................................................. 26
2.3.2 Definition and Restatement Clues ....................................................................................... 28
2.3.3 Synonym Clues ..................................................................................................................... 30
2.3.4 Antonym Clues ..................................................................................................................... 32
2.3.5 Example Clues ...................................................................................................................... 34
2.3.6 Experience/Description Clues .............................................................................................. 36
2.3.7 Uncommon Meaning ................................................................................... 38
2.3.8 Practice ........................................................................................................ 40
2.3.9 Important Prefix and Roots ................................................................. 52
Sentence Comprehension ................................................................................ 54
3.1 Structure Analysis........................................................................................ 54
3.1.1 Simple Sentence ................................................................................................................... 54
3.1.2 Annotation Method ................................................................................. 68
3.1.3 Compound and Complex Sentence .................................................. 72
3

Version 3.0

[]

3.1.4 Practice......................................................................................................... 76
3.1.5 Grammatical Feature ........................................................................... 82
3.2 Speed Reading and Comprehension ............................................................... 88
3.2.1 Skimming, Scanning, and Scrolling ....................................................................................... 88
3.2.2 Reading in Group Words ...................................................................................................... 90
3.2.3 Internal Structure Skimming ........................................................ 94
3.3 Critical Reading ........................................................................................... 98
3.3.1 Semantic Transformation and Logical Relationship ................. 98
3.3.2 Semantic Information ........................................................................ 106
3.4 Extra Practice .................................................................................................... 112
3.5 Extra Instructions- ..................................................................................... 121
Paragraph Analysis ......................................................................................... 126
4.1 Topical Structure ....................................................................................... 126
4.1.1 Structure of Paragraph....................................................................................................... 126
4.1.2 Topic ................................................................................................................................... 128
4.1.3 Main Idea ........................................................................................................................... 136
4.1.4 Topic Sentence ................................................................................................................... 142
4.1.5 Position of Topic Sentence ................................................................................................. 148
4.1.6 Supporting Details .............................................................................................................. 156
4.1.7 Note Taking ................................................................................................ 160
4.2 Internal Relationships of Paragraph.......................................................... 162
4.2.1 Reference ................................................................................................... 162
4.2.2 Coherence .................................................................................................. 170
4.2.3 Inference .................................................................................................... 178
4.2.4 Purpose Understanding ..................................................................... 182
Passage Organization ..................................................................................... 190
5.1 Organization Patterns ............................................................................... 190
5.1.1 Time Order ......................................................................................................................... 190
5.1.2 Classification ...................................................................................................................... 194
5.1.3 Definition ........................................................................................................................... 196
5.1.4 Cause and Effect................................................................................................................. 198
5.1.5 Comparison and Contrast .................................................................................................. 200
5.2 Additional Practice .................................................................................... 202
4

[] Version 3.0
Approaching Themes ................................................................................. 208
6.1 Natural and Physical Science................................................................. 208
6.1.1 Desert Formation ............................................................................................................... 208
6.1.2 Geology and Landscape ..................................................................................................... 214
6.1.3 Green Icebergs ................................................................................................................... 220
6.2 Biological Sciences ............................................................................................ 227
6.2.1 The origins of Cetaceans .................................................................................................... 227
6.2.2 Swimming Machines .......................................................................................................... 231
6.2.3 Opportunists and Competitors .......................................................................................... 237
6.3 Arts ............................................................................................................................ 242
6.3.1 Applied Arts and Fine Arts ................................................................................................. 242
6.3.2 Loie Fuller ........................................................................................................................... 246
6.3.3 Lascaux Cave Paintings....................................................................................................... 252
6.4 History and Humanities......................................................................... 258
6.4.1 Early cinema ....................................................................................................................... 258
6.4.2 Artisans and Industrialization ............................................................................................ 264
6.4.3 Nineteenth-Century Politics in the United States .............................................................. 270
6.5 Social Sciences .................................................................................................. 276
6.5.1 Aggression .......................................................................................................................... 276
6.5.2 The Expression of Emotions ............................................................................................... 282
Answer Key & Explanation .................................................................................................................. 288
Chapter 1......................................................................................................................................... 288
Chapter 2......................................................................................................................................... 290
Chapter 3......................................................................................................................................... 295
Chapter 4......................................................................................................................................... 301
Chapter 5......................................................................................................................................... 305
Chapter 6......................................................................................................................................... 307

Version 3.0

[]

[] Version 3.0

20134530

Steve Jobs Stay hungryStay foolish

Outman

somebody

ABCD

Version 3.0

[]

iBT
1.1 Key Features

Section
Reading

Number of Questions
3~4 Passages
Approximately 700 words each
12~14 questions per passage
Listening 2~3 Conversations
12~25 exchanges each (3 mins)
5 questions per conversation
4~6 Lectures
500~800 words each (3~5 mins)
6 questions per lecture
BREAK
Speaking 2 Independent Tasks
(preparation: 15 sec/ response: 45 sec)
1 personal experience
1 personal choice/opinion
2 Integrated Tasks: Read-Listen-Speak
(preparation: 30 sec/ response: 60 sec)
1 campus situation topic
Reading: 75~100 words (45 secs)
Conversation: 150~180 words (60~80 secs)
1 academic course topic
Reading: 75~100 words (45 secs)
Lecture: 150~220 words (60~90 secs)
2 Integrated Tasks: Listen-Speak
(preparation: 20 sec/ response: 60 sec)
1 campus situation topic
Conversation: 180~220 words (60~90 secs)
1 academic course topic
Lecture: 230~280 words (90~120 secs)
Writing
1 Integrated Task: Read-Listen-Write (20 mins)
Reading: 230~300 words (3 mins)
Lecture: 230~300 words (2 mins)
A summary of 150~225 words
1 Independent Task (30 mins)
A minimum 300-word essay

Timing
60~80
mins

Score
30 points

60~90
mins

30 points

10 mins 0 point

20 mins 30 points

50 mins 30 points

[] Version 3.0

TOEFL Orientation
The TOEFL (Test Of English as a Foreign Language) is a test of an individuals ability to
use and understand English in an academic setting designed and administered by ETS
(Educational Testing Service). It was developed to address the problem on ensuring English
language prociency for non-native speakers wishing to study at American universities. It
has become an admission requirement for non-native English speakers at many Englishspeaking colleges and universities. Additionally, institutions such as government agencies,
licensing bodies, businesses, or scholarship programs may require this test. A TOEFL score is
valid for two years and then will no longer be officially reported. 1

Key Features of TOEFL

TOEFL iBT Reading Structure

3~4
700~800 /
12~14 /
20 /
12~4
0~30

Transfer Table of Raw points to Scaled Score 2


Raw Point
45
44
43
42
41
40
39
38
37
36
35
34
33
32
1
2

Scaled Score
30
30
29
29
29
28
28
27
27
26
26
25
24
23

Raw Point
31
30
29
28
27
26
25
24
23
22
21
20
19
18

Scaled Score
23
22
21
20
19
18
17
16
15
14
13
12
11
9

Test and Score Summary for TOEFL Internet Based Test, Educational Testing Service, 2011
The Official Guide to the New TOEFL iBT, Third Edition, Educational Testing Service.

Version 3.0

[]

From pBT to iBT



10

[] Version 3.0

1.2 In-Depth Analysis


1.2.1
Vocabulary and
grammar

Major ideas and


important information

Relative importance
of ideas

Organization of
ideas and concept

1.2.2 Classification of the Questions

Basic Comprehension Questions


1. Vocabulary questions (3 to 5 questions per set)
To identify the meanings of words and phrases in the reading passage
2. Factual Information questions (3 to 6 questions per set)
To identify the specic information that is explicitly stated in the passage
3. Negative Factual Information questions (0 to 2 questions per set)
To identify what information is NOT mentioned in the passage
4. Reference questions (0 to 2 questions per set)
To identify the referential relationship between the words in the passage
5. Sentence Simplication questions (0 to1 question per set)
To choose the sentence that best paraphrases the essential information in the highlighted
sentence

Inference Questions
6. Inference questions (0 to 2 questions per set)
To identify an idea that is not explicitly stated but strongly implied in the passage
7. Rhetorical Purpose questions(0 to 2 questions per set)
To identify why the author uses particular words, phrases, or sentences
8. Insert Text question (0 to 1 question per set)
To identify where the best place for an example sentence would be in the passage

Reading to Learn Questions


9. Prose Summary (0 to 1 question per set)
To complete a summary chart with major ideas from the passage
Worth up to 2 points, and partial credit is given
10.

Fill in a Table (0 to 1 question per set)


To identify and organize the major ideas of the passage into table categories
Worth up to 3 points for tables with 5 correct answers
Worth up to 4 points for tables with 7 correct answers
11

Version 3.0

[]

1.2.3 Subject Classification


Natural and physical sciences 30%
Astronomy (
) and Astrophysics
Chemistry
Geology (
) , Geography, and Geosciences
Marine (
) Sciences and Oceanography
Meteorology (
) and Atmospheric Science
Physics
Agricultural Sciences
Biological Sciences 20%
Biological and Biomedical Sciences
Anatomy (
)
Biochemistry
Botany (
) and Plant Sciences
Cellular (
), Molecular (
) and Structural Biology
Ecology (
), Environmental Biology and Evolutionary (
Entomology (
)
Genetics (
), Developmental Biology and Reproductive (
Marine Biology
Microbiological (
) Sciences
Neuroscience (
)
Nutrition (
)
Paleontology (
) & Zoology (
)
Arts and American History 20%
Applied Arts and Design
Architecture (
)
Art and Art History
Film, Television and Video
Music & Performing Arts
Social Sciences 10%
Communication and Media
Economics
Political Science and International Affairs
Psychology (
)
Social Sciences and Sociology (
)
Anthropology (
) & Archaeology (
)
Humanities 10%
History & Humanities
Language and Literature
Linguistic (
) Studies
Philosophy (
) and Ethics (
)
Biography and Others 10%

12

) Theory
) Biology

[] Version 3.0

Comfort Zone
The comfort zone is a behavioral state within which a person operates in an anxiety-neutral
condition, using a limited set of behaviors to deliver a steady level of performance, usually
without a sense of risk. A persons personality can be described by his or her comfort zones.
Highly successful persons may routinely step outside their comfort zones, to accomplish what
they wish. A comfort zone is a type of mental conditioning that causes a person to create and
operate mental boundaries. Such boundaries create an unfounded sense of security. Like
inertia, a person who has established a comfort zone in a particular axis of his or her life, will
tend to stay within that zone without stepping outside of it. To step outside a comfort zone, a
person must experience the new and different responses that then occur within their
environment. 3
Question: Comfort Zone ?
Answer: ______________________________________________________________

1.2.4 Sources of Contents





Wikipedia......
3

Adapted from http://en.wikipedia.org/wiki/Comfort_zone

13

Version 3.0

1.3

[]

Additional Remarks
TOEFL iBT Total and Section Score Means- by Geographic Region: Asia

Rank
1
2
3
4
5
6
7
7
9
10
11
11
13

Country
Singapore
India
Pakistan
Malaysia
Philippines
Bangladesh
Hong Kong
Republic of Korea
Sri Lanka
Kazakhstan
Indonesia
DPR of Korea
China

Reading
24
22
21
22
21
20
19
21
19
18
19
19
20

Listening
25
23
22
22
22
20
20
20
20
20
19
19
18

TOEFL

www.usnews.com
www.nytimes.com
www.ted.com
www.yeeyan.com
www.jstor.org
www.wikipedia.org
www.gter.net
www.taisha.org
14

Speaking
24
23
24
21
23
21
21
20
21
21
20
20
19

Writing
26
23
23
24
23
22
22
21
21
20
21
20
20

Total
99
92
90
89
88
83
82
82
81
79
78
78
77

[] Version 3.0

1.4

Test Yourself
THE LONG-TERM STABILITY OF ECOSYSTEMS 4

Plant communities assemble themselves flexibly, and their particular structure depends on the
specific history of the area. Ecologists use the term succession to refer to the changes that
happen in plant communities and ecosystems over time. The first community in a succession
is called a pioneer community, while the long-lived community at the end of succession is
called a climax community. Pioneer and successional plant communities are said to change
over periods from 1 to 500 years. These changes--in plant numbers and the mix of species-are cumulative. Climax communities themselves change but over periods of time greater than
about 500 years.
An ecologist who studies a pond today may well find it relatively unchanged in a years time.
Individual fish may be replaced, but the number of fish will tend to be the same from one
year to the next. We can say that the properties of an ecosystem are more stable than the
individual organisms that compose the ecosystem.
At one time, ecologists believed that species diversity made ecosystems stable. They believed
that the greater the diversity the more stable the ecosystem. Support for this idea came from
the observation that long-lasting climax communities usually have more complex food webs
and more species diversity than pioneer communities. Ecologists concluded that the apparent
stability of climax ecosystems depended on their complexity. To take an extreme example,
farmlands dominated by a single crop are so unstable that one year of bad weather or the
invasion of a single pest can destroy the entire crop. In contrast, a complex climax
community, such as a temperate forest, will tolerate considerable damage from weather to
pests.
The question of ecosystem stability is complicated, however. The first problem is that
ecologists do not all agree what stability means. Stability can be defined as simply lack of
change. In that case, the climax community would be considered the most stable, since, by
definition, it changes the least over time. Alternatively, stability can be defined as the speed
with which an ecosystem returns to a particular form following a major disturbance, such as
fire. This kind of stability is also called resilience. In that case, climax communities would be
the most fragile and the least stable, since they can require hundreds of years to return to the
climax state.
Even the kind of stability defined as simple lack of change is not always associated with
maximum diversity. At least in temperate zones, maximum diversity is often found in midsuccessional stages, not in the climax community. Once a redwood forest matures, for
example, the kinds of species and the number of individuals growing on the forest floor are
reduced. In general, diversity, by itself, does not ensure stability. Mathematical models of
ecosystems likewise suggest that diversity does not guarantee ecosystem stability--just the
4

Adapted from

15

Version 3.0

[]

opposite, in fact. A more complicated system is, in general, more likely than a simple system
to break down. A fifteen-speed racing bicycle is more likely to break down than a childs
tricycle.
Ecologists are especially interested to know what factors contribute to the resilience of
communities because climax communities all over the world are being severely damaged or
destroyed by human activities. The destruction caused by the volcanic explosion of Mount St.
Helens, in the northwestern United States, for example, pales in comparison to the destruction
caused by humans. We need to know what aspects of a community are most important to the
communitys resistance to destruction, as well as its recovery.
Many ecologists now think that the relative long-term stability of climax communities comes
not from diversity but from the patchiness of the environment, an environment that varies
from place to place supports more kinds of organisms than an environment that is uniform. A
local population that goes extinct is quickly replaced by immigrants from an adjacent
community. Even if the new population is of a different species, it can approximately fill the
niche vacated by the extinct population and keep the food web intact.

Questions
Paragraph 1: Plant communities assemble themselves flexibly, and their particular structure
depends on the specific history of the area. Ecologists use the term succession to refer to
the changes that happen in plant communities and ecosystems over time. The first community
in a succession is called a pioneer community, while the long-lived community at the end of
succession is called a climax community. Pioneer and successional plant communities are
said to change over periods from 1 to 500 years. These changes--in plant numbers and the
mix of species-are cumulative. Climax communities themselves change but over periods of
time greater than about 500 years.
1. The word particular in the passage is closest in meaning to
A. Natural
B. Final
C. Specific
D. Complex
2. According to paragraph 1, which of the following is NOT true of climax communities?
A. They occur at the end of a succession.
B. They last longer than any other type of community.
C. The numbers of plants in them and the mix of species do not change
D. They remain stable for at least 500 years at a time.

16

[] Version 3.0
Paragraph 2: An ecologist who studies a pond today may well find it relatively unchanged
in a years time. Individual fish may be replaced, but the number of fish will tend to be the
same from one year to the next. We can say that the properties of an ecosystem are more
stable than the individual organisms that compose the ecosystem.
3. According to paragraph 2, which of the following principles of ecosystems can be learned
by studying a pond?
A. Ecosystem properties change more slowly than individuals in the system.
B. The stability of an ecosystem tends to change as individuals are replaced.
C. Individual organisms are stable from one year to the next.
D. A change in the members of an organism does not affect an ecosystems properties
Paragraph 3: At one time, ecologists believed that species diversity made ecosystems stable.
They believed that the greater the diversity the more stable the ecosystem. Support for this
idea came from the observation that long-lasting climax communities usually have more
complex food webs and more species diversity than pioneer communities. Ecologists
concluded that the apparent stability of climax ecosystems depended on their complexity. To
take an extreme example, farmlands dominated by a single crop are so unstable that one year
of bad weather or the invasion of a single pest can destroy the entire crop. In contrast, a
complex climax community, such as a temperate forest, will tolerate considerable damage
from weather to pests.
4. According to paragraph 3, ecologists once believed that which of the following illustrated
the most stable ecosystems?
A. Pioneer communities
B. Climax communities
C. Single-crop farmlands
D. Successional plant communities
Paragraph 4: The question of ecosystem stability is complicated, however. The first problem
is that ecologists do not all agree what stability means. Stability can be defined as simply
lack of change. In that case, the climax community would be considered the most stable,
since, by definition, it changes the least over time. Alternatively, stability can be defined as
the speed with which an ecosystem returns to a particular form following a major disturbance,
such as fire. This kind of stability is also called resilience. In that case, climax communities
would be the most fragile and the least stable, since they can require hundreds of years to
return to the climax state.
5. According to paragraph 4, why is the question of ecosystem stability complicated?
A. The reasons for ecosystem change are not always clear.
B. Ecologists often confuse the word stability with the word resilience.
C. The exact meaning of the word stability is debated by ecologists.
D. There are many different answers to ecological questions.
17

Version 3.0

[]

6. According to paragraph 4, which of the following is true of climax communities?


A. They are more resilient than pioneer communities.
B. They can be considered both the most and the least stable communities.
C. They are stable because they recover quickly after major disturbances.
D. They are the most resilient communities because they change the least over time.
Paragraph 5: Even the kind of stability defined as simple lack of change is not always
associated with maximum diversity. At least in temperate zones, maximum diversity is often
found in mid-successional stages, not in the climax community. Once a redwood forest
matures, for example, the kinds of species and the number of individuals growing on the
forest floor are reduced. In general, diversity, by itself, does not ensure stability.
Mathematical models of ecosystems likewise suggest that diversity does not guarantee
ecosystem stability--just the opposite, in fact. A more complicated system is, in general, more
likely than a simple system to break down. A fifteen-speed racing bicycle is more likely to
break down than a childs tricycle.

7. Which of the following can be inferred from paragraph 5 about redwood forests?
A. They become less stable as they mature.
B. They support many species when they reach climax.
C. They are found in temperate zones.
D. They have reduced diversity during mid-successional stages.
8. The word guarantee in the passage is closest in meaning to
A. Increase
B. Ensure
C. Favor
D. Complicate
9. In paragraph 5, why does the author provide the information that A fifteen-speed racing
bicycle is more likely to break down than a childs tricycle?
A. To illustrate a general principle about the stability of systems by using an everyday
example
B. To demonstrate that an understanding of stability in ecosystems can be applied to help
understand stability in other situations
C. To make a comparison that supports the claim that, in general, stability increases with
diversity
D. To provide an example that contradicts mathematical models of ecosystems

18

[] Version 3.0
Paragraph 6: Ecologists are especially interested to know what factors contribute to the
resilience of communities because climax communities all over the world are being severely
damaged or destroyed by human activities. The destruction caused by the volcanic explosion
of Mount St. Helens, in the northwestern United States, for example, pales in comparison to
the destruction caused by humans. We need to know what aspects of a community are most
important to the communitys resistance to destruction, as well as its recovery.
10. The word pales in the passage is closest in meaning to
A. Increases proportionally
B. Differs
C. Loses significance
D. Is common
Paragraph 7: Many ecologists now think that the relative long-term stability of climax
communities comes not from diversity but from the patchiness of the environment, an
environment that varies from place to place supports more kinds of organisms than an
environment that is uniform. A local population that goes extinct is quickly replaced by
immigrants from an adjacent community. Even if the new population is of a different species,
it can approximately fill the niche vacated by the extinct population and keep the food web
intact.
11. Which of the sentences below best expresses the essential information in the highlighted
sentence in the passage? Incurred choices change the meaning in important ways or leave
out essential information.
A. Ecologists now think that the stability of an environment is a result of diversity rather
than patchiness.
B. Patchy environments that vary from place to place do not often have high species
diversity.
C. Uniform environments cannot be climax communities because they do not support as
many types of organisms as patchy environments.
D. A patchy environment is thought to increase stability because it is able to support a
wide variety of organisms.
12. The word adjacent in the passage is closest in meaning to
A. Foreign
B. Stable
C. Fluid
D. Neighboring

19

Version 3.0

[]

Paragraph 6: Ecologists are especially interested to know what factors contribute to the
resilience of communities because climax communities all over the world are being severely
damaged or destroyed by human activities. The destruction caused by the volcanic
explosion of Mount St. Helens, in the northwestern United States, for example, pales in
comparison to the destruction caused by humans. We need to know what aspects of a
community are most important to the communitys resistance to destruction, as well as its
recovery.
13. Look at the four squares [] that indicate where the following sentence could be added to
the passage.
In fact, damage to the environment by humans is often much more severe than damage
by natural events and processes.
Where would the sentence best fit? Click on a square to add the sentence to the passage.
14. Directions: An introductory sentence for a brief summary of the passage is provided below.
Complete the summary by selecting the THREE answer choices that express the most
important ideas in the passage. Some sentences do not belong in the summary because
they express ideas that are not presented in the passage or are minor ideas in the passage.
This question is worth 2 points.
The process of succession and the stability of a climax community can change over time.

Answer choices
A. The changes that occur in an ecosystem from the pioneer to the climax community
can be seen in one human generation.
B. A high degree of species diversity does not always result in a stable ecosystem.
C. The level of resilience in a plant community contributes to its long-term stability.
D. Ecologists agree that climax communities are the most stable types of ecosystems.
E. Disagreements over the meaning of the term stability make it difficult to identify
the most stable ecosystems.
F. The resilience of climax communities makes them resistant to destruction caused by
humans.

20

[] Version 3.0

1.5

Defect Analysis

A. 30-60 words/min
B. 60-90 words/min

C. 90-120 words/min
D. > 120 words/min

A. < 10 mins
B. 10-20 mins

C. 20-30 mins
D. > 30 mins

A.
B.

C.
D.

A.
B.

C.
D.

A.
C.
B. D.

A.
B.

C.
D.

A.
B.

C.
D.

A.
C.
B. D.

21

Version 3.0

[]

1.

2.

3.

4.

statement
inference

5.

6.

passageparagraphsentenceword
word:
sentence
paragraph
passage

22

[] Version 3.0

Vocabulary Workshop
2.1 Basic Recognition
Filling in the blanks with one of the words listed below

derived
annihilation
imminent
advocate
innovative
components
dearth
facilitate
stimulant
repercussions

1. Knowing that the arrival of the president was (


), officials were scurrying around at
a frantic pace trying to make sure that everything was in order and ready for his arrival.
2. Changes in the regulations governing the countrys banks had enormous and devastating
(
) on taxpayers.
3. The fear of civil war in Iraq persists because the Sunni and Shia sects of Islam feel so
much hostility toward each other that their goal is not cooperation but (
).
4. Many people still believe that playing Mozart for infants will improve their IQ; however,
there is a(n) (
) of hard evidence proving such claims.
5. Throughout his Supreme Court career, Chief Justice Earl Warren was a strong (
for the rights of the individual.
6. The journalist who was supposed to (
miserably at the job.
7. People think that alcohol is a(n) (
8.

Opium is a drug (

) the debate between the two candidates failed

),but actually it has the opposite effect.

) from poppies.

9. The directions suggested that not all of the (


the box.

) for the hot tub had been included in

10. Steve Jobs, the head of the Apple, was from the beginning, a(n) (
was always one step ahead of his competitors.
23

) thinker who

Version 3.0

2.2

[]

Word Memorization and Tricks

2.2.1 Word-Associative Memorization

2.2.2 Root and Affix Memorization


Root provides words with their essential or main meanings. The root meaning of a word
never changes no matter what prexes (beginnings of words) or sufxes (endings of words)
are added on to it, e.g., farm, farmer, farming
Afx: Elements attached to a root.
Prex: Prexes dont affect a words core meaning. But they do modify the words
meaning as a whole, e.g., social and antisocial; annual and biannual
Sufx: Most sufxes tell you less about word meaning and more about word function,
changing adjectives to nouns or nouns to adverbs, e.g., dry and dryness; grace and
gracefully.

24

[] Version 3.0

2.2.3 Mechanical Memorization

45
1
L1~3
*L1~3

2
L4~6
*L1~3
*L4~6

3
L7~9
*L4~6
*L7~9
10
L28~30
*L7~9
*L19~21
*L25~27
*L28~30
17
*L7~9
*L28~30
*L40~42

4
L10~12
*L1~3
*L7~9
*L10~12
11
L31~33
*L10~12
*L22~24
*L28~30
*L31~33
18
*L10~12
*L31~33
*L43~45

5
L13~15
*L4~6
*L10~12
*L13~15
12
L34~36
*L13~15
*L25~27
*L31~33
*L34~36
19
*L13~15
*L34~36

6
L16~18
*L7~9
*L13~15
*L16~18
13
L37~39
*L16~18
*L28~30
*L34~36
*L37~39
20
*L16~18
*L37~39

7
L19~21
*L10~12
*L16~18
*L19~21
14
L40~42
*L19~21
*L31~33
*L37~39
*L40~42
21
*L19~21
*L40~42

8
L22~24
*L1~3
*L13~15
*L19~21
*L22~24
15
L43~45
*L1~3
*L22~24
*L34~36
*L40~42
*L43~45
22
*L22~24
*L43~45
29
*L43~45
36
*L19~21
43
*L40~42

9
L25~27
*L4~6
*L16~18
*L22~24
*L25~27
16
*L4~6
*L25~27
*L37~39
*L43~45

23
*L25~27

24
*L28~30

25
*L31~33

26
*L34~36

27
*L37~39

28
*L40~42

30
*L1~3
37
*L22~24
44
*L43~45

31
*L4~6
38
*L25~27

32
*L7~9
39
*L28~30

33
*L10~12
40
*L31~33

34
*L13~15
41
*L34~36

35
*L16~18
42
*L37~39

25

Version 3.0

[]

2.3 Vocabulary Questions


2.3.1 Root and Affix Clues
1. As Philadelphia grew from a small town into a city in the 18th century, it became an
increasingly important marketing center for a vast and growing agricultural hinterland. 5
The word hinterland in the passage is closest in meaning to
(A) tradition
(B) association
(C) produce
(D) region
2. Other dimensions along which the two groups differ markedly are density and composition.
The densities of the terrestrial planets average about 5 times the density of water, whereas the
Jovian planets have densities that average only 1.5 times the density of water. 6
The word markedly in the passage is closest in meaning to
(A) essentially
(B) typically
(C) consistently
(D) noticeably
3. The ice trade grew with the growth of cities. Ice was used in hotels, taverns, and hospitals,
and by some forward-looking city dealers in fresh meat, fresh sh, and butter. 7
The phrase forward-looking in the passage is closest in meaning to
(A) progressive
(B) popular
(C) thrifty
(D) well-established
4. With additional time, pressure, and refrozen meltwater from above, the small rn granules
become larger, interlocked crystals of blue glacial ice.

The word interlocked in the passage is closest in meaning to


(A) intermediate
(B) linked
(C) frozen
(D) fully developed
5

Adapted from TOEFL PBT, 2001.01, Educational Testing Services.


Adapted from Planets in sour solar system, TPO 16, Educational Testing Services.
7
Adapted from The Origin of Refrigerators, TOEFL PBT, 1996.01, Educational Testing Services.
8
Adapted from Glacier Formation, TPO 15, Educational Testing Services.
6

26

[] Version 3.0
Vocabulary Enlargement
agricultural
terrestrial
Jovian
tavern
dealer
rn
granule
glacial

[grkltrl]
[trstrl]
[duvin]
[tv()n]
[dil]
[frn]
[grnjul]
[lel]

Sentence Pattern
Grow into
The pictures below show, how insects grow from eggs into adults.

Cell Stems have the amazing ability to transform and grow into other types of cells

Practical Translation Training


__________
________________________
By meeting the legitimate demands of its citizens for ______ and ______, Chinas
leaders can help their country ________ a modern, prosperous, and confident nation.
Further ReadingTerrestrial VS. Jovian
Terrestrial planets are those planets which are
predominantly composed of silicate rocks. The examples
of terrestrial planets in the solar system are Mercury,
Venus, Earth and Mars. These planets resemble the planet
Earth to a significant extent, and hence they are referred to
as Earth-like or terrestrial (derived from the Latin word
the Earth - terra).
Jovian planets are the planets which are not composed of any solid matter. Technically
speaking, the planets which have 10 times more mass than the Earth are classified as the
Jovian planets. The examples in our solar system include planets like the Jupiter, Saturn,
Uranus and Neptune. These planets are referred to as the Jovian planets owing to their stark
resemblance to the planet Jupiter, the largest planet in the solar system. 9
9

Adapted from Abhijit Naik, Jovian Planets VS. Terrestrial Planets, www.buzzle.com.

27

Version 3.0

[]

2.3.2 Definition and Restatement Clues


These clues provide a formal definition/restatement of the word. They usually follow signal
words/phrases: or, means, refers to, is called, colon, semicolon, dash, and parentheses.
Eg. We all know that ants are ferocious eaters. They differ widely in their diets and may be
carnivorous, herbivorous, or omnivorous, meaning that some eat meat; others eat only plants
while others eat a combination of both.
Today, Id like to talk about some of the changes land can undergo, specifically
desertification, process through which land becomes part of a desert.
5. Combines were also coming into use on the great wheat ranches in California and the
Pacific Northwest. These ponderous machines sometimes pulled by as many as 40 horses
reaped the grain, threshed it, and bagged it, all in one simultaneous operation.
The word ponderous in the passage is closest in meaning to
(A) advanced
(B) heavy
(C) complex
(D) rapid
6. Yet it would be wrong to suggest that Rome was somehow a junior partner in Greco-Roman
civilization. The Roman genius was projected into new spheresespecially into those of law,
military organization, administration, and engineering.
The word spheres in the passage is closest in meaning to
(A) abilities
(B) areas
(C) combinations
(D) models
7. The coach observed his teams bizarre behavior (odd actions such as slurring their speech,
making unnecessary fumbles, and trying to run on wobbly knees) and concluded that the
players were suffering from dehydration.
The word bizarre in the passage is closest in meaning to
(A) unusual
(B) a flea market and bake sale
(C) appropriate
(D) ordinary
8. In the seventeenth century the organ, the clavichord, and the harpsichord became the chief
instruments of the keyboard group, a supremacy they maintained until the piano supplanted
them at the end of the eighteenth century.
The word supremacy in the passage is closest in meaning to
(A) a suggestion
(B) an improvement
(C) a dominance
(D) a development.
28

[] Version 3.0
Vocabulary Enlargement
reap
thresh
microscopic
skeleton
slur
fumble
wobbly
insult
Witty

[rip]
[r]
[makrskpk]
[skltn]
[sl]
[fmbl]
[wbli]
[nslt]
[wti]

Sentence Pattern
Come into use
Recently computers have come into use for storage and retrieval of these data.

The more recently introduced method has now come into more widespread use.

Practical Translation Training



The acronym __________________________________________________________
Further ReadingBRICS
BRICS, originally "BRIC" before the inclusion of South Africa in 2010, is the title of an
association of emerging national economies: Brazil, Russia, India, China and South Africa.
With the possible exception of Russia, the BRICS members are all developing or newly
industrialized countries, but they are distinguished by their large, fast-growing economies
and significant influence on regional and global affairs. As of 2013, the five BRICS
countries represent almost 3 billion people, with a combined nominal GDP of US$14.9
trillion, and an estimated US$4 trillion in combined foreign reserves. Presently, South
Africa holds the chair of the BRICS group. 10

10

Adapted from http://en.wikipedia.org/wiki/BRICS

29

Version 3.0

[]

2.3.3 Synonym Clues


Synonym clues use a familiar word with a similar meaning or a synonym to help show the
meaning of an unknown word.
Jays feelings about rap music were ambivalent. He admired the clever rhyming lyrics but
detested the ideas they conveyed.
During World War 2, the British repeatedly tried to decipher the Germans code. However,
they couldnt figure it out until mathematician Alan Turing discovered the key to the code.
9. To commemorate the 4th of July, members of our community remember and honor the event
by participating in a main street parade full of crepe-papered bicycles, handshaking
politicians, and homemade floats.
The word commemorate in the passage is closest in meaning to
(A) honor or observe an occasion
(B) harm the memory of
(C) use a memory tool for studying more effectively
(D) form a committee for a particular purpose
10. If all of ones money is spent on clothes, there may be none left to buy food or go to the
movies. Similarly, a plant or animal cannot squander all its energy on growing a big body if
none would be left over for reproduction, for this is the surest way to extinction.
The word squander in the passage is closest in meaning to
(A) extend
(B) transform
(C) activate
(D) waste
11. Among all the abilities with which an individual may be endowed, musical talent appears
earliest in life. Very young children can exhibit musical precocity for different reasons. Some
develop exceptional skill as a result of a well-designed instructional regime, such as the
Suzuki method for the violin.
The word precocity in the passage is closest in meaning to
(A) strong interest
(B) good luck
(C) advanced skill
(D) personal style
12. His recollections of the past were vague at best; thoughts of the past were so depressing he
did not wish to retrieve them.
The word recollections in the passage is closest in meaning to
(A) words
(B) insults
(C) purchases
(D) memories
30

[] Version 3.0
Vocabulary Enlargement
crepe paper
homemade
epidermis
waxy
spiky
trichome
larvae
vague
retrieve

[krep] [pep]
[hmmed]
[epdms]
[wks]
[spaki]
[trkom]
[lrvi]
[veg]
[rtriv]

Sentence Pattern
As a result of
Workers approaching retirement should be saving more, as a result of low rates.

Practical Translation Training



_______________________, humans should___________________________________
Further ReadingIndependence Day (film) 11
Independence Day is a 1996 American military science
fiction disaster film about an alien invasion of Earth.
The narrative focuses on a disparate group of people
who converge in the Nevada desert and, along with the
rest of the human population, participate in a lastchance counterattack on July 4, the same date as the
Independence Day holiday in the United States. It was
directed by German director Roland Emmerich, who
co-wrote the script with producer Dean Devlin.
While promoting Stargate in Europe, Emmerich came up with the idea for the film when
fielding a question about his own belief in the existence of alien life. He and Devlin decided
to incorporate a large-scale attack when noticing that aliens in most invasion films travel long
distances in outer space only to remain hidden when reaching Earth. Principal photography
for the film began in July 1995 in New York City, and the film was officially completed on
June 20, 1996.
11

Adapted from http://en.wikipedia.org/wiki/Independence_Day_(film)

31

Version 3.0

[]

2.3.4 Antonym Clues


An antonym clue provides a word with an opposite meaning to the unknown word. It is
useful to look at signal words like but, however, in contrast, on the other hand, though, while.
13. His patients thought of the doctor as altruistic. However, the nurses he worked with did not
consider him so selfless.
The word altruistic in the passage is closest in meaning to
(A) selfish
(B) giving
(C) impatient
(D) religious
14. Within a very short time, however, the incongruity of playing lively music to a solemn film
became apparent, and film pianists began to take some care in matching their pieces to the
mood of the film.
The word solemn is closest in meaning to
(A) simple
(B) serious
(C) short
(D) silent
15. In contrast to her normally hopeful nature, Annie grew more pessimistic about her career
choice after each test in TOEFL exam.
The word pessimistic is closest in meaning to
(A) characterized by a hopeful outlook
(B) cheerful in daily work
(C) annoying
(D) characterized by a negative outlook
16. The main conflict in the novel is between a mother who places a high value on hard work and
honor and a son who repudiates his mothers values instead preferring the easier path to
fortune and celebrity.
The word repudiates is closest in meaning to
(A) refuses to accept
(B) lives up to
(C) tries to understand
(D) makes the best of
17. Thomas Hancock, an English settler, wrote thanking his plant supplier for a gift of some tulip
bulbs from England, but his letter the following year grumbled that they were all dead.
The word grumbled in the passage is closest in meaning to
(A) denied
(B) warned
(C) complained
(D) explained
32

[] Version 3.0
Vocabulary Enlargement
impatient
rebellion
audacious

[mpe()nt]
[rbeljn]
[des]

Sentence Pattern
If anything
If anything, his father worked with even more enthusiasm than usual

Come up with
If Republicans do not come up with a reasonable set of compromise measures over the
next few weeks, he says that he intends to forge ahead anyway.

Practical Translation Training



Everyone, regardless of age or background, ____________________________________
Further Reading2004 French-Ivorian Clashes
In 2002, a civil war broke out in Cte dIvoire between Ivorian military and other forces loyal to
Laurent Gbagbo, the Ivorian president. Although most of the fighting ended by late 2004, the country
remained split in two, with a rebel-held north and a government-held south. On 6 November 2004,
Gbagbo ordered air strikes on Ivorian rebels. At least one Ivorian Sukhoi Su-25 bomber attacked a
French peacekeeping position in the rebel town of Bouak at 1 pm, killing nine French soldiers and
wounding 31.
On 7 November, armed mobs of Ivorians loyal to the government took to the streets of Abidjan to
protest against France. Crowds of young Ivorians attacked a residential district made up of French
citizens, which had to be evacuated by airlift as mobs burst into their apartment buildings. Armored
cars carried armed protesters to join the fight, and French helicopters flew over Abidjan and dropped
concussion grenades, while French armored vehicles carried troops to put down the riots. Protesters
erected burning roadblocks, and French gunboats were positioned beneath the bridges. Fighting
continued, and, by Sunday, French forces were still not in control of the city. At Abidjans airport,
French and Ivorian troops exchanged fire, and a French military plane was reportedly damaged. As
the riots in the streets continued, French soldiers opened fire on Ivorian rioters; the French
Government stated that 20 were killed while Ivorian authorities placed the death toll at 60. 12
12

Adapted from http://en.wikipedia.org/wiki/2004_French-Ivorian_clashes

33

Version 3.0

[]

2.3.5 Example Clues


Example clues provide examples of the unknown word. The clue is often introduced by
signal words such as include, for example, for instance, such as, and like.
18. The British aristocracy doesnt generally have a way with ordinary people. Princess Diana
was a spectacular exception to that rule. 13
The word aristocracy is closest in meaning to
(A) theater
(B) nobility
(C) kings
(D) folks
19. Joan of Arc was valiant to the end, refusing to show any fear as men with torches prepared to
burn her at the stake.

14

The word valiant is closest in meaning to


(A) mad
(B) brave
(C) quiet
(D) outraged
20. The 1862 Homestead Act, designed to give free land to the poor, produced a lot of fraudulent
activity. For instance, large landowners would send their employees to apply for deeds to the
free land; then the landless employees would sign over the deeds to their wealthy employers.
The word fraudulent is closest in meaning to
(A) unusual
(B) brilliant
(C) illegal
(D) amusing
21. To save money, one might adopt more frugal practices, including reusing plastic bags, buying
items on sale, packing a lunch instead of eating fast food each day, and renting movies
instead of going out on weekends.
The word frugal is closest in meaning to
(A) costly
(B) economical
(C) expensive
(D) boring
13

Diana (1 July 1961 31 August 1997), the first wife of Charles, Prince of Wales. She was also well known for
her fund-raising work for international charities and as an eminent celebrity of the late 20th century.
14
Saint Joan of Arc (1412 1431) is a folk heroine of France. She led the French army to several important
victories during the Hundred Years' War

34

[] Version 3.0
Vocabulary Enlargement
spectacular
exception
outrage
Deeds
adopt

[spektkjl]
[kspn]
[atred]
[did]
[dpt]

Sentence Pattern
Have a way with
Do you have a way with people an ability to calm down someone who is angry?
--?
Apply for
What qualifications should I have to apply for this major?

Practical Translation Training



Youve got to admit, _________________________________________

Students may__________________________, but here, they______________________
Further ReadingHundred Years War
The Hundred Years War was a series of conflicts waged from 1337 to 1453 between the Kingdom of
England and the Kingdom of France for control of the French throne. The war had its roots in a
dynastic disagreement dating back to the time of William the Conqueror, who became King of
England in 1066 while retaining possession of the Duchy of Normandy in France. As the rulers of
Normandy and other lands on the continent, the English kings owed feudal homage to the king of
France. In 1337, Edward III of England refused to pay homage to Philip VI of France, leading the
French king to claim confiscation of Edwards lands in Aquitaine. Edward responded by declaring
that he, not Philip, was the rightful king of France, a claim dating to 1328, when Charles IV of France
died without a male heir. Edward was the closest male relative of Charles IV as son of Isabella of
France, daughter of Philip IV of France and sister of Charles IV. But instead, Philip VI, the son of
Philip IVs younger brother, Charles of Valois, was crowned king of France in accordance with Salic
Law, which disqualified female succession and the succession of males descended through female
lines. The question of legal succession to the French crown was central to the war over generations. 15
15

Adapted from http://en.wikipedia.org/wiki/Hundred_Years'_War

35

Version 3.0

[]

2.3.6 Experience/Description Clues


These clues rely on your experience and common knowledge to understand an unknown
word. You need to use your logic and reasoning skills.
22. Asked a difcult question by a student at the rally, the senator pondered before answering.
The word pondered is closest in meaning to
(A) murmured
(B) wrote
(C) laughed
(D) thought
23. In 1980, with Democratic President Jimmy Carters popularity abysmally low, Ronald
Reagan, who promised a break with the past, won the presidency.
The word abysmally is closest in meaning to
(A) climbing
(B) cheerfully
(C) horribly
(D) randomly
24. Doctors often prescribe them drugs for hyperactive children to lengthen attention span; to
narcoleptic to keep them awake, to sufferers of short-term depression to cheer them up.
The word narcoleptic is closest in meaning to
(A) salesclerks
(B) people who sleep excessively
(C) excessive eaters
(D) rock stars
25. One result of rent control is a decrease in the construction of new rental units. Rent Controls
have artificially depressed the most important long-term determinant of profitability rents.
The word depressed is closest in meaning to
(A) saddened
(B) created
(C) lowered
(D) defeated
26. When the gas reaches the air, it comes into contact with available oxygen from atmosphere
and combines with the oxygen to produce nitrogen dioxide (NO2), which is a gas with a
brownish hue.
The word hue is closest in meaning to
(A) color
(B) odor
(C) thickness
(D) smoke
36

[] Version 3.0
Vocabulary Enlargement
rally
hyperactive
intellectual
frenzy

[rli]
[haprktv]
[,ntlktul]
[frenz]

Further ReadingIran Hostage Crisis 16


The Iran hostage crisis was a diplomatic crisis between Iran and the United States. Fifty-two
Americans were held hostage for 444 days (November 4, 1979 to January 20, 1981), after a
group of Islamist students and militants supporting the Iranian Revolution took over the
American Embassy in Tehran. President Carter called the hostages "victims of terrorism and
anarchy," adding that "the United States will not yield to blackmail."
The crisis has been described as an entanglement of "vengeance and mutual
incomprehension." In Iran, the hostage taking was widely seen as a blow against the United
States and its influence in Iran, its perceived attempts to undermine the Iranian Revolution,
and its longstanding support of the recently overthrown Shah Mohammad Reza Pahlavi of
Iran. Following his overthrow, the Shah was allowed into the US for medical treatment. In
the United States, the hostage-taking was seen as an outrage violating a centuries-old
principle of international law granting diplomats immunity from arrest and diplomatic
compounds inviolability.
The episode reached a climax when, after failed attempts
to negotiate a release, the United States military
attempted a rescue operation off the USS Nimitz. On
April 24, 1980, Operation Eagle Claw resulted in a
failed mission, the deaths of eight American servicemen,
one Iranian civilian, and the destruction of two aircraft.
On July 27, 1980, the former Shah died; then, in
September, Iraq invaded Iran. These two events led the
Iranian government to enter negotiations with the U.S.,
with Algeria acting as a mediator. The hostages were
formally released into United States custody the day
after the signing of the Algiers Accords, just minutes
after the new American president Ronald Reagan was
sworn into office.
Considered a "pivotal episode" in the history of IranUnited States relations, political
analysts cite the crisis as having weighed heavily on U.S. President Jimmy Carters
presidency and run for reelection in the 1980 presidential election. In Iran, the crisis
strengthened the prestige of the Ayatollah Ruhollah Khomeini and the political power of
those who supported theocracy and opposed any normalization of relations with the West.
The crisis also marked the beginning of U.S. legal action, or economic sanctions against Iran,
that further weakened ties between Iran and the United States.
16

Adapted from http://en.wikipedia.org/wiki/Iran_hostage_crisis

37

Version 3.0

[]

2.3.7 Uncommon Meaning

27. The Native Americans of northern California were highly skilled in basketry, using grasses,
barks, and roots to fashion articles of all sorts and sizes-not only trays, containers, and
cooking pots, but hats, boats, fish traps, and baby carriers. 17
The word fashion in the passage is closest in meaning to
(A) maintain
(B) organize
(C) trade
(D) create
28. His big break came with the novel So big (1924), which was awarded the Pulitzer in
literature.
The word break in the passage is closest in meaning to
(A) rupture
(B) revelation
(C) opportunity
(D) rest
29. A snowfall consists of myriads of minute ice crystals that fall to the ground in the form of
frozen precipitation.
The word minute in the passage is closest in meaning to
(A) tiny
(B) quick
(C) clear
(D) sharp
30. In seeking to describe the origins of theater, one must rely primarily on speculation, since
there is little concrete evidence on which to draw. The most widely accepted theory,
championed by anthropologists in the late nineteenth and early twentieth centuries, envisions
theater as emerging out of myth and ritual. 18
The word championed is closest in meaning to
(A) changed
(B) debated
(C) created
(D) supported

17
18

Adapted from TOEFL PBT, 1999.01, Educational Testing Services.


Adapted from The origin of Theater, TPO 1, Educational Testing Services.

38

[] Version 3.0
Vocabulary Enlargement
meteorologist
myriad
precipitation
igneous
obsidian

[mitrldst]
[mrd]
[pr,spte()n]
[gns]
[bsdn]

Sentence Pattern
Be skilled in
Life is like a boat. Everybody must learn to be skilled in boating.

Practical Translation Training



You might be__________________, but______________________________________
Brain FoodEnglish Language Puzzles
1. What occurs twice in a lifetime, but once in every year. Twice in a week but never in a day?
2. Which word, if pronounced right, is wrong, but if pronounced wrong is right?
3. An Arab sheikh tells his two sons to race their camels to a distant city to see who will
inherit his fortune. The one whose camel is slower will win. The brothers, after wandering
aimlessly for days, ask a wise man for advise. After hearing the advice they jump on the
camels and race as fast as they can to the city. What does the wise man say?
4. Dont cheat! Because if you did, the test would be no fun. I promise, there are no tricks to
the test. Read the sentence below and count the Fs in that sentence. Count them ONLY
ONCE. Do not go back and count them again. See solutions for your score.
FINISHED FILES ARE THE RESULT OF YEARS OF SCIENTIFIC STUDY
COMBINED WITH THE EXPERIENCE OF YEARS.
5. What is strange about these sentences?
Was it a car or a cat I saw?
Warsaw was raw.
No lemons, no melon.
Dennis and Edna sinned.
39

Version 3.0

[]

2.3.8 Practice
1. In lowland country almost any spot on the ground may overlie what was once the bed of a
river that has since become buried by soil; if they are now below the waters upper surface
(the water table), the gravels and sands of the former riverbed, and its sandbars, will be
saturated with groundwater.
The word overlie is closest in meaning to
(A) cover
(B) change
(C) separate
(D) surround
2. But neither the human imitative instinct nor a penchant for fantasy by itself leads to an
autonomous theater. Therefore, additional explanations are needed.
The word penchant in the passage is closest in meaning to
(A) compromise
(B) inclination
(C) tradition
(D) respect
3. At the upper timberline the trees begin to become twisted and deformed. This is particularly
true for trees in the middle and upper latitudes, which tend to attain greater heights on ridges,
whereas in the tropics the trees reach their greater heights in the valleys.
The word attain in the passage is closest in meaning to
(A) require
(B) resist
(C) achieve
(D) endure
4. Much of the worlds great architecture has been constructed of stone because of its beauty,
permanence, and availability. In the past, whole cities grew from the arduous task of cutting
and piling stone upon.
The word arduous in the passage is closest in meaning to
(A) difficult
(B) necessary
(C) skilled
(D) shared

40

[] Version 3.0
Vocabulary Enlargement
gravel
imitative
timberline
ridge

[rvl]
[mttv]
[tmblan]
[rd]

Sentence Pattern
Tend to
She stayed at home and tended to her mother.

We all tend to like those similar to us.

This policy tends to the improvement of economic results.

Practical Translation Training



They _____________________________________________they know all the opinions.
Brain FoodEnglish Language Puzzles
1. Of
those
numbers
whose
English
representation in capital letters consists only of
straight lines, only one number has a value
equal to the number of straight line segments
required to write it out. What number is this?

2. What word is pronounced differently when the


first letter is capitalized?

3. Punctuate the following so it makes sense: "That that is is that that is not is not is not that
it it is."

4. What word begins with h, ends with n, contains six letters, and contains eight words
besides itself without transposing a single letter?
41

Version 3.0

[]

5. The first wells were drilled into the Ogallala during the drought years of the early 1930s.
The ensuing rapid expansion of irrigation agriculture, especially from the 1950s onward,
transformed the economy of the region. 19
The word ensuing in the passage is closest in meaning to
(A) continuing
(B) surprising
(C) initial
(D) subsequent
6. The numbers of deer have fluctuated markedly since the entry of Europeans into Puget Sound
country. The early explorers and settlers told of abundant deer in the early 1800s and yet
almost in the same breath bemoaned the lack of this succulent game animal. 20
The phrase in the same breath in the passage is closest in meaning to
(A) impatiently
(B) humorously
(C) continuously
(D) immediately
7. Gas pressure gradually dies out, and oil is pumped from the well. Water or steam may be
pumped down adjacent wells to help push the oil out. 21
The word adjacent in the passage is closest in meaning to
(A) nearby
(B) existing
(C) special
(D) deep
8. Hydroponics has applications beyond basic research, since it facilitates the growing of
greenhouse vegetables during winter. Acroponics, a technique in which plants are suspended
and the roots misted with a nutrient solution, is another method for growing plants without
soil. 22
The word facilitates in the passage is closest in meaning to
(A) slows down
(B) affects
(C) makes easier
(D) focuses on
19

Adapted from
Adapted from
21
Adapted from
22
Adapted from
20

42

[] Version 3.0
Vocabulary Enlargement
drought
bemoan
succulent
nutrient

[drat]
[bmn]
[skjl()nt]
[njutrnt]

Sentence Pattern
Die out
Would the common H7N9 die out if we all stayed at home?

When he says that if you leave them be, they will die out, thats simply not true.

Practical Translation Training



_______________________________________________________________________
Further ReadingBird Flu 23
"Bird flu" is a phrase refers to an illness caused by any of many different strains of influenza
viruses that have adapted to a specific host. All known viruses that cause influenza in birds
belong to the species influenza A virus. All subtypes (but not all strains of all subtypes) of
influenza A virus are adapted to birds, which is why for many purposes avian flu virus is the
influenza A virus. (Note, however, that the "A" does not stand for "avian").
Adaptation is not exclusive. Being adapted towards a particular species does not preclude
adaptations, or partial adaptations, towards infecting different species. In this way, strains of
influenza viruses are adapted to multiple species, though may be preferential towards a
particular host. For example, viruses responsible for influenza pandemics are adapted to both
humans and birds. Recent influenza research into the genes of the Spanish flu virus shows it
to have genes adapted to both birds and humans, with more of its genes from birds than less
deadly later pandemic strains.
While its most highly pathogenic strain (H5N1) had been spreading throughout Asia since
2003, avian influenza reached Europe in 2005, and the Middle East, as well as Africa, the
following year. On January 22, 2012, China reported its second human death due to bird flu
in a month following other fatalities in Vietnam and Cambodia.
23

Adapted from http://en.wikipedia.org/wiki/Avian_influenza_virus

43

Version 3.0

[]

9. Contrary to these theorists, the overwhelming evidence of physical anthropology, linguistics,


and archaeology shows that the Pacific islanders came from Southeast Asia and were skilled
enough as navigators to sail against the prevailing winds and currents. 24
The word overwhelming in the passage is closest in meaning to
(A) powerful
(B) favorable
(C) current
(D) reasonable
10. During a geologically brief 100-million-year period, all modern animal groups (along with
other animals that are now extinct) evolved. This rapid origin and diversification of animals
is often referred to as the Cambrian explosion. 25
The word diversification in the passage is closest in meaning to
(A) emergence of many varieties
(B) steady decline in number
(C) gradual increase in body size
(D) sudden disappearance
11. Smith used mail coaches to travel as much as 10000 miles per year. In 1815 he published the
first modern geological map A Map of the Strata of England and Wales with a Part of
Scotland, map so meticulously researched that it can still be used today. 26
The word meticulously in the passage is closest in meaning to
(A) carefully
(B) quickly
(C) frequently
(D) obviously
12. In 1769 in a little town in Oxford shire, England, a child with the very ordinary name of
William Smith was born into the poor family of a village blacksmith. He received
rudimentary village schooling but mostly he roamed his uncles farm collecting the fossils
that were so abundant in the rocks of the Cotswold hills.
The word rudimentary in the passage is closest in meaning to
(A) thorough
(B) strict
(C) basic
(D) occasional
24

Adapted from
Adapted from
26
Adapted from
25

44

[] Version 3.0
Vocabulary Enlargement
anthropology
linguistics
archaeology

[nrpldi]
[lwstks]
[,rkldi]

Sentence Pattern
Refer to
Study the example and refer to the explanation below.

And if this sentence is true then Santa does not exist, in which case the name Santa
does not refer to any actual thing.

Practical Translation Training



Jim records his workout__________________________________________________
_____________________________________________________________________
Further ReadingThe Cambrian Explosion
Most major animal groups appear for the first time in the fossil
record some 545 million years ago on the geological time scale in
a relatively short period of time known as the Cambrian
explosion. Of great worry to Darwin, the explanation of this
sudden, apparent explosion persists as a source of numerous
major debates in paleobiology. While some scientists believe
there was indeed an explosion of diversity (the so-called
punctuated equilibrium theory elaborated by Nils Eldredge the
late Stephen J. Gould - Models In Paleobiology, 1972), others
believe that such rapid acceleration of evolution is not possible;
they posit that there was an extended period of evolutionary progression of all the animal groups,
the evidence for which is lost in the all but nonexistent precambrian fossil record. Early complex
animals in the Paleozoic may have been nearly microscopic. Apparent fossil animals smaller than
0.2 mm have been found in the Doushantuo Formation, China, forty to fifty-five million years
before the Cambrian (Chen et al. 2004). Much of the early evolution could have simply been too
small to see, much less preserve. Modern molecular technologies (genomics and other omics),
through comparing nucleic acid and amino acid sequences across living species, are enabling the
identification of genetic components and patterns stingily conserved by evolution, from those in
which times of evolutionary branching of the tree of life can be inferred.
45

Version 3.0

[]

13. lr has not been common at Earths since the very beginning of the planets history. Because it
usually exists in a metallic state, it was preferentially incorporated in Earths core as the
planet cooled and consolidated. lr is found in high concentrations in some meteorites, in
which the solar systems original chemical composition is preserved. Even today,
microscopic meteorites continually bombard Earth, falling on both land and sea. 27
The word bombard in the passage is closest in meaning to
(A) approach
(B) strike
(C) pass
(D) circle
14. The coastal hypothesis suggests an economy based on marine mammal hunting, saltwater
fishing gathering, and the use of watercraft. Because of the barrier of ice to the east, the
Pacific Ocean to the west, and populated areas to the north, there may have been a greater
impetus for people to move in a southerly direction. 28
The word impetus in the passage is closest in meaning to
(A) chance
(B) protection
(C) possibility
(D) incentive
15. These new characteristics offered better ways for the seeds to move to new habitats. They
could travel through the air, float in water, and lie dormant for many months. 29
The word dormant in the passage is closest in meaning to
(A) hidden
(B) inactive
(C) underground
(D) preserved
16. Spores light enough to float on the breezes were carried thousands of miles from more
ancient lands and deposited at random across the bare mountain flanks.
The phrase at random in the passage is closest in meaning to
(A) finally
(B) over a long period of time
(C) successfully
(D) without a definite pattern
27

Adapted from
Adapted from
29
Adapted from
28

46

[] Version 3.0
Vocabulary Enlargement
metallic
preferentially
meteorite
watercraft
breeze
flank

[mtlk]
[prefrenli]
[mitrat]
[wtkrft]
[briz]
[flk]

Sentence Pattern
Offer better way
The technique may offer scientists a better way to study autism in older children.

They also offered a better way to provide their products to distributors and customers.

Practical Translation Training


Google
Today, the company announced an update to Google Maps that__________________
_____________________________________________________________________
Further ReadingMeteorite 30
A meteorite is a meteoroid originating in outer space that survives impact with the Earths surface. A
meteorites size can range from small to extremely large. When a meteoroid enters the atmosphere,
frictional, pressure, and chemical interactions with the atmospheric gases cause the body to heat up
and emit light, thus forming a fireball, also known as a meteor or shooting/falling star. The term
bolide refers to either an extraterrestrial body that collides with the Earth, or to an exceptionally bright,
fireball-like meteor regardless of whether it ultimately impacts the surface.
More generally, a meteorite on the surface of any celestial body is a natural object that has come from
elsewhere in space. Meteorites have been found on the Moon and Mars. Meteorites that are recovered
after being observed as they transited the atmosphere or impacted the Earth are called falls. All other
meteorites are known as finds. As of February 2010, there are approximately 1,086 witnessed falls
having specimens in the worlds collections. In contrast, there are more than 38,660 well-documented
meteorite finds.
Meteorites have traditionally been divided into three broad categories: stony meteorites are rocks,
mainly composed of silicate minerals; iron meteorites are largely composed of metallic iron-nickel;
and, stony-iron meteorites contain large amounts of both metallic and rocky material. Modern
classification schemes divide meteorites into groups according to their structure, chemical and
isotopic composition and mineralogy. Meteorites smaller than 2mm are classified as micro-meteorites.
30

Adapted from http://en.wikipedia.org/wiki/Meteorite

47

Version 3.0

[]

17. Beyond that, the triumph of recorded sound has overshadowed the rich diversity of
technological and aesthetic experiments with the visual image that were going forward
simultaneously in the 1920s. 31
The word overshadowed in the passage is closest in meaning to
(A) distracted from
(B) explained
(C) conducted
(D) coordinated with
18. Water does not remain immobile in an aquifer but can seep out at springs or leak into other
aquifers. The rate of movement may be very slow: in the Indus plain, the movement of saline
(salty) ground waters has still not reached equilibrium after 70 years of being tapped. 32
The word immobile in the passage is closest in meaning to
(A) enclosed
(B) permanent
(C) motionless
(D) intact
19. Heart and respiration monitors provide the investigator with the number of heartbeats or
breaths taken when a new stimulus is presented. Numerical increases are used as quantifiable
indicators of heightened interest in the new stimulus. 33
The word quantifiable in the passage is closest in meaning to
(A) visual
(B) permanent
(C) meaningful
(D) measurable
20. The Baka is a nomadic group living in the rain forests of Cameroon, Congo, Gabon, and the
Central African Republic. They have an average height of 1.5 m and are considered to be
pygmies. However, they do not like this term and prefer to be called by their tribal name, the
Bake.
The word nomadic in the passage is closest in meaning to
(A) settled
(B) travelling
(C) dangerous
(D) cheerful
31

Adapted from
Adapted from
33
Adapted from
32

48

[] Version 3.0
Vocabulary Enlargement
aesthetic
equilibrium
respiration

[isetk]
[,ikwlbrm]
[,rspren]

Sentence Pattern
Prefer to do
Everyone has certain likes and dislikes, and there is no denying that we all prefer to
do the things that we like.

Practical Translation Training


10
While _________________________________________________, were guessing that
_______________________________________________________________________
Further ReadingBaka people 34
The Baka are a hunter-gatherer people. Groups establish
temporary camps of huts constructed of bowed branches covered
in large leaves (though today more and more homes are
constructed following Bantu methods). The men hunt and trap in
the surrounding forest, using poisoned arrows and spears to great
effect. They sometimes obtain honey from beehives in the forest
canopy. The men also fish using chemicals obtained from
crushed plant material. Using fast-moving river water, they
disperse the chemical downstream. This non-toxic chemical
deprives fish of oxygen, making them float to the surface and
easily collected by Baka men. Another method of fishing,
performed only by women, is dam fishing, in which water is
removed from a dammed area and fish are taken from the
exposed ground. Women also gather wild fruits and nuts or practice beekeeping while tending to the
children. The group remains in one area until it is hunted out then abandon the camp for a different
portion of the forest. The group is communal and makes decisions by consensus.
The Baka people are skilled in using various plants in which they may wash out chemicals to use or
mash it into a pulp etc. to treat illness and infertility. Childrens health is of a particular concern, as
they are particularly susceptible to disease, often resulting in death. Their skills in this traditional
medicine are such that even non-Baka often seek out their healers for treatment.
34

Adapted from http://en.wikipedia.org/wiki/Baka_(Cameroon_and_Gabon)

49

Version 3.0

[]

2
1) The President began to address the Senate on a regular basis.

2) Articles for nearly every household activity and ornament could be bought in
Rockingham ware.

3) In the mid-eighteenth century, painters had been willing to assume such artisan-related
tasks as varnishing, gilding teaching, keeping shops, and painting wheel carriages, houses,
and signs.

4) The bill is unlikely to pass the House of Representatives, where Republican leaders
oppose it.

5) It had over 2,000 apartment complexes, a great market, a large number of industrial
workshops, an administrative center, a number of massive religious edifices, and a regular
grid pattern of streets and buildings.

6) Rent controls have artificially depressed the most important long-term determinant of
profitability rents.

7) Environmental sounds, such as thunder, and electronically generated hisses and blips
can be recorded, manipulated, and then incorporated into a musical composition. But
composers also draw new sounds from voices and nonelectric instruments.

50

[] Version 3.0
8) Along with market days, the institution of twice-yearly fairs persisted in Philadelphia
even after similar trading days had been discontinued in other colonial cities.

9) Clay is far too small to sieve accurately; therefore, in soils with a high proportion of
clay, the fine particles are measured on the basis of their settling velocity when suspended in
water.

10) Chemical evidence shows that platelets (minute flat portions) of ice form in the water
and then accrete and stick to the bottom of the ice shelf to form a slush (partially melted
snow).

11) Most machines, no matter how complex they may be, are combinations of the lever
and the inclined plane.

12) The Art Nouveau style was a major force in the decorative arts from 1895 until 1915,
although its influence continued throughout the mid-1920's. It was eventually to be overtaken
by a new school of thought known as Functionalism that had been present since the turn of
the century.

13) Hazen methodically screened and cultured scores of soil samples.

14) Tears spring into her eyes as she starts telling them what had happened.

15) Unpainted wooden furniture weathers to a gray color.

51

Version 3.0

[]

2.3.9 Important Prefix and Roots


Meaning
Example
Prefix
Ab
Am
Anti
Bene
Bi
Circum
Con
Contra
De
Di
Dis
Du
En
Ex
Inter
Ir,il
Mal, male
Mini
Mono
Multi
Neo
Omni
Para
Per
Poly
Post
Pre
Pro
Re
Retro
Sub
Super
Sur
Syn
Tele
Trans
uni
52

[] Version 3.0
Root
Anthrop
Astro
Aqua
Audi
Bio
Cap
Carn
Ceed/Cees
Ceive/ Cept
Chron
Cide
Clam/claim
Cred
Demo
Dom
Dict
Duct
Gen
Geo
Hemo
Hydro
Lat
Lumen
Luc
Magni
Man(u)
Ortho
Path
Photo
Port
Reg/rect
Simil
Spec
Strict
Terra
Theo
Thermo
Ven
Verb
Ver/vert
Viv/vit
Voc
Vor

Meaning

Example

53

Version 3.0

[]

Sentence Comprehension
3.1 Structure Analysis

3.1.1 Simple Sentence


____________________________________________________________
+
+
+
+
_______________, _____________
3.1.1.1 Basic Sentence Patterns

Subject + Vi +


Subject + Vt + Object + +


Subject + link.Verb + Predicative + +


Subject + Vt + Object 1 + Object 2 + + +


Subject + Vt + Object + Complement + + +

feel, look, sound, smell, taste, seem, appear......


remain, continue, stay, keep......
become, go, run, get, turn, fall, come, grow......
54

[] Version 3.0
Vocabulary Enlargement
naive
subject
object
predicative

[naiv]
[sbdekt]
[bdkt]
[prdketv]

..

Further ReadingDoes every language use Subject-Verb-Object 35


In linguistic typology, subjectverbobject (SVO) is a sentence structure where the subject comes
first, the verb second and the object third. Languages may be classified according to the dominant
sequence of these elements. It is the second most common order found in the world, after SOV, and
together, they account for more than 75% of the worlds languages. It is also the most common order
developed in Creole languages, suggesting that it may be somehow more initially obvious to human
psychology.
Albanian, Arabic, Assyrian (VSO and VOS are also followed, depending on the person), Berber,
Bulgarian, Chinese, English, Estonian, Finnish, French, Ganda, Greek, Hausa, Hebrew, Italian,
Javanese, Kashmiri, Khmer, Latvian, Macedonian, Polish, Portuguese, Quiche, Rotuman, Russian,
Serbo-Croatian, Spanish, Swahili, Thai, Vietnamese, Yoruba and Zulu are examples of languages that
can follow an SVO pattern. The label is often used for ergative languages which do not have subjects,
but have an agentverbobject order. The Romance languages also follow SVO construction,
except for certain constructions in many of them in which a pronoun functions as the object (e.g.
French: je taime, Italian: (io) ti amo, Spanish: (yo) te amo or Portuguese: "(eu)amo-te", meaning "I
you love" in English). All of the Scandinavian languages follow this order also but change to VSO
when asking a question. Arabic and Hebrew will occasionally use an SVO pattern with sentences
with subject pronouns (e.g. Arabic , Hebrew:
, lit. "I love you."). However the
subject pronouns here are grammatically unnecessary and most other constructions suggest that both
languages are VSO languages at their core, though Modern Hebrew generally uses SVO
construction as well as the modern varieties of Arabic. Other SVO languages, such as English, can
also use an OSV structure in certain literary styles, such as poetry.
An example of SVO order in English is: Andy ate oranges.
In this, Andy is the subject, ate is the verb, oranges is the object.
Some languages are more complicated. For example, Russian, Finnish and Hungarian languages allow
all possible combinations: SVO, OVS, SOV, OSV, VSO, VOS. Word order is often changed to
emphasize a different part of the sentence. or to change the nuance of the meaning. In Polish, a
word/phrase can be brought to the front or, less commonly, put to the back of a sentence or clause to
add emphasis e.g. "Roweru ci nie kupi" (I wont buy you a bicycle), "Od pitej czekam" (Ive been
waiting since five).
In German and in Dutch, SVO in main clauses coexists with SOV in subordinate clauses. English
developed from such a reordering language and still bears traces of this word order, for example in
locative inversion ("In the garden sat a cat") and some clauses beginning with negative expressions:
"only" ("only then do we find X"), "not only" ("not only did he storm away but also slammed the
door"), "under no circumstances" ("under no circumstances are the students allowed to use a mobile
phone"), "on no account" and the like. In these cases do-support may or may not be required,
depending on the construction.
35

Adapted from http://en.wikipedia.org/wiki/Word_order

55

Version 3.0

[]

___________________________________
_____________

Ving

Subject

Object

Predicative

Complement

Ved
To do

1. _________ many times, but he still couldnt understand it.


A. Having been told
B. Having told
C. He had been told
D. Though he had been told
2. _________ many times, he still couldnt understand it.
A. Having been told
B. Having told
C. He had been told
D. Though he had told
ing ed
3. ____________ in the mountains for a week, the two students were finally saved by the
local police.
A. Having lost
B. Lost
C. Being lost
D. Losing

56

[] Version 3.0
4.

__________, I really believe that Id prefer not to make any change now.
A. Considered all the possibilities
B. Taking all the possibilities into consideration
C. Taken all the possibilities into consideration
D. Giving all the possibilities
-ing -ing

5. The discovery of new evidence led to ______________.


A. the thief having been caught
B. the thief to be caught
C. catch the thief
D. the thief being caught
6. At the beginning of class, the noise of desks ____ could be heard outside the classroom.
A. opened and closed
B. to be opened and closed
C. being opened and closed
D. to open and close
-ing to do -ed
7. ___________ in a heavy traffic jam is quite an unpleasant experience.
A. Caught
B. Having caught
C. Being caught
D. To catch
8. _____________twice a year, whether it is a car or a bus or a truck, is the rule that every
driver must obey in this city.
A. Examining
B. Examined
C. Being examined
D. Having been examined
notnot to do,not doing,not having
done, not to have done
9. What worried him most was ______ to visit his sick child.
A. his being not allowed
B. his not being allowed
C. his not allowing
D. having not been allowed
57

Version 3.0

[]

3.1.1.2 Modifier
Attribute
____________

Adjective Phrase

Prep Phrase

Ving/Ved Phrase

The infinitive


which, who, whom, that, as +

whose + (

when, where, why

+ in which, on which

n./pron. +

Which/that/who + V + O
Which/that/whom + S + V
Where/when/why + SVO
58

[] Version 3.0

Practice of Attribute (Attributive Clause)


1. How happy we are! The holiday we have been looking forward _______ at last.
A. to has come
B. to have come
C. to having come
D. has come
2. Last month, part of Southeast Asia was struck by floods, from _____ effects the people are
still suffering.
A. that

B. whose

C. those D. what

3. Mark was a student at this university from 1999 to 2003, __________ he studied very hard
and was made Chairman of the Students Union.
A. during which time

B. for which time

C. during whose time

D. by that time

4. I work in a business ______ almost everyone is waiting for a great chance.


A. how

B. which

C. where

D. that

5. The English play ________ my students acted at the New Years party was a great success.
A. for which

B. at which

C. in which

D. on which

6. New York, ________ I visited last year, is a nice old city.


A. that

B. which

C. when

D. in which

7. I can think of many cases _______students obviously knew a lot of English words and
expressions but couldnt write a good essay.
A. why

B. which

C. as

D. where

8. George Orwell, ________ was Eric Arthur, wrote many political novels and essays.

9.

A. the real name

B. what his real name

C. his real name

D. whose real name

_______is reported in the newspaper, talks between the two countries are making
A. It

B. As

C. That

D. What

10. American women usually identify their best friend as someone _______ they can talk
frequently.
A. who

B. as

C. about which

D. with whom

11. Anyway, that evening, _______ Ill tell you more about later, I ended up staying at
Rachels place.
A. when

B. where

C. what

D. which

12. The factory ____ the students visited last week is the one ____ I have worked for these
years.
A. that; which
B. when; in which
C. /; where
D. which; that

59

Version 3.0

[]

His brother who is now a doctor always


encourages him to go to college

3
4

The robot which is very smart makes me


happy

which/that ______

His brother, who is now a doctor,


always encourages him to go to
college
_______
_______
The robot is very smart, which makes
me very happy
____________

Which/that _______
_________

They were spurred by the inflation of the 1970s, which, combined with Californias
rapid population growth, pushed housing prices, as well as rents, to record levels. 36
__________________
_______________________
__________________________________________________________
Now, scientists have data from satellites and ground-based observations from which we
know that the aurora brilliance is an immense electrical discharge similar to that
occurring in a neon sign. 37
__________________
_______________________
__________________________________________________________
The canopy, the upper level of the trees in the rain forest, holds a plethora of climbing
mammals of moderately large size, which may include monkeys, cats, civets, and
porcupines. 38
__________________
_______________________
__________________________________________________________
36

Adapted from TOEFL pBT, 1997. 01, Educational Testing Services


Adapted from TOEFL pBT, 1999. 05, Educational Testing Services
38
Adapted from TOEFL pBT, 2000. 05, Educational Testing Services
37

60

[] Version 3.0
Vocabulary Enlargement
spur
inflation
neon
canopy
plethora
civet
porcupine

[sp]
[nfle()n]
[nin]
[knpi]
[plr]
[svt]
[prkjpan]

Sentence Pattern
Combine with
When theyre combined with oxygen, this energy is released as heat and light.

Practical Translation Training



Demographic and epidemiological transitions__________________________________
_______________________________________________to create new trends.
Further ReadingThe Great Inflation of the 1970s 39
Its the 1970s, and the stock market is a mess. It loses 40% in an 18-month period, and for
close to a decade few people want anything to do with stocks. Economic growth is weak,
which results in rising unemployment that eventually reaches double-digits. The easy-money
policies of the American central bank, which were designed to generate full employment, by
the early 1970s, also caused high inflation. The central bank, under different leadership,
would later reverse its policies, raising interest rates to some 20%, a number once considered
usurious. For interest-sensitive industries, such as housing and cars, rising interest rates cause
a calamity. With interest rates skyrocketing, many people are priced out of new cars and
homes.
The great inflation was blamed on oil prices, currency speculators, greedy businessmen and
avaricious union leaders. However, it is clear that monetary policies, which financed huge
budget deficits and were supported by political leaders, were the cause. This mess was proof
of what Milton Friedman said in "Money Mischief. Episodes in Monetary History" (1994):
inflation is always "a monetary phenomenon." The great inflation, and the recession that
followed, wrecked many businesses and hurt countless individuals. Interestingly, John
Connolly, the Nixon-installed treasury secretary without formal economics training, later
declared personal bankruptcy.
39

Adapted from The Great Inflation of the seventies: what really happened? By E Nelson

61

Version 3.0

[]

Appositive

____________
//or/of
The canopy, the upper level of the trees in the rain forest, holds a plethora of climbing
mammals of moderately large size, which may include monkeys, cats, and civets.
This record has already provided insights into the patterns and causes of past climatic
change information that may be used to predict future climates. 40
Did he give you a pound, or a hundred pence?
Columbia University, in the city of New York

+ that/whether +
Although leaders are often thought to be people with unusual personal ability, decades
of research have failed to produce consistent evidence that there is any category of
natural leaders. 41

The desperate plight of the South has eclipsed the fact that reconstruction had to be
undertaken also in the North. 42

that
The fact that half of the known species are thought to inhabit the worlds rain forests
does not seem surprising, considering the huge numbers of insects that comprise the
bulk of the species. 43

_____________________________________________________________________

_____________________________________________________________________
40

Adapted from TOEFL pBT, 1995. 08, Educational Testing Services.


Adapted from TOEFL pBT, 2000. 05, Educational Testing Services.
42
Adapted from TOEFL pBT, 1995. 08, Educational Testing Services.
43
Adapted from TOEFL pBT, 1995. 12, Educational Testing Services.
41

62

[] Version 3.0
Vocabulary Enlargement
plight
eclipse
bulk

[plat]
[klps]
[blk]

Sentence Pattern
Fail to
Many people fail to do this and wonder why they cannot concentrate.

Many obese people complain that others believe they are overweight simply because
they eat too much or fail to exercise.

Practical Translation Training



____________________________________________________simply

because

they

associate themselves with the wrong people.


Further ReadingLincoln (2012 film) 44
Lincoln recounts President Abraham Lincolns efforts, during January 1865, to obtain passage for the
Thirteenth Amendment to the United States Constitution in the United States House of
Representatives, which would formally abolish slavery in the country.
Expecting the Civil War to end within a month but concerned that his
1863 Emancipation Proclamation may be discarded by the courts once
the war has concluded and the 13th Amendment defeated by the
returning slave states, Lincoln feels it is imperative to pass the
amendment by the end of January, thus removing any possibility that
slaves who have already been freed may be re-enslaved. The Radical
Republicans fear the amendment will merely be defeated by some who
wish to delay its passage; the support of the amendment by Republicans
in the border states is not yet assured either, since they prioritize the
issue of ending the war. Even if all of them are ultimately brought on
board, the amendment will still require the support of several
Democratic congressmen if it is to pass. With dozens of Democrats
having just become lame ducks after losing their re-election campaigns
in the fall of 1864, some of Lincolns advisors believe that he should
wait until the new Republican-heavy Congress is seated, presumably giving the amendment an easier
road to passage. Lincoln, however, remains adamant about having the amendment in place and the
issue of slavery settled before the war is concluded and the southern states readmitted into the Union.
44

Adapted from http://en.wikipedia.org/wiki/Lincoln_(2012_film)

63

Version 3.0

[]

Parenthesis

_____________
///Ving/
Though we are accustomed to speaking of the films made before 1927 as silent
the film has never been, in the full sense of the word, silent. 45
This preference for exogamy, Gutman suggests, may have derived from West African
rules governing marriage, which, though they differed from one tribal group to
another, all involved some kind of prohibition against unions with close kin.
What we today call America folk art was, indeed, art of, by, and for ordinary,
everyday "folks" who, with increasing prosperity and leisure, created a market for art
of all kinds, and especially for portraits.
Although Gutman admits that forced separation by sale was frequent, he shows that
the slaves preference, revealed most clearly on plantations where sale was infrequent,
was very much for stable monogamy.
Adverbial

______________
///

Adverb
Prep Phrase
Ving/Ved Phrase
The Infinitive

45

Adapted from TOEFL pBT, 1995. 12, Educational Testing Services.

64

[] Version 3.0
Sentence Pattern
(Be) accustomed to
Not everyone on the team was accustomed to helicopter assaults.

Practical Translation Training



___________________________and readily acknowledges how complicated the brain is.
Further ReadingThe Artist (2012 silent film) 46
In 1927, silent film star George Valentin is posing for pictures
outside the premiere of his latest hit film when a young woman,
Peppy Miller, accidentally bumps into him. Valentin reacts with
humor to the accident and shows off with Peppy for the cameras.
The next day, Peppy finds herself on the front page of Variety with
the headline "Whos That Girl?" Later, Peppy auditions as a dancer
and is spotted by Valentin, who insists that she have a part in
Kinograph Studios next production, despite objections from the
studio boss, Al Zimmer. While performing a scene together,
Valentin and Peppy show great chemistry, despite her being merely
an extra. With a little guidance from Valentin, Peppy slowly rises
through the industry, earning more prominent starring roles.
Two years later, Zimmer announces the end of production of silent
films at Kinograph Studios, but Valentin is dismissive, insisting that
sound is just a fad. In a dream, Valentin begins hearing sounds from his environment (as does the
audience), but cannot speak himself, then wakes up in a sweat. He decides to produce and direct his
own silent film, financing it himself. The film opens within a few days of Peppys new sound film as
well as the 1929 Stock Market Crash. Now Valentins only chance of avoiding bankruptcy is for his
film to be a hit. Unfortunately audiences flock to Peppys film instead and Valentin is ruined. His wife,
Doris, kicks him out, and he moves into an apartment with his valet/chauffeur, Clifton, and his dog.
Peppy goes on to become a major Hollywood star.
Later, the bankrupt Valentin is forced to auction off all of his personal effects, and fires Clifton,
telling him to get another job, while Peppy insists to Zimmer that Valentin co-star in her next film,
threatening to quit Kinograph if Zimmer does not agree to her terms. After Valentin learns to his
dismay that it had been Peppy who had purchased all his auctioned effects, he returns in despair to his
burnt-out apartment. Peppy arrives, panicked, and finds that Valentin is about to attempt suicide. The
two reconcile, and remembering Valentins superb dancing ability, Peppy persuades Zimmer to let
them make a musical together.
Now the audience hears sound for the second time, as the film starts rolling for a dance scene with
Peppy and Valentin and their tap-dancing can be heard. Once the choreography is complete, the two
dancers are heard panting. The director of the musical calls out audibly, "Cut!" to which Zimmer adds:
"Perfect. Beautiful. Could you give me one more?" Valentin, in his only audible line, replies "With
pleasure!"
46

Adapted from http://en.wikipedia.org/wiki/The_Artist_(film)

65

Version 3.0

[]

Adverbial Clause

Time Clauses

when, before, after, since,


while, as, as long as, until,
till, etc.
where, wherever, anywhere,
everywhere
if, unless, lest

Her goldfish died when she was


young.

Place Clauses
Conditional Clauses
Purpose Clauses
Reason Clauses

in order to, so that, in order


that
because, since, as, given

Result Clauses

so...that...

Concession Clauses

although, though, while

Manner Clauses

as, like, the way

He said he was happy where he was.


If they lose weight during an illness,
they soon regain it afterwards.
They had to take some of his land so
that they could extend the churchyard.
I couldnt feel anger against her
because I liked her too much.
My suitcase had become so damaged
that the lid would not stay closed.
I used to read a lot although I dont
get much time for books now.
I was never allowed to do things as I
wanted to do them.

n. + Ving/Ved
n. + Adj.
n. + Adv.
n. + Inf.
n. + Prep Phrase

Weather permitting, we will have a picnic tomorrow.


It stood silent in the noon sunlight, its door open.
The meeting over, all of us went home.
We divided the work, he to clean the windows.
I followed him here, climbed in, sword in hand.

Practice
1. Not far from the school there was a garden, _________ owner seated in it playing chess
with his little grandson every afternoon.
A. its
B. whose
C. which
D. that
2. The country has already sent up three unmanned spacecraft, the most recent _______ at
the end of last March.
A. has been launched

B. having been launched

C. being launched

D. to be launched
66

[] Version 3.0
Vocabulary Enlargement
churchyard
Lid

[ttjrd]
[ld]

Sentence Pattern
Send up
Now big plans in China are being made to send up more satellites and even build a
space station.

Could you send up a Coke and a salad?

Practical Translation Training


5

Iraq government announced that _____________________________________________


______________________________________________to help reconstruction efforts.
Further Reading

Black tea
Green tea
Strong (Weak) tea
Brick tea
Black coffee
White coffee
Instant coffee
Cocoa
Whole milk
Milk curd
Powder milk
Condensed milk
Malted milk
Mineral water
Soft drink
Soda water
Orangeade
Fruit juice
Liquor

67

Sweet wine
Dry sweet wine
Yellow wine
White wine
Red wine, port
Sherry
Vermouth
Rum
Champagne
Brandy
Cider
Whisky
Gin
Vodka
Punch
Drought beer
Sprite
Seven-up
Fanta

Version 3.0

[]

3.1.2 Annotation Method



/
<>
()
[]


1. The teacher with two of his students is walking into the classroom.
2. There is an old man coming here.
3. To do todays homework without the teachers help is very difficult.

1. A single idea from the human mind can build cities.
2. Cinderella didnt do her chores last night.
3. Life is like a box of chocolate.
4. Everything you see exits together in a delicate balance.
5. Flying Snow and Broken sword have teamed up for assassination.
6. Then Richard Parker, the awful, fierce companion of my torment, moved forward and
disappeared from my life.
7. Fear can hold you prisoner; hope can set you free.
8. Soon they all became obsessed with the Ring.
9. Go through the wall and you can find the Platform Nine and Three-Quarters.
10. A monster named Sphinx sitting in the middle of the road said he had a riddle for
passersby.

1. You must pay good attention to your pronunciation.
2. The old man was feeling very tired.
3. Some of the students in the school want to go swimming, how about you?
4. The old man sitting at the gate said he was ill.
5. Soon they all became interested in the subject.
6. Go across the bridge and you will find the museum on the left.
7. They didnt know who Father Christmas really is.
8. She was the first to learn about it.
68

[] Version 3.0
Sentence Pattern
Pay attention to
Oppose extravagant eating and drinking and pay attention to thrift and economy.

Practical Translation Training

With limited time and attention, we have to ____________________________________
_______________________________________________________________________
Further ReadingHistory of Father Christmas 47
In England the earliest known personification of Christmas
dates from the mid-15th century. A carol attributed to Richard
Smart, Rector of Plymtree from 1435 to 1477, takes the form
of a sung dialogue between a choir and a figure representing
Christmas,variously addressed as Nowell, Sir Christemas
and my lord Christemas. Giving news of Christs birth,
Christmas encourages everyone to celebrate: "Buvez bien par
toute la campagnie,/Make good cheer and be right merry."
However, the specific depiction of Christmas as a merry old
man began in the early 17th century. The rise of puritanism
had led to increasing condemnation of the traditions handed
down from pre-Reformation times, especially feasting and
drinking. As debate intensified, those writing in support of the
traditional celebrations often personified Christmas as a
venerable, kindly old gentleman, given to good cheer but not
excess. They referred to this personification as "Christmas",
"Old Christmas" or "Father Christmas".
During the mid-17th century, the debate about the celebration of Christmas became politically
charged, with Royalists adopting a pro-Christmas stance and radical puritans striving to ban the
festival entirely. Early in 1646 an anonymous satirical author wrote The Arraignment, Conviction and
Imprisoning of Christmas, in which a Royalist lady is frantically searching for Father Christmas: this
was followed months later by the Royalist poet John Taylors The Complaint of Christmas, in which
Father Christmas mournfully visits puritan towns but sees "...no sign or token of any Holy Day". A
book dating from the time of the Commonwealth, The Vindication of CHRISTMAS or, His Twelve
Years Observations upon the Times (London, 1652), involved "Old Christmas" advocating a merry,
alcoholic Christmas and casting aspersions on the charitable motives of the ruling Puritans. In a
similar vein, a humorous pamphlet of 1686 by Josiah King presents Father Christmas as the
personification of festive traditions pre-dating the puritan commonwealth. He is described as an
elderly gentleman of cheerful appearance, "who when he came looked so smug and pleasant, his
cherry cheeks appeared through his thin milk white locks, like flushing Roses with snow white
Tiffany". His character is associated with feasting, hospitality and generosity to the poor.
47

Adapted from http://en.wikipedia.org/wiki/Father_Christmas

69

Version 3.0

[]


1. The boy to write this letter needs a pen.
2. The girl who is reading needs the pen which you bought yesterday.
3. I am waiting for the sound of the other shoe!

1. There was a big smile on her face.
2. Every night he heard the noise upstairs.
3. The man on the motorbike was traveling too fast.
4. With the medicine box under her arm, Miss Li hurried off.
5. I am afraid that if youve lost it, you must pay for it.
6. The students followed Uncle Wang to see the other machine.

1. Totally without light and subjected to intense pressures hundreds of times greater than at
the Earths surface, the deep-ocean bottom is a hostile environment to humans, in some
ways as forbidding and remote as the void of outer space. 48


2. A useful definition of an air pollutant is a compound added directly or indirectly by
humans to the atmosphere in such quantities as to affect humans, animals, vegetation, or
materials adversely. 49


3. In a period characterized by the abandonment of so much of the realistic tradition by
authors such as John Barth, Donald Barthelme, and Thomas Pynchon, Joyce Carol Oates
has seemed at times determinedly old-fashioned in her insistence on the essentially
mimetic quality of her fiction


4. Elizabeth Blackwell, the first woman medical doctor in the United States, founded the
New York Infirmary, an institution that has always had a completely female medical staff.

48
49

Adapted from TOEFL pBT, 1995. 08, Educational Testing Services


Adapted from TOEFL pBT, 1995. 08, Educational Testing Services

70

[] Version 3.0
Vocabulary Enlargement
hostile
forbidding
void
realistic
essentially
mimetic

[hstal]
[fbd]
[vd]
[rilstk]
[snli]
[mmtk]

Sentence Pattern
Such as to
We are not such a fool as to believe him

Characterized by
His work is characterized by lack of attention to detail.

Many obese people complain that others believe they are overweight simply because
they eat too much or fail to exercise.

Practical Translation Training



_______________________________________________________that simulate racing.
Brain FoodCan you guess the missing word?
52 W__ in a Y___
Did you guess right? The correct sentence should read 52 weeks in a year. Take a look at
the following ten sentences and try and guess the missing words
4 s___ in a y___.
30 d___ in S___.
32 t___ in a m___.
26 l___ in the E___ a___.
50 s___ on the A___ f___.
206 b___ in the h___ b___.
88 k___ on a p___.
52 c___ in a p___ of p___ c___.
9. 11 p___ in a f___ t___.
10. 44 P___ of A___.

1.
2.
3.
4.
5.
6.
7.
8.

71

Version 3.0

[]

3.1.3 Compound and Complex Sentence

=1

=1

= +

3.1.3.1
Compound Sentence: FANBOYS
A compound sentence contains two independent clauses joined by a coordinator. The
coordinators are as follows: for, and, nor, but, or, yet, so or punctuations like: colon, semicolon, and dash.
Coordinator

and, bothand, neithernor, not onlybut also, as well as

but, while, yet, however, whereas, nevertheless

or, or else, otherwise, eitheror, rather than

so, for, thus, hence

Example
1. The railroad could be and was a despoiler of nature; furthermore, in its manifestation of
speed and noise, it might be a despoiler of human nature as well.

2. For a number of years the selection of music for each film program rested entirely in the
hands of the conductor or leader of the orchestra, and very often the principal qualification
for holding such a position was not skill or taste so much as the ownership of a large
personal library of musical pieces. 50



90
50

Adapted from TOEFL pBT, 1995. 12, Educational Testing Services

72

[] Version 3.0
Vocabulary Enlargement
conductor
orchestra
qualification

[kndkt]

[rkstr]

[kwlfke()n]

Sentence Pattern
Rest in
The hopes for republican government, he knew, rested in his hands.

When power rested in one man, King Louis, all sorts complained of oppression, and
the nobility, middle, and moneyed men called on the poor to help.

Further ReadingMovie Music 51


Accustomed though we are to speaking of the films made before 1927 as silent, the film has never
been, in the full sense of the word, silent. From the very beginning, music was regarded as an
indispensable accompaniment; when the Lumiere films were shown at the first public film exhibition
in the United States in February 1896, they were accompanied by piano improvisations on popular
tunes. At first, the music played bore no special relationship to the films; an accompaniment of any
kind was sufficient. Within a very short time, however, the incongruity of playing lively music to a
solemn film became apparent, and film pianists began to take some care in matching their pieces to
the mood of the film.
As movie theaters grew in number and importance, a violinist, and perhaps a cellist would be added to
the pianist in certain cases, and in the larger movie theaters small orchestras were formed. For a
number of years the selection of music for each film program rested entirely in the hands of the
conductor or leader of the orchestra, and very often the principal qualification for holding such a
position was not skill or taste so much as the ownership of a large personal library of musical pieces.
Since the conductor seldom saw the films until the night before they were to be shown (if indeed, the
conductor was lucky enough to see them then), the musical arrangement was normally improvised in
the greatest hurry.
To help meet this difficulty, film distributing companies started the practice of publishing suggestions
for musical accompaniments. In 1909, for example, the Edison Company began issuing with their
films such indications of mood as pleasant, sad, lively. The suggestions became more explicit,
and so emerged the musical cue sheet containing indications of mood, the titles of suitable pieces of
music, and precise directions to show where one piece led into the next.
Certain films had music especially composed for them. The most famous of these early special scores
was that composed and arranged for D.W Griffiths film Birth of a Nation, which was released in
1915.
51

Adapted from TOEFL pBT, 1995. 12, Educational Testing Services

73

Version 3.0

[]

3.1.3.2
Complex Sentence
A complex sentence has an independent clause joined by one or more dependent clauses. A
complex sentence always has a subordinator such as because, since, after, although, or when
or a relative pronoun such as that, who, which.


DC: SVO V O
S V DC: SVO
S, DC: SVO, V O
S V O, DC: SVO

S, DC: SVO, V O
S V O, DC: SVO

DC: SVO, S V O
S V O, DC: SVO
a
that that

That Chinese cannot learn English well is ridiculous.


that it
It is an undeniable fact that Taiwan is part of China.
b
The idea that we invited him is quite good.
He put forward a problem whether we stay or leave
There are numerous unsubstantiated reports that natural vitamins are superior to
synthetic ones, that fertilized eggs are nutritionally superior to unfertilized eggs, that
untreated grains are better than fumigated grains and the like
c

American women have come a long way toward social roles for women which are as
dignified and responsible as those for men.
Tourists take special interest in the relics displayed in the exhibition hall, which have
been unearthed recently.
74

[] Version 3.0
Practice
1. His father was not satisfied with _______ he had.
2. The question is ________ will come here.
3. We got the news _______ our team won the basketball match.
4.

_________ he has disappeared worries his parents.

5. Ill never forget the day ___________ I joined the Party.


6. Ill never forget the days _________ we spent on the seaside with you.
7. It is almost five years ________ we saw each other last time.
8. His plan was such a good one _________ we all agreed to accept it.
9. We're just trying to reach a point ___________ both sides will sit down together and talk.
10. The American Civil War lasted four years ___________ the North won in the end.
11. Many people ______ had seem the film were afraid to go to the forest when they
remembered the scenes in ______ people were eaten by the tiger.
12. I'd like to send you a pair of compasses, ______ were a present given to me by my sister
on my last birthday and I think it will be of great help to your study.
13. There is little evidence to support the fact _______ human or plant life would not be able
to exist without this essential liquid.
14. ______ she was getting me settled into a tiny but clean room the head of the village was
tying up his horse to my car to pull it to a small town some kilometers away _______ there
was a garage.
15. People find life in Canberra relaxing and pleasant. The city is surrounded by beautiful
mountains _______ people can ski. There is a large man _______ made lake lined with
beaches and parks in the center of Canberra.
16. Years ago I worked with the Notco Company as the head of the designing department.
One of the best designers was called Steve King, (1)________ was clever and hardworking. We didn't know his secret (2)________ one day when he had an accident. His
legs were badly hurt and he was taken to the hospital. (3)________ the doctor wanted to
examine his legs, he refused to take off his trousers. Then he told the doctor his secret
(4)________ he was a woman. She had pretended to be a man in order to get a job she
liked. That night (5)________ I went to visit her, I discovered her secret. The next day the
company had a meeting about the problem of Ms King. (6)________ the company had
never had women engineers working, we finally decided to let her stay. From then on,
more women workers were employed by the Nocto Company.
75

Version 3.0

[]

3.1.4 Practice
1. A series of mechanical improvements continuing well into the nineteenth century,
including the introduction of pedals to sustain tone or to soften it, the perfection of a metal
frame and steel wire of the finest quality, finally produced an instrument capable of myriad
tonal effects from the most delicate harmonies to an almost orchestral fullness of sound,
from a liquid, singing tone to a sharp, percussive brilliance. 52

2. As a result, claims that eating a diet consisting entirely of organically grown foods prevents
or cures disease or provides other benefits to health have become widely publicized and
form the basis for folklore.

3. The sex ratio will be favored which maximizes the number of descendants an individual
will have and hence the number of gene copies transmitted.

4. people who believe that aggression is necessary and justified-as during wartime-are likely
to act aggressivelywhereas people who believe that a particular War or act of aggression
is unjust, who think that aggression is never justified, are less likely to behave
aggressively. 53

5. Finally, for the many small mammals that supplement their insect diet with fruits or seeds
an inability to span open gaps between tree crowns may be problematic, since trees that
yield these foods can be sparse.

52
53

Adapted from TOEFL pBT, 1995. 10, Educational Testing Services.


Adapted from Aggression, OG Practice Set 4, Educational Testing Services.

76

[] Version 3.0
Vocabulary Enlargement
delicate
Harmony
percussive
justified

[delkt]
[hmn]
[pksv]
[dstfad]

Sentence Pattern
Capable of
In the wake of developments in science and technology, man has become more
capable of conquering nature.

Be responsive to
Our ability to be responsive to changes in the epidemic is a central factor to success.

Practical Translation Training



Through this growth ____________________________________________________
Further ReadingThe origin of Piano
For a century and a half the piano has been one of the most popular solo instruments for Western
music. Unlike string and wind instrument, the piano is completely self-sufficient, as it is able to play
both the melody and its accompanying harmony at the same time. For this reason, it became the
favorite household instrument of the nineteenth century.
The ancestry of the piano can be traced to the early keyboard instruments of the fifteenth and
sixteenth centuries the spinet, the dulcimer, and the virginal. In the seventeenth century the organ,
the clavichord, and the harpsichord became the chief instruments of the keyboard group, a supremacy
they maintained until the piano supplanted them at the end of the eighteenth century. The clavichords
tone was metallic and never powerful; nevertheless, because of the variety of tone possible to it, many
composers found the clavichord a sympathetic instrument for concert use, but the character of the tone
could not be varied save by mechanical or structural devices.
The piano was perfected in the early eighteenth century by a harpsichord maker in Italy (though
musicologists point out several previous instances of the instrument). This instrument was called a
piano e forte (soft and loud), to indicate its dynamic versatility; its strings were struck by a recoiling
hammer with a felt-padded head. The wires were much heavier in the earlier instruments. A series of
mechanical improvements continuing well into the nineteenth century, including the introduction of
pedals to sustain tone or to soften it, the perfection of a metal frame and steel wire of the finest quality,
finally produced an instrument capable of myriad tonal effects from the most delicate harmonies to an
almost orchestral fullness of sound, from a liquid, singing tone to a sharp, percussive brilliance.
77

Version 3.0

[]

6. The Whigs were strongest in the towns, cities, and those rural areas that were fully
integrated into the market economy, whereas Democrats dominated areas of semisubsistence farming that were more isolated and languishing economically. 54

7. The experiments that follow in the next section bring us to the conclusion that while a
reasonably good dictionary that fits all is indeed within reach, extracting state-of-the-art
denoising performance calls for a more complex model that uses several dictionaries
switched by content an option we do not explore in this work.

8. Sheldon, is proving that you are single-handedly smarter than everyone else so important
that you would rather lose by yourself than win as part of a team? 55

9. What is particularly meaningful to anthropologists is the realization that although the


materials available to a society may to some extent limit or influence what it can do
artistically, the materials by no means determine what is done. 56

10. The agricultural revolution stimulated many in the countryside to seek a new life in the city
and made it possible for fewer farmers to feed the large concentrations of people needed to
provide a workforce for growing numbers of factories. 57

54

th

Adapted from 19 century politics in the United States, OG Test 1


Adapted from The Big Bang Theory, Season 1, Episode 13.
56
Adapted from TOEFL pBT, 1999. 08, Educational Testing Services.
57
Adapted from TOEFL pBT, 1999. 05, Educational Testing Services.
55

78

[] Version 3.0
Vocabulary Enlargement
languishing
artistically

[lw]
[rtstkli]

Sentence Pattern
To some extent
To some extent, this will happen in South America as well.

By no means
However, these notations were by no means a standard.

Practical Translation Training



_______________________________________________________________________
Further ReadingDomestic Migration in United States
Matching the influx of foreign immigrants into the larger cities of the United States during the late
nineteenth century was a domestic migration, from town and farm to city, within the United States.
The country had been overwhelmingly rural at the beginning of the century, with less than 5 percent
of Americans living in large towns or cities. The proportion of urban population began to grow
remarkably after 1840, increasing from 11 percent that year to 28 percent by 1880 and to 46 percent
by 1900. A country with only 6 cities boasting a population of more than 8,000 in 1800 had become
one with 545 such cities in 1900. Of these, 26 had a population of more than 100,000 including 3 that
held more than a million people. Much of the migration producing an urban society came from
smaller towns within the United States, but the combination of new immigrants and old American
"settlers" on Americas "urban frontier" in the late nineteenth century proved extraordinary.
The growth of cities and the process of industrialization fed on each other. The agricultural revolution
stimulated many in the countryside to seek a new life in the city and made it possible for fewer
farmers to feed the large concentrations of people needed to provide a workforce for growing numbers
of factories. Cities also provided ready and convenient markets for the products of industry, and huge
contracts in transportation and construction-as well as the expanded market in consumer goodsallowed continued growth of the urban sector of the overall economy of the United States.
Technological developments further stimulated the process of urbanization. One example is the
Bessemer converter which provided steel girders for the construction of skyscrapers. The refining of
crude oil into kerosene, and later the development of electric lighting as well as of the telephone,
brought additional comforts to urban areas that were unavailable to rural Americans and helped attract
many of them from the farms into the cities. In every era the lure of the city included a major
psychological element for country people; the bustle and social interaction of urban life seemed
particularly intriguing to those raised in rural isolation.
79

Version 3.0

[]

11. A white dwarf, in contrast, has gone through a long period when it burns hydrogen,
followed by another long period in which it burns the helium created by burning of
hydrogen and ends up with a core that consists mostly of oxygen and carbon with a thin
layer of hydrogen surrounding the core.

12. Implicit in it is an aesthetic principle as well: that the medium has certain qualities of
beauty and expressiveness with which sculptors must bring their own aesthetic
sensibilities into harmony.

13. Among the species of seabirds that use the windswept cliffs of the Atlantic coast of
Canada in the summer to mate, lay eggs, and rear their young are common murres,
Atlantic puffins, black-legged kittiwakes, and northern gannets.

14. The argument that humans, even in prehistoric times, had some number sense, at least to
the extent of recognizing the concepts of more and less when some objects were added to
or taken away from a small group, seems fair.

15. Moreover, in addition to its being a transportation pathway equipped with a mammoth
physical plant of tracks, signals, crossings, bridges, and junctions, plus telegraph and
telephone lines, the railroad nurtured factory complexes, coat piles, warehouses, and
generating stations, forming along its right-of-way what has aptly been called the
metropolitan corridor of the American landscape.

80

[] Version 3.0
16. The activities observed in a number of "pioneer" cities sponsoring art in public placesa
broadening exploration of public sites, an increasing awareness among both sponsors and
the public of the varieties of contemporary artistic practice, and a growing public
enthusiasmare increasingly characteristic of cities across the country.

17. Unable to finance road construction, states turned for help to private companies, organized
by merchants and land speculators who had a personal interest in improved
communications with the interior.

18. Contrary to the conclusions of Thor Heyerdahl's Kon-Tiki expedition of 1946, the
evidence of plant dispersal, archaeology, linguistics, and genetics now shows quite
conclusively that the Pacific Islands were not populated from the east by South Americans
who drifted on balsa-wood rafts and the prevailing wind and current, but from the west, by
groups from mainland Asia who gradually spread from island to island out into the Pacific.

19. Anyone who has handled a fossilized bone knows that it is usually not exactly like its
modern counterpart, the most obvious difference being that it is often much heavier.

20. However, for many years, physicists thought that atoms and molecules always were much
more likely to emit light spontaneously and that stimulated emission thus always would be
much weaker.

81

Version 3.0

[]

3.1.5 Grammatical Feature


3.1.5.1
Hyperbaton

____________hardly, barely, scarcely, no sooner...than..., rarely,


not only...but also, seldom, not until, little, in no way, by no means, in no case, never,
on no account, under no circumstances, nor
Hardly could I understand what he said.
Little did I think of the consequences.
Neither at this meeting nor at the previous one did anyone raise the problem.
Not until 1866 was the first fully successful transatlantic cable finally laid.
Hardly had he begun to speak when the audience interrupted him.
Not in the least did she like him.
__________
Only outside the Earth's atmosphere is it safe for a space vessel to attain extremely
high velocity.
As inevitably as human culture has changed with the passing of time, so has the
environment.
than as
Mammals have a larger, more well-developed brain than do other animals.
More first-borns have served in the U.S. Congress and as U.S. presidents than have
those born in other birth-order positions.
As
Jack likes sports, as do many of his friends.
Young as he is, he knows a lot.


Even summer insects keep silence for me, silent is Cambridge tonight.
Marching at the head were Jack and his friends.
To be particularly considered are the following questions.
1. Basic to any understanding of Canada in the 20 years after the Second World War is the
countrys impressive population growth. 58
2. Coincident with concerns about the accelerating loss of species and habitats has been a
growing appreciation of the importance of biological diversity, the number of species in a
particular ecosystem, to the health of the Earth and human well-being. 59

58
59

Adapted from TOEFL pBT, 1995. 08, Educational Testing Services.


Adapted from TOEFL pBT, 1995. 12, Educational Testing Services.

82

[] Version 3.0
Sentence Pattern
Basic to
Pets are as basic to American culture as hot dogs or apple pie.

And yet, though it is basic to human thought, causality is a notion shrouded in


mystery and controversy.

Coincident with
This bubble is roughly coincident with real estate bubbles in the United Kingdom,
Germany and even South Korea.

Practical Translation Training



_______________________________________________________________________
Further ReadingGrowth in Canadas population60
Basic to any understanding of Canada in 20 years after the Second World War is the countrys
impressive population growth. For every three Canadians in 1945, there were over five in 1996. In
September 1966 Canadas population passed the 20 million mark. Most of this surging growth came
from natural increase. The depression of the 1930s and the war had held back marriages and the
catching up process began after 1945. The baby boom continued through the decade of the 1950s,
producing a population increase of nearly fifteen percent in the five years from 1951 to 1956. This
rate of increase had been exceeded only once before in Canadas history, in the decade before 1911,
when the prairies were being settled. Undoubtedly, the good economic conditions of the 1950s
supported a growth in the population, but the expansion also derived from a trend toward earlier
marriages and an increase in the average size of families. In 1957 the Canadian birth rate stood at 28
per thousand, one of the highest in the world.
After the peak year of 1957, the birth rate in Canada began to decline. It continued falling until in
1966 it stood at the lowest level in 25 years. Partly this decline reflected the low level of births during
the depression and the war, but it was also caused by changes in Canadian society. Young people
were staying at school longer, more women were working, young married couples were buying
automobiles or houses before starting families, rising living standards were cutting down the size of
families. It appeared that Canada was once more falling in step with the trend toward smaller families
that had occurred all through the Western world since the time of the Industrial Revolution.
Although the growth in Canadas population has slowed down by 1966 (the increase in the first half
of the 1960s was only nine percent). Another large population wave was coming over the horizon. It
would be composed of the children who were born during the period of the high birth rate prior to
1957.

60

Adapted from TOEFL pBT, 1995. 08, Educational Testing Services.

83

Version 3.0

[]

3.1.5.2
Emphatic Sentence

It + is/was + + that/who + ......


Example: Pi played with Richard Parker in the boat for one whole year.
(1) Pi:
(2) Richard Parker:
(3) Boat:
(4) one whole year:

It + V... + that/to do/doing +

S + V + it + adj. + that/to do/doing +
it /
Example
3. It was in the cities that the elements that can be associated with modern capitalism first
appearedthe use of money and commercial paper in place of barter, open competition in
place of social deference and hierarchy, with an attendant rise in social disorder, and the
appearance of factories using coal or water power in place of independent crafts people
working with hand tools. 61

2. It was she, a Baltimore printer, who published the first official copies of the Declaration,
the first copies that included the names of its signers and therefore heralded the support of
all thirteen colonies 62



61
62

Adapted from TOEFL pBT, 1999.01, Educational Testing Services


Adapted from TOEFL pBT, 1997.10, Educational Testing Services

84

[] Version 3.0
Vocabulary Enlargement
capitalism
barter
hierarchy
herald

[kptlzm]
[bt]
[harrki]
[her()ld]

Sentence Pattern
In place of
Almost all restaurants can give you a salad in place of French fries or potatoes.

These facilities can have weather maps analyzed in place of a forecaster.

Practical Translation Training



_______________________________________________________________________
Further ReadingCopies of Declaration of Independence 63
If you look closely at some of the early copies of the Declaration of Independence, beyond the
flourished signature of John Hancock and the other 55 men who signed it, you will also find the name
of one woman, Mary Katherine Goddard. It was she, a Baltimore printer, who published the first
official copies of the Declaration, the first copies that included the names of its signers and therefore
heralded the support of all thirteen colonies.
Mary Goddard first got into printing at the age of twenty-four when her brother opened a printing
shop in Providence, Rhode Island, in 1762. When he proceeded to get into trouble with his partners
and creditors, it was Mary Goddard and her mother who were left to run the shop. In 1765 they began
publishing the Providence Gazette, a weekly newspaper. Similar problems seemed to follow her
brother as he opened businesses in Philadelphia and again in Baltimore. Each time Ms. Goddard was
brought in to run the newspapers. After starting Baltimores first newspaper, The Maryland Journal, in
1773, her brother went broke trying to organize a colonial postal service. While he was in debtors
prison. Mary Katherine Goddards name appeared on the newspapers masthead for the first time.
When the Continental Congress fled there from Philadelphia in 1776, it commissioned Ms. Goddard
to print the first official version of the Declaration of Independence in January 1777. After printing
the documents, she herself paid the post riders to deliver the Declaration throughout the colonies.
During the American Revolution, Mary Goddard continued to publish Baltimores only newspaper,
which one historian claimed was "second to none among the colonies". She was also the citys
postmaster from 1775 to 1789 - appointed by Benjamin Franklin - and is considered to be the first
woman to hold a federal position.
63

Adapted from TOEFL pBT, 1997.10, Educational Testing Services

85

Version 3.0

[]

3.1.5.3
Subjunctive Mood

Were/did

should/would be/do

had been/done

should/would have been/done

should be/do were

should/would be/do

were
If I were you, I might tell the truth.
If I had had much money last year, I should have bought a house.
If he were to come, I could ask him about it.
If there were to be something wrong, they would let you know at once.

He would be dead now if the doctor had not operated on him right away.
If I were rich, I would have bought a house last year.
were, had should, if

Were I you, I would not do it.


Were I to meet him tomorrow, I should ask him about it.
Had I the time, I would go.
Should you change your mind, no one would blame you.

butorwithoutbut forotherwise
He was ill, or he would have come.
Without electricity, there would be no modern industry.
But for his help, I could not have finished it so early.
If it were not for this facultythey would devour all the food available in short time
and would probably starve themselves out of existence.
Indeed, had it not been for the superb preservation of these fossils, they might well
have been classified as dinosaurs.

86

[] Version 3.0
Vocabulary Enlargement
Faculty
Devour
Available
Starve

['fklti]
[d'va]
['velbl]
[strv]

Practical Translation Training


In the second case, pollinators (insects, birds) obtain food from the flowering plant, and
the plant has its pollen distributed and seeds dispersed much more efficiently than they
would be if they were carried by the wind only.

Further Reading
Unlike those available for painting, the opportunities to exhibit sculpture in the United States
around the turn of the twentieth century were quite scarce. There was almost no room for
sculpture at the influential Fine Arts Societys 57th Street Galleries in New York. As late as
1905, the Monumental News, a journal dedicated to the promotion of sculpture, lamented,
Exhibitions of sculptors works are so comparatively rare. In response to this dire
predicament, the sculptor Frederick W. Ruckstull and Charles de Kay, art editor of the
newspaper The New York Times, founded the National Sculpture Society (NSS) in 1893, the
first organization dedicated solely to the advancement of sculpture. Incorporated in 1896 to
promote sculptural production and encourage the exhibition and sale of the plastic arts, the
National Sculpture Society (NSS) elected John Quincy Adams Ward (1830-1910), the
prestigious sculptor of public monuments, to serve as its first president, an office he held
from 1893 to 1905. During the last twenty years of his life, Ward dedicated much time to
public and private organizations that promoted public art. To the end, he headed the NSS
committee that oversaw the sculptural decoration of the Library of Congress Reading Room
in Washington D.C. as well as the building and decorating of the Dewey Arch----a monument
in New York to honor Admiral George Dewey. He was a champion of the City Beautiful
Movement---an effort to increase the presence of urban art---and defended the central role
that sculpture played in its national program. The National Sculpture Society promoted the
production of sculpture by standardizing procedures for competitions, enhancing the
professional status of sculptors, and encouraging commissions for American sculpture in
homes, public buildings, parks, and squares. Moreover, it included members in its
organization who were not sculptors, hoping to close the gap between artists and the great
body of the people, not merely well-to-do patrons, but the working public. The NSS
encouraged the commission and purchase of sculptures for both private consumption---home
and garden ---and for public enjoyment---parks and squares. Through this campaign, smallscale sculptures---either reductions of monumental artworks or smaller-sized originals---were brought to the attention of an interested public. 64
64

Adapted from TOEFL pBT, 2005.08, Educational Testing Services

87

Version 3.0

[]

3.2 Speed Reading and Comprehension


3.2.1 Skimming, Scanning, and Scrolling
Skimming
Skimming is used to quickly identify the main ideas of a text. When you read the newspaper,
youre probably not reading it word-by-word, instead youre scanning the text. Skimming is
done at a speed three to four times faster than normal reading. People often skim when they
have lots of material to read in a limited amount of time. Use skimming when you want to see
if an article may be of interest in your research. There are many strategies that can be used
when skimming. Some people read the first and last paragraphs using headings, summarizes
and other organizers as they move down the page or screen. You might read the title, subtitles,
subheading, and illustrations. Consider reading the first sentence of each paragraph. This
technique is useful when youre seeking specific information rather than reading for
comprehension. Skimming works well to find dates, names, and places.

Focus on the key words and look for main points.

Decide if the text contains information they need or want to know.

Do not read it word-by-word, instead read in groups of words.

Scanning
Scanning is a technique you often use to search for key words or ideas. In most cases,
you know what you are looking for, so you are focused on finding a particular answer.

Moving your eyes quickly down the page seeking specific words and phrases.

Once youve scanned the document, you might go back and skim it.

Scrolling

Scrolling is sliding the text across displayer vertically. Scrolling does not change the
layout of the text, but moves the users view across what is apparently a larger image
that is not wholly seen.

Scroll the text to get an overview of the article when skimming

Scroll the text to seek the specific details when reading paragraph.

88

[] Version 3.0
Further ReadingMethods of Reading 65
There are several types and methods of reading, with differing rates that can be attained for
each, for different kinds of material and purposes:
Sub-vocalized reading combines sight reading with internal sounding of the words as if
spoken. Advocates of speed reading claim it can be a bad habit that slows reading and
comprehension. These claims are currently backed only by controversial, sometimes nonexistent scientific research.
Speed Reading is a collection of methods for increasing reading speed without an
unacceptable reduction in comprehension or retention. It is closely connected to speed
learning.
Proofreading is a kind of reading for the purpose of detecting typographical errors. One can
learn to do it rapidly, and professional proofreaders typically acquire the ability to do so at
high rates, faster for some kinds of material than for others, while they may largely suspend
comprehension while doing so, except when needed to select among several possible words
that a suspected typographic error allows.
Structure-Proposition-Evaluation (SPE) method, popularized by Mortimer Adler in How
to Read a Book, mainly for non-fiction treatise, in which one reads a writing in three passes:
(1) for the structure of the work, which might be represented by an outline; (2) for the logical
propositions made, organized into chains of inference; and (3) for evaluation of the merits of
the arguments and conclusions.
This method involves suspended judgment of the work or its arguments until they are fully
understood.
Survey-Question-Read-Recite-Review (SQ3R) method, often taught in public schools,
which involves reading toward being able to teach what is read, and would be appropriate for
instructors preparing to teach material without having to refer to notes during the lecture.
Multiple Intelligences-based methods, which draw upon the readers diverse ways of
thinking and knowing to enrich his or her appreciation of the text. Reading is fundamentally a
linguistic activity: one can basically comprehend a text without resorting to other
intelligences, such as the visual (e.g., mentally "seeing" characters or events described),
auditory (e.g., reading aloud or mentally "hearing" sounds described), or even the logical
intelligence (e.g., considering "what if" scenarios or predicting how the text will unfold based
on context clues).
However, most readers already use several intelligences while reading and making a habit of
doing so in a more disciplined manner -- i.e., constantly, or after every paragraph -- can result
in more vivid, memorable experience."
65

Adapted from http://en.wikipedia.org/wiki/Reading_(process)

89

Version 3.0

[]

3.2.2 Reading in Group Words


(Definition of Sense Group)
We speak to share information. When we speak, we want the listener to understand what we
say. Often we speak without visual aid, in the form of a written script, for instance. Unless we
speak clearly, the message cannot be shared with the listener fully.
The span of memory is also important. The speaker cannot be legible unless he or she speaks
only a small group of words at a time. Longer stretches cannot be retained by memory.
Therefore, the speaker will facilitate the listener to share the message completely by giving
him or her clues to brief units of meaning, each at a time, in order to reach the total meaning
of the utterance without confusion. Each brief unit, discretely spoken, is called a sense group
and the clue to understand it is the silence (or pause) that occurs before and after it. The
meaning of an utterance will depend on how we identify sense groups with the pauses. For
example, the meaning of the following utterance changes according to sense groups:
The government of South Africa said the Zambian President has grossly neglected the
incidence of AIDS.
Note sense groups and note how they alter the meaning:
The government of South Africa said the Zambian President has grossly neglected
the incidence of AIDS
The government of South Africa said the Zambian President has grossly neglected
the incidence of AIDS

His tolerance, his kindliness, his stout common sense, had made him an object of suspicion
when the treason of the king and the court clique had given the extreme radicals their chance
to get hold of the government and kill their opponents. 66
His tolerance, his kindliness, his stout common sense, had made him an object of suspicion
when the treason of the king and the court clique had given the extreme radicals their chance
to get hold of the government and kill their opponents.
66

Adapted from The story of mankind, by Hendrik Van Loon

90

[] Version 3.0
Vocabulary Enlargement
kindliness
stout
treason
clique

[kandlns]
[stat]
[trizn]
[klik]

Sentence Pattern
Get hold of
Ill explain, and youll soon get hold of the idea.

As military high-explosives have become harder to get hold of, terrorists have moved
to commercially available materials.

Further ReadingThe Fall of Rome (excerpts) 67


THE text-books of ancient History give the date 476 as the year in which Rome fell, because in that
year the last emperor was driven off his throne. But Rome, which was not built in a day, took a long
time falling. The process was so slow and so gradual that most Romans did not realise how their old
world was coming to an end. They complained about the unrest of the times -- they grumbled about the
high prices of food and about the low wages of the workmen -- they cursed the profiteers who had a
monopoly of the grain and the wool and the gold coin. Occasionally they rebelled against an unusually
rapacious governor. But the majority of the people during the first four centuries of our era ate and
drank (whatever their purse allowed them to buy) and hated or loved (according to their nature) and
went to the theatre (whenever there was a free show of fighting gladiators) or starved in the slums of
the big cities, utterly ignorant of the fact that their empire had outlived its usefulness and was doomed
to perish.
How could they realize the threatened danger? Rome made a fine showing of outward glory. Wellpaved roads connected the different provinces; the imperial police were active and showed little
tenderness for highwaymen. The frontier was closely guarded against the savage tribes who seemed to
be occupying the waste lands of northern Europe. The whole world was paying tribute to the mighty
city of Rome, and a score of able men were working day and night to undo the mistakes of the past and
bring about a return to the happier conditions of the early Republic.
But the underlying causes of the decay of the State, of which I have told you in a former chapter, had
not been removed and reform therefore was impossible. Rome was, first and last and all the time, a
city-state as Athens and Corinth had been city-states in ancient Hellas. It had been able to dominate the
Italian peninsula. But Rome as the ruler of the entire civilized world was a political impossibility and
could not endure. Her young men were killed in her endless wars. Her farmers were ruined by long
military service and by taxation. They either became professional beggars or hired themselves out to
rich landowners who gave them board and lodging in exchange for their services and made them
serfs, those unfortunate human beings who are neither slaves nor freemen, but who have become
part of the soil upon which they work, like so many cows, and the trees. The Empire, the State, had
become everything. The common citizen had dwindled down to less than nothing. As for the slaves,
they had heard the words that were spoken by Paul. They had accepted the message of the humble
carpenter of Nazareth. They did not rebel against their masters. (To be continued)
67

Adapted from The story of mankind, by Hendrik Van Loon

91

Version 3.0

[]

1. Research has shown that aggressive behavior can be learned through a combination of
modeling and positive reinforcement of the aggressive behavior and that children are
influenced by the combined forces of observing aggressive behavior in parents, pears, or
fictional role models and of noting either positive reinforcement for the aggressive behavior
or, minimally, a lack of negative reinforcement for the behavior.

2. The difference between the chemical composition of the cell walls of fungi and those of
plants is of enormous importance because it enables the tips of the growing hyphae, the
threadlike cells of the fungus, to secrete enzymes that break down the walls of plant cells
without having any effect on those of the fungus itself.

3. Despite the gathering of more information about ordinary women during the nineteenth
century, most of the writing about women conformed to the great women theory of
history, just as much of mainstream American history concentrated on great men.

4. A long-held view of the history of the English colonies that became the United States has
been that Englands policy toward these colonies before 1763 was dictated by commercial
interests and that a change to a more imperial policy, dominated by expansionist militarist
objectives, generated the tensions that ultimately led to the American Revolution.

92

[] Version 3.0
Vocabulary Enlargement
hypha
enzyme
imperial

[haf]
[enzam]
[mprl]

Sentence Pattern
Conform to
Science makes sense of the world by showing how things conform to its hypotheses.

If you disagree and dont conform to their expectations, some of them get irritated.

Further ReadingWomen and the contribution in 17th century 68
During the seventeenth and eighteenth centuries, almost nothing was written about the contributions
of women during the colonial period and the early history of the newly formed United States. Lacking
the right to vote and absent from the seats of power, women line were not considered an important
force in history. Anne Bradstreet wrote some significant poetry in the seventeenth century, Mercy
Otis Warren produced the best contemporary history of the American Revolution, and Abigail Adams
penned important letters showing she exercised great political influence over her husband, John, the
second President of the United States. But little or no notice was taken of these contributions. During
these Centuries, women remained invisible in history books.
Throughout the nineteenth century, this lack of visibility continued, despite the efforts of female
authors writing about women. These writers, like most of their male counterparts, were amateur
historians. Their writings were celebratory in nature, and they were uncritical in their selection and
use of sources.
During the nineteenth century, however, certain feminists showed a keen sense of history by
keeping records of activities in which women were engaged. National, regional, and local womens
organizations compiled accounts of their doings. Personal correspondence, newspaper clippings, and
souvenirs were saved and stored. These sources from the core of the two greatest collections of
womens history in the United States one at the Elizabeth and Arthur Schlesinger Library at Radcliffe
College, and the other the Sophia Smith Collection at Smith College. Such sources have provided
valuable materials for later Generations of historians.
Despite the gathering of more information about ordinary women during the nineteenth Century,
most of the writing about women conformed to the great women theory of History, just as much of
mainstream American history concentrated on great men. To demonstrate that women were making
significant contributions to American life, female authors singled out women leaders and wrote
biographies, or else important women produced their autobiographies. Most of these leaders were
involved in public life as reformers, activists working for womens right to vote, or authors, and were
not representative at all of the great of ordinary woman. The lives of ordinary people continued,
generally, to be untold in the American histories being published.
68

Adapted from TOEFL pBT, 2000.05, Educational Testing Services

93

Version 3.0

[]

3.2.3 Internal Structure Skimming


__________________________________
Orientation is simply facing in the right direction; navigation involves finding ones
way from point A to point B. 69
Fossils are the remains and traces (such as footprints or other marks) of ancient plant and
animal life that are more than 10,000 years old. 70
Musical theater is a form of entertainment that tells stories through songs, words, and
dancing.

______________________________________
Soluble ions such as calcium, sodium, potassium, and some magnesium are dissolved
and transported. Insoluble ions such as aluminum, iron, and silicon stay where they are
and form the thin, fertile skin of soil on which vegetation can grow. 71

A few art collectors, such as Tames Bowdoin III of Boston, William Byrd of Virginia,
and the Aliens and Hamiltons of Philadelphia introduced European art traditions to those
colonists privileged to visit their galleries, especially aspiring artists, and established in
their respective communities the idea of the value of art and the need for institutions
devoted to its encouragement. 72

_______________________________________
This new design concept, coupled with the sharp postwar reactions to the styles and
conventions of the preceding decades, created an entirely new public taste which caused
Art Nouveau types of glass to fall out of favor. 73

Rocks are composed of elements and compounds of elements. Feldspar, which is the
most abundant mineral on the Earths surface, is basically made up of the oxides silica
and alumina combined with alkalies like potassium and some so-called impurities such
as iron. 74

69

Adapted from Orientation and Navigation, TPO 11, Educational Testing Services
Adapted from TOEFL pBT, 1997.12, Educational Testing Services
71
Adapted from TOEFL pBT, 1998.10, Educational Testing Services
72
Adapted from TOEFL pBT, 1998.10, Educational Testing Services
73
Adapted from TOEFL pBT, 2000.05, Educational Testing Services
74
Adapted from TOEFL pBT, 2002.05, Educational Testing Services
70

94

[] Version 3.0
Vocabulary Enlargement
dissolve
aspiring
imperial
Calcium
Sodium
Potassium
magnesium

[dzlv]
[spar]
[mprl]
[klsm]
[sodm]
[ptsm]
[mgnizm]

Sentence Pattern
Be privileged to
Stark: People vote me. Id be privileged to serve them.

Ill be privileged to finish my career at Liverpool.

Further ReadingThe Art Nouveau style


The end of the nineteenth century and the early years of the twentieth century were marked by the
development of an international Art Nouveau style, characterized by sinuous Lines, floral and
vegetable motifs, and soft evanescent coloration. The Art Nouveau style was an eclectic one, bringing
together elements of Japanese art, motifs of ancient cultures, and natural forms. The glass objects of
this style were elegant in outline, although often deliberately distorted, with pale or iridescent surfaces.
A favored device of the style was to imitate the iridescent surface seen on ancient glass that had been
buried. Much of the Art Nouveau glass produced during the years of its greatest popularity had been
generically termed art glass. Art glass was intended for decorative purposes and relied for its effect
pon carefully chosen color combinations and innovative techniques.
France produced a number of outstanding exponents of the Art Nouveau style; among the most
celebrated was Emile Galle (1846-1904). In the United States, Louis Comfort Tiffany (1843-1933)
was the most noted exponent of this style, producing a great variety of Glass forms and surfaces,
which were widely copied in their time and are highly prized today. Tiffany was a brilliant designer,
successfully combining ancient Egyptian, Japanese, and Persian motifs.
The Art Nouveau style was a major force in the decorative arts from 1895 until 1915, although its
influence continued throughout the mid-1920s. It was eventually to be overtaken by a new school of
thought known as Functionalism that had been present since the turn of the century. At first restricted
to a small avant-garde group of architects and designers, Functionalism emerged as the dominant
influence upon designers after the First World War. The basic tenet of the movement-that function
should determine from-was not a new concept. Soon a distinct aesthetic code evolved: from should be
simple, surfaces plain, and any ornament should be based on geometric relationships. This new design
concept, coupled with the sharp postwar reactions to the styles and conventions of the preceding
decades, created an entirely new public taste which caused Art Nouveau types of glass to fall out of
favor. The new taste demanded dramatic effects of contrast, stark outline and complex textural
surfaces.
95

Version 3.0

[]

_________________________________________________
However, unlike the cases of sea otters and pinnipeds (seals, sea lions, and walruses,
whose limbs are functional both on land and at sea), it is not easy to envision what the
first whales looked like. 75
According to their digestive systems, these herbivores can be divided into two categories:
the non-ruminants (such as the zebra, which has a digestive system like a horse) and the
ruminants (such as the wildebeest, topi, and gazelle, which are like the cow). 76
A of B of C__________________________________________________
Coincident with concerns about the accelerating loss of species and habitats has been a
growing appreciation of the importance of biological diversity, the number of species in a
particular ecosystem, to the health of the Earth and human well-being. 77
____________________________________________________

Not A but B
Not A so much as B
Rather than A, B
More B than A
In addition to A, B

As much A as B
As B, A
Like B, so A
Just as B, A
It is not just B, A

Cooperation, rather than evoking a characteristic at the opposite extreme of human nature
from competition, is in reality a necessary factor in competition.
As the roles men and women played in society became more rigidly defined, so did the
roles they played in the home.
If one begins by examining why ancients refer to Amazons, it becomes clear that ancient
Greek descriptions of such societies were meant not so much to represent observed
historical fact-real Amazonian societies-but rather to offer moral lessons on the supposed
outcome of womens rule in their own society.
In the twenties, jazz became the hottest new thing in dance music, much as ragtime had at
the turn of the century, and as would rhythm and blues in the fifties, rock in the fifties, and
disco in the seventies. 78
Expressive leaders are less concerned with the overall goals of the group than with
providing emotional support to group members and attempting to minimize tension and
conflict among them.
75

Adapted from Origin of Cetaceans, OG Practice Set 1, Educational Testing Services


Adapted from Feeding Habits of East African Herbivores, OG Test 1
77
Adapted from TOEFL pBT, 1995.12, Educational Testing Services
78
Adapted from TOEFL pBT, 1998.01, Educational Testing Services
76

96

[] Version 3.0
Vocabulary Enlargement
pinniped
seal
walrus
ruminant
Wilderbeest
Topi
Gazelle

[pnped]
[sil]
[wlrs]
[rumnnt]
[wldbist]
[tp]
[gzl]

Further ReadingThe development of Jazz 79


The development of jazz can be seen as part of the larger continuum of American popular
music, especially dance music. In the twenties, jazz became the hottest new thing in dance
music, much as ragtime had at the turn of the century, and as would rhythm and blues in the
fifties, rock in the fifties, and disco in the seventies.
But two characteristics distinguish jazz from
other dance music. The first is improvisation,
the changing of a musical phrase according
to the players inspiration. Like all artists,
jazz musicians strive for an individual style,
and the improvise or paraphrased is a jazz
musicians main opportunity to display his or
her individuality. In early jazz, musicians
often improvised melodies collectively, thus
creating a kind of polyphony. There was little
soloing as such, although some New Orleans
players, particularly cornet player Buddy
Bolden, achieved local fame for their ability to improvise a solo. Later the idea of the choruslong or multichorus solo took hold. Louis Armstrongs instrumental brilliance, demonstrated
through extended solos, was a major influence in this development.
Even in the early twenties, however, some jazz bands had featured soloists. Similarly, show
orchestras and carnival bands often included one or two such get-off musicians.
Unimproved, completely structured jazz does exist, but the ability of the best jazz musicians
to create music of great cohesion and beauty during performance has been a hallmark of the
music and its major source of inspiration and change.
The second distinguishing characteristic of jazz is a rhythmic drive that was initially called
"hot" and later "swing". In playing hot , a musician consciously departs from strict meter to
create a relaxed sense of phrasing that also emphasizes the underlying rhythms.("Rough" tone
and use of moderate vibrato also contributed to a hot sound.) Not all jazz is hot, however,
many early bands played unadorned published arrangements of popular songs. Still, the
proclivity to play hot distinguished the jazz musician from other instrumentalists.
79

Adapted from TOEFL pBT, 1998.01, Educational Testing Services

97

Version 3.0

[]

3.3 Critical Reading


3.3.1 Semantic Transformation and Logical Relationship
Paraphrase
A paraphrase is a restatement of the meaning of a text or passage using other words.

Original Sentence
Most birds have body
temperatures between 40 and 42
degrees Celsius, while most
mammals have body
temperatures between 36 and 38
degrees.

If we factor in the quality of life,


then the carrying capacity of
Earth will be much smaller than
if we simply estimate how much
food it takes to avoid starvation.

Although her early theatrical


career had included stints as an
actress, she was not primarily
interested in storytelling or
expressing emotions through
dance; the drama of her dancing
emanated from her visual
effects.

Paraphrase
The body temperatures of
most mammals are
between 36 and 38
degrees Celsius, whereas
those of birds are
between 40 and 42
degrees.
If the quality of life is
considered, Earth can
support fewer people, and
more if avoiding
starvation is most
important.
Fuller used visual effects
to dramatize the stories
and emotions expressed
in her work.

Keys to Solution
Determine the precise meaning of the highlighted sentence. Dividing the sentence into
meaningful parts helps you to grasp its correct meaning.
See if the sentence makes a reference to an idea outside of the sentence. If so, you need
to find what this reference word indicates.
Identify key ideas and eliminate unimportant information. Information that provides
examples or equivalent ideas, such as appositive phrases, is unimportant information.
Decide which answer choice best paraphrases only the key information from the original
sentence. You need to understand how key ideas relate to one another.
Eliminate answer choices if they leave out important pieces of information, confuse the
relationship between the ideas, or generally alter the meaning of the original sentence.
98

[] Version 3.0
Vocabulary Enlargement
celsius
starvation

[slss]
[stven]

Further ReadingDesert Animals 80


Large animals that inhabit the desert have
evolved a number of adaptations for
reducing the effects of extreme heat. One
adaptation is to be light in color, and to
reflect rather than absorb the Suns rays.
Desert mammals also depart from the
normal
mammalian
practice
of
maintaining a constant body temperature.
Instead of trying to keep down the body
temperature deep inside the body, which
would involve the expenditure of water
and energy, desert mammals allow their
temperatures to rise to what would
normally be fever height, and
temperatures as high as 46 degrees
Celsius have been measured in Grants
gazelles. The overheated body then cools
down during the cold desert night, and
indeed the temperature may fall unusually low by dawn, as low as 34 degrees Celsius in the
camel. This is an advantage since the heat of the first few hours of daylight is absorbed in
warming up the body, and an excessive buildup of heat does not begin until well into the day.
Another strategy of large desert animals is to tolerate the loss of body water to a point that
would be fatal for non-adapted animals. The camel can lose up to 30 percent of its body
weight as water without harm to itself, whereas human beings die after losing only 12 to 13
percent of their body weight. An equally important adaptation is the ability to replenish this
water loss at one drink. Desert animals can drink prodigious volumes in a short time, and
camels have been known to imbibe over 100 liters in a few minutes. A very dehydrated
person, on the other hand, cannot drink enough water to rehydrate at one session, because the
human stomach is not sufficiently big and because a too rapid dilution of the body fluids
causes death from water intoxication. The tolerance of water loss is of obvious advantage in
the desert, as animals do not have to remain near a water hole but can obtain food from
grazing sparse and far-flung pastures. Desert-adapted mammals have the further ability to
feed normally when extremely dehydrated, it is a common experience in people that appetite
is lost even under conditions of moderate thirst.
80

Adapted from TOEFL pBT, 1997.01, Educational Testing Services

99

Version 3.0

[]

Function
Illustrate
Negative
Explain
Give reasons
Show result
Compare
Contrast
Add
Limit
Emphasize

Transitions
For example, for instance, make evident, to illustrate, such, such
as
few, little, ignore, fail to, hardly
at this point, furthermore, how, in fact, in this case
as a result of, because of, due to, one reason is, since
accordingly, as a result, consequently, otherwise, therefore, thus
both, equally important, like, the same, similarly, similar to
although, conversely, however, in contrast, instead, nevertheless,
on the contrary, on the other hand, rather, unlike, whereas, while
also, another, as well as, finally, furthermore, moreover, not
only...but also, too
although, but, except for, even though, however, yet
certainly, clearly, indeed, in fact, most importantly, surely

Illustrate
Wind velocity also increases with altitude and may cause serious stress for trees, as is made
evident by the deformed shapes at high altitudes.
Which of the sentences below best expresses the essential information in the highlighted
sentence in the passage? Incorrect choices change the meaning in important ways or leave out
essential information.
A. Because of their deformed shapes at high altitudes, trees are not likely to be seriously
harmed by the strong winds typical of those altitudes.
B. As altitude increases, the velocity of winds increase, leading to a serious decrease in the
number of trees found at high altitudes.
C. The deformed shapes of trees at high altitudes show that wind velocity, which increase
with altitude, can cause serious hardship for trees.
D. Increased wind velocity at high altitudes deforms the shapes of trees, and this may cause
serious stress for trees.
Negative
Few people, however, regardless of their feelings towards snails, know a great deal of them.
Which of the sentences below best expresses the essential information in the highlighted
sentence in the passage? Incorrect choices change the meaning in important ways or leave out
essential information.
A. Some people dont know how they feel about snails.
B. Most people are quite knowledgeable about snails.
C. Most people are quite ignorant about snails.
D. Snails live in only a few specific areas of the world.

100

[] Version 3.0
Further ReadingTimberline vegetation on mountains
The transition from forest to treeless tundra on a mountain slope is often a dramatic one.
Within a vertical distance of just a few tens of meters, trees disappear as a life-form and are
replaced by low shrubs, herbs, and grasses. This rapid zone of transition is called the upper
timberline or tree line. In many semiarid areas there is also a lower timberline where the
forest passes into steppe or desert at its lower edge, usually because of a lack of moisture.
The upper timberline, like the snow line, is highest in the tropics and lowest in the Polar
Regions. It ranges from sea level in the Polar Regions to 4,500 meters in the dry subtropics
and 3,500-4,500 meters in the moist tropics. Timberline trees are normally evergreens,
suggesting that these have some advantage over deciduous trees (those that lose their leaves)
in the extreme environments of the upper timberline. There are some areas, however, where
broadleaf deciduous trees form the timberline. Species of birch, for example, may occur at
the timberline in parts of the Himalayas.
At the upper timberline the trees begin to become twisted and deformed. This is particularly
true for trees in the middle and upper latitudes, which tend to attain greater heights on ridges,
whereas in the tropics the trees reach their greater heights in the valleys. This is because
middle- and upper- latitude timberlines are strongly influenced by the duration and depth of
the snow cover. As the snow is deeper and lasts longer in the valleys, trees tend to attain
greater heights on the ridges, even though they are more exposed to high- velocity winds and
poor, thin soils there. In the tropics, the valleys appear to be more favorable because they are
less prone to dry out, they have less frost, and they have deeper soils.
There is still no universally agreed-on explanation for why there should be such a dramatic
cessation of tree growth at the upper timberline. Various environmental factors may play a
role. Too much snow, for example, can smother trees, and avalanches and snow creep can
damage or destroy them. Late-lying snow reduces the effective growing season to the point
where seedlings cannot establish themselves. Wind velocity also increases with altitude and
may cause serious stress for trees, as is made evident by the deformed shapes at high altitudes.
Some scientists have proposed that the presence of increasing levels of ultraviolet light with
elevation may play a role, while browsing and grazing animals like the ibex may be another
contributing factor. Probably the most important environmental factor is temperature, for if
the growing season is too short and temperatures are too low, tree shoots and buds cannot
mature sufficiently to survive the winter months.
Above the tree line there is a zone that is generally called alpine tundra. Immediately adjacent
to the timberline, the tundra consists of a fairly complete cover of low-lying shrubs, herbs,
and grasses, while higher up the number and diversity of species decrease until there is much
bare ground with occasional mosses and lichens and some prostrate cushion plants. Some
plants can even survive in favorable microhabitats above the snow line. The highest plants in
the world occur at around 6,100 meters on Makalu in the Himalayas. At this great height,
rocks, warmed by the sun, melt small snowdrifts.
The most striking characteristic of the plants of the alpine zone is their low growth form. This
enables them to avoid the worst rigors of high winds and permits them to make use of the
higher temperatures immediately adjacent to the ground surface. In an area where low
temperatures are limiting to life, the importance of the additional heat near the surface is
crucial. The low growth form can also permit the plants to take advantage of the insulation
provided by a winter snow cover. In the equatorial mountains the low growth form is less
prevalent

101

Version 3.0

[]

Explain
Proponents point to features such as the terraced beaches shown in one image, which could
conceivably have been left behind as a lake or ocean evaporated and the shoreline receded.
But detractors maintain that the terraces could also have been created by geological activity,
perhaps related to the geologic forces that depressed the Northern Hemisphere far below the
level of the south, in which case they have nothing whatever to do with Martian water. 81
A. But detractors argue that geological activity may be responsible for the water associated
with the terraces.
B. But detractors argue that the terraces may have been formed by geological activity rather
than by the presence of water.
C. But detractors argue that the terraces may be related to geological forces in the Northern
Hemisphere of Mars, rather than to Martian water in the south.
D. But detractors argue that geological forces depressed the Northern Hemisphere so far
below the level of the south that the terraces could not have been formed by water.
Give Reasons
He refused to develop projection technology, reasoning that if he made and sold projectors,
then exhibitors would purchase only one machine-a projector-from him instead of several. 82
A. Edison was more interested in developing a variety of machines than in developing a
technology based on only one
B. Edison refused to word on projection technology because he did not think exhibitors
would replace their projectors with newer machines.
C. Edison did not want to develop projection technology because it limited the number of
machines he could sell.
D. Edison would not develop projection technology unless exhibitors agree to purchase
more than one projector from him.
Show Results
The American Civil War began with the surrender of the Northern army of the garrison at
Fort Sumter. This military post was located in the city of Charleston, the hotbed of Southern
sedition, and thus capturing it was a huge morale boost for the Southern states. 83
A. The Southerners felt defeated when they gave up Fort Sumter.
B. The Southerners would not lose faith in their victory although they lost Fort Sumter.
C. The Southerners felt motivated by the win at Fort Sumter.
D. The Southerners knew that the win at Fort Sumter predicted a series of victories over the
North.
81

Adapted from Running water on mars, TPO 8, Educational Testing Services


Adapted from The Official Guide to the New TOEFL iBT, Third Edition, Educational Testing Service.
83
Adapted from Longman iBT General Course for the TOEFL Reading, 2nd edition, by Ji-Yeon LEE, published by
Pearson Education Korea, Ltd.
82

102

[] Version 3.0
Vocabulary Enlargement
terrace
detractor
hotbed
morale

[ters]
[dtrkt]
[htbed]
[mrl]

Sentence Pattern
Have nothing to do with
But all these good personality traits have nothing to do with being a good teacher

Practical Translation Training



These proposals _______________________________for reinventing market capitalism.
Further Reading: Urban growth in Europe and in America 84
The principal difference between urban growth in Europe and in the American colonies was the slow
evolution of cities in the former and their rapid growth in the latter. In Europe they grew over a period
of centuries from town economies to their present urban structure. In North America, they started as
wilderness communities and developed to mature urbanisms in little more than a century.
In the early colonial day in North America, small cities sprang up along the Atlantic Coastline, mostly
in what are now New America, small cities sprang up along the Atlantic United States and in the
lower Saint Lawrence valley in Canada. This was natural because these areas were nearest England
and France, particularly England, from which most capital goods (assets such as equipment) and
many consumer goods were imported Merchandising establishments were, accordingly,
advantageously located in port cities from which goods could be readily distributed to interior
settlements. Here, too, were the favored locations for processing raw materials prior to export. Boston,
Philadelphia, New York, Montreal, and other cities flourished, and, as the colonies grew, these cities
increased in importance.
This was less true in the colonial South, where life centered around large farms, known as plantations,
rather than around towns, as was the case in the areas further north along the Atlantic coastline. The
local isolation and the economic self-sufficiency of the plantations were antagonistic to the
development of the towns. The plantations maintained their independence because they were located
on navigable streams and each had a wharf accessible to the small shipping of that day. In face, one of
the strongest factors in the selection of plantation land was the desire to have it front on a water
highway.
When the United States became an independent nation in 1776, it did not have a single city as large as
50,000 inhabitants, but by 1820 it had a city of more than 10,000 people, and by 1880 it had recorded
a city of over one million. It was not until after 1823, after the mechanization of the spinning had
weaving industries that cities started drawing young people away from farms. Such migration was
particularly rapid following the Civil War (1861-1865).
84

Adapted from TOEFL pBT, 2000.08, Educational Testing Services

103

Version 3.0

[]

Compare and Contrast


The whale retained a tail and lacked a fluke, the major means of locomotion in modern
cetaceans. The structure of the backbone shows, however, that Ambulocetus swam like
modern whales by moving the rear portion of its body up and down, even though a fluke was
missing. 85
A. Even though Ambulocetus swam by moving its body up and down, it did not have a
backbone.
B. The backbone of Ambulocetus, which allowed it to swim, provides evidence of its
missing fluke.
C. Although Ambulocetus had no fluke, its backbone structure shows that it swam like
modern whales.
D. By moving the rear parts of their bodies up and down, modern whales swim in a different
way from the way Ambulocetus swam.
Limit
He noticed that as they fluttered around in the cage, they often launched themselves in the
direction of their normal migratory route. He then set up experiments with caged starlings
and found that their orientation was in fact, in the proper migratory direction except when the
sky was overcast, at which times there was no clear direction to their restless movements. 86
A. Experiments revealed that caged starlings displayed a lack of directional sense and
restless movements.
B. Experiments revealed that caged starlings were unable to orient themselves in the
direction of their normal migratory route.
C. Experiments revealed that the restless movement of caged starlings had no clear
direction.
D. Experiments revealed that caged starlings orientation was accurate unless the weather
was overcast.
Emphasize
Tornadoes are natures most violent storms. The United Kingdom experiences an average of
32 tornadoes each year which is the highest frequency of reported tornadoes per unit area in
the world but the deadliest and most violent tornadoes occur in the United States. 87
A. The United Kingdom reports the deadliest number of tornadoes per year.
B. The United States has the highest annual frequency of tornadoes per year.
C. For its size the United States has 32 tornadoes per year.
D. The United States suffers more than any other country due to the strength of its
tornadoes.
85

Adapted from The Origins of Cetaceans, OG Practice Set 1, Educational Testing Services
Adapted from Orientation and Navigation, TPO 11, Educational Testing Services
87
Adapted from Longman iBT General Course for the TOEFL Reading, 2nd edition, by Ji-Yeon LEE, published by
Pearson Education Korea, Ltd.
86

104

[] Version 3.0
Vocabulary Enlargement
fluke
cetacean
starling
migratory

[fluk]
[sten]
[strl]
[martri]

Sentence Pattern
Set up
The government announced the setting up of a special fund.

A fund will be set up for the orphans.

Practical Translation Training



_______________________________________________________________________
Further Reading: Many Types of Weather 88
Many of the most flexible examples of tool use in animals come from primates (the order that
includes humans, apes, and monkeys). For example, many wild primates use objects to
threaten outsiders. But there are many examples of tool use by other mammals, as well as by
birds and other types of animals.
Tools are used by many species in the capture or preparation of food. Chimpanzees use sticks
and poles to bring out ants and termites from their hiding places. Among the most complex
tool use observed in the wild is the use of stones by Ivory Coast chimpanzees to crack nuts
open. They select a large flat stone as an anvil (a heavy block on which to place the nuts) and
a smaller stone as a hammer. Stones suitable for use as anvils are not easy to find, and often a
chimpanzee may carry a haul of nuts more than 40 meters to find a suitable anvil. The use of
tools in chimpanzees is especially interesting because these animals sometimes modify tools
to make them better suited for their intended purpose. To make a twig more effective for
digging out termites, for example, a chimp may first strip it of its leaves. Surprisingly, there is
also a species of bird that uses sticks to probe holes in the search for insects. One of the
species of Galapagos finch, the woodpecker finch, picks up or breaks off a twig, cactus spine,
or leaf stem. This primitive tool is then held in the beak and used to probe for insects in holes
in trees that the bird cannot probe directly with its beak. Birds have been seen to carry twigs
from tree to tree searching for prey. Tools may also be used for defense. Hermit crabs grab
sea anemones with their claws and use them as weapons to repel their enemies. Studies have
demonstrated that these crabs significantly improve their chances against predators such as
octopus by means of this tactic. Also, many species of forest-dwelling primates defend
themselves by throwing objects, including stones, at intruders.
88

Adapted from TOEFL pBT, 2004.08, Educational Testing Services

105

Version 3.0

[]

3.3.2 Semantic Information

Keys to Solution
Identify the key word in the question, and scan the passage for this word or
corresponding idea. Carefully read the related information.
The correct answer is a restatement of particular information from the passage. Therefore,
while the wording may be different, the similar ideas should be apparent.
Exclude answer choices that is contradicted by the passage or not mentioned in the
passage, or is stated in the passage but is irrelevant to the question.
According to paragraph 1, which of
the following is true of the Late
Cretaceous climate?
A. Summers were very warm and
winters were very cold.
B. Shallow seas on the continents
caused
frequent
temperature
changes.
C. The climate was very similar to
todays climate.
D. The climate did not change
dramatically from season to
season.
According to the passage, a highly
significant factor in the
development of realist and
naturalist literature was
A. the Civil War
B. a recognition that romanticism was
unpopular
C. an increased interest in the study of
common speech
D. an economic depression

According to the passage,


Krzysztof Penderecki is known for
which of the following practices?
A. Using tones that are clumped
together
B. Combining traditional and
nontradinonal instruments
C. Seating musicians in unusual areas
of an auditorium
D. Playing Western music for nonWestern audiences

Paleontologists have argued for a long time that the


demise of the dinosaurs was caused by climatic
alterations associated with slow changes in the positions
of continents and seas resulting from plate tectonics. Off
and on throughout the Cretaceous (the last period of the
Mesozoic era, during which dinosaurs flourished), large
shallow seas covered extensive areas of the continents.
Data from diverse sources, including geochemical
evidence preserved in seafloor sediments, indicate that
the Late Cretaceous climate was milder than todays. The
days were not too hot, nor the nights too cold. The
summers were not too warm, nor the winters too frigid.
The shallow seas on the continents probably buffered the
temperature of the nearby air, keeping it relatively
constant.

The economic depression in the late-nineteenthcentury United States contributed significantly to a


growing movement in literature toward realism and
naturalism. After the 1870s, a number of important
authors began to reject the romanticism that had
prevailed immediately following the Civil War of
1861-1865 and turned instead to realism.

Non-Western music typically divides and interval


between two pitches more finely than western music
does, thereby producing a greater number of distinct
tones, or microtones, within the same interval.
Composers such as Krzysztof Penderecki create
sound that borders on electronic noise through tone
clustersclosely spaced tones played together and
heard as a mass, block, or band of sound.

106

[] Version 3.0
Vocabulary Enlargement
tectonics
cretaceous
buffer
tundra
tributary
formidable

[tektnks]
[krtes]
[bf]
[tndr]
[trbjt()r]
[frmdbl]

[]
[]

Sentence Pattern
Associated with ..
Cigarette smoking has been associated with lung cancer.

Close off
The stakes are higher when people are approaching the age when options tend to close
off and lifelong commitments must be made.

Practical Translation Training



Love is a telephone________________________________________________________
Further ReadingVolcanic fire and glacial ice 89
Volcanic fire and glacial ice are natural enemies. Eruptions at glaciated volcanoes typically destroy
ice fields as they did in 1980 when 70 percent of Mount Saint Helens ice cover was demolished.
During long dormant intervals, glaciers gain the upper hand cutting deeply into volcanic cones and
eventually reducing them to rubble. Only rarely do these competing forces of heat and cold operate in
perfect balance to create a phenomenon such as the steam caves at Mount Rainier National Park.
Located inside Rainiers two ice-filled summit craters, these caves form a labyrinth of tunnels and
vaulted chambers about one and one -half miles in total length. Their creation depends on an unusual
combination of factors that nature almost never brings together in one place. The cave-making recipe
calls for a steady emission of volcanic gas and heat, a heavy annual snowfall at an elevation high
enough to keep it from melting during the summer, and a bowl-shaped crater to hold the snow.
Snow accumulating yearly in Rainiers summit craters is compacted and compressed into a dense
form of ice called firm, a substance midway between ordinary ice and the denser crystalline ice that
makes up glaciers. Heat rising from numerous openings (called fumaroles) along the inner crater walls
melts out chambers between the rocky walls and the overlying ice pack. Circulating currents of warm
air then melt additional openings in the firm ice, eventually connecting the individual chambers and,
in in the large of Rainiers two craters, forming a continuous passageway that extends two-thirds of
the way around the craters interior
89

Adapted from TOEFL pBT, 1996.05, Educational Testing Services

107

Version 3.0

[]

According to paragraph 1, all of There is a quality of cohesiveness about the Roman world
the following are controls that held that applied neither to Greece nor perhaps to any other
together the roman world EXCEPT civilization, ancient or modern. Like the stone of Roman
A. administrative and legal systems
B. the presence of the military
C. a common language
D. transportation networks

According
to
the
cognitive
approach described in paragraphs
7 and 8, all of the following may
influence the decision whether to
act aggressively EXCEPT a
persons
A. Moral values

wall, which were held together both by the regularity of


the design and by that peculiarly powerful Roman
cement, so the various parts of the Roman realm were
bonded into a massive, monolithic entity by physical,
organizational, and psychological controls. The physical
bonds included the network of military garrisons, which
were stationed in every province, and the network of
stone-built roads that linked the provinces with Rome.
The organizational bonds were based on the common
principles of law and administration and on the universal
army of officials who enforced common standards of
conduct. The psychological controls were built on fear
and punishment on the absolute certainty that anyone or
anything that threatened the authority of Rome would be
utterly destroyed.
Paragraph 7: The Cognitive psychologists assert that our
behavior is influenced by our values, by the ways in
which we interpret our situations and by choice. For
example, people who believe that aggression is necessary
and justified-as during wartime-are likely to act
aggressively, whereas people who believe that a particular
war or act of aggression is unjust, or who think that
aggression is never justified, are less likely to behave
aggressively.

B. Previous experiences with


aggression

Paragraph 8: One cognitive theory suggests that


aggravating and painful events trigger unpleasant
feelings. These feelings, in turn, can lead to aggressive
action, but not automatically. Cognitive factors intervene.
C. Instinct to avoid aggression
People decide whether they will act aggressively or not on
D. Beliefs about other peoples the basis of factors such as their experiences with
aggression and their interpretation of other peoples
intentions
motives. Supporting evidence comes from research
showing that aggressive people often distort other
peoples motives. For example, they assume that other
people mean them harm when they do not.

Keys to Solution
Identify the key words from the question, and then scan the passage to find these key
words in their original context.
As with factual information questions, the correct answers will be paraphrased. It is
important to recognize corresponding ideas.
Use the process of elimination. By identifying correct statements of information one by
one, you will eventually find the incorrect one.
108

[] Version 3.0
Vocabulary Enlargement
cohesiveness
garrison
cognitive
aggravate

[kohisvns]
[grs()n]
[kntv]
[grvet]

Sentence Pattern
Based on
All of the work we do is based on science.

I choose yes or no based on my goals.

Practical Translation Training



For some teachers, ________________________________________________________
Further ReadingThe Renaissance 90
The French word renaissance means rebirth. It was first used in 1855 by the historian Jules Michelet
in his History of France, then adopted by historians of culture, by art historians, and eventually by
music historians, all of whom applied it to European culture during the 150 years spanning 1450-1600.
The concept of rebirth was appropriate to this period of European history because of the renewed
interest in ancient Greek and Roman culture that began in Italy and then spread throughout Europe.
Scholars and artists of the fifteenth and sixteenth centuries wanted to restore the learning and ideals of
the classical civilizations of Greece and Rome. To these scholars this meant a return to human-as
opposed to spiritual-values. Fulfillment in life-as opposed to concern about an afterlife- became a
desirable goal, and expressing the entire range of human emotions and enjoying the pleasures of the
senses were no longer frowned on. Artists and writers now turned to secular as well as religious
subject matter and sought to make their works understandable and appealing.
These changes in outlook deeply affected the musical culture of the Renaissance period--how people
thought about music as well as the way music was composed, experienced, discussed, and
disseminated. They could see the architectural monuments, sculptures, plays, and poems that were
being rediscovered, but they could not actually hear ancient music-although they could read the
writings of classical philosophers, poets, essayists, and music theorists that were becoming available
in translation. They learned about the power of ancient music to move the listener and wondered why
modern music did not have the same effect. For example, the influential religious leader Bernardino
Cirillo expressed disappointment with the learned music of his time. He urged musicians to follow the
example of the sculptors, painters, architects, and scholars who had rediscovered ancient art and
literature. The musical Renaissance in Europe was more a general cultural movement and state of
mind than a specific set of musical techniques. Furthermore, music changed so rapidly during this
century and a half-though at different rates in different countries-that we cannot define a single
Renaissance style.
90

Adapted from TOEFL pBT, 2004.10, Educational Testing Services

109

Version 3.0

[]


:
:


:
:
:
:
:
:
:

:
:
:
:
:
:

:
:
:
+ :
:

110

[] Version 3.0
Sentence Pattern
Lack of
The lack of funds posed difficulties for this organization last year.

I extend sincere apologies to the president for this lack of civility.

Lead to
A prolonged loneliness can lead you to believe that you are depressed, or, in some
circumstances, can lead to depression.

This type of situation may lead to massive disappointment for both of you and injure
what you have.

Practical Translation Training



_______________________________________________________________________
Brain Food
There is a common English word that is nine letters long. Each time you remove a letter from
it, it still remains an English word from nine letters right down to a single letter. What is
the original word, and what are the words that it becomes after removing one letter at a time?
Answer Example: Startling
Remove the l, and the word becomes: starting
Remove one t, and the word becomes: staring
Remove the a, and the word becomes: string
Remove the r, and the word becomes: sting
Remove the other t, and the word becomes: sing
Remove the g, and the word becomes: sin
Remove the s, and the word becomes: in
Remove the n, and the word becomes: I
Can you figure out another answer?
111

Version 3.0

[]

3.4 Extra Practice


Exercise 1: Sentence Simplification
Which of the sentences below best expresses the essential information in the highlighted
sentence in the passage? Incorrect choices change the meaning in important ways or leave
out essential information.
Oaks are native to the northern hemisphere.They include deciduous and evergreen species,
extending from cold latitudes to tropical Asia and the Americas.
A. All the deciduous and evergreen trees in Asia and the Americas are oaks.
B. Deciduous and evergreen oaks are found in cold and hot Asia and the Americas.
C. Oaks are deciduous or evergreen trees found in both cold and hot regions.
D. Deciduous and evergreen species of oaks survive best at cold or hot latitudes.

All of these factors led to a war between the North and South which would be the bloodiest in
the nations history.
A. There are many reasons for the war in which more people died than any other war.
B. During the war, there are more casualties in the south and north than in the east and west.
C. Only one reason contributed to the war which is the worst in the world history.
D. Many people died because of lack of blood transfusion which led to a war.

The people on both sides suffered, and that suffering was worse because it was inflicted by
people who had been their countrymen, and even their brothers.
A. If the war had not been with their brothers, there would have been no pain.
B. If the war had not taken place in the countryside, there would have been less pain.
C. If the war had been with other nations, the results would have been worse
D. If the war had not been with the same race, the pain would have been much less.

There can be no argument, however, that the Civil War was the most painful period in
American history.
A. Discussing the matter is worthless to everybody.
B. The vast majority of people accept the statement of fact.
C. It is impossible to hold a different opinion.
D. It is illegal to hold a different opinion.

112

[] Version 3.0

Exercise 2
Color in textiles is produced by dyeing, by printing, or by painting. Until the nineteenth
century, all dyes were derived from vegetable or, more rarely, animal or mineral sources,
Since madder plants could be grown practically everywhere, the roots of some species of the
madder plant family were used from the earliest period to produce a whole range of reds. Red
animal dyes, derived! from certain species of scale insects, were also highly valued from
ancient times through the Middle Ages. Blues were obtained from indigo, which was widely
cultivated in India and exported from there, and from woad, a plant common in Europe and
also used in the Near East from the beginning of the Christian era. Before the first, nonfading
"solid" green was invented in the early nineteenth century, greens were achieved by the
overdyeing or overprinting of yellow and blue. However, yellow dyes whether from weld or
some other plant source such as saffron or turmeric, invariably fade or disappear. This
accounts for the bluish tinge of what were once bright greens in, for example, woven tapestry.
The range of natural colors was hugely expanded and, indeed, superseded by the chemical
dyes developed during the eighteen hundreds. By 1900 a complete range of synthetic colors
had been evolved, many of them reaching a standard of resistance to fading from exposure to
light and to washing that greatly exceeded that of natural dyestuffs. Since then, the petroleum
industry has added many new chemicals, and from these other types of dyestuffs have been
developed. Much of the research in dyes was stimulated by the peculiarities of some of the
new synthetic fibers- Acetate rayon, for example, seemed at first to have no affinity for dyes
and a new range of dyes had to be developed; nylon and Terylene presented similar problems.
The printing of textiles has involved a number of distinct methods. With the exception of
printing patterns directly onto the cloth, whether by block, roller, or screen, all of these are
based on dyeing; that is, the immersion of the fabric in a dye bath.
1. The passage mainly discusses the
A. development of synthetic colors foe textiles during the nineteenth century
B. advantages of chemical dyes over dyes derived from plants and animals
C. differences between dyeing textiles and printing them
D. history of the use of natural and chemical dyes to color textiles
2. According to the passage, what was the source of most textile dyes that were used before
the nineteenth century?
A. Animals
B. Minerals
C. Plants
D. Chemicals

113

Version 3.0

[]

3. What was the advantage of using madder plants for different shades of red?
A. It was possible to cultivate madder plants in almost every location.
B. Madder plants produced brighter colors than other plant sources.
C. Plant sources produced more lasting colors than animal sources.
D. Dyes derived from the madder plants were easier to work with than other dyes.

4. The word invariably in paragraph 1 is closest in meaning to


A. without exception
B. steadily
C. after some time
D. noticeably
5. It can be inferred from the passage that the green areas in woven tapestries developed a
bluish tinge because
A. a darker color, like blue, dominates a light color, like yellow
B. light changed some of the green dye used in the tapestries to blue
C. the yellow dye. that was used in the tapestries had faded
D. the dyes used to color woven tapestries were made from minerals
6. The word superseded in paragraph 2 is closest in meaning to
A. Strengthened
B. Improved
C. Replaced
D. Complemented
7. According to the passage, how did chemical dyes compare to natural dyes?
A. The chemical dyes had less attractive colors.
B. The chemical dyes were less easy to use.
C. The chemical dyes lost their brightness more quickly when exposed to light.
D. The chemical dyes held up belter after washing.
8. According to the passage, what problem led to the development of new dyes after 1900?
A. Previously developed dyes did not work on new types of fibers.
B. Dyes derived from petroleum caused damage to new synthetic fibers.
C. New synthetic fibers required brighter colors tijan natural fibers did.
D. New fabrics easily lost their colors when washed.
114

[] Version 3.0

Exercise 391
Printmaking is the generic term for a number of processes, of which woodcut and engraving
are two prime examples. Prints are made by pressing a sheet of paper (or other material)
against an image-bearing surface to which ink has been applied. When the paper is removed,
the image adheres to it, but in reverse.
The woodcut had been used in China from the fifth century A.D. for applying patterns to
textiles. The process was not introduced into Europe until the fourteenth century, first for
textile decoration and then for printing on paper. Woodcuts are created by a relief process;
first, the artist takes a block of wood, which has been sawed parallel to the grain, covers it
with a white ground, and then draws the image in ink. The background is carved away,
leaving the design area slightly raised. The woodblock is inked, and the ink adheres to the
raised image. It is then transferred to damp paper either by hand or with a printing press.
Engraving, which grew out of the goldsmiths art, originated in Germany and northern Italy
in the middle of the fifteenth century. It is an intaglio process (from Italian intagliare, "to
carve"). The image is incised into a highly polished metal plate, usually copper, with a
cutting instrument, or burin. The artist inks the plate and wipes it clean so that some ink
remains in the incised grooves. An impression is made on damp paper in a printing press,
with sufficient pressure being applied so that the paper picks up the ink.
Both woodcut and engraving have distinctive characteristics. Engraving lends itself to subtle
modeling and shading through the use of fine lines. Hatching and cross-hatching determine
the degree of light and shade in a print. Woodcuts tend to be more linear, with sharper
contrasts between light and dark. Printmaking is well suited to the production of multiple
images. A set of multiples is called an edition. Both methods can yield several hundred goodquality prints before the original block or plate begins to show signs of wear. Mass
production of prints in the sixteenth century made images available, at a lower cost, to a
much broader public than before.
1. The word prime in paragraph 1 is closest in meaning to
A. principal
B. complex
C. general
D. recent
2. The word incised in paragraph 3 is closest in meaning to
A. burned
B. cut
C. framed
D. baked

91

Adapted from TOEFL pBT, 2004.05, Educational Testing Services.

115

Version 3.0

[]

3. The word distinctive in paragraph 4 is closest in meaning to


A. Unique
B. Accurate
C. Irregular
D. Similar
4. According to the passage, all of the following are true about engraving EXCEPT that it
A. developed from the art of the goldsmiths
B. requires that the paper be cut with a burin
C. originated in the fifteenth century
D. involves carving into a metal plate
5. The word yield in paragraph 4 is closest in meaning to
A. Imitate
B. Produce
C. Revise
D. Contrast
6. According to the passage, what do woodcut and engraving have in common?
A. Their designs are slightly raised.
B. They achieve contrast through hatching and cross-hatching.
C. They were first used in Europe.
D. They allow multiple copies to be produced from one original.
7. According to the author, what made it possible for members of the general public to own
prints in the sixteenth century?
A. Prints could be made at low cost.
B. The quality of paper and ink had improved.
C. Many people became involved in the printmaking industry.
D. Decreased demand for prints kept prices affordable.
8. According to the passage, all of the following are true about prints EXCEPT that they
A. can be reproduced on materials other than paper
B. are created from a reversed image
C. show variations between light and dark shades
D. require a printing press
116

[] Version 3.0

Exercise 4
The Rococo art movement emerged during the 18th century in France as a reaction to the
very dark and oppressive Baroque style. While Baroque engaged very serious and heavy
themes, Rococo delighted in taking a playful approach to art. Its motifs are lighthearted and
joyful, the color palette employed soft and pale. In contrast to the religious figures and heroic
battlefields depicted by Baroque artists, Rococo artists were more interested in representing
the carefree and opulent life of aristocrats and the nobility. The main focus of Rococo art is
romance and nature, for which reason it was often criticized as being frivolous and vain. It
played to the fashion of the times, and its purpose was more to entertain than to educate or
inspire. The Roman Catholic Church looked down on the worldliness of Rococo art as an
offense to God and earnest moral principles. It considered Rococo art as artificial, obtrusive
and obviously lacking in taste.
In spite of all these negative comments, Rococo emerged as one of the leading artistic
movements of its time. Its love for shell-like or curved forms gave the style its name. Thus
the term Rococo is a combination of the French word rocaille, which means shell, and
the Italian word barocco, which designates the Baroque style. At first, it was used in a
derogatory way meaning old-fashioned. However, as soon as the 19th century, Rococo was
accepted by critics as a major period in art history. It was predominantly used in the
decorative arts of its times, such as furniture, porcelain figurines and metalwork. It then made
its way into large-scale architectural designs, as well as sculptures and paintings. Rococo
influences can also be found in the court music of the 18th century, which is often referred to
as the Galante Style.
Rococo was first introduced in the interiors design of the court of French king Louis XV. At
that time, the French nobility was tired of the rich heavy designs of the Baroque. The dark
motifs of the old era gave way to the light naturalness and the curved patterns of the new
style. The main promoter of this courtly art was interior designer Nicolas Pineau. When the
court moved from Versailles to Paris, Pineau was put in charge of decorating the royal palace
according to the new tastes of the French aristocracy. His delicate, intricate patterns soon
established themselves as a leading art trend. Rococo spread from France to Germany and
Italy as well as Austria and Bohemia. It never caught on in England, however, where it was
often ridiculed as extravagant French taste.
1. The Rococo art movement emerged during the 18th century in France as a reaction to the
very dark and oppressive Baroque style.
Which of the sentences below best expresses the essential information in the highlighted
sentence in the passage? Incorrect choices change the meaning in important ways or leave out
essential information.
A. Rococo art was more serious than Baroque art.
B. Rococo art was less serious than Baroque art.
C. Rococo was just as serious an art form as Baroque.
D. Rococo often outplayed Baroque in seriousness.
117

Version 3.0

[]

2. It played to the fashion of the times, and its purpose was more to entertain than to educate or
inspire.
Which of the sentences below best expresses the essential information in the highlighted
sentence in the passage? Incorrect choices change the meaning in important ways or leave out
essential information.
A. Rococo was a fashion style that wanted to be entertaining and not educational.
B. Rococo was a fashionable style that wanted to be entertaining and not educational.
C. Rococo was a style in fashion at a time that wanted to be entertaining and not
educational.
D. Rococo was a style that kept up with what was fashionable by being entertaining and not
educational.
3. In spite of all these negative comments, Rococo emerged as one of the leading artistic
movements of its time.
Which of the sentences below best expresses the essential information in the highlighted
sentence in the passage? Incorrect choices change the meaning in important ways or leave out
essential information.
A. Rococo disregarded all negative comments and became the most important art style of its
time.
B. Rococo disregarded all negative comments and became one of the most important art
styles of its time.
C. Rococo became one of the most important art styles of its time even though it was very
criticized by many.
D. Rococo became one of the most important art styles of its time even though it was
criticized by the Church.

4. However, as soon as the 19th century, Rococo was accepted by critics as a major period in art
history.
Which of the sentences below best expresses the essential information in the highlighted
sentence in the passage? Incorrect choices change the meaning in important ways or leave out
essential information.
A. Although critics considered Rococo an old-fashioned style at first, they soon recognized
its important place in art history.
B. Critics considered Rococo an old-fashioned style in the 19th century.
C. Although critics knew Rococo was important, they criticized its style.
D. Although critics consider Rococo an old-fashioned style, they have to recognize its
important place in art history.

118

[] Version 3.0

Exercise 592
In the United States, many social reformers in the late nineteenth century demonstrated a
concern for improved housing conditions for workers. George Pullman (1831-1897), the
wealthy industrialist who introduced luxury railway cars with beds, built his model city called
Pullman in 1880 to address housing problems caused by Chicagos industrialization.
Construction the town, Pullman hoped to produce an ideal environment that would help
attract workers of a superior type to the railway car industry and retain them. Pullman
inhabitants were expected to embody values of thrift, industry, and morality. They were
taught to develop propriety and good manners, cleanliness and neatness of appearance,
diligence, and self-improvement through education and savings. Like the brick clock tower
that dominated the town center, Pullman kept a regulatory eye on his workers.
In its first five years, this new experiment in industrial life received little criticism, except
form radical political groups. Crediting the town of Pullman with producing a new type of
dependable and ambitious worker in a rationally ordered environment, reformers, at first,
praised it as a successful model for modern industrial life. However, after 1885, with the high
gloss of the experiment dulled, it became clear that the residents of Pullman had honest
grievances about the overcharging of rent and other services.
In 1893, The Worlds Columbian Exposition, an exhibition that aimed to promote American
cultural, economical, and technological development, and in which George Pullman was a
major investor, was held in Chicago. The town of Pullman became a popular tourist stop,
attracting more than its share of curious travelers. There were 10,000 foreign visitors alone
during the exposition year. In fact, the first Baedeker Travel Guide to the United States
advised visitors to tour Pullman. Frequent trains and trolley cars connected the fairgrounds of
the exposition with the town, and on several occasions, George Pullman himself guided the
tours. Construction a fantastic environment for the benefit of tourists, he made sure that any
real tensions between his office and the working inhabitants of the town were rendered
invisible to the tourist gaze.
1. According to the passage, which of the following led to the creation of two town of
Pullman?
(A) A surplus of railway workers in Chicago
(B) Housing problems caused by industrialization
(C) George Pullmans decision to stop producing railway cars
(D) The opening of the Worlds Columbian Exposition

92

Adapted from TOEFL pBT, 2005.08, Educational Testing Services

119

Version 3.0

[]

2. According to the passage, George Pullman expected that the city of Pullman would
(A) impress social reformers
(B) satisfy radical political groups
(C) soon develop housing problems
(D) draw workers to the railway car industry
3. According to the passage, Pullman inhabitants were taught to do all of the following
EXCEPT
(A) have good manners
(B) become active in town politics
(C) value education
(D) save money
4. According to the passage, what did George Pullman do to promote tourism in the town of
Pullman?
(A) He personally showed tourists around the town.
(B) He published a travel guide to the town,
(C) He started to invest in the towns cultural development.
(D) He built a new road connecting it to the Worlds Columbian Exposition.
5. The passage suggests that George Pullman worked to hide which of the following from
tourists?
(A) His role as an investor in the Worlds Columbian Exposition
(B) His conflicts with the inhabitants of the town of Pullman
(C) His efforts to promote the town of Pullman
(D) His lack of knowledge about how the inhabitants of Pullman really lived
6. The word retain in paragraph 1 is closest in meaning to
(A) house
(B) train
(C) keep
(D) reward
7. The word grievances in paragraph 2 is closest in meaning to
(A) stories
(B) opinions
(C) findings
(D) complaints
120

[] Version 3.0

3.5 Extra Instructions-

1.

2.

3.

4.

5.

1.
2.
3. ()

121

Version 3.0

1.

[]

_____________________________________
()

2. _____________________________________


_______________________

3.

_____________________________________

_________________________
__________________

4. _______________________________________
1 It appeared that Canada was once more falling in step with the trend toward smaller
families that had occurred all through the Western world since the time of the Industrial
Revolution.

122

[] Version 3.0
2 The different uses to which societies put these materials are of interest to anthropologists
who may ask, for example, why people choose to use clay and not copper when both items
are available.

3 The methods that a community devises to perpetuate itself come into being to preserve
aspects of the cultural legacy that that community perceives as essential.

1.

1.1

Barry Goldwater, the junior senator from Arizona, received the Republican
nomination in 1964.

1 The fact that some bones are heavily weathered shows that some bodies
remained above the surface for weeks or months.
2 In its early history, the violin had a dull and rather quiet tone resulting from the
fact that the strings were thick and were attached to the body of the instrument very
loosely.
1.2

1.3

3 The thought came to him that maybe the enemy had fled the city.
2.

1.
1 an apple on the table
2 a book of Shakespeare/importance/mine
2.
an instrument capable of myriad tonal effects
3.
something new
123

Version 3.0

[]

4. ______________________________
alive; awake; afraid
1 a cat alive
2 the boy afraid of exams
5. ______________________________
a way to solve the problem
6. _______________________________
1 a dog barking at the sun
2 power given by the people
3 The older painters, most of whom were born before 1835, practiced in a mode
often self-taught and monopolized by landscape subject matter and were securely
established in and fostered by the reigning American art organization, the National
Academy of Design.
7. ______________________________
1 The stump speech, a political speech given by traveling politicians and lasting
3/2 to 2 hours, which characterized nineteenth-century political discourse, has given way
to the 30-second advertisement and the 10 second "sound bite" in broadcast news.
3
3.1

3.2

It was she, a Baltimore printer, who published the first official copies of the
Declaration, the first copies that included the names of its signers and therefore heralded
the support of all thirteen colonies.
4
4.1
___________________
4.2
___________________________

4.3
124

[] Version 3.0

___________________________

1)

1 In no way could I help you with your Japanese grammar question.


2 Not until I got home did I realize that my shoes were untied.
2)

3 Only after he had spoken out the word did he realize he had made a big mistake.
3)

4 So rarely does a comet appear visible to the naked eye that when one does, it is
considered a major event.
4)
5 Accustomed though we are to speaking of the films made before 1927 as
silent, the film has never been, in the full sense of the word, silent.

1 Equally important is the fact that the execution of multiple-step tasks is accomplished in
a series-parallel sequence.

2 Surrounding the column are three sepals and three petals, sometimes easily recognizable
as such, often distorted into gorgeous, weird, but always functional shapes.

3 Herein lay the beginning of what ultimately turned from ignorance to denial of the value
of nutritional in medicine.

4 No less impressive than the invention of the laser was the development of the wheel.
5 Basic to any understanding of Canada in the 20 years after the Second World War is the
countrys impressive population growth.
6 To the coal mine came a company of soldiers with orders from the headquarters to
rescue the trapped miners.
125

Version 3.0

[]

Paragraph Analysis
4.1 Topical Structure
4.1.1 Structure of Paragraph
The reading section of the TOEFL iBT measure test takers ability to understand
university-level academic texts.
_____________________________________________
Types of TOEFL Reading Passages
1. Exposition: Material that provides an explanation of a topic
2. Argumentation: Material that presents a point of view about a topic and provides
evidence to support it
3. Historical narrative: An account of a past event or of a persons life, narrated or written
by someone else
Structure of Academic Texts
Introduction
Introductory statement (something catchy or provocative)
Topic or Main Idea
Topic Sentences 1, 2, 3......(optional)
Body
paragraph 1 (used to support the essay)
Topic Sentence 1
Supporting ideas, evidences, analysis, examples......
paragraph 2 (used to support the essay)
Topic Sentence 2
Supporting ideas, evidences, analysis, examples......
paragraph 3 (used to support the essay)
Topic Sentence 3
Supporting ideas, evidences, analysis, examples......
Conclusion(wrapping things up)
Re-summarize the Topic or Main ideas
Moves from specific to general

126

[] Version 3.0
Further ReadingIdentifying Topics, Main Ideas, and Supporting Details
Understanding the topic, the gist, or the larger conceptual framework of a textbook chapter, an article,
a paragraph, a sentence or a passage is a sophisticated reading task. Being able to draw conclusions,
evaluate, and critically interpret articles or chapters is important for overall comprehension in college
reading. Textbook chapters, articles, paragraphs, sentences, or passages all have topics and main ideas.
The topic is the broad, general theme or message. It is what some call the subject. The main idea is
the key concept being expressed. Details, major and minor, support the main idea by telling how,
what, when, where, why, how much, or how many. Locating the topic, main idea, and supporting
details helps you understand the point(s) the writer is attempting to express. Identifying the
relationship between these will increase your comprehension.

Grasping the Main Idea


A paragraph is a group of sentences related to a particular topic, or central theme. Every paragraph
has a key concept or main idea. The main idea is the most important piece of information the author
wants you to know about the concept of that paragraph.
When authors write they have an idea in mind that they are trying to get across. This is especially true
as authors compose paragraphs. An author organizes each paragraphs main idea and supporting
details in support of the topic or theme, and each paragraph supports the paragraph preceding it.
A writer will state his/her main idea explicitly somewhere in the paragraph. That main idea may be
stated at the beginning of the paragraph, in the middle, or at the end. The sentence in which the main
idea is stated is the topic sentence of that paragraph.
The topic sentence announces the general theme (or portion of the theme) to be dealt with in the
paragraph. Although the topic sentence may appear anywhere in the paragraph, it is usually first - and
for a very good reason. This sentence provides the focus for the writer while writing and for the reader
while reading. When you find the topic sentence, be sure to underline it so that it will stand out not
only now, but also later when you review.

Identifying the Topic


The first thing you must be able to do to get at the main idea of a paragraph is to identify the topic the subject of the paragraph. Think of the paragraph as a wheel with the topic being the hub - the
central core around which the whole wheel (or paragraph) spins. Your strategy for topic identification
is simply to ask yourself the question, What is this about? Keep asking yourself that question as you
read a paragraph, until the answer to your question becomes clear. Sometimes you can spot the topic
by looking for a word or two that repeat. Usually you can state the topic in a few words.
Let us try this topic-finding strategy. Reread the first paragraph on this page - the first paragraph
under the heading Grasping the Main Idea. Ask yourself the question, What is this paragraph
about? To answer, say to yourself in your mind, The author keeps talking about paragraphs and the
way they are designed. This must be the topic - paragraph organization. Reread the second paragraph
of the same section. Ask yourself What is this paragraph about? Did you say to yourself, This
paragraph is about different ways to organize a paragraph? That is the topic. Next, reread the third
paragraph and see if you can find the topic of the paragraph. How? Write the topic in the margin next
to this paragraph. Remember, getting the main idea of a paragraph is crucial to reading.
The bulk of an expository paragraph is made up of supporting sentences (major and minor details),
which help to explain or prove the main idea. These sentences present facts, reasons, examples,
definitions, comparison, contrasts, and other pertinent details. They are most important because they
sell the main idea. The last sentence of a paragraph is likely to be a concluding sentence. It is used to
sum up a discussion, to emphasize a point, or to restate all or part of the topic sentence so as to bring
the paragraph to a close. The last sentence may also be a transitional sentence leading to the next
paragraph.
127

Version 3.0

[]

4.1.2 Topic
Definition
It is the person, place, event, or idea repeatedly mentioned or referred to throughout the
reading.
Key Features
The topic never pops up and then disappears. Its a constant presence in the passage.
Although one word can sometimes sum up the topic, youll often need several words to
express it.
Identifying Topics
Early in the 19th century, the swelling numbers of urban poor had given rise to a new kind
of politician, the boss, who listened to his urban constituents and lobbied to improve their
lot. The boss presided over the citys machinean unofficial political organization
designed to keep a particular party or faction in office. Whether officially serving as mayor
or not, the boss, assisted by local ward of precinct captains, wielded enormous influence in
city government. Often a former saloon-keeper or labor leader, the boss knew his
constituents well. 93
Q. Which is the TOPIC of the paragraph above? The Boss or the Party Machine?
A.____________________________________________________________________
How to find out the Topic
Your strategy for topic identification is simply to ask yourself the question, What is this
about? Keep asking yourself that question as you read a paragraph, until the answer to
your question becomes clear. Sometimes you can spot the topic by looking for a word or
two that repeat. Usually you can state the topic in a few words.
A precise topic is general enough to sum up most of whats discussed in the passage but
narrow enough to exclude what is not discussed. In general, references to the topic tend
to increase as the passage or reading unfolds.
Early in the 19th century, the swelling numbers of urban poor had given rise to a new kind
of politician, the boss, who listened to his urban constituents and lobbied to improve their
lot. The boss presided over the citys machinean unofficial political organization
designed to keep a particular party or faction in office. Whether officially serving as mayor
or not, the boss, assisted by local ward of precinct captains, wielded enormous influence in
city government. Often a former saloon-keeper or labor leader, the boss knew his
constituents well.
93

Adapted from Boyer et al., The Enduring Vision, p. 578

128

[] Version 3.0
Vocabulary Enlargement
swelling
lobby
faction
precinct
wield

[swel]
[lb]
[fkn]
[priskt]
[wild]

Sentence Pattern
Preside over
It is uncertain whether this president will ever again preside over such a ceremony.

The real motive is that these leaders didnt want to preside over the first EU default

Give rise to
Otherwise, that may give rise to regional protectionism or even trade conflicts or wars.

Further Reading: U.S. in the 19th century


In the United States, many social reformers in the late nineteenth century demonstrated a concern for
improved housing conditions for workers. George Pullman (1831-1897), the wealthy industrialist who
introduced luxury railway cars with beds, built his model city called Pullman in 1880 to address
housing problems caused by Chicagos industrialization. Construction the town, Pullman hoped to
produce an ideal environment that would help attract workers of a superior type to the railway car
industry and retain them. Pullman inhabitants were expected to embody values of thrift, industry, and
morality. They were taught to develop propriety and good manners, cleanliness and neatness of
appearance, diligence, and self-improvement through education and savings. Like the brick clock
tower that dominated the town center, Pullman kept a regulatory eye on his workers.
In its first five years, this new experiment in industrial life received little criticism, except form
radical political groups. Crediting the town of Pullman with producing a new type of dependable and
ambitious worker in a rationally ordered environment, reformers, at first, praised it as a successful
model for modern industrial life. However, after 1885, with the high gloss of the experiment dulled, it
became clear that the residents of Pullman had honest grievances about the overcharging of rent and
other services.
In 1893, The Worlds Columbian Exposition, an exhibition that aimed to promote American cultural,
economical, and technological development, and in which George Pullman was a major investor, was
held in Chicago. The town of Pullman became a popular tourist stop, attracting more than its share of
curious travelers. There were 10,000 foreign visitors alone during the exposition year. In fact, the first
Baedeker Travel Guide to the United States advised visitors to tour Pullman. Frequent trains and
trolley cars connected the fairgrounds of the exposition with the town, and on several occasions,
George Pullman himself guided the tours. Construction a fantastic environment for the benefit of
tourists, he made sure that any real tensions between his office and the working inhabitants of the
town were rendered invisible to the tourist gaze.
129

Version 3.0

[]

Finding out the topic of the following paragraph by selecting, filling, or


summarizing.
Exercise 1 94
Toward the end of the elementary school years, girls tend to become significantly taller than
boys of the same age. The difference results partly from girls earlier puberty and partly from
the timing of the growth spurt associated with puberty. Inevitably, these changes in height at
puberty create temporary embarrassment for some boys and girls. Their embarrassment stems
from the fact that late childhood is when most children become aware of social expectations
about attractiveness, for instance, the idea that men should be taller than women. This
awareness of social expectation about height dawns at precisely the time when many children
of both sexes exhibit a very different pattern, with some girls being taller than boys.
Q. Which is the TOPIC of the paragraph above?
A. height changes in girls at puberty
B. height changes during puberty
C. puberty
D. stereotypes about height
Exercise 2
Is there a cause and effect relationship between violence on television and the problem of
violence among todays young people. No says the entertainment industry, yes says almost
all of the research. The American Medical Association, The American Psychological
Association, The American Academy of Pediatrics, and the National Institute of Mental
Health have all funded studies that linked violence on the screen with violence in real life.
According to researcher L.Rowell Huesmann from the University of Michigan, letting
children watch lots of television is like smoking. Both increase the potential for disaster. As
Huesmann puts it, Just as every cigarette increases the chance that someday you will get
lung cancer, every exposure to violence increases the chances that someday a child will
behave more violently than he or she otherwise would. According to Dale Kunkel, a
professor of communications at the University of California, more than 1000 studies have
been done on the effect of media violence and all of them concluded that television violence
contributes to real-world violence. 95
Q. The topic of this paragraph is
The effect of televised violence__________________________________________________

94

Adapted from Seifert and Hoffnung, Child and Adolescent Development, p. 336
Quotations from Laurie Mifflin, Many Researchers Say Link is Already Clear, The New York Times, May 9,
1999, p. 23.

95

130

[] Version 3.0
Vocabulary Enlargement
puberty
spurt
Pediatrics

[pjubt]
[spt]
[,piditrks]

Further ReadingThe Role of Television in Politics 96


Television has transformed politics in
the United States by changing the way
in which information is disseminated,
by altering political campaigns, and by
changing citizens patterns of response
to politics. By giving citizens
independent access to the candidates,
television diminished the role of the
political party in the selection of the
major party candidates. By centering
politics on the person of the candidate,
television accelerated the citizens
focus on character rather than issues.
Television has altered the forms of political communication as well. The messages on which most of
us rely are briefer than they once were. The stump speech, a political speech given by traveling
politicians and lasting 3/2 to 2 hours, which characterized nineteenth-century political discourse, has
given way to the 30-second advertisement and the 10 second "sound bite" in broadcast news.
Increasingly the audience for speeches is not that standing in front of the politician but rather the
viewing audience who will hear and see a snippet of the speech on the news.
In these abbreviated forms, much of what constituted the traditional political discourse of earlier ages
has been lost. In 15 or 30 seconds, a speaker cannot establish the historical context that shaped the
issue in question, cannot detail the probable causes of the problem, and cannot examine alternative
proposals to argue that one is preferable to others. In snippets, politicians assert but do not argue.
Because television is an intimate medium, speaking through it require a changed political style that
was more conversational, personal, and visual than that of the old-style stump speech. Reliance on
television means that increasingly our political world contains memorable pictures rather than
memorable words. Schools teach us to analyze words and print. However, in a word in which politics
is increasingly visual, informed citizenship requires a new set of skills.
Recognizing the power of televisions pictures, politicians craft televisual, staged events, called
pseudo-event, designed to attract media coverage. Much of the political activity we see on television
news has been crafted by politicians, their speechwriters, and their public relations advisers for
televised consumption. Sound bites in news and answers to questions in debates increasingly sound
like advertisements.

96

Adapted from TOEFL pBT, 1999.05, Educational Testing Services.

131

Version 3.0

[]

Exercise 3 97
Every year troops of researchers journey to the worlds rain forests. Although they spend
some of their time on the forest floor, they spend more of it rummaging through the canopy,
or the tops of rain forest trees. For it is the forests canopy that contains its greatest riches.
This is where researchers can find the berries, fruits, flowers, and seeds that they have come
to discover. Some of the rain forest researchers are funded by perfume companies. They
come in pursuit of flowers with scents that might become the basis for a new perfume. Others
have been flown to the rain forest by food companies desperate for a new taste sensation that
will help them corner the worlds markets. Medical researchers come to hunt for plants that
might help cure diseases. As Kelsey Downum, a biologist at Florida International University,
says about the rain forest: About 60 percent of all our medicines originally come from plants;
there is much more to be found. Aware of the riches that can be found in the rain forests
canopy, the researchers know something else as well. The forest is disappearing as the trees
are cut down for wood and the jungle cleared for farming.
Q. Summarize the topic for the paragraph above
A._________________________________________________________________________
Exercise 4
Although Benedict Arnold, an American Major General during the Revolutionary War,
served his country heroically in several early battles, he is now known as a villain whose
name is a synonym for "traitor." Why did an outstanding officer who fought bravely for
independence turn his back on America in 1780 and plot to sell the fort at West Point to the
British? Several factors led Arnold to commit treason. First of all, the ambitious Arnold
became bitter about several setbacks to his career. He was passed over for important
promotions, and he alienated several of his superiors with his jealous rivalry. He had a huge
ego and craved recognition and public admiration, and he believed the Continental Congress
had betrayed him by denying him the official honors he had earned. Arnold was also devoted
to his own financial self-interest. He tried to increase his income through legal and illegal
means, including inappropriate use of his position to engage in trade. Therefore, when the
British promised him $20,000 along with rewards of rank and honor in exchange for West
Point, which was under his command, he managed to convince himself that America would
be better off under English rule and switched his loyalty in exchange for his own personal
gain.
Q. Which is the TOPIC of the paragraph above?
A. Benedict Arnolds service in the Revolutionary Army
B. Benedict Arnolds greed
C. Reasons behind Benedict Arnolds treason
97

Quotation from Marlise Simons, Eau de Rain Forest, The New York Times, May 2, 1999, p. 61

132

[] Version 3.0
Vocabulary Enlargement
rummage
traitor
setback
rivalry

[rmd
[tret]
[setbk]
[ravlri]

Sentence Pattern
In pursuit of
Many are stubborn in pursuit of the path they have chosen, few in pursuit of the goal.

Practical Translation Training



_______________________________________________________________________
Further Reading: Benedict Arnold
Benedict Arnold was a general during the American Revolutionary War who originally fought for
the American Continental Army but defected to the British Army. While a general on the American
side, he obtained command of the fort at West Point, New York, and planned to surrender it to the
British forces. After the plan was exposed in September 1780, he was commissioned into the British
Army as a brigadier general.
Born in Connecticut, Arnold was a merchant operating ships on the Atlantic Ocean when the war
broke out in 1775. After joining the growing army outside Boston, he distinguished himself through
acts of intelligence and bravery. His actions included the Capture of Fort Ticonderoga in 1775,
defensive and delaying tactics despite losing the Battle of Valcour Island on Lake Champlain in 1776,
and key actions during the pivotal Battles of Saratoga in 1777, in which he suffered leg injuries that
ended his combat career for several years.
Despite Arnolds successes, he was passed over for promotion by the Continental Congress while
other officers claimed credit for some of his accomplishments. Adversaries in military and political
circles brought charges of corruption or other malfeasance, but most often he was acquitted in formal
inquiries. Congress investigated his accounts and found he was indebted to Congress after spending
much of his own money on the war effort. Frustrated and bitter, Arnold decided to change sides in
1779, and opened secret negotiations with the British. In July 1780, he was offered, continued to
pursue and was awarded command of West Point. Arnolds scheme to surrender the fort to the British
was exposed when American forces captured British Major John Andr carrying papers that revealed
the plot. Upon learning of Andrs capture, Arnold fled down the Hudson River to the British sloopof-war Vulture, narrowly avoiding capture by the forces of George Washington. Arnold received a
commission as a brigadier general in the British Army, an annual pension of 360, and a lump sum of
over 6,000. He led British forces on raids in Virginia, and nearly captured Thomas Jefferson, and
against New London and Groton, Connecticut. Because of the way he changed sides, his name
quickly became a byword in the United States for treason or betrayal. His conflicting legacy is
recalled in the ambiguous nature of some of the memorials that have been placed in his honor.
133

Version 3.0

[]

Exercise 5
Polygamy, a marital state involving one man with several, simultaneous wives, has a long
history in America. In the early 1840s, Mormon founder Joseph Smith encouraged the
practice of polygamy because he believed that conceiving as many children as possible
allowed one to achieve the highest levels of heaven. Many of his Mormon followers settled in
the remote territory of Utah during the 1800s. Utah formally banned polygamy in order to
gain entry into the United States in 1896. However, outlawing the practice did not end it;
polygamy continued underground. Although Utah jailed polygamists until the 1950s,
tolerance for the practice grew, and polygamists were left alone during the second half of the
20th century. Today, an estimated 20,000 to 100,000 people live in polygamous families, and
conversions and a high birth rate continue to increase those numbers. Polygamys defenders
argue not only that the practice is part of Utahs history, but also that it represents just another
lifestyle choice no different from other non-traditional households like same-sex marriages or
single-parent homes
Q. Which is the TOPIC of the paragraph above?
A. Polygamy, past and present
B. Joseph Smith, the founder of Mormonism
C. The presence of polygamy in Utah
Exercise 6
Large numbers of Americans believe that some people possess psychic ability. As a result,
they call the Psychic Friends Network. They have their palms and Tarot cards read for
information about the future. They visit mediums who profess the ability to talk to deceased
loved ones. And they usually believe what the psychics tell them because they want the
information to be true. However, they may not be aware of the techniques psychics use to get
their "insights." Most psychics have perfected the ability to read subtle signals, or clues, that
people give them. They typically start a session by stating a few facts while carefully
observing the clients reactions, such as eye movements or changes in facial expression. Then,
they probe for information by asking the client questions, and they veer quickly from any
errors they make. For instance, if a psychic incorrectly guesses that the client is married, he
or she would immediately shift focus to someone the client knows who is married. Psychics
gather every bit of information they can from the clients responses and tend to speak in a
rambling style, throwing out impressions and watching the client for signs that theyve hit the
mark. They also count on peoples tendencies to forget the inaccuracies of a session and to
focus on whatever the psychic got right.
Q. Which is the TOPIC of the paragraph above?
A. The Psychic Friends Network
B. Tricks used by psychics
C. The increase in psychic phenomena
134

[] Version 3.0
Vocabulary Enlargement
polygamy
jail
psychic
veer
rambling

[plmi]
[del]
[sakk]
[v]
[rmbl]

Sentence Pattern
Hit the mark
Great post, you really hit the mark, I just dont understand why people quite get it.

Practical Translation Training



_______________________________________________________________________
Further Reading: Types of Psychic abilities
Animal Telepathy: The ability to communicate with (but not command or influence)
various kinds of creatures. Think "pet psychic".
Astral Projection: The ability to leave ones body and travel in spirit to another location.
Automatic Writing: Writing through the subconscious mind without conscious thought, or
through the guidance of an outside intelligence.
Channeling: Associated with mediums, this is the ability to act as a channel or vessel for
an outside intelligence.
Clairaudience: Put simply, this type of ability is used to hear what is "inaudible". For
example, someone with this ability could be a thousand miles way and "hear" a loved ones
cry of distress.
Clairvoyance: Usually confused with Precognition, this ability actually has much more in
common with "Remote Viewing", True clairvoyance is not the ability to see into the future,
but the psychic ability to see visions of that which is hidden or far away.
Clairsentience: In this instance the psychic has an insight or "knowing" of and a hidden or
forgotten fact.
Divination: A broad term that includes fortune telling, precognition, prophesy, and other
methods used in an effort to predict the future.
Dowsing: Also known as "water witching", dowsing involves the use of a rod, sticks, or
pendulum to locate water or lost objects.
Empathy: The talent to sense the needs, drives, and emotions of another. As with Aura
Reading, psychic ability can often reveal itself through the development of empathy.
Levitation - The ability to cause ones body to hover off the ground. One of the more welldocumented cases of levitation involved St. Theresa of Avila during the 16th century.
135

Version 3.0

[]

4.1.3 Main Idea


Definition
The main idea of a passage or reading is the central thought or message. In contrast to the
term topic, which refers to the subject under discussion, the term main idea refers to the point
or thought being expressed about the TOPIC.
Difference between TOPIC and MAIN IDEA

Key Features
As soon as you can define the topic, ask yourself What general point does the author
want to make about this topic? Once you can answer that question, you have more than
likely found the main idea.
Most main ideas are stated or suggested early on in a reading; pay special attention to the
first third of any passage, article, or chapter. Thats where you are likely to get the best
statement or clearest expression of the main idea.
Pay attention to any idea that is repeated in different ways. If an author returns to the
same thought in several different sentences or paragraphs, that idea is the main or central
thought under discussion.
Once you feel sure you have found the main idea, test it. Ask yourself if the examples,
reasons, statistics, studies, and facts included in the reading lend themselves as evidence
or explanation in support of the main idea you have in mind. If they do, your
comprehension is right on target. If they dont, you might want to revise your first notion
about the authors main idea.
Example
Both buyers and sellers benefit from branding. Because brands are easily recognizable, they
reduce the amount of time buyers must spend shopping; buyers can quickly identify the
brands they prefer. Choosing particular brands such as Tommy Hilfiger, Polo, Nautica, and
Nike can be a way of expressing oneself. When buyers are unable to evaluate a products
characteristics, brands can help them judge the quality of the product. For example, most
buyers arent able to judge the quality of stereo components but may be guided by a wellrespected brand name. Brands can symbolize a certain quality level to a customer. Brands
thus help reduce a buyers perceived risk of purchase. Finally, customers may receive a
psychological reward that comes from owning a brand that symbolizes status. The lexus
brand is an example. 98
TOPIC: _________________________________________________________________
Main Idea: ______________________________________________________________
98

Adapted from Pride, Hughes and Kapoor, Business, p.394.

136

[] Version 3.0
Further ReadingThe importance of Branding in Business 99
So what is branding, exactly? Branding is successfully creating a brand identity that will not only
make your business more appealing in comparison to your competitors, but it will also convince
consumers that in a sea of prospects, your business is the only one capable of satisfying their needs.
Many different components can go into developing a brand, including eye-catching designs and a
unique name; however, a brand encompasses more than just a logo. Its what clients take away from
the experience of working with you. Its what your company stands for and is known for within your
market. Coming up with a brand means coming up with what your company promises to deliver,
whether thats perfectly seasoned gourmet pizzas or impeccably tailored shirts. Branding includes the
overall style of your company and the meaning it has to clients. Ideally, it would motivate people to
buy your products or use your services.
If you dont define your brand, you create the possibility that someone else will do it for you,
including your competitors. Defining you brand allows you to controlor at least influencehow
others perceive you. An effective brand marketing strategy minimizes your companys chances of
fading into the background and solidifies your reputation as a veritable force to be reckoned with.
Strong branding demonstrates that a company aspires to something greater than just business as usual.
By raising your status from a basic commodity to a brand, consumers will be willing to pay a
premium for your services or products.
Ultimately, branding is perhaps most essential because it increases the chances that people will
remember you, even as the battle for consumers attention (and money) rages on. A fully-developed
brand identity will push your business to the forefront of consumers minds, making them more likely
to recognize you, enlist your services, and remain loyal long after the conclusion of your professional
relationship. It takes a lot of time to build (and maintain) these valuable customer relationships. Let
me give you a few examples of successful branding:
Coca Cola: The majority of people instantly recognize this brand. Their logo, the Christmas songs,
TV commercials, ads and cans are a part of the Coca Cola branding strategy. This brand has been
around for decades, allowing its message to go from generation to generation, and all of Coca Colas
efforts have paid off. Wouldnt you agree it is quite hard to imagine a world without Coca Cola?
McDonalds: Remember when you were a kid and going to McDonalds was a treat? Of course you
do! This is why you will probably take your (future) children to come and have a quick bite here
every once in a while. McDonalds introduced Happy Meals for children, including a small toy in
every meal box. A smart move, given the fact that competitors soon offered similar deals.
Furthermore, they incorporated the word Mac/Mc into their product names, like McNuggets or
the famous Big Mac. These decisions have had major influence on the way customers perceive the
brand.
Apple: Apple has found a way to become one of the most popular brands in the world by branding
themselves successfully. Mac computers arent for just anyone, nor are the iPad or the iPhone. The
late Steve Jobs was known for his way of presenting new products to the public and used his black
turtleneck sweater as part of his strategy. Even the use of putting an i in their product names is all
part of Apples branding strategy. The designs of the products, the campaigns and product launches
are all in sync with Apples style and its branding.

99

Adapted from EIC social media team

137

Version 3.0

[]

Finding out the main idea of the following paragraph


Exercise 1
Financial genius James Big Jim Fisk (1834-1872) died of gunshot wound when he was
only thirty-seven years old. During his brief lifetime, Fisk earned and lost huge sums of
money, much of it through bribery and theft. During the Civil War, he smuggled cotton from
the South to the North. He also printed and sold phony bonds to gain control of the wildly
profitable Erie Railroad. Then he bankrupted the railroad while gaining a personal fortune for
himself. In 1869, Fisks attempts to take over the gold market led to financial panic and the
collapse of the stock market. Oddly enough, Fisk seemed rather proud of his wicked ways,
saying Some people are born to be good; other people to be bad. I was born to be bad. A
lover of the ladies, Fisk was killed in a fight with a rival over the affections of actress Josie
Mansfield. 100
Main Idea:
A. Big Jim Fisk liked pretty women a little too much for his own good.
B. In his pursuit of wealth, James Fisk never let law or morality stand in his way.
C. James Fisk did not have a long life, but that did not stop him from making a great deal of
money.
D. James Fisk was born to be bad.
Exercise 2
For the ancient Romans, taking a bath was a very special occasion. Because they considered
bathing a social opportunity, they constructed huge public baths that put our modern-day
indoor pools and spas to shame. Not only were the baths themselves lavishly decorated, they
were also surrounded by shops, libraries, and lounges so that a person could shop, read or
chat after bathing. The famed Baths of Caracalla, for example, offered Roman citizens
massages and saunas in addition to a gymnasium and gardens for after-bath walks in lovely
surroundings. Art lovers that they were, the Romans also frequently built art galleries into
their bathing facilities. There were also kitchens, where food was prepared to serve hungry
bathers. Although initially men and women bathed separately, mixed baths became the
fashion until 500 A.D., when the coming of Christianity brought the public baths to an end. 101
Main Idea:
A. The ancient Romans were the first to lead a life of pure luxury.
B. If the Romans had spent more time governing and less time bathing, the Roman Empire
would still exist today.
C. The ancient Romans made luxury and socializing a part of bathing.
D. Mixed baths were fashionable in the ancient Romans.

100
101

Adapted from Armento et al., A More Perfect Union, p. 472


Adapted from Charles Panati, Extraordinary Origins of Everyday Things, p. 200

138

[] Version 3.0
Vocabulary Enlargement
bribery
smuggle
lounge
massage
sauna

[brabri]
[smgl]
[land]
[ms]
[sn]

Sentence Pattern
Put to shame
They have all been put to shame by a boy who, while playing truant.

Practical Translation Training



_______________________________________________________________________
Further Reading: Role of social bathing in Classical Rome
Public baths fulfilled many of the needs and desires of
Roman citizens in classical times and were the center of
Roman social life. These extravagant structures were
places where one could relax, enjoy art, discuss politics,
and converse with others about the issues of the day.
These great "palaces of entertainment served all tastes
and admitted almost anyone," both rich and poor.
In early Roman history, bathing was done every nine
days and was not seen as a priority. During the 2nd
century BCE, however, the Greek custom of regular
bathing reached Rome. It gradually became a daily practice for Roman citizens, regardless of their
financial standing, to frequent public baths. Small bathhouses called balneae opened up throughout
the empire. The balneae were privately owned baths, accessible to the public for a small fee. Later
came the majestic and renown thermae. These large bathhouses were owned by the state and
sometimes covered several city blocks. Funding for the building and upkeep of the public baths was
expected of the wealthy
Men and women bathing together was discouraged. Thus, women either had separate bathing facilities,
or, more commonly, had different bathing hours. However, public bathing for women was viewed as
rather middle class, since the wealthy, aristocratic women owned private steam baths and were waited
upon by their own personal slaves. Even though women had the worst bathing hours (morning till
noon), they still had to pay twice the amount as males to enter the public baths.
The exact hours of the baths opening in the morning and closing at night are not clearly known and
are much debated upon. However, it is known that the opening was announced by the ringing of the
bath bell. Though, patrons were allowed to enter the exercise grounds before this bell was
sounded.The bathing process was lengthy, usually lasting several hours. After patrons were done
exercising and bathing, they departed for home to dine, planning to return the next day.
139

Version 3.0

[]

Exercise 3
Birds have long played a central role in superstitions. However, the role birds have played
varies greatly. While crows were thought to be in league with the devil, blue birds were
usually considered signs of good fortune. Blue jays, in contrast, were seldom assigned a
positive role in superstition and legend. Most of the time, they were considered companions
to the devil. According to one ancient superstition, blue jays were never seen on Fridays.
Friday was their day to meet with the devil and pass on any useful gossip about souls who
might be ready to go astray. Owls, in contrast, have played a number of different roles, some
good, some bad. In several superstitions they are portrayed as wise counselors; in others they
are a sign that death is near.
Main Idea:
A. Despite having the same color, blue jays and blue birds have played very different roles
in superstitions
B. Birds turn up frequently in superstitions as signs of both good and evil.
C. Hardly a superstition exists that doesnt have a bird in it.
D. Owls were considered playing different roles.

Exercise 4
French and American cookbooks obviously share the same subject: They both deal with the
preparation of food. But thats where the similarity ends. American recipes are very exact,
while French recipes are open to personal variation. In American cookbooks, the teaspoon of
sugar, for instance, is described as rounded or flat. French cookbooks are inclined to be less
precise. The French cookbook writer thinks nothing of listing a "sprinkling of pepper" or a
"pinch of salt." French cookbooks are also likely to tell the reader to "season according to
taste." American cookbooks, in contrast, dont seem to have as much faith in their readers
ability to get the recipe right from taste. They tell readers exactly how much seasoning to use.

Main Idea:
A. Despite a common topic, French and American cookbooks differ in the way they give
directions.
B. Because they are better cooks, the French write cookbooks that trust their readers
judgment
C. American cookbooks dont have much faith in the cooks ability to determine the right
taste.
D. French cookbooks and recipes are open to personal variation.

140

[] Version 3.0
Vocabulary Enlargement
superstition
counselor
sprinkling

[,supstn]
[kansl]
[sprkl]

Sentence Pattern
Inclined to
We sympathize with the poor, so we are naturally inclined to help.

Practical Translation Training



_______________________________________________________________________
Further ReadingNest-building of Birds 102
The origins of nest-building remain obscure, but current observation of nest-building
activities provide evidence of their evolution. Clues to this evolutionary process can be found
in the activities of play and in the behavior and movements of birds during mating, such as
incessant pulling at strips of vegetation or scraping of the soil. During the early days of the
reproductive cycle, the birds seem only to play with the building materials. In preparation for
mating, they engage in activities that resemble nest-building, and continue these activities
throughout and even after the mating cycle. Effective attempts at construction occur only
after mating.
Although nest-building is an instinctive ability, there is considerable adaptability in both site
selection and use of materials, especially with those species which build quite elaborate
constructions. Furthermore, some element of learning is often evident since younger birds do
not build as well as their practiced elders. Young ravens, for example, first attempt to build
with sticks of quite unsuitable size, while a jackdaws first nest includes virtually any
movable object. The novelist John Steinbeck recorded the contents of a young osprey nest
built in his garden, which included three shirts, a bath towel, and one arrow.
Birds also display remarkable behavior in collecting building materials. Crows have been
seen to tear off stout green twigs, and sparrowhawks will dive purposefully onto a branch
until it snaps and then hang upside down to break it off. Golden eagles, over generations of
work, construct enormous nests. One of these, examined after it had been dislodged by high
winds, weighed almost two tons and included foundation branches almost two meters long.
The carrying capacity of the eagles, however, is only relative to their size and most birds are
able to carry an extra load of just over twenty percent of their body weight.

102

Adapted from TOEFL pBT, 2003.01, Educational Testing Services.

141

Version 3.0

[]

4.1.4 Topic Sentence


Definition
A sentence that states the main idea of a paragraph or of a larger unit of discourse.
Key Features
A topic sentence introduces words and terms that are repeated, referred to, or further
explained and illustrated by other sentences in the paragraph.
The idea expressed in a topic sentence is threaded throughout the paragraph; it doesnt
appear just once and then disappear.
A topic sentence generally sums up a paragraphs overall point or message.

Example:
The 1920s began on a note of economic optimism. However, by the end of the decade,
America was sinking into an economic depression that left the country reeling. Automobile,
sales, the heart of the early twenties consumer boom, were bottoming out. Housing starts fell,
along with manufacturing output. In the fall of 1929, the stock market crashed with investors
losing as much money as the government had spent during all of World War I. Those without
investments on Wall Street were facing even grimmer prospects. Jobs were disappearing as
factories and businesses closed their doors. For many working people, it wasnt clear if they
would have a roof over their head or enough food to feed themselves and their families.
Topic Sentence:
However, by the end of the decade, America was sinking into an economic depression that
left the country reeling
Supporting Details:
Decreasing car sales
Failing investment
Job losses
142

[] Version 3.0
Vocabulary Enlargement
optimism
reel
Grim

[ptmzm]
[ril]
[rm]

Sentence Pattern
Sink into
Japan will not actually sink into recession, despite GDP growth all but disappearing.

Practical Translation Training



_______________________________________________________________________
Further Reading: Water projects in the depressions
Water projects in the United States gained a new
rationale in the 1930s as the nation suffered its worst
economic depression and the Great Plains region
suffered its worst drought in recorded history. As the
economy sank into a deep depression and
unemployment rates increased, the political climate for
direct federal government involvement in water projects
improved. President Franklin Roosevehs first 100 days
in office brought a number of new laws to deal with the
severe economic depression that became known as the Great Depression. Two of these laws, the
Tennessee Valley Authority Act of 1933 and the National Recovery Act of 1933 (NIRA), had
particular significance for water resource development. The natural pattern of the Tennessee River
was characterized by large spring flows that produced destructive floods and low summer flows that
inhibited navigation. The intensily and frequency of the events discouraged development and
contributed to persistent poverty in the valley. To counter these natural obstacles, the Tennessee
Valley Authority Act of 1933 created the Tennessee Valley Authority (TVA), a public agency with
broad powers to promote development in the region, including the authority to build dams and
reservoirs and to generate and sell hydroelectric power. The TVA is a unique institution in that it
brings all the water-related functions of the federal government under a single body. The TVA used
its authority to transform the Tennessee River into one of the most highly regulated rivers in the world
within about two decades. The TVA inherited the Wilson Dam, and by the beginning of the Second
World War, it had completed six additional multipurpose dams with power plants and locks for
navigation. Investments in dams and hydropower facilities within the Tennessee Valley also received
high priority during the war. The NIRA authorized the creation of the Public Works Administration to
create jobs while undertaking work of benefit to the community. The NIRA also gave the United
States President unprecedented powers to initiate public works, including water projects. The Public
Works Administration provided loans and grants to state and local governments and to federal
agencies for municipal waterworks, sewage plants, irrigation, flood control, and waterpower projects.
143

Version 3.0

[]

Finding out the topic sentence of the following paragraph


Exercise 1: Alfred A. Tomatis (1920-2001) was one of the first educational researchers to be
interested in the Mozart effect. Tomatis used the phrase to describe the increase in intellectual
ability that supposedly occurs when children listen to the music of eighteenth-century composer
Wolfgang Amadeus Mozart. Although for a while now, the media has celebrated the Mozart
Effect as if it were proven fact, there is little hard evidence it exists. The idea that there really
was such a thing as a Mozart Effect originated with physicist Gordon Shaw and learning
researcher Frances Rauscher. Working with students from the University of California at Irvine,
Rauscher and Shaw played Mozart to a few dozen subjects. Then they administered intelligence
tests. The tests suggested a temporary increase in I.Q., which was attributed to the music listened
to before testing. As a result of their work, Shaw and Rauscher have become famous. They are
so well-known that they have founded their own institute, the Music Intelligence Neural
Development Institute. The media, never quick to examine the scientific evidence for
sensational claims, has made it seem as if belief in the Mozart Effect is widely shared by the
scientific community. It isnt. Thats because no one has ever been able to repeat Shaw and
Rauschers results. 103
A. Alfred A. Tomatis (1920-2001) was one of the first educational researchers to be interested
in the Mozart effect.
B. Tomatis used the phrase to describe the increase in intellectual development that supposedly
occurs when children listen to the music of eighteenth-century composer Wolfgang Mozart.
C. Although for a while now, the media has celebrated the Mozart Effect as if it were a proven
fact, it turns out that there is little hard evidence it exists.
Exercise 2: Mount Everest, the towering mountain located between the two countries of Tibet
and Nepal, is Earths highest point above sea level. The mountains challenging and aweinspiring height is one reason why so many people have been obsessed by the desire to climb it
and died trying. In 1924, British explorers George Mallory and Andrew Irvine cheerily set off to
climb Everests heights, disappeared from view, and were never seen again. Mallorys body was
discovered in 1999, but there was no indication of whether or not he had made it to the top.
Since that time, its estimated that around 2,000 climbs have been attempted with 180 people
dying as a result. The worse year for deaths was 1996, when twelve people lost their lives while
trying to summit. Most recently, in May of 2006, a New Zealander named David Sharp made it
to the top in his third summit attempt, but he died of cold and oxygen deprivation around 1,000
feet into his descent. 104
A. Mount Everest, the towering mountain located between the two countries of Tibet and
Nepal, is Earths highest point above sea level.
B. The mountains challenging and awe-inspiring height is one reason why so many people
have been obsessed by the desire to climb it and died trying.
C. In 1924, British explorers George Mallory and Andrew Irvine cheerily set off to climb
Everests height, disappeared from view, and were never seen again.
103
104

Adapted from Robert Todd Carroll. The Skeptics Dictionary, New Jersey: Wiley, 2003, p.233
Adapted from www.factmonster.com/spot/everest2.html

144

[] Version 3.0
Further Reading: Mozart Effect 105
Society has often hoped for a quick and simple way of increasing intelligence. In 1993, such a possibility
was offered by researchers at the University of California, Irvine. Rauscher, Shaw, and Ky (1993) found
that a group of 36 college undergraduates improved their spatial-temporal intelligence (the ability to
mentally manipulate objects in three-dimensional space) after listening to 10 minutes of a Mozart sonata.
Results showed that student IQ scores improved by 8-9 points and lasted for 10-15 minutes. The
findings, which were later dubbed the Mozart effect, have spawned both criticism and support for
musics ability to alter intelligence.
The original Mozart effect researchers based their rationale on the Trion model of the cerebral cortex.
The cerebral cortex is a part of the brain that helps with, among other things, motor control, speech,
memory, and auditory reception. The Trion model, developed by Shaw, showed that similar neural
firings patterns occur when listening to music and performing spatial tasks. Rauscher and Shaw
hypothesized that listening to certain types of complex music may "warm-up" neural transmitters inside
the cerebral cortex and thereby improve spatial performance.
Other researchers have been wary of the findings presented by Rauscher and colleagues. Instead, Mozart
effect critics have claimed the spatial intelligence increase to be nothing more than a shift in participants
arousal, which then produces better spatial test scores. In essence, their argument is that listening to
Mozarts music causes either an increase or decrease in someones arousal and mood to a level that is
more optimal for testing. Personal preference for the music heard in Mozart effect testing may also be a
possible influence on increasing spatial test scores.
The first Mozart effect publication showed participants spatial intelligence scores improved by 8-9
points, by far the largest increase reported in the literature. A meta-analysis of Mozart effect research
showed an increase of 1.4 general IQ points between participants listening to Mozart or silence. However,
this IQ score included studies that did not strictly measure for spatial intelligence. When analyzing
studies that only used spatial intelligence, results showed an increase of 2.1 general IQ points. The IQ
scores created by the meta-analysis may not be particularly promising since they are less than the normal
standard deviation (15 points) found in the Fourth Edition of the Stanford-Binet Intelligence Test, which
includes the prominent spatial subtest used in Mozart effect research. Additionally, the meta-analysis IQ
scores were not specifically calculated for spatial ability, but instead present an overall intelligence
increase, which is beyond the scope intended by the original Mozart effect authors.
The majority of Mozart effect research has been conducted on college undergraduates. Although some
media outlets have claimed Mozarts music improves a babys intelligence, as yet, no test involving the
possible influence of this music on infants spatial intelligence has been conducted. The youngest group
of participants that have been tested had a mean age of 11.95 years. Currently, no testing has been done
on older adults, adolescents, or other developmental levels.
The music used in the first Mozart effect experiment was Mozarts Sonata for two pianos in D-major,
K.448. This has continued to be the primary song used in Mozart effect research. Other styles of also
been attempted such as Yanni, which was believed to have similar musical properties as Mozart,
minimalist music by Philip Glass, the dance group Aqua, and pieces by Albinoni and Schubert. Some
non-musical selections have also been compared to Mozart music, such as a George Carlin comedy
performance and a Stephen King story read aloud. Of these different selections, the music of Yanni and
Mozart have been the only styles to increase spatial scores. To date, no published research on the Mozart
effect has used any other non-classical musical pieces.
105

Adapted from Martin Jones, The Mozart Effect

145

Version 3.0

[]

Exercise 3: When Kentucky Derby winner, Barbaro, shattered his leg at the start of the
Preakness Stakes in 2006, everyone in horse racing appeared stunned. Barbaro, a two-yearold racing wonder, had been expected to win. Instead he suffered irreparable injuries and had
to withdraw. Yet when one considers the damage racing inflicts on young horses, its hard to
imagine why everyone, from the owners to the fans, seemed so shocked. Horse racing is a
dangerous and deadly sport for the horses, no matter how eager and spirited the animals
might look at the starting gate. According to Susan Stover, a horse veterinarian at the
University of California at Davis, horses sustain fatal injuries in 1.5 of every 1000 starts. The
only wonder is that they do not suffer more injuries. These are young horses, whose bodies
undergo almost unimaginable stress. As Jim Orsini, who treated Barbaro pointed out, ...at
high speed, a horses leg bones can actually deform, and keep deforming until they or the
ligaments or tendons eventually fail. Orsini ought to know. He is a professional horse
surgeon. He works at Pennsylvanias New Bolton Center, where Barbaro was sent to recover.
Add to the stress of regular racing the fact that race horses are bred to be thin-bonedthe
thin bones help them break out from the starting gate more quicklyand you have a recipe
for disaster. No wonder, Elliot Katz, a former veterinarian and president of In Defense of
Animals, a San-Franciscobased animal-rights group, calls horse racing a killer sport. 106
A. When Kentucky Derby winner, Barbaro, shattered his leg at the start of the Preakness
Stakes in 2006, everyone in horse racing appeared stunned.
B. Horse racing is a dangerous and deadly sport for the horses, no matter how eager and
spirited the animals might look at the starting gate.
C. According to Susan Stover, a horse veterinarian at the University of California at Davis,
horses sustain fatal injuries in 1.5 of every 1000 starts.
Exercise 4: In 1974, Congress passed the Family Educational Rights and Privacy Act. Also
known as the Buckley amendment, the new legislation specified who was allowed to view
student records. The Buckley amendment also defined the conditions under which the records
might be seen. Not surprisingly, parents of school-age children were the biggest winners from
the passage of the amendment, which gave them access to their childrens records and
evaluations. Previously they had been kept from seeing the official judgments that so
powerfully affected their childrens lives. For example, prior to the new legislation, parents
had not been allowed to see paperwork justifying why children were held back or placed in a
special education class. After the legislation was passed, any school denying parents the right
to review their childrens records could lose federal funding.
A. In 1974, Congress passed the Family Educational Rights and Privacy Act.
B. Also known as the Buckley amendment, the new legislation specified who was allowed
to view student records.
C. Not surprisingly, parents of school-age children were the biggest winners from the
passage of the Buckley amendment, which gave them access to their childrens records
and evaluations.
106

Adapted from Alfred Lubrano. Horse Racing is Still Saddled by Cruelty Issue. Philadelphia Inquirer, May 27,
2006, p.10

146

[] Version 3.0
Vocabulary Enlargement
derby
ligament
tendon

[dbi]
[lmnt]
[tndn]

Sentence Pattern
Inflict on
Dont inflict your ideas on me.

Practical Translation Training



___________________________in the selfish pursuit of _________________________
Further Reading: Education in the United States

107

It is commonly believed that in the United States that school is where people to get an
education. Nevertheless, it has been said that today children interrupt their education to go to
school. The distinction between schooling and education implied by this remark is important.
Education is much more open-ended and all-inclusive than schooling. Education knows no
bounds. It can take place anywhere, whether in the shower or on the job, whether in a kitchen
or on a tractor. It includes both the formal leaning that takes place in school sand the whole
universe of informal leaning. The agents of education can range from a revered grandparent
to the people debating politics on the radio, from a child to a distinguished scientist. Whereas
schooling has a certain predictability, education quite often produces surprises. A chance
conversation with stranger may lead a person to discover how little is known of other
religions. People are engaged in education from infancy on. Education, then, is a very broad,
inclusive term. It is a lifelong process, a process that starts long before the start of school, and
one that should be an integral part of ones entire life.
Schooling, on the other hand, is a specific, formalized process, whose general pattern varies
little from one setting to the next. Throughout a country, children arrive at school at
approximately the same time, take assigned seats, are taught by an adult, use similar
textbooks, do homework, take exams, and so on. The slices of reality that are to be learned,
whether they are the alphabet or an understanding of the workings of governments, have
usually been limited by the boundaries of the subject being taught.
For example, high schools students know that they are not likely to find out in their classes
the truth about political problems in their communities or what the newest filmmakers are
experimenting with. There are definite conditions surrounding the formalized process of
schooling.
107

Adapted from TOEFL pBT, 1996.12, Educational Testing Services.

147

Version 3.0

[]

4.1.5 Position of Topic Sentence


At or near the beginning
At the first sentence
At the second sentence

Exercise 1 108
Other dimensions along which the two groups differ markedly are density and composition.
The densities of the terrestrial planets average about 5 times the density of water, whereas the
Jovian planets have densities that average only 1.5 times the density of water. One of the
outer planets, Saturn, has a density of only 0.7 that of water, which means that Saturn would
float in water. Variations in the composition of the planets are largely responsible for the
density differences. The substances that make up both groups of planets are divided into
three groupsgases, rocks, and icesbased on their melting points. The terrestrial planets
are mostly rocks: dense rocky and metallic material, with minor amounts of gases. The
Jovian planets, on the other hand, contain a large percentage of the gases hydrogen and
helium, with varying amounts of ices: mostly water, ammonia, and methane ices.
Sentence ______ is the topic sentence
Exercise 2 109
How does U.S. family leave policy compare with that in other countries ? Most
industrialized countries have much more generous policies than those in the United States,
providing not only more extended leaves of absence so that working parents can take care of
their young children but also financial support for part or all of the leave period. In Sweden,
Germany, and France, one parent can take a paid infant-care leave supported by the employer
or a social insurance fund. If both parents choose to continue to work, they are guaranteed
access to high-quality daycare for their child. In Sweden, either the mother or father is
entitled to a twelve-month paid leave to stay home with a new infant. The parent on leave
is reimbursed 90 percent of her of his salary for the first nine months following the childs
birth, receive $ 150 per month for the next three months, and then is allowed to continue with
an unpaid leave and a job guarantee for six additional months until the child is eighteen
months old.
Sentence _______ is the topic sentence
108
109

Adapted from Planets and our solar system, TPO 16, Educational Testing Services
Adapted from Seifert and Hoffnung, Child and Adolescent Development, p.222.

148

[] Version 3.0
Vocabulary Enlargement
ammonia
methane
reimburse

[mon]
[men]
[,rimbs]

Sentence Pattern
Be entitled to
You may be entitled to reclaim some of the tax you paid last year.

Practical Translation Training



__________________________________________________________________is that
_______________________________________________________________________
Further ReadingUS Federal Holidays
Date
January 1
Third Monday in
January
First January 20
following a
Presidential
election
Third Monday in
February

Official Name
New Years Day
Birthday of Dr. Martin
Luther King, Jr.
Inauguration Day

Last Monday in
May

Memorial Day

July 4

Independence Day

First Monday in
September

Labor Day

Second Monday
in October
November 11
Fourth Thursday
in November

Columbus Day

December 25

Christmas

Washingtons
Birthday/Presidents
Day

Veteran Day
Thanksgiving Day

Remarks
Celebrates beginning of the Gregorian calendar year.
Honors Dr. Martin Luther King, Jr., Civil Rights leader
Observed only by federal government employees in
Washington, D.C., and the border counties of Maryland
and Virginia to relieve congestion that occurs with this
major event.
Washingtons Birthday was first declared a federal
holiday by an 1879 act of Congress. The Uniform
Holidays Act, 1968, shifted the date of the
commemoration of Washingtons Birthday from
February 22 to the third Monday in February
Honors the nations war dead from the Civil War
onwards; marks the unofficial beginning of the summer
season
Celebrates the signing of the Declaration of
Independence from British rule
Celebrates the achievements of workers and the labor
movement; marks the unofficial end of the summer
season
Honors Christopher Columbus, traditional discoverer of
the Americas.
Honors all veterans of the United States armed forces.
Traditionally celebrates the giving of thanks for the
autumn harvest. Traditionally includes the sharing of a
turkey dinner.
commemoration of the birth of Jesus of Nazareth

149

Version 3.0

[]

Exercise 3 110
Henry Ford is probably best known as the man who invented the gasoline-powered
automobile . In fact, Fords real passion was not cars but soybeans . From Fords
perspective, soybeans were the solution to most of humanitys problems. A soybean diet
would, be thought, cure numerous ills. Thus, every meal served in the Ford home included
one soybean dish and a big pitcher of soymilk. But, in his mind, soybeans were not just
good nutrition. Ford was convinced they could also revolutionize manufacturing. In the
1930s, Ford built three soybean-processing plants.What was produced in those plants?
Paint, fittings, knobs, and horn buttons for cars, all made from a soybean-oil base.
Sentence _______ is the topic sentence
Exercise 4 111
Although American society has always been divided into competing and cooperating interests,
the development of formal interest group organizations began only in the late 1800s. The
first formal interest groups were organizations of farmers, such as the Grange, founded in
1867. Farmers groups were followed by the organization of workers into labor unions in
the late 1800s . These organizationlike the many groups that followeddeveloped in
response to changing social, economic, technological, and political conditions . Farm
organizations, for example, developed in response to the depression of the 1870s, which was
particularly devastating to the farmers in the South and West. Similarly, labor unions
developed after the industrial revolution created a permanent wagelabor force with little
political or economic power of their won.
Sentence ________ is the topic sentence
Exercise 5
Moreover, getting petroleum out of the ground and from under the sea and to the consumer
can create environmental problems anywhere along the line. Pipelines carrying oil can be
broken by faults or landslides, causing serious oil spills. Spillage from huge oil-carrying
cargo ships, called tankers, involved in collisions or accidental groundings (such as the one
off Alaska in 1989) can create oil slicks at sea. Offshore platforms may also lose oil, creating
oil slicks that drift ashore and foul the beaches, harming the environment. Sometimes, the
ground at an oil field may subside as oil is removed. The Wilmington field near Long Beach,
California, has subsided nine meters in 50 years; protective barriers have had to be built to
prevent seawater from flooding the area. Finally, the refining and burning of petroleum and
its products can cause air pollution. Advancing technology and strict laws, however, are
helping control some of these adverse environmental effects.
110
111

Adapted from Lanaine Flemming, Reading Keys, 3rd edition


Adapted from Lasser, American Politics, p.208.

150

[] Version 3.0
Vocabulary Enlargement
soybean
hookworm

[s,bin]
[hkwm]

Sentence Pattern
Live off
A man who all his life had lived off his father.

The injured climbers had only meager supplies of water and peanuts to live off.

Practical Translation Training



_______________________________________________________________________
Further Reading: Diets in US before mid-nineteenth century
Before the mid-nineteenth century, people in the United States ate most foods only in season. Drying,
smoking, and salting could preserve meat for a short time, but the availability of fresh meat, like that
of fresh milk, was very limited; there was no way to prevent spoilage. But in 1810 a French inventor
named Nicolas Appert developed the cooking-and-sealing process of canning. And in the 1850s an
American named Gail Borden developed a means of condensing and preserving milk. Canned goods
and condensed milk became more common during the 1860s, but supplies remained low because
cans had to be made by hand. By 1880, however, inventors had fashioned stamping and soldering
machines that mass-produced cans from tinplate. Suddenly all kinds of food could be preserved and
bought at all times of the year.
Other trends and inventions had also helped make it possible for Americans to vary their daily diets.
Growing urban populations created demand that encouraged fruit and vegetable farmers to raise more
produce. Railroad refrigerator cars enabled growers and meat packers to ship perishables great
distances and to preserve them for longer periods. Thus, by the 1890s, northern city dwellers could
enjoy southern and western strawberries, grapes, and tomatoes, previously available for a month at
most, for up to six months of the year. In addition, increased use of iceboxes enabled families to store
perishables. An easy means of producing ice commercially had been invented I the 1870s, and by
1900 the nation had more than two thousand commercial ice plants, most of which made home
deliveries. The icebox became a fixture in most homes and remained so until the mechanized
refrigerator replaced it in the 1920s and 1930s.
Almost everyone now had a more diversified diet. Some people continued to eat mainly foods that
were heavy in starches or carbohydrates, and not everyone could afford meat. Nevertheless, many
families could take advantage of previously unavailable fruits, vegetables, and dairy products to
achieve more varied fare.

151

Version 3.0

[]

Near the Middle

Exercise 1 112
Contrary to the myth, Americans did not invent baseball in Cooperstown, New York, in 1839
. As an English game called rounders, the pastime had existed in one form or another
since the seventeenth century . But if Americans did not create baseball, they
unquestionably took the game and turned it into a major professional sport. The first
organized baseball team, the New York Knickerbockers, was formed in 1845. In the 1860s,
Americans codified the rules, and the sport assumed its modern form. Overhand pitches,
for instance, replaced underhand tosses . In the same decade, Promoters organized
professional clubs and began to charge admission and compete for players.
Sentence ________ is the topic sentence
Exercise 2 113
Web logs, or as they are more commonly known, blogs regularly record in cyberspace the
personal and political views of the people who post them. To hear diehard bloggers tell it,
their regular, sometimes daily, postings have brought about a cultural revolution. Whether
the blogger is conservative or progressive, the message seems to be the same: Traditional
news sources are no longer of interest. People are getting their news from blogs instead
While there is some small truth to that claim, its still pretty obvious that newspapers
continue to sell and that many Americans still watch the nightly news. However, thats not
to say that bloggers havent had a powerful impact . On the contrary, blogs have
enormously enlarged public access to information about key current events. For instance,
in 2004, it was a blogger who proclaimed, correctly as it turned out, that a news story about
George W. Bushs national guard record was based on doctored evidence. In another
incident that showed blogger ingenuity, book editor Russ Kick beat out experienced
journalists after he read that the U.S. military was clamping down on press photos of coffins
arriving in the states from Iraq. Outraged by what he saw as censorship, Kick immediately
filed a Freedom of Information Act request. As a result of his request, Kick got a CD from
the air force showing photos of the coffins coming home. He then posted the photos on his
web log to the embarrassment of members of the national press, who were forced to beat a
path to his door. No one among conventional news journalists had thought to ask whether
the military had pictures. It took a blogger to do it.
Sentence _________ is the topic sentence
112
113

Adapted from Lanaine Flemming, Reading Keys, 3rd edition


Adapted from www.time.com/time/personoftheyear/2004/poymoments.html

152

[] Version 3.0
Vocabulary Enlargement
myth
diehard

[m]
[dahrd]

Sentence Pattern
Clamp down on
The mayor had become ineffectual in the struggle to clamp down on drugs

Recently, decrees were issued to clamp down on terrorism.


.
Practical Translation Training

_____________________________________________________________________
Further Reading: The growth of Baseball in U.S. 114
Before the Civil War, baseball competed for public interest with cricket and regional variants of
baseball, notably town ball played in Philadelphia and the Massachusetts Game played in New
England. In the 1860s, aided by the War, "New York" style baseball expanded into a national game,
as its first governing body, The National Association of Base Ball Players was formed. The NABBP
soon expanded into a true national organization, although most of the strongest clubs remained those
based in the northeastern part of the country. In its 12-year history as an amateur league, the Brooklyn
Atlantics won seven championships, establishing themselves as the first true dynasty in the sport,
although, the New York Mutuals were widely considered to be one of the best teams of the era as well.
By the end of 1865, almost 100 clubs were members of the NABBP. By 1867, it ballooned to over
400 members, including some clubs from as far away as San Francisco and Louisiana. One of these
clubs, the Chicago White Stockings, won the championship in 1870. Today known as the Chicago
Cubs, they are the oldest team in American organized sports. Because of this growth, regional and
state organizations began to assume a more prominent role in the governance of the sport.
The NABBP of America was initially established upon principles of amateurism. However, even early
in its history some star players, such as James Creighton of Excelsior, received compensation, either
secretly or indirectly. In 1866, the NABBP investigated Athletic of Philadelphia for paying three
players including Lip Pike, but ultimately took no action against either the club or the players. To
address this growing practice, and to restore integrity to the game, at its December 1868 meeting the
NABBP established a professional category for the 1869 season. Clubs desiring to pay players were
now free to declare themselves professional.
The Cincinnati Red Stockings were the first to so declare themselves as openly professional, and were
easily the most aggressive in recruiting the best available players. Twelve clubs, including most of the
strongest clubs in the NABBP, ultimately declared themselves professional for the 1869 season.
114

Adapted from http://en.wikipedia.org/wiki/History_of_baseball_in_the_United_States

153

Version 3.0

[]

Near the End


Exercise 1
Fathers of children born outside marriage are sometimes pictured as lacking a sense of
responsibility for their actions. At other times, they are portrayed as people who would like
to be responsible for their children if only they were given a chance, yet in reality, some
fathers do take responsibility for their children and others would do so if they could. Then
there are those who flatly refuse all responsibility. The truth is that no one pattern or
picture fits all fathers of children born outside of marriage.
Sentence ________ is the topic sentence
Exercise 2
In general, when men talk to one another, they like to trade sports opinions or discuss hobbies
. Men make statements like No one can take the Giants this year or My circular saw is
already rusty Overall, they tend to talk more about political than personal relationships.
Professional problems or challenges are also hot topics-a proposal that may not get through a
committee, a union vote that did not go well. If men do mention their wives or families,
they are likely to be brief. They dont go into depth or detail. The wife is fine; the
children are good. In other words, men dont usually talk about their personal lives, at least
not to one another.
Sentence _________ is the topic sentence
Exercise 3
Speculation on the origin of these Pacific islanders began as soon as outsiders encountered
them. In the absence of solid linguistic, archaeological, and biological data, many fanciful
and mutually exclusive theories were devised. Pacific islanders were variously thought to
have come from North America, South America, Egypt, Israel, and India, as well as
Southeast Asia. Many older theories implicitly deprecated the navigational abilities and
overall cultural creativity of the Pacific islanders. For example, British anthropologists G.
Elliot Smith and W. J. Perry assumed that only Egyptians would have been skilled enough to
navigate and colonize the Pacific. They inferred that the Egyptians even crossed the Pacific to
found the great civilizations of the New World (North and South America). In1947
Norwegian adventurer Thor Heyerdahl drifted on a balsa-log raft westward with the winds
and currents across the Pacific from South America to prove his theory that Pacific islanders
were Native Americans (also called American Indians). Later Heyerdahl suggested that the
Pacific was peopled by three migrations: by Native Americans from the Pacific Northwest of
North America drifting to Hawaii, by Peruvians drifting to Easter Island, and by Melanesians.
In 1969 he crossed the Atlantic in an Egyptian style reed boat to prove Egyptian influences in
the Americas. Contrary to these theorists, the overwhelming evidence of physical
anthropology, linguistics, and archaeology shows that the Pacific islanders came from
Southeast Asia and were skilled enough as navigators to sail against the prevailing winds and
currents.
154

[] Version 3.0
Vocabulary Enlargement
rusty
nominee
harassment

[rsti]
[nmni]
[hrsmnt]

Sentence Pattern
Get through
If you can get through all that, youre almost in the clear.

Such a radical proposal would never get through parliament

Practical Translation Training



___________________________________________________________________________
Further Reading: The Homestead Act of 1862 115
The Homestead Act of 1862 gave beads of families or individuals aged twenty-one or older the right
to own 160 acres of public land in the western United States after five years of residence and
improvement. This law was intended to provide land for small farmers and to prevent land from being
bought for resale at a profit or being owned by large landholders. An early amendment to the act even
prevented husbands and wives from filing separate claims. The West, land reformers had assumed,
would soon contain many 160-acre family farms. They were doomed to disappointment. Most
landless Americans were too poor to become farmers even when they could obtain land without cost.
The expense of moving a family to the ever-receding frontier exceeded the means of many, and the
cost of tools, draft animals, a wagon, a well, fencing, and of building the simplest house, might come
to $1,000---a formidable barrier. As for the industrial workers for whom the free land was supposed to
provide a "safety valve," they had neither the skills nor the inclination to become farmers.
Homesteaders usually came from districts not far removed from frontier conditions. And despite the
intent of the law, speculators often managed to obtain large tracts. They hired people to stake out
claims, falsely swear that they had fulfilled the conditions laid down in the law for obtaining legal title,
and then deed the land over to their employers.
Furthermore, 160 acres were not enough for raising livestock or for the kind of commercial
agriculture that was developing west of the Mississippi. The national government made a feeble
attempt to make larger holdings available to homesteaders by passing the Timber Culture Act of 1873,
which permitted individuals to claim an additional 160 acres if they would agree to plant a quarter of
it in trees within ten years. This law proved helpful to some farmers in the largely treeless states of
Kansas, Nebraska, and the Dakotas. Nevertheless, fewer than 25 percent of the 245,000 who took up
land under the Act obtained final title to the property.
115

Adapted from TOEFL pBT, 2002.05, Educational Testing Services.

155

Version 3.0

[]

4.1.6 Supporting Details


Definition
The materials used to supply the reasons, dates, examples, statistics, facts, studies, and so on
that make the main idea both clear and convincing.

Example: Read the topic sentence and choose two sentences as supporting details
Topic Sentence:
To this day, Fred Shuttlesworth remains an unsung hero of the civil rights movement.
Supporting Details:
A. No matter what happened, Fred Shuttlesworth was always convinced that he was
doing Gods work.
B. The Ku Klux Klan tried to stop Shuttlesworths organizing activities by bombing his
house, but he refused to give in.
C. Although the intrepid Shuttlesworth was active in every aspect of the civil rights
movement, the media preferred to focus on the bravery of Martin Luther King Jr.,
whose eloquence made him so quotable.
D. Diane McWhorters book Carry Me Home, an account of both her Alabama
childhood and the civil rights movement, won a Pulitzer Prize.

Explanation: ______________________________________________________________

Key Features
Supporting details are the authors way of guiding readers to the appropriate meaning.
They are the authors way of saying to readers, I mean this and not that.
The form of supporting details varies. Details can range in form from reasons to statistics.
The main idea is what causes supporting details to vary. When the main idea changes, so
do the details that develop it.

Paragraph = (Background) + Topic Sentence + N


i=1 Detailsi

Paragraph = Topic Sentence1 + N


i=1 Detailsi + Topic Sentence2
Paragraph = N
i=1 Detailsi + Topic Sentence

Paragraph = N
i=1 Detailsi

156

[] Version 3.0
Vocabulary Enlargement
unsung hero
intrepid
eloquence

[n'trpd]
['elkwns]

Further Reading: Overview: Three Klans


The First Klan was founded in 1865 in Pulaski, Tennessee, by six veterans of the Confederate Army.
The name is probably from the Greek word kuklos which means circle, suggesting a circle or band of
brothers. Although there was no organizational structure above the local level, similar groups arose
across the South adopted the same name and methods. Klan groups spread throughout the South as an
insurgent movement during the Reconstruction era in the United States. As a secret vigilante group,
the Klan targeted freedmen and their allies; it sought to restore white supremacy by threats and
violence, including murder, against black and white Republicans. In 1870 and 1871, the federal
government passed the Force Acts, which were used to prosecute Klan crimes. Prosecution of Klan
crimes and enforcement of the Force Acts suppressed Klan activity. In 1874 and later, however,
newly organized and openly active paramilitary organizations, such as the White League and the Red
Shirts, started a fresh round of violence aimed at suppressing blacks' voting and running Republicans
out of office. These contributed to segregationist white Democrats regaining political power in all the
Southern states by 1877.
In 1915, the Second Klan was founded in Atlanta, Georgia. Starting in 1921, it adopted a modern
business system of recruiting (which paid most of the initiation fee and costume charges as
commissions to the organizers) and grew rapidly nationwide at a time of prosperity. Reflecting the
social tensions of urban industrialization and vastly increased immigration, its membership grew most
rapidly in cities, and spread out of the South to the Midwest and West. The second KKK preached
One Hundred Percent Americanism and demanded the purification of politics, calling for strict
morality and better enforcement of prohibition. Its official rhetoric focused on the threat of the
Catholic Church, using anti-Catholicism and nativism. Its appeal was directed exclusively at white
Protestants. Some local groups took part in attacks on private houses and carried out other violent
activities. The violent episodes were generally in the South. The second Klan was a formal fraternal
organization, with a national and state structure. At its peak in the mid-1920s, the organization
claimed to include about 15% of the nation's eligible population, approximately 45 million men.
Internal divisions, criminal behavior by leaders, and external opposition brought about a collapse in
membership, which had dropped to about 30,000 by 1930. It finally faded away in the 1940s. Klan
organizers also operated in Canada, especially in Saskatchewan in 1926-28, where members of the
Klan attacked immigrants from Eastern Europe.
The Ku Klux Klan name was used by many independent local groups opposing the Civil Rights
Movement and desegregation, especially in the 1950s and 1960s. During this period, they often forged
alliances with Southern police departments, as in Birmingham, Alabama; or with governor's offices,
as with George Wallace of Alabama. Several members of KKK groups were convicted of murder in
the deaths of civil rights workers and children in the bombing of the 16th Street Baptist Church in
Birmingham. Today, researchers estimate that there may be 150 Klan chapters with upwards of 5,000
members nationwide.

157

Version 3.0

[]

Exercise 1: Which details does NOT support this topic sentence?


Topic Sentence:
Belief in traditional religion, along with church attendance, has been declining in the
United States, but spirituality is alive and well and taking many different forms.
Supporting Details
A. In Memphis, an Episcopal church added a special and very untraditional service that
blends ancient Irish music with communion.
B. One of the results of the immigration reform of 1965 was that immigration brought
with it new religious practices that took hold in the U.S.
C. Out of 1,004 Americans polled by Newsweek and Beliefnet.com, 57 percent said that
spirituality was an important part of their lives.
D. In the same poll, 70 percent said that their spouses shared their beliefs.
Exercise 2: Which details does NOT support this topic sentence?
Topic Sentence:
According to a 1999 Texas law, judges have a right to impose a public punishment for
certain crimes
Supporting Details:
A. Those convicted of drunk driving can be made to stand at busy intersections with a
sign identifying their crime.
B. All 50 states have laws requiring sex offenders to alert communities to their presence.
C. One Texas judge has ordered registered sex offenders to display signs in front of their
homes; the signs say Registered Sex Offender Lives Here.
D. Probation officers say that the Texas law seems to be successful at stopping crime
because those on probation are saying they will do anything to avoid public
punishment.
Exercise 3: Which details does NOT support this topic sentence?
Topic Sentence:
Some athletes are only interested in the pursuit of extreme sports, where the risks include
not just injury but death.
Supporting Details:
A. Despite the high fatality rate, BASE jumpers leap from buildings, cliffs, and bridges,
relying solely on a parachute to save them.
B. Paraglider pilots use nylon wings to take off from mountaintops and float away on
wind currents that they hope will carry them safely to a landing place, but there is no
guarantee.
C. Not all scientists are convinced that a love of risk-taking (or not) is part of our genetic
makeup.
D. Despite the high rate of injuries common to the sport, snowboardings popularity
continues to grow.
158

[] Version 3.0
Vocabulary Enlargement
spirituality
fatality
parachute

[,sprtu'lti]
[f'tlti]
['pr't]

Further Reading: An introduction on Extreme Sport


Extreme sports is the generic label used for sports that in some way go beyondin endurance, testing
the limits, or dangertraditional sports. The label was promoted in the 1990s by athletes, promoters,
the sports equipment industry, and advertisers to bring mainstream attention to these sports and, at the
same time, to differentiate them from related sports. While most sports now classified as extreme have
all existed for some time and most have governing bodies and regional and world competitions,
extreme sports as a category of sports achieved their highest level of public exposure in 1995 with the
First Extreme Games held in June in Newport, Rhode Island. These games were given 45 hours of
week-long coverage on the ESPN (Entertainment and Sports Programming Network) cable network
and have been popular ever since. The games included street luge, eco-challenge, skysurfing, sport
climbing, barefoot water ski jumping, in-line skating, BMX dirt biking, mountain biking, and bungy
jumping.
What differentiates these sports from the related ones varies with
each activity and might involve changes in the rules, the
combining of different events, use of different equipment and
venues, and a greater degree of risk to the athletes. Additionally,
some extreme sports have been called outlaw sports because
they have been banned in some jurisdictions as too dangerous. In
street luge, for example, the participants lie flat on their backs on
wheeled luges or rails and race downhill on roads. When these
roads are used also by pedestrians, bikers, and vehicles, the sport
is dangerous for both the competitors and others. Perhaps the
common feature of extreme sports and what makes them extreme are requirements of the activities
that make the sport especially dangerous and/or that test the limits of human physical ability or
endurance. Extreme sports are international with the Extreme or X Games attracting competitors from
dozens of nations. Different sports have developed in different nationsskysurfing in France,
barefoot ski jumping in Australia, and some sports such as street luge, sport climbing, eco-challenge,
and mountain biking draw participants from a variety of nations.
For the most part, however, competitors are from Western nations,
with most coming from the United States, Australia, Canada, Germany,
New Zealand, France, and England. Some sports, such as sport
climbing and eco-challenge, draw competitors who are in their thirties
and forties, but most competitors in the other sports are usually in their
twenties or even teens.Most extreme sports draw only male
competitors, with only eco-challenge and sport climbing having
significant numbers of women competitors. Sport climbing is the only
extreme sport with separate divisions for men and women. All extreme
sports, except for eco-challenge and skysurfing, are individual sports.

159

Version 3.0

[]

4.1.7 Note Taking

160

[] Version 3.0

Practice
Potash (the old name for potassium carbonate) is one of
the two alkalis (the other being soda, sodium carbonate) that
were used from remote antiquity in the making of glass, and
from the early Middle Ages in the making of soap: the
former being the product of heating a mixture of alkali and
sand, the latter a product of alkali and vegetable oil. Their
importance in the communities of colonial North America
need hardly be stressed.
Potash and soda are not interchangeable for all purposes,
but for glass- or soap-making either would do. Soda was
obtained largely from the ashes of certain Mediterranean sea
plants, potash from those of inland vegetation. Hence potash
was more familiar to the early European settlers of the North
American continent.
The settlement at Jamestown in Virginia was in many
ways a microcosm of the economy of colonial North
America, and potash was one of its first concerns. It was
required for the glassworks, the first factory in the British
colonies, and was produced in sufficient quantity to permit
the inclusion of potash in the first cargo shipped out of
Jamestown. The second ship to arrive in the settlement from
England included among its passengers experts in potash
making.
The method of making potash was simple enough. Logs
was piled up and burned in the open, and the ashes collected.
The ashes were placed in a barrel with holes in the bottom,
and water was poured over them. The solution draining from
the barrel was boiled down in iron kettles. The resulting
mass was further heated to fuse the mass into what was
called potash.
In North America, potash making quickly became an
adjunct to the clearing of land for agriculture, for it was
estimated that as much as half the cost of clearing land could
be recovered by the sale of potash. Some potash was
exported from Maine and New Hampshire in the
seventeenth century, but the market turned out to be mainly
domestic, consisting mostly of shipments from the northern
to the southern colonies. For despite the beginning of the
trade at Jamestown and such encouragements as a series of
acts "to encourage the making of potash," beginning in 1707
in South Carolina, the softwoods in the South proved to be
poor sources of the substance.
161

Version 3.0

[]

4.2 Internal Relationships of Paragraph


4.2.1 Reference

Subject Pronouns
Object Pronouns
Possessive Pronouns
Demonstrative Pronouns
Relative Pronouns
Other Pronouns

He, she it, they


Him, her, it, them
His, her, its, their
This, that, these, those
Who, whom, which, whose
All, another, any, both, each, either, a few, many, most,
neither, none, one, others, several, some

Personal Pronouns(Subject/Object/Possessive)
The first and only tidal power plant in North America is the Annapolis Royal Generating
Station. It was originally constructed in 1984 to look at alternative methods for
generating electricity. However, it has also created changes to the local environment.
Demonstrative Pronouns
Nearly every other animal has a relatively heavy coat of fur to protect it from the weather.
When humans began changing their environment by wearing clothes and building homes,
this became unnecessary.
Relative Pronouns
Vygotsky proposed that a child rises through the zone by a process called scaffolding, in
which a mentor provides guided, step-by-step instruction.
Other Pronouns
The kit fox resembles its distant cousin, the gray fox. Both have thick coats and bushy
tails.
Pro-forms
The Nile waters were found to be choked with plant matter. That awful condition was
caused by a shortage of rainfall in the Niles watershed.
Keys to Solution
Carefully read the sentence that contains the pronoun and check the surrounding text for
the referent. The referent will not always be the choice closet to the reference word, and
may appear before or after the pronoun. The correct referent will be the only logical
replacement for the pronoun

162

[] Version 3.0
Vocabulary Enlargement
scaffolding
bushy
choke
watershed

[skfld]
[bi]
[tk]
[wted]

Sentence Pattern
Protect from
Wild forests should be well protecedt from excessive deforestion.
.
Further Reading: The Genesis of Life116
Life originated in the early seas less than a billion years after the Earth was formed. Yet
another three billion years were to pass before the first plants and animals appeared on the
continents. Lifes transition from the sea to the land was perhaps as much of an evolutionary
challenge as was the genesis of life.
What forms of life were able to make such a drastic change in lifestyle? The traditional view
of the first terrestrial organisms is based on megafossils-relatively large specimens of
essentially whole plants and animal. Vascular plants, related to modern seed plants and ferns,
left the first comprehensive megafossil record. Because of this, it has been commonly
assumed that the sequence of terrestrialization reflected the evolution of modern terrestrial
ecosystems. In this view, primitive vascular plants first colonized the margins of continental
waters, followed by animals that fed on the plants, and lastly by animals that preyed on the
plant-eater. Moreover, the megafossils suggest that terrestrial life appeared and diversified
explosively near the boundary between the Silurian and the Devonian periods, a little more
than 400 million years ago.
Recently, however, paleontologists have been taking a closer look at the sediments below this
Silurian-Devonian geological boundary. It turns out that some fossils can be extracted from
these sediments by putting the rocks in an acid bath. The technique has uncovered new
evidence from sediments that were deposited near the shores of the ancient oceans-plant
microfossils and microscopic pieces of small animals. In many instances the specimens are
less than one-tenth of a millimeter in diameter. Although they were entombed in the rocks for
hundreds of millions of years, many of the fossils consist of the organic remains of the
organism.
These newly discovered fossils have not only revealed the existence of previously known
organisms, but have also pushed back these dates for the invasion of land by multicellular
organisms. Our views about the nature of the early plant and animal communities are now
being revised. And with those revisions come new speculations about the first terrestrial lifeforms.
116

Adapted from TOEFL pBT, 1996.12, Educational Testing Services.

163

Version 3.0

[]

Practice
Exercise 1
Many millions of years after ferns evolved (but long before the Hawaiian Island were born
from the sea), another kind of flora evolved on Earth: the seed-bearing plants. This was a
wonderful biological invention. The seed has an outer coating that surrounds the genetic
material of the new plant, and inside this covering is a concentrated supply of nutrients. Thus,
the speeds chances of survival are greatly enhanced over those of the naked spore.
The word This in the passage refers to
A. the spread of ferns and mosses in Hawaii
B. the creation of the Hawaiian Islands
C. the evolution of ferns
D. the development of plants that produce seeds
Exercise 2
But the myths that have grown up around the rites may continue as part of the groups oral
tradition and may even come to be acted out under conditions divorced from these rites.
When this occurs, the first step has been taken toward theater as an autonomous activity, and
thereafter entertainment and aesthetic values may gradually replace the former mystical and
socially efficacious concerns.
The word this in the passage refers to
A. The acting out of rites
B. The divorce of ritual performers from the rest of society
C. The separation of myths from rites
D. The celebration of supernatural forces
Exercise 3
What is particularly meaningful to anthropologists is the realization that although the
materials available to a society may to some extent limit or influence what it can do
artistically, the materials by no means determine what is done. Why does the artist in
Japanese society rake sand into patterns; and the artist in Roman society melt sand to form
glass?
The word it in the passage refers to
A. Realization
B. Society
C. Extent
D. influence

164

[] Version 3.0
Vocabulary Enlargement
fern
Spore
aesthetic
efficacious

[fn]
[spr]
[stk]
[,efkes]

Sentence Pattern
Act out
Can you act out the feeling of sorrow in body language?

Practical Translation Training



_____________________________________________________________________
Further Reading: How people live is by mirroring the environment 117
Perhaps the most obvious way artistic creation reflect how people live is by mirroring the
environment-the materials and technologies available to a culture. Stone, wood, tree bark,
clay, and sand are generally available materials. In addition, depending on the locality, other
resources may be accessible: shells, horns, gold, copper, and silver. The different uses to
which societies put these materials are of interest to anthropologists who may ask, for
example, why a people choose to use clay and not copper when both items are available.
Although there are no conclusive answers yet, the way in which a society views its
environment is sometimes apparent in its choice and use of artistic materials. The use of
certain metals, for example, may be reserved for ceremonial objects of special importance. Or
the belief in the supernatural powers of a stone or tree may cause a sculptor to be sensitive to
that material.
What is particularly meaningful to anthropologist is the realization that although the materials
available to a society may to some extent limit or influence what it can do artistically, the
materials by no means determine what is done. Why does the artist in Japanese society rake
sand into patterns; and the artist in Roman society melt sand to form glass? Moreover, even
when the same material is used in the same way by members of different societies, the form
or style of the work varies enormously from culture to culture. A society may simply choose
to represent objects or phenomena that are important to its population. An examination of the
art of the Middle Ages tells us something about the medieval preoccupation with theological
doctrine. In addition to revealing the primary concerns of a society, the content of that
societys art may also reflect the cultures social stratification.
117

Adapted from TOEFL pBT, 1999.08, Educational Testing Services.

165

Version 3.0

[]

Exercise 4
The necessary space is there, however, in many forms. The commonest spaces are those
among the particles--sand grains and tiny pebbles--of loose, unconsolidated sand and gravel.
Beds of this material, out of sight beneath the soil, are common. They are found wherever
fast rivers carrying loads of coarse sediment once flowed. For example, as the great ice sheets
that covered North America during the last ice age steadily melted away, huge volumes of
water flowed from them. The water was always laden with pebbles, gravel, and sand, known
as glacial outwash, that was deposited as the flow slowed down.
The phrase glacial outwash in the passage refers to
A. Fast rivers
B. Glaciers
C. The huge volumes of water created by glacial melting
D. The particles carried in water from melting glaciers.
Exercise 5
Jupiter and other giant planets are of a low-density type quite distinct from the terrestrial
planets: they are composed predominantly of such substances as hydrogen, helium, ammonia,
and methane.
The word they in the passage refers to
A. nuclear reactions
B. giant planets
C. terrestrial
D. substances
Exercise 6
If the physical barriers of the plant are breached, then preformed chemicals may inhibit or kill
the intruder, and plant tissues contain a diverse array of toxic or potentially toxic substances,
such as resins, tannins, glycosides, and alkaloids, many of which are highly effective
deterrents to insects that feed on plants.
The word which in the passage refers to
A. tissues
B. substances
C. barriers
D. alkaloids

166

[] Version 3.0
Vocabulary Enlargement
culminate
stoppage

[klmnet]
[stpd]

Sentence Pattern
Laden with
The air was laden with spices - the mingled fragrance of many grasses and flowers.
,.
Practical Translation Training

_____________________________________________________________________
Further Reading: Surge Glaciers
During most of their lives, surge glaciers behave like normal glaciers, traveling perhaps only
a couple of inches per day. However, at intervals of 10 to 100 years, these glaciers move
forward up to 100 times faster than usual. The surge often progresses along a glacier like a
great wave, proceeding from one section to another. Sub-glacial streams of melt water
pressure under the glacier might lift it off its bed, overcoming the friction between ice and
rock, thus freeing the glacier, which rapidly sliders downhill Surge glaciers also might be
influenced by the climate, volcanic heat, or earthquakes. However, many of these glaciers
exist in the same area as normal glaciers, often almost side by side.
Some 800 years ago, Alaskas Hubbard Glacier advanced toward the sea, retreated, and
advanced again 500 years later. Since 1895, this 70-mile-long river of ice has been flowing
steadily toward the Gulf of Alaska at a rate of approximately 200 feet per year. In June 1986,
however, the glacier surged ahead as much as 47 feet a day. Meanwhile, a western tributary,
called Valerie Glacier, advanced up to 112 feet a day. Hubbards surge closed off Russell
Fiord with a formidable ice dam, some 2,500 feet wide and up to 800 feet high, whose caged
waters threatened the town of Yakutat to the south.
About 20 similar glaciers around the Gulf of Alaska are heading toward the sea. If enough
surge glaciers reach the ocean and raise sea levels, West Antarctic ice shelves could rise off
the seafloor and become adrift. A flood of ice would then surge into the Southern Sea. With
the continued rise in sea level, more ice would plunge into the ocean, causing sea levels to
rise even higher, which in turn would release more ice and set in motion a vicious cycle. The
additional sea ice floating toward the tropics would increase Earths albedo and lower global
temperatures, perhaps enough to initiate a new ice age. This situation appears to have
occurred at the end of the last warm interglacial (the time between glaciations), called the
Sangamon, when sea ice cooled the ocean dramatically, spawning the beginning of the Ice
Age.
167

Version 3.0

[]

Exercise 7
Aviculturists, people who raise birds for commercial sale, have not yet learned how to
simulate the natural incubation of parrot eggs in the wild. They continue to look for better
ways to increase eggs production and to improve chick survival rates.
The word They in the passage refers to
A. birds
B. aviculturists
C. eggs
D. rates

Exercise 8
Some of their baskets were completely covered with shell pendants; others with feathers that
made the baskets surfaces as soft as the breasts of birds.
The word others in the passage refers to
A. masters
B. baskets
C. pendants
D. surfaces

Exercise 9
Encouraged by the example of certain Americans of European descent such as Thomas
Eakins, Robert Henri, and George Lukes, who had included persons of African descent in
their paintings as serious studies rather than as trivial or sentimental stereotypes, African
American artists of this period set about creating a new portrayal of themselves and their
lives in the United States. As they began to strive for social and cultural independence, their
attitudes toward themselves changed, and, to some extent, other segments of American
society began to change their attitudes toward them.
The word them in the passage refers to
A. Americans of European descent
B. paintings
C. African American artists
D. attitudes

168

[] Version 3.0
Vocabulary Enlargement
Aviculturists
Pendant
Trivial
Chick

['ev'kltrst]
['pndnt]
['trvl]
[tk]

Sentence Pattern
Set about
They immediately set about to create a specialized, step by step methodology to
address this.

Further Reading: Aviculturists


Aviculturists, people who raise birds for commercial sale, have not yet learned how to
simulate the natural incubation of parrot eggs in the wild. They continue to look for better
ways to increase egg production and to improve chick survival rates.
When parrots incubate their eggs in the wild, the temperature and humidity of the nest are
controlled naturally. Heat is transferred from the bird's skin to the top portion of the eggshell,
leaving the sides and bottom of the egg at a cooler temperature. This temperature gradient
may be vital to successful hatching. Nest construction can contribute to this temperature
gradient Nests of loosely arranged sticks, rocks, or dirt are cooler in temperature at the
bottom where the egg contacts the nesting material. Such nests also act as humidity regulators
by allowing rain to drain into the bottom sections of the nest so that the eggs are not in direct
contact with the water. As the water that collects in the bottom of the nest evaporates, the
water vapor rises and is heated by the incubating bird, which adds significant humidity to the
incubation environment.
In artificial incubation programs, aviculturists remove eggs from the nests of parrots and
incubate them under laboratory conditions. Most commercial incubators heat the eggs fairly
evenly from top to bottom, thus ignoring the bird's method of natural incubation, and perhaps
reducing the viability and survivability of the hatching chicks. When incubators are not used,
aviculturists sometimes suspend wooden boxes outdoors to use as nests in which to place
eggs. In areas where weather can become cold after eggs are laid, it is very important to
maintain a deep foundation of nesting material to act as insulator against the cold bottom of
the box. If eggs rest against the wooden bottom in extremely cold weather conditions, they
can become chilled to a point where the embryo can no longer survive. Similarly, these boxes
should be protected from direct sunlight to avoid high temperatures that are also fatal to the
growing embryo. Nesting material should be added in sufficient amounts to avoid both
extreme temperature situations mentioned above and assure that the eggs have a soft, secure
place to rest.

169

Version 3.0

[]

4.2.2 Coherence
Recognizing Coherence
____________________

Example 1: :
1) It turned from very bad to exceedingly good.
2) Soon after Jefferson had introduced the tomato to American society, recipes combining
the newly fashionable tomato with the equally fashionable and exotic sauce known as
ketchup began to appear.
3) Thomas Jefferson, who cultivated the tomato in his gardens at Monticello and served
dishes containing tomatoes at lavish feasts, often receives credit for changing the
reputation of the tomato.
4) By the middle of the 19th century, both the tomato and tomato ketchup were staples of
the American kitchen.
Example 2: Where would the bold sentence best fit for the four squares 118
Kindergartners are quite skillful with language. Providing a sharing time gives children a
natural opportunity for talking. However, many will need help in becoming good listeners.
Some sort of rotation scheme is usually necessary to divide talking opportunities between the
talkative and silent extremes. Teachers can provide activities or experiences for less
confident children to talk about, such as a field trip, a book, or a film.
Most of them like to talk, especially in front of a group.

Keys to Solution
Dont waste time looking for spots where the sentence will clear up an illogical or
incoherent discussion. Instead, look for places where it simply provides additional
relevant information.
See if there are any reference words such as pronouns and pro-forms in the insertion
sentence. Look for such referents in surrounding sentences, and these will indicate where
the insertion sentence belongs.
See if there are any transitional words or phrases in the insertion sentence. Such
transitions indicate the function of an insertion sentence.
Double check the paragraph after inserting
118

Adapted from Nancy Gallagher, Deltas key to the next generation TOEFL test: Advanced skill practice for
the iBT, Delta Publishing Company.

170

[] Version 3.0
Vocabulary Enlargement
exotic
ketchup
lavish
feast
staple

[ztk]
[ktp]
[lv]
[fist]
[step()l]

Further Reading:Education in the U.S. 119


As the twentieth century began, the importance of formal education in the United States
increased The frontier had mostly disappeared and by 1910 most Americans lived in towns
and cities. Industrialization and the bureaucratization of economic life combined with a new
emphasis upon credentials and expertise to make schooling increasingly important for
economic and social mobility. Increasingly, too, schools were viewed as the most important
means of integrating immigrants into American society.
The arrival of a great wave of southern and eastern European immigrants at the turn of the
century coincided with and contributed to an enormous expansion of formal schooling. By
1920 schooling to age fourteen or beyond was compulsory in most states, and the school year
was greatly lengthened. Kindergartens, vacation schools, extracurricular activities, and
vocational education and counseling extended the influence of public schools over the lives
of students, many of whom in the larger industrial cities were the children of immigrants.
Classes for adult immigrants were sponsored by public schools, corporations, unions,
churches, settlement houses, and other agencies.
Reformers early in the twentieth century suggested that education programs should suit the
needs of specific populations. Immigrant women were one such population. Schools tried to
educate young women so they could occupy productive places in the urban industrial
economy, and one place many educators considered appropriate for women was the home.
Although looking after the house and family was familiar to immigrant women, American
education gave homemaking a new definition. In preindustrial economies, homemaking had
meant the production as well as the consumption of goods, and it commonly included
income-producing activities both inside and outside the home, in the highly industrialized
early-twentieth-century United States, however, overproduction rather than scarcity was
becoming a problem. Thus, the ideal American homemaker was viewed as a consumer rather
than a producer. Schools trained women to be consumer homemakers cooking, shopping,
decorating, and caring for children efficiently in their own homes, or if economic necessity
demanded, as employees in the homes of others. Subsequent reforms have made these
notions seem quite out-of-date.
119

Adapted from TOEFL pBT, 2000.01, Educational Testing Services.

171

Version 3.0

[]


this, these, theirsuch, another, other, they, it

Exercise 1 120
Impacts by meteorites represent one mechanism that could cause global catastrophes and
seriously influence the evolution of life all over the planet. According to some estimates,
the majority of all extinctions of species may be due to such impacts. Such a perspective
fundamentally changes our view of biological evolution. The standard criterion for the
survival of a species is its success in competing with other species and adapting to slowly
changing environments. Yet an equally important criterion is the ability of a species to
survive random global ecological catastrophes due to impacts.
This is the criterion emphasized by Darwins theory of evolution by natural selection.
Exercise 2
This traditional Native American dish was quite a novelty to newcomers to the Americas.
Columbus and his sailors found natives in the West Indies wearing popcorn necklaces, and
explorer Hernando Corts described the use of popcorn amulets in the religious ceremonies
of the Aztecs. According to legendary descriptions of the celebratory meal, Quadequina,
the brother of Chief Massasoit, contributed several deerskin bags of popcorn to the
celebration.
A century after these early explorers, the Pilgrims at Plymouth may have been
introduced to popcorn at the first Thanksgiving dinner.


some/suchother

later, older, former, latter, earlier, new, old
however, yet, other, even if/though
/
thus, therefore, hence, consequently, accordingly
120

Adapted from Meteorite Impact and Dinosaur Extinction, TOEFL Official Model Exam

172

[] Version 3.0
Vocabulary Enlargement
meteorite
catastrophe
perception

[mitrat]
[ktstrfi]
[psep()n]

Sentence Pattern
Compete with
He can take advantage of the wiles of as a skilled negotiator to compete with others

Practical Translation Training


,
_____________________________________________________________________
_____________________________________________________________________
Further Reading: Recognizable Meteorites

121

The most easily recognizable meteorites are the iron variety, although they only represent about 5
percent of all meteorite falls. They are composed of iron and nickel along with sulfur, carbon, and
traces of other elements. Their composition is thought to the similar to that of Earths iron core and
indeed they might have once made up the core of a large planetoid that disintegrated long ago. Due to
their dense structure, iron meteorites have the best chance of surviving an impact, and most are found
by farmers plowing their fields.
One of the best hunting grounds for meteorites is on the glaciers of Antarctica where the dark stones
stand out in stark contrast to the white snow and ice. When meteorites fall on the continent they are
embedded in the moving ice sheets. At places where the glaciers move upward against mountain
ranges, meteorites are left exposed on the surface. Some of the meteorites that have landed in
Antarctica are believed to have conic from the Moon and even as far away as Mars, when large
impacts blasted out chunks of material and hurled them toward Earth.
Perhaps the worlds largest source of meteorites is the Nullarbor Plain, an area of limestone that
stretches for 400 miles along the southern coast of Western and South Australia. The pale, smooth
desert plain provides a perfect backdrop for spotting meteorites, which are usually dark brown of
black. Since very little erosion takes place, the meteorites are well preserved and are found just where
they landed. Over 1,000. fragments from 150 meteorites that fell during the last 20,000 years have
been recovered. One large iron meteorite, called the Mundrabilla meteorite, weighed more than 11
tons.
Stony meteorites, called chordates, are the most common type and make up more than 90 percent of
all falls . But because they are similar to Earth materials and therefore erode easily, they are often
difficult to find. Among the most ancient bodies in the solar system are the carbonaceous chondrites
that also contain carbon compounds that might have been the precursors of life on Earth.
121

Adapted from TOEFL pBT, 1998.08, Educational Testing Services.

173

Version 3.0

[]

Exercise 3 122
What do you remember about your life before you were three? Few people can remember
anything that happened to them in their early years Adults memories of the next few
years also tend to be scanty Most people remember only a few eventsusually ones that
were meaningful and distinctivesuch as being hospitalized or a siblings birth.
Other important occasions are school graduations and weddings.
Exercise 4 123
It is unlikely in the foreseeable future that marijuana will be fully legalized in the U.S.
In 1996, voters in both California and Arizona agreed to a bill for marijuana to be legalized
for such medical purposes. It means that doctors and patients will be protected from criminal
prosecution when marijuana is prescribed for serious conditions. Other states, such as
Alaska, have not legalized drug, but have decriminalized it such that possession of small
amounts is only a minor offense that does not carry a police record.
Some states, however, do permit its use for medical purposes when it comes
accompanied by a doctors prescription.
Exercise 5 124
Because they are always swimming, tunas simply have to open their mouths and water is
forced in and over their gills. Accordingly, they have lost most of the muscles that other
fishes use to suck in water and push it past the gills. In fact, tunas must swim to breathe.
They must also keep swimming to keep from sinking, since most have largely or completely
lost the swim bladder, the gas-filled sac that helps most other fish remain buoyant.
Consequently, tunas do not need to suck in water.
Exercise 6
One method of popping corn involved skewering an ear of corn on a stick and roasting it
until the kernels popped off the ear. Corn was also popped by first cutting the kernels off
the cob, throwing them into a fire, and gathering them as they popped out of the fire. In a
final method for popping corn, sand and unpopped kernels of corn were mixed together in a
cooking pot and heated until the corn popped to the surface of the sand in the pot.
Native Americans have been popping corn for at least 5,000 years, using a variety of
different methods.
122

Adapted from Infantile Amnesia, TPO 6, Educational Testing Services.


Adapted from Ji-Yeon LEE, Marijuana, Longman iBT general course for the TOEFL Reading, Person Education
Korea, Ltd
124
Adapted from Swimming Machine, OG Practice Set 6
123

174

[] Version 3.0
Vocabulary Enlargement
marijuana
decriminalize
gill
bladder
sac
buoyant

[,mrwn]
[dikrmnlaz]
[l]
[bld]
[sk]
[bnt]

Sentence Pattern
Prescribe for
What punishment does the law prescribe for this crime?

Further Reading: Can animal remember past events? 125


Some animal behaviorists argue that certain animals can remember past events, anticipate

future ones, make plans and choices, and coordinate activities within a group. These
scientists, however, are cautious about the extent to which animals can be credited with
conscious processing.
Explanations of animal behavior that leave out any sort of consciousness at all and ascribe
actions entirely to instinct leave many questions unanswered. One example of such
unexplained behavior: Honeybees communicate the sources of nectar to one another by doing
a dance in a figure-eight pattern. The orientation of the dance conveys the position of the
food relative to the suns position in the sky, and the speed of the dance tells how far the food
source is from the hive. Most researchers assume that the ability to perform and encode the
dance is innate and shows no special intelligence. But in one study, when experimenters kept
changing the site of the food source, each time moving the food 25 percent farther from the
previous site, foraging honeybees began to anticipate where the food source would appear
next. When the researchers arrived at the new location, they would find the bees circling the
spot, waiting for their food. No one has yet explained how bees, whose brains weigh four tenthousandths of an ounce, could have inferred the location of the new site.
Other behaviors that may indicate some cognition include tool use. Many animals, like the
otter who uses a stone to crack mussel shells, are capable of using objects in the natural
environment as rudimentary tools. One researcher has found that mother chimpanzees
occasionally show their young how to use tools to open hard nuts. In one study, chimpanzees
compared two pairs of food wells containing chocolate chips. One pair might contain, say,
five chips and three chips, the other our chips and three chips. Allowed to choose which pair
they wanted, the chimpanzees almost always chose the one with the higher total, showing
some sort of summing ability. Other chimpanzees have learned to use numerals to label
quantities of items and do simple sums.
125

Adapted from TOEFL pBT, 2002.10, Educational Testing Services.

175

Version 3.0

[]

Exercise 7
These people probably originated in eastern Nigeria. Their migration may have been set in
motion by an increase in population caused by a movement of peoples fleeing the desiccation,
or drying up, of the Sahara. They spoke a language, prior-Bantu (Bantu means the
people), which is the parent tongue of a language of a large number of Bantu languages still
spoken throughout sub-Sahara Africa. Why and how these people spread out into central and
southern Africa remains a mystery, but archaeologists believe that their iron weapons allowed
them to conquer their hunting-gathering opponents, who still used stone implements. Still,
the process is uncertain, and peaceful migrationor simply rapid demographic growthmay
have also caused the Bantu explosion.
These people had a significant linguistic impact on the continent as well.
Exercise 8
Although origin in ritual has long been the most popular, it is by no means the only theory
about how the theater came into being. Storytelling has been proposed as one alternative.
Under this theory, relating and listening to stories are seen as fundamental human pleasures.
Thus, the recalling of an event (a hunt, battle, or other feat) is elaborated through the
narrators pantomime and impersonation and eventually through each role being assumed by
a different person).
To enhance their listeners enjoyment, storytellers continually make their stories more
engaging and memorable.
Exercise 9
Ecologists are especially interested to know what factors contribute to the resilience of
communities because climax communities all over the world are being severely damaged or
destroyed by human activities. The destruction caused by the volcanic explosion of Mount
St. Helens, in the northwestern United States, for example, pales in comparison to the
destruction caused by humans. We need to know what aspects of a community are most
important to the communitys resistance to destruction, as well as its recovery.
In fact, damage to the environment by humans is often much more severe than damage
by natural events and processes.

176

[] Version 3.0
Vocabulary Enlargement
desiccation
demographic
pantomime
elaborate
resilience

[,dsken]
[,dmrfk]
[pntmam]
[lbret]
[rzlns]

Sentence Pattern
Spread out
Laser light does not spread out even at great distances.
.
Practical Translation Training
,.
______________________________________________________________________
Further Reading:Beginning of Drama in Ancient Greece 126
There are many theories about the beginning of drama in ancient Greece. The one most widely
accepted today is based on the assumption that drama evolved from ritual. The argument for this view
goes as follows. In the beginning, human beings viewed the natural forces of the world, even the
seasonal changes, as unpredictable, and they sought through various means, to control these unknown
and feared powers. Those measures which appeared to bring the desired results were then retained and
repeated until they hardened into fixed rituals. Eventually stories arose which explained or veiled the
mysteries of the rites. As time passed some rituals were abandoned, but the stories, later called myths,
persisted and provided material for art and drama.
Those who believe that drama evolved out of ritual also argue that those rites contained the seed of
theater because music, dance, masks, and costumes were almost always used. Furthermore, a suitable
site had to be provided for performances, and when the entire community did not participate, a clear
division was usually made between the "acting area" and the "auditorium." In addition, there were
performers, and since considerable importance was attached to avoiding mistakes in the enactment of
rites, religious leaders usually assumed that task. Wearing masks and costumes, they often
impersonated other people, animals, or supernatural beings, and mimed the desired effect - success in
hunt or battle, the coming rain, the revival of the Sun - as an actor might. Eventually such dramatic
representations were separated from religious activities.
Another theory traces the theaters origin from the human interest in storytelling. According to this
view, tales (about the hunt, war, or other feats) are gradually elaborated, at first through the use of
impersonation, action, and dialogue by a narrator and then through the assumption of each of the roles
by a different person. A closely related theory traces theater to those dances that are primarily
rhythmical and gymnastic or that are imitations of animal movements and sounds.
126

Adapted from TOEFL pBT, 1995.08, Educational Testing Services.

177

Version 3.0

[]

4.2.3 Inference
Features

____________________

Exercise 1 127 : Even the kind of stability defined as simple lack of change is not always
associated with maximum diversity. At least in temperate zones, maximum diversity is often
found in mid-successional stages, not in the climax community. Once a redwood forest
matures, for example, the kinds of species and the number of individuals growing on the
forest floor are reduced. In general, diversity, by itself, does not ensure stability.
Mathematical models of ecosystems likewise suggest that diversity does not guarantee
ecosystem stability-just the opposite. In fact, a more complicate system, in general, more
likely than a simple system to break down. (A fifteen-speed racing bicycle is more likely to
break down than a childs tricycle.)
Which of the following can be inferred from paragraph 5 about redwood forests?
A. They become less stable as they mature.
B. They support many species when they reach climax.
C. They are found in temperate zones.
D. They have reduced diversity during mid-successional stages.

Keys to Solution
Find a key word in the question and scan the passage for the key word or corresponding
idea. Read the related information carefully.
It is essential to have a thorough understanding of the passage. An inference should
always be made based on the facts stated in the text.
Do not look for answer clues outside of the passage. Personal experiences, beliefs, or
values will not help you make correct inferences. Everything you need is provided in
the passage

127

Adapted from Long term stability of ecosystem, TPO 2, Education Testing Services

178

[] Version 3.0
Vocabulary Enlargement
successional
climax

[sksnl]
[klamks]

Sentence Pattern
Associate with
You will compromise your good name if you associate with these people.
.
Practical Translation Training
.
______________________________________________________________________
Further Reading: Biological Diversity 128
Coincident with concerns about the accelerating loss of species and habitats has been a growing
appreciation of the importance of biological diversity, the number of species in a particular ecosystem,
to the health of the Earth and human well-being. Much has been written about the diversity of
terrestrial organisms, particularly the exceptionally rich life associated with tropical rain-forest
habitats. Relatively little has been said, however, about diversity of life in the sea even though coral
reef systems are comparable to rain forests in terms of richness of life.
An alien exploring Earth would probably give priority to the planets dominants, most-distinctive
feature-the ocean. Humans have a bias toward land that sometimes gets in the way of truly examining
global issues. Seen from far away, it is easy to realize that landmasses occupy only one-third of the
Earths surface. Given that two-thirds of the Earths surface is water and that marine life lives at all
levels of the ocean, the total three-dimensional living space of the ocean is perhaps 100 times greater
than that of land and contains more than 90 percent of all life on Earth even though the ocean has
fewer distinct species.
The fact that half of the known species are thought to inhabit the worlds rain forests does not seem
surprising, considering the huge numbers of insects that comprise the bulk of the species. One
scientist found many different species of ants in just one tree from a rain forest. While every species is
different from every other species, their genetic makeup constrains them to be insects and to share
similar characteristics with 750,000 species of insects. If basic, broad categories such as phyla and
classes are given more emphasis than differentiating between species, then the greatest diversity of
life is unquestionably the sea. Nearly every major type of plant and animal has some representation
there.
To appreciated fully the diversity and abundance of life in the sea, it helps to think small. Every
spoonful of ocean water contains life, on the order of 100 to 100,000 bacterial cells plus assorted
microscopic plants and animals, including larvae of organisms ranging from sponges and corals to
starfish and clams and much more.
128

Adapted from TOEFL pBT, 1995.12, Educational Testing Services.

179

Version 3.0

[]

Exercise 2: Groundwater is the word used to describe water that saturates the ground, filling
all the available spaces. By far the most abundant type of groundwater is meteoric water; this
is the groundwater that circulates as part of the water cycle. Ordinary meteoric water is water
that has soaked into the ground from the surface, from precipitation (rain and snow) and from
lakes and streams. There it remains, sometimes for long periods, before emerging at the
surface again. At first thought it seems incredible that there can be enough space in the
solid ground underfoot to hold all this water.
Which of the following can be inferred about the ground that we walk on?
A. It cannot hold rainwater for long periods of time.
B. It prevents most groundwater from circulating.
C. It has the capacity to store large amounts of water.
D. It absorbs most of the water it contains from rivers.
Exercise 3: One of the most puzzling aspects of the paintings is their location. Other rock
paintings--for example, those of Bushmen in South Africa--are either located near cave
entrances or completely in the open. Cave paintings in France and Spain, however, are in
recesses and caverns far removed from original cave entrances. This means that artists were
forced to work in cramped spaces and without sources of natural light. It also implies that
whoever made them did not want them to be easily found. Since cave dwellers normally lived
close to entrances, there must have been some reason why so many generations of Lascaux
cave dwellers hid their art.
What can be inferred about cave painters in France and Spain?
A. They also painted rocks outside caves.
B. They did not live close to the cave entrances.
C. They developed their own sources of light to use while painting.
D. Their painting practices did not last for many years.
Exercise 4: They breathe through lungs, not through gills, and given birth to live young.
Their streamlined bodies, the absence of hind legs, and the presence of a fluke and blowhole
cannot disguise their affinitieswith land-dwelling mammals. However, unlike the cases of sea
otters and pinnipeds (seals sea lions, and walruses, whose limbs are functional both on land
and at sea), it is not easy to envision what the first whales looked like.
Which of the following can be inferred from paragraph 1 about early sea otters?
A. It is not difficult to imagine what they looked like
B. There were great numbers of them
C. The lived in the sea only
D. They did not leave many fossil remains

180

[] Version 3.0
Vocabulary Enlargement
meteoric
recesses
cramped
dismantle

[,mitrk]
[riss]
[krmpt]
[dsmntl]

Sentence Pattern
Soak into
The magnitude of the problem finally soaked into his mind.

Further ReadingArchaeological Records


Archaeological recordspaintings, drawings, and carvings of humans engaged in activities
involving the use of handsindicate that humans have been predominantly right-handed for
more than 5,000 years. In ancient Egyptian artwork, for examples, the right hand is depicted
as the dominant one in about 90 percent of the example. Fracture or wear patterns on tools
also indicate that a majority of ancient people were right-handed. Cro-Magnon cave paintings
some 27,000 years old commonly show outlines of human hands made by placing one hand
against the cave wall and applying paint with the other. Children today make similar outlines
of their hands with crayons on paper. With few exceptions, left hands of Cro-Magnons are
displayed on cave walls, indicating that the paintings were usually done by right-handers.
Anthropological evidence pushes the record of handedness in early human ancestors back to
at least 1.4 million years ago. One important line of evidence comes from flaking patterns of
stone cores used in tool making: implements flaked with a clockwise motion (indicating a
right-handed toolmaker) can be distinguished from those flaked with a counter-clockwise
rotation (indicating a left-handed toolmaker).
Even scratches found on fossil human teeth offer clues. Ancient humans are thought to have
cut meat into strips by holding it between their teeth and slicing it with stone knives, as do the
present-day Inuit. Occasionally the knives slip and leave scratches on the users teeth.
Scratches made with a left-to-right stroke direction (by right-handers) are more common than
scratches in the opposite direction (made by left-handers).
Still other evidence comes from cranial morphology: scientists think that physical differences
between the right and left sides of the interior of the skull indicate subtle physical differences
between the two sides of the brain. The variation between the hemispheres corresponds to
which side of the body is used to perform specific activities. Such studies, as well as studies
of tool use, indicate that right-or left-sided dominance is not exclusive to modem Homo
sapiens. Populations of Neanderthals, such as Homo erectus and Homo habilis, seem to have
been predominantly right-handed, as we are.

181

Version 3.0

[]

4.2.4 Purpose Understanding


Classification of the Purpose
Writing meant to Inform
Impersonal means of conveying a feeling, an attitude, or an emotion
Do not use emotionally charged or highly connotative language
Relies heavily on factual statements and dont vary according to person or location
The main idea suggest no judgment
Writing meant to Persuade
Focus on developing an opinion and use facts only to serve that opinion
Likely to use emotionally charged language
Even seemingly factual statements are likely to include some opinion
The main idea takes a stand

Exercise 1 129
Celebrity photographers, also known as paparazzi, have gone too far in their quest to snap
pictures of the stars. Everyoneeven celebritiesshould have the right to privacy, yet
photographers routinely hound famous people everywhere they go. The creepiest of these
photographers are not above following their prey into bathrooms or climbing trees to get
shots of celebrities in their own homes. This constant invasion forces the rich and famous to
become rude and mean, setting a bad example for their often youthful fans. Even worse,
though, the paparazzis behavior is frequently downright dangerous and irresponsible. One
newspaper recently reported that pop singer Britney Spears was forced to drive 90 miles an
hour in an attempt to outrun relentless photographers. And who can forget that in 1997,
Englands Princess Diana was killed in a car accident while her driver tried to evade the
paparazzi in a high-speed chase. People may crave pictures of the stars, but most readers
certainly would not want them at the expense of a celebritys safety. Newspapers and
magazines should cease paying for these kinds of photos while photographers who put
celebrities in harms way should be arrested for stalking.
The authors purpose is
A. to inform
B. to persuade

129

Adapted from Michael Paul, Lenoir, NC, Paparazzi Go Much Too Far, USA Today, September 9,
2004, p. 12A

182

[] Version 3.0
Vocabulary Enlargement
paparazzi
creepy
stalking

[,pprtsi]
[kripi]
[stk]

Sentence Pattern
At the expense of
One countrys success need not come at the expense of another.

Practical Translation Training



______________________________________________________________________
______________________________________________________________________
Further ReadingPaparazzi by Lady Gaga
We are the crowd
Were c-comin out
Got my flash on its true
Need that picture of you
Its so magical
Wed be so fantastico

Chase you down until you love me


Papa-Paparazzi
Ill be your girl
Backstage at your show
Velvet ropes and guitars
Yeah, cause youre my rock star
In between the sets
Eyeliner and cigarettes

Leather and jeans


Garage Glamorous
Not sure what it means
But this photo of us it dont have a price
Ready for those flashing lights
Cause you know that baby I...

Shadow is burnt
Yellow dance and we turn
My lashes are dry
Purple teardrops I cry
It dont have a price
Loving you is cherry pie
Cause you know that baby I...

Im your biggest fan


Ill follow you until you love me
Papa-Paparazzi
Baby theres no other superstar
You know that Ill be your...
Papa-Paparazzi
Promise Ill be kind
But I wont stop until that boy is mine
Baby youll be famous

Real good, We dance in the studio


Snap Snap to that shit on the radio
Dont stop for anyone
Were plastic, but we still have fun

183

Version 3.0

[]

Exercise 2
Joy and sadness are experienced by people in all cultures around the world, but how can we
tell when other people are happy or despondent? It turns out that the expression of many
emotions may be universal. Smiling is apparently a universal sign of friendliness and
approval. Baring the teeth in a hostile way, as noted by Charles Darwin in the 19th century,
may be a universal sign of anger.
The author mentions Baring the teeth in a hostile way in order to
A. Differentiate one possible meaning of a particular facial experssion from other meanings

of it
B. Support Darwins theory of evolution
C. Provide an example of a facial expression whose meaning is widely understood
D. Contrast a facial expression that is easily understood with other facial expressions
Exercise 3
Literary criticism is the study, discussion, and interpretation of literature. Nowadays, most
literary critics use some form of literary theory to appraise novels, poems, and plays. Literary
theory is based on certain philosophical ideas that critics use when they discuss certain books
or poems. These philosophical ideas might include Marxism, feminism, or realism. Most
professional literary critics are university professors or reporters who write for literary
magazines. An example of a Marxist literary critic might be a university professor who
examines The Little Prince based upon social theories created by the economist and
philosopher Karl Marx. Therefore, he would argue that all of the characters that the Little
Prince meets in his travels act greedily because humans are basically all greedy. This is a
simplified concept of Karl Marx.
Why does the author mention Marxism, feminism, or realism?
A. To argue that philosophical perspectives must be considered in literary criticism
B. To name a few perspectives on which literary theory can be based
C. To show what philosophical ideas professional critics study
D. To prove that literary critics cannot discuss literature without philosophy
Keys to Solution
Read the question and locate the section of the passage that contains that particular
information. As you read, figure out how this information relates to the main idea. For
example, the question may deal with information like that:
Exemplifies or supports a main idea
Clarifies an unfamiliar concept that a paragraph discusses
Refutes or supports a certain argument
Compares or contrasts one thing with another
Provides a solution or answer to a problem
184

[] Version 3.0
Vocabulary Enlargement
appraise
Marxism
feminism
realism
greedily

[prez]
[ma:ksizm
[femnz()m]
[rlz()m]
[gri:dili]

Further Reading: U.S. writers in the 19th century 130


The economic depression in the late-nineteenth-century United States contributed
significantly to a growing movement in literature toward realism and naturalism. After the
1870s, a number of important authors began to reject the romanticism that had prevailed
immediately following the Civil War of 1861-1865 and turned instead to realism.
Determined to portray life as it was, with fidelity to real life and accurate representation
without idealization, they studied local dialects, wrote stories which focused on life in
specific regions of the country, and emphasized the "true" relationships between people. In
doing so, they reflected broader trends in the society, such as industrialization, evolutionary
theory which emphasized the effect of the environment on humans, and the influence of
science.
Realists such as Joel Chandler Harris and Ellen Glasgow depicted life in the South; Hamlin
Garland described life on the Great Plains; and Sarah One Jewett wrote about everyday life in
rural New England. Another realist, Bret Harte, achieved fame with stories that portrayed
local life in the California mining camps.
Samuel Clemens, who adopted the pen name Mark Twain, became the countrys most
outstanding realist author, observing life around him with a humorous and skeptical eye. In
his stories and novels, Twain drew on his own experiences and used dialect and common
speech instead of literary language, touching off a major change in American prose style.
Other writers became impatient even with realism. Pushing evolutionary theory to its limits,
they wrote of a world in which a cruel and merciless environment determined human fate.
These writers, called naturalists, often focused on economic hardship, studying people
struggling with poverty, and other aspects of urban and industrial life. Naturalists brought to
their writing a passion for direct and honest experience.
Theodore Dreiser, the foremost naturalist writer, in novels such as Sister Carrie, grimly
portrayed a dark world in which human beings were tossed about by forces beyond their
understanding or control. Dreiser thought that writers should tell the truth about human
affairs, not fabricate romance, and Sister Carrie, he said, was not intended as a piece of
literary craftsmanship, but was a picture of conditions.

130

Adapted from TOEFL pBT, 2001.08, Educational Testing Services.

185

Version 3.0

[]

Exercise 4
Paragraph 5: Even the kind of stability defined as simple lack of change is not always
associated with maximum diversity. At least in temperate zones, maximum diversity is often
found in mid-successional stages, not in the climax community. Once a redwood forest
matures, for example, the kinds of species and the number of individuals growing on the
forest floor are reduced. In general, diversity, by itself, does not ensure stability.
Mathematical models of ecosystems likewise suggest that diversity does not guarantee
ecosystem stability--just the opposite, in fact. A more complicated system is, in general, more
likely than a simple system to break down. A fifteen-speed racing bicycle is more likely to
break down than a childs tricycle
In paragraph 5, why does the author provide the information that (A fifteen-speed racing
bicycle is more likely to break down than a childs tricycle)?
A. To illustrate a general principle about the stability of systems by using an everyday
example
B. To demonstrate that an understanding of stability in ecosystems can be applied to help
understand stability in other situations
C. To make a comparison that supports the claim that, in general, stability increases with
diversity
D. To provide an example that contradicts mathematical models of ecosystems

Exercise 5
Many millions of years after ferns evolved (but long before the Hawaiian Island were born
from the sea), another kind of flora evolved on Earth: the seed-bearing plants. This was a
wonderful biological invention. The seed has an outer coating that surrounds the genetic
material of the new plant, and inside this covering is a concentrated supply of nutrients. Thus,
the speeds chances of survival are greatly enhanced over those of the naked spore. One type
of seed-bearing plant, the angiosperm, includes all forms of blooming vegetation. In the
angiosperm the seeds are wrapped in an additional layer of covering. Some of these coats are
hard-like the shell of-a nut for extra protection. Some are soft and tempting, like a peach or a
cherry. In some angiosperm the seeds are equipped with gossamer wings, like the dandelion
and milkweed seeds. These new characteristics offered better ways for the seeds to move to
new habitats. They could travel through the air, float in water, and lie dormant for many
months.
Why does the author mention a nut, a peach, and a cherry?
A. To indicate that some seeds are less likely to survive than others
B. To point out that many angiosperms can be eaten
C. To provide examples of blooming plants
D. To illustrate the variety of coverings among angiosperm seeds

186

[] Version 3.0
Vocabulary Enlargement
angiosperm
wrap
gossamer
dormant

[ndo,spm]
[rp]
[gsm]
[drmnt]

Sentence Pattern
Be equipped with
Our nation is equipped with new tools to monitor the terrorists movements, freeze
their finances and break up their plots.

Further Reading:
Growing tightly packed together and collectively weaving a dense canopy of branches, a
stand of red alder trees can totally dominate a site to the exclusion of almost everything else.
Certain species such as salmonberry and sword ferns have Line adapted to the limited
sunlight dappling through the canopy, but few evergreen trees will survive there; still fewer
can compete with the early prodigious growth of alders. A Douglas fir tree reaches its
maximum rate of growth ten years later than an alder, and if the two of them begin life at the
same time, the alder quickly outgrows and dominates the Douglas fir. After an alder canopy
has closed, the Douglas fir suffers a marked decrease in growth, often dying within seven
years. Even more shade-tolerant species of trees such as hemlock may remain badly
suppressed beneath aggressive young alders.
Companies engaged in intensive timber cropping naturally take a dim view of alders
suppressing more valuable evergreen trees. But times are changing; a new generation of
foresters seems better prepared to Include in their management plans consideration of the
vital ecological role alders, play.
Among the alders valuable ecological contributions is its capacity to fix nitrogen in nitrogendeficient soils. Alder roots contain clusters of nitrogen-fixing nodules like those found on
legumes such as beans. in addition, newly developing soils exposed by recent glacier retreat
and planted with alders show that these trees are applying the equivalent of ten bags of highnitrogen fertilizer to each hectare per year. Other chemical changes to soil in which they are
growing include a lowering of the base content and rise In soil acidity, as well as a substantial
addition of carbon and calcium to the soil.
Another important role many alders play in the wild, particularly in mountainous areas, is to
check the rush of water during spring melt. In Japan and elsewhere, the trees are planted to
stabilize soil on steep mountain slopes. Similarly, alders have been planted to stabilize and
rehabilitate waste material left over from old mines, flood deposits, and landslide areas in
both Europe and Asia.
187

Version 3.0

[]

Paragraph Practice 1
1.According to the passage, the inventors of

written language in Mesopotamia were


probably the
A.
B.
C.
D.

The Sumerian Writing

Babylonians
Subarians
Akkadians
Sumerian

2.The word subjugated in paragraph 1 is


closest in meaning to
A.
B.
C.
D.

distinguished
segregated
concentrated
conquered

3.The phrase synonymous with in paragraph1


is closest in meaning to
A.
B.
C.
D.

equivalent to
important for
respected in
familiar with

4.According to the passage, by the year 3000


B.C. the Sumerians had already done all of the
following EXCEPT:
A. They had abandoned the area north of the
Persian Gulf.
B. They had established themselves in cities.
C. They had started to communicate through
writing
D. They had created bronze tools and
weapons
5. Which of the following can be inferred from
the passage concerning the Sumerians?
A. They were descendants of the Persians
B. They were the first people to cultivate the
valley of the Tigris.
C. They were accomplished musicians.
D. They had the beginnings of an economy.
188

Archaeological discoveries have led some


scholars to believe that the first
Mesopotamian inventors of writing may
have been a people the later Babylonians
called Subarians. According to tradition,
they came from the north and moved into
Uruk in the south. By about 3100 B.C. ,
they were apparently subjugated in
southern Mesopotamia by the Sumerians,
whose name became synonymous with
the region immediately north of the Presian
Gulf, in the fertile lower valleys of the
Tigris and Euphrates. Here the Sumerians
were already well established by the year
3000 B.C. They had invented bronze, an
alloy that could be cast in molds, out of
which they made tools and weapons. They
lived in cities, and they had begun to
acquire and use capital. Perhaps most
important, the Sumerians adapted writing
(probably from the Subarians) into a
flexible tool of communication.

[] Version 3.0
Paragraph Practice 2
1. According to the author , all of the following
evidence relating to the first birds was found
EXCEPT
A. nesting materials
B. four skeletons in good condition
C. two fragmented skeletons
D. a single feather
2.
A.
B.
C.
D.

The word preserved is closest in meaning to


confused with others
gradually weakened
protected from destruction
lost permanently

3. It can be inferred from the passage that the


Archaeopteryx were classified as birds on the
basis of
A. imprints of bones
B. imprints of feathers
C. the neck structure
D. skeletons
4.
A.
B.
C.
D.

The word they refers to


indications
fossils
dinosaurs
characteristics

5.
A.
B.
C.

Why does the author mention a crow ?


to indicate the size of Archaeopteryx
to specify the age of the Archaeopteryx fossils
to explain the evolutionary history of
Archaeopteryx
D. to demonstrate the superiority of the theropod
to Archaeopteryx

6. It can be inferred from the passage that


theropods were
A. dinosaurs
B. birds
C. Archaeopteryx
D. crows

189

Early Birds

The first birds appeared during late


Jurassic times. These birds are known from
four very good skeletons, two incomplete
skeletons, and an isolated feather, all from
the Solnhofen limestone of Bavaria,
Germany. This fine-grained rock, which is
extensively quarried for lithographic stone,
was evidently deposited in a shallow coral
lagoon of a tropical sea, and flying
vertebrates occasionally fell into the water
and were buried by the fine limy mud, to be
preserved with remarkable detail. In this
way, the late Jurassic bird skeletons, which
have been named Archaeopteryx, were
fossilized. And not only were the bones
preserved in these skeletons, but also were
imprints of the feathers. If the indications of
feathers had not been preserved in
association with Archaeopteryx, it is likely
that these fossils would have been classified
among the dinosaurs, for they show
numerous
theropod
characteristics.
Archaeopteryx were animals about the size
of a crow, with an archeosaurian type of
skull, a long neck, a compact body
banlanced on a pair of strong hind limbs,
and a long tail. The forelimbs were enlarged
and obviously functioned as wings.

Version 3.0

[]

Passage Organization
5.1 Organization Patterns
5.1.1 Time Order
5.1.1.1 Process Pattern
Writers use the process pattern to explain step by step how something functions, develops, or
occurs.
Exercise 1 131
Rain and warm weather stimulate the growth of seeds. When water enters a seed, the seed
leaves get wet and start to swell. Then the seed cover splits, allowing the root to grow out of
the seed. Next, the root grows downward and absorbs water. Finally, the plant grows up out
of the ground, where it is exposed to sunshine.
Process: the growth of seeds
Transitional Clues: 3

Transitional Words
After
After a while
Afterward
As a result
At the end
At this point
During this stage (phase)

Finally
First
Following the step
In a few days
In the end
In the final stage (phase)
In the first stage (phase)

Next
Now
Once
Second/third
Then
Within days
Within minutes

Exercise 2 132
Moths pass through four distinct stages of development: egg, larva, pupa, and adult. The
process begins when the female lays her eggs. Within a few days, the eggs hatch and the
larvae appear. During this stage, the eggs and larvae are often eaten by birds and other insects.
In the pupa stage, wings begin to form and reproductive organs begin to develop. At this
point many larvae spin cocoons for protection. In the final stage, the insects leave the
cocoons. Within minutes, they expand their wings and are ready to fly.
Process:
Transitional Clues:
131
132

Adapted from Tanzer, Biology and Human Progress, p.313


Adapted from Curitis, The Second Nature of Things, p.156

190

[] Version 3.0
Vocabulary Enlargement
moth
larva
pupa
hatch
cocoon

[m]
[lrv]
[pjup]
[ht]
[kkn]

[] []
[]

Sentence Pattern
Pass through
A caterpillar must pass through the cocoon stage to become a butterfly.
.

Further Reading: Eusocial insects 133


What unusual or unique biological train led to the remarkable diversification and
unchallenged success of the ants forever 50 million years? The answer appears to be that they
were the first group of predatory eusocial insects that both lived and foraged primarily in the
soil and in rotting vegetation on the ground. Eusocial refers to a form of insect society
characterized by specialization of tasks and cooperative care of the young; it is rare among
insects. Richly organized colonies of the land made possible by eusociality enjoy several key
advantages over solitary individuals.
Under most circumstances groups of workers are better able to forage for food and defend the
nest, because they can switch from individual to group response and back again swiftly and
according to need. When a food object or nest intruder is too large for one individual to
handle, nestmates can be quickly assembled by alarm or recruitment signals. Equally
important is the fact that the execution of multiple-step tasks is accomplished in a seriesparallel sequence. That is, individual ants can specialize in particular steps, moving from one
object (such as a larva to be fed) to another (a second larva to be fed). They do not need to
carry each task to completion from start to finish. for example, to check the larva first, then
collect the food, then feed the larva. Hence, if each link in the chain has many workers in
attendance, a sense directed at any particular object is less likely to fail. Moreover, ants
specializing in particular labor categories typically constitute a caste specialized by age or
body form or both. There has been some documentation of the superiority in performance and
net energetic yield of various castes for their modal tasks, although careful experimental
studies are still relatively few. What makes ants unusual in the company of eusocial insects is
the fact that they are the only eusocial predators (predators are animals that capture and feed
on other animals) occupying the soil and ground litter. The eusocial termites live in the same
places as ants and also have wingless workers, but they feed almost exclusively on dead
vegetation.
133

Adapted from TOEFL pBT, 2001.10, Educational Testing Services

191

Version 3.0

[]

5.1.1.2 Sequence of Dates and Events


Sequence of Dates and Events introduced in the order in which they occurred in real time.
Key Features
Trace a series of dates and events considered remarkable or unusual.
Make some general point about an event, a career, or a life.
Describe the events leading up to or following a significant historical happening.

Example 1 134
Between 1993 and 2001, the United States became intimately acquainted with the horrors of
terrorism. On February 16, 1993, the first terrorist attack on U.S. soil took place when a truck
bomb exploded at the World Trade Center in New York City. On April 19, 1995, a
homemade bomb exploded outside the Murrah Federal Office Building in Oklahoma City,
killing 168 people. On November 13, 1995, a bomb left in a van parked at U.S. military
headquarters in Riyadh, Saudi Arabia, exploded and killed 7 people. On June 25, 1996, a
truck bomb exploded at U.S. military headquarters in Dhahran, Saudi Arabia, killing 19
Americans. On August 7, 1998, two more truck bombs exploded at U.S. embassies in East
Africa, killing 258 and injuring over 5,000. On October 12, 2000, a small boat laden with
explosives blew up alongside the navy ship USS Cole, killing 17 U.S. sailors. And finally, on
September 11, 2001, the World Trade Center was destroyed and the Pentagon damaged when
terrorists hijacked four airliners and succeeded.

02/16/1993
04/19/1995
11/13/1995
06/25/1996
08/07/1998
10/12/2000
09/11/2001

Truck bomb explodes at WTC


Bomb explode outside Oklahomas MFOB and kills 168
Bomb explodes at Riyadh military headquarters and kills 7
Truck bomb explodes in Dhahran, Saudi Arabia, killing 19
Two truck bombs kill 258 and injure over 5,000
Boat blows up alongside the USS Cole and kills 17
WDC is destroyed and Pentagon damaged by terrorists

Transitional Words
Before/after/by the _____ century
At a later time
Between ____ and _____
By the year _____
From _____ to _____

134

Then in _______
On/in the days/weeks/months
Folloing ________
In the years since _______
Until _______

Adapted from Susan Page, A decade of Terrorism, USA Today, Nov.12, 2001, p.9A.

192

[] Version 3.0
Further Reading: September 11 attacks
At 8:46 a.m., five hijackers crashed American Airlines Flight 11 into the World Trade Centers North
Tower (1 WTC), and at 9:03 a.m., another five hijackers crashed United Airlines Flight 175 into the
South Tower (2 WTC). Five hijackers flew American Airlines Flight 77 into the Pentagon at 9:37 a.m.
A fourth flight, United Airlines Flight 93, under the control of four hijackers, crashed near
Shanksville, Pennsylvania, southeast of Pittsburgh, at 10:03 a.m. after the passengers fought the
hijackers. Flight 93s target is believed to have been either the Capitol or the White House. Flight 93s
cockpit voice recorder revealed crew and passengers tried to seize control of the plane from the
hijackers after learning through phone calls that similarly hijacked planes had been crashed into
buildings that morning. Once it became evident to
the hijackers that the passengers might regain
control of the plane, the hijackers rolled the plane
and intentionally crashed it.
Some passengers and crew members who called
from the aircraft using the cabin airphone service
and mobile phones provided details: several
hijackers aboard each plane; they used mace, tear
gas, or pepper spray to overcome attendants; and
some people aboard had been stabbed. Reports
indicated hijackers stabbed and killed pilots, flight
attendants, and one or more passengers. In their
final report, the 9/11 Commission found the
hijackers had recently purchased multi-function
hand tools and assorted knives and blades. A flight
attendant on Flight 11, a passenger on Flight 175,
and passengers on Flight 93 said the hijackers had
bombs, but one of the passengers said he thought the bombs were fake. The FBI found no traces of
explosives at the crash sites, and the 9/11 Commission concluded that the bombs were probably fake.
Three buildings in the World Trade Center complex collapsed due to fire-induced structural failure.
The South Tower collapsed at 9:59 a.m. after burning for 56 minutes in a fire caused by the impact of
United Airlines Flight 175 and the explosion of its fuel. The North Tower collapsed at 10:28 a.m.
after burning for 102 minutes. When the North Tower collapsed, debris fell on the nearby 7 World
Trade Center building (7 WTC), damaging it and starting fires. These fires burned for hours,
compromising the buildings structural integrity, and 7 WTC collapsed at 5:21 p.m. The Pentagon
sustained major damage.
At 9:40 a.m., the FAA grounded all aircraft within the continental U.S., and aircraft already in flight
were told to land immediately. All international civilian aircraft were either turned back or redirected
to airports in Canada or Mexico, and all international flights were banned from landing on U.S. soil
for three days. The attacks created widespread confusion among news organizations and air traffic
controllers. Among the unconfirmed and often contradictory news reports aired throughout the day,
one of the most prevalent said a car bomb had been detonated at the U.S. State Departments
headquarters in Washington, D.C. Another jetDelta Air Lines Flight 1989was suspected of
having been hijacked, but the aircraft responded to controllers and landed safely in Cleveland, Ohio.
193

Version 3.0

[]

5.1.2 Classification
Classification pattern divide a larger group into smaller categories. Then they define and
describe each category, as the author of the following passage has done.
Key Features
The topic sentence usually announces that a large group can be divided into smaller
subgroups.
Classification topic sentences frequently identify the method used to create the categories.
Classification paragraphs always describe each category mentioned in the topic sentence.
Exercise 1
A symbiotic relationship is an interaction between two or more species in which one species
lives in or on another species. There are three main types of symbiotic relationships:
parasitism, commensalism, and mutualism. The first and the third can be key factors in the
structure of a biological community; that is, all the populations of organisms living together
and potentially interacting in a particular area. Parasitism is a kind of predator-prey
relationship in which one organism, the parasite, derives its food at the expense of its
symbiotic associate, the host. In commensalism, one partner benefits without significantly
affecting the other. The third type of symbiosis, mutualism, benefits both partners in the
relationship.
Q. What larger group is divided into smaller subgroups or categories?
Q. How many subgroups are mentioned?
Q. Name and describe each subgroup

Exercise 2
There is still no universally agreed-on explanation for why there should be such a dramatic
cessation of tree growth at the upper timberline. Various environmental factors may play a
role. Too much snow, for example, can smother trees, and avalanches and snow creep can
damage or destroy them. Late-lying snow reduces the effective growing season to the point
where seedlings cannot establish themselves. Wind velocity also increases with altitude and
may cause serious stress for trees, as is made evident by the deformed shapes at high altitudes.
Some scientists have proposed that the presence of increasing levels of ultraviolet light with
elevation may play a role, while browsing and grazing animals like the ibex may be another
contributing factor. Probably the most important environmental factor is temperature, for if
the growing season is too short and temperatures are too low, tree shoots and buds cannot
mature sufficiently to survive the winter months.
194

[] Version 3.0
Vocabulary Enlargement
parasitism
[prsatzm]
commensalism [kmnslzm]
mutualism
[mjutlz()m]

Further Reading: Burning theory 135


A seventeenth-century theory of burning proposed that anything that burns must contain
material that the theorists called "phlogiston." Burning was explained as the release of
phlogiston from the combustible material to the air. Air was thought essential, since it had to
provide a home for the released phlogiston. There would be a limit to the phlogiston transfer,
since a given volume of air could absorb only so much phlogiston. When the air had become
saturated, no additional amounts of phlogiston could leave the combustible substance, and the
burning would stop. Burning would also stop when the combustible substance was emptied of
all its phlogiston.
Although the phlogiston theory was
self-consistent, it was awkward because
it required that imaginative, even
mysterious, properties be ascribed to
phlogiston. Phlogiston was elusive. No
one had ever isolated it and
experimentally
determined
its
properties. At times it seemed to show a
negative weight: the residue left after
burning weighed more than the material
before burning. This was true, for
example, when magnesium burned.
Sometimes phlogiston seemed to show a positive weight, when, for example, wood burned,
the ash weighed less than the starting material. And since so little residue was left when
alcohol, kerosene, or high-grade coal burned, these obviously different materials were
thought to be pure or nearly pure phlogiston.
In the eighteenth century, Antoine Lavoisier, on the basis of careful experimentation, was led
to propose a different theory of burning, one that required a constituent of air- later shown to
be oxygen- for combustion. Since the weight of the oxygen is always added, the weight of the
products of combustion, including the evolved gases, would always be greater than the
weight of the starting material.
Lavoisiers interpretation was more reasonable and straightforward than that of the
phlogiston theorists. The phlogiston theory, always clumsy, became suspect, eventually fell
into scientific disrepute, and was replaced by new ideas.
135

Adapted from TOEFL pBT, 1998.08, Educational Testing Services.

195

Version 3.0

[]

5.1.3 Definition
Definition pattern usually begin with the word or term being defined.
Key Features
The definition of a key term is the topic sentence of a paragraph or larger unit
At least one example is provided for the definition
Theres also a sentence or two explaining how the term differs from other similar terms
Exercise 1 136
Litigation is an adversarial kind of communication in which a dispute is settled by a judge,
who decides who is lying. Litigation is usually conducted by lawyers who call witnesses and
question them to reveal information. Litigants must follow a prescribed set of rules that have
been established by law and interpreted by the person in charge, usually a judge.
What term is defined in the paragraph?
In your own words, what is the definition of that term?
Describe the example that the author uses to illustrate the definition

Exercise 2 137
Phishing is a computer term for the act of tricking people into revealing information like
passwords and credit card numbers. People who phish set up seemingly legitimate websites
and e-mail addresses. Then they send out e-mail messages that request information for some
official purpose. In this way, online scam artists fish for details they can use for criminal
activities. For example, in the first phishing attacks in the mid-1990s, con artists posing as
AOL staff members sent messages to potential victims to ask them to verify your account
or confirm billing information. Once victims replied and sent back their AOL passwords,
the con artist could use the accounts for illegal purposes.
What term is defined in the paragraph?
In your own words, what is the definition of that term?
Describe the example that the author uses to illustrate the definition

136
137

Adapted from Berko, Wolvin, and Wolvin, Communicating, p.198.


Adapted from http://en.wikipedia.org/wiki/phishing

196

[] Version 3.0
Vocabulary Enlargement
litigation
phishng
scam

[ltge()n]
[f]
[skm]

Further Reading: Early phishing on AOL 138


Phishing on AOL was closely associated with the warez community that exchanged pirated
software and the hacking scene that perpetrated credit card fraud and other online crimes.
After AOL brought in measures in late 1995 to prevent using fake, algorithmically generated
credit card numbers to open accounts, AOL crackers resorted to phishing for legitimate
accounts and exploiting AOL.
A phisher might pose as an AOL staff member and send an instant message to a potential
victim, asking him to reveal his password. In order to lure the victim into giving up sensitive
information the message might include imperatives such as verify your account or confirm
billing information. Once the victim had revealed the password, the attacker could access
and use the victims account for fraudulent purposes or spamming. Both phishing and
warezing on AOL generally required custom-written programs, such as AOHell. Phishing
became so prevalent on AOL that they added a line on all instant messages stating: "no one
working at AOL will ask for your password or billing information", though even this didnt
prevent some people from giving away their passwords and personal information if they read
and believed the IM first. A user using both an AIM account and an AOL account from an
ISP simultaneously could phish AOL members with relative impunity as internet AIM
accounts could be used by non-AOL internet members and could not be actioned (i.e.reported to AOL TOS department for disciplinary action.)
Eventually, AOLs policy enforcement with respect to phishing and warez became stricter
and forced pirated software off AOL servers. AOL simultaneously developed a system to
promptly deactivate accounts involved in phishing, often before the victims could respond.
The shutting down of the warez scene on AOL caused most phishers to leave the service.
The term phishing is said to have been coined by well known mid-1990s spammer and
hacker Khan C Smith, and its use was quickly adopted by warez groups throughout AOL.
AOL enforcement would detect words used in aol chat rooms to suspend the accounts
individuals involved in pirating software and trading stolen accounts. The term was used
because <>< is the single most common tag of HTML that was found in all chat transcripts
naturally, and as such could not be detected or filtered by AOL staff. The symbol <>< was
replaced for any wording that referred to stolen credit cards, accounts, or illegal activity.
Since the symbol looked like a fish, and due to the popularity of phreaking, it was adapted as
phishing.

138

Adapted from http://en.wikipedia.org/wiki/Phishing

197

Version 3.0

[]

5.1.4 Cause and Effect


It explains how one thing, idea, or event leads to or produces another.
Key Features
Paragraph relying on the cause and effect pattern explain how one event led to or
produced another
The cause and effect pattern may focus on causes, effects, or a mix of both
Exercise 1
Although southem Maya areas received more rainfall than northern areas, problems of water
were paradoxically more severe in the wet south. While that made things hard for ancient
Maya living in the south, it has also made things hard for modem archaeologists who have
difficulty understanding why ancient droughts caused bigger problems in the wet south than
in the dry north. The likely explanation is that an area of underground freshwater underlies
the Yucatan Peninsula, but surface elevation increases from north to south, so that as one
moves south the land surface lies increasingly higher above the water table. In the northern
peninsula the elevation is sufficiently low that the ancient Maya were able to reach the water
table at deep sinkholes called cenotes, or at deep caves. In low-elevation north coastal areas
without sinkholes, the Maya would have been able to get down to the water table by digging
wells up to 75 feet (22 meters) deep. But much of the south lies too high above the water
table for cenotes or wells to reach down to it. Making matters worse, most of the Yucatan
Peninsula consists of karst, a porous sponge-like limestone terrain where rain runs straight
into the ground and where little or no surface water remains available.
Exercise 2
In addition to exploring the possible antecedents of theater, scholars have also theorized
about the motives that led people to develop theater. Why did theater develop, and why was it
valued after it ceased to fulfill the function of ritual? Most answers fall back on the theories
about the human mind and basic human needs. One, set forth by Aristotle in the fourth
century B.C., sees humans as naturally imitativeas taking pleasure in imitating persons,
things, and actions and in seeing such imitations. Another, advanced in the twentieth century,
suggests that humans have a gift for fantasy, through which they seek to reshape reality into
more satisfying forms than those encountered in daily life. Thus, fantasy or fiction (of which
drama is one form) permits people to objectify their anxieties and fears, confront them, and
fulfill their hopes in fiction if not fact. The theater, then, is one tool whereby people define
and understand their world or escape from unpleasant realities.
Which of the following best describes the organization of paragraph 5?
(A) The author presents two theories for a historical phenomenon.
(B) The author argues against theories expressed earlier in the passage.
(C) The author argues for replacing older theories with a new one.
(D) The author points out problems with two popular theories.
198

[] Version 3.0
Vocabulary Enlargement
pretzel
munch
intermix
fetus
morphine
heroin
methadone
aspirin

[prets()l]
[mnt]
[ntmks]
[fits]
[mrfin]
[hron]
[mdon]
[sp()rn]

Further Reading
Ethology is concerned with the study of adaptive, or survival, value of behavior and its
Evolutionary history. Ethological theory began to be applied to research on children in the
1960s but has become even more influential today. The origins of ethology can be traced to
the work of Darwin. Its modern foundations were laid by two European zoologists, Konrad
Lorenz and Niko Tinbergen.
Watching the behaviors diverse animal species in their natural habitats, Lorenz, and
Tinbergen observed behavior patterns that promote survival. The most well-known of these is
imprinting, the carly following behavior of certain baby birds that ensures that the young will
stay close to their mother and be fed and protected from danger. Imprinting takes place
during an early, restricted time period of development. If the mother goose is not present
during this time, but an object resembling her in important features is, young goslings may
imprint on it instead.
Observations of imprinting led to major concept that has been applied in child Development
the critical period. It refers to a limited times span during which the child is biologically
prepared to acquire certain adaptive behaviors but needs the support of suitably stimulating
environment. Many researchers have conducted studies to find out whether complex
congnitive and social behaviors must be learned during restricted time periods. for example,
if children are deprived of adequate food or physical and social stimulation during the early
years of life, will their intelligence be permanently impaired? If language is not mastered
during the preschool years, is the childs capacity to acquire it reduced?
Inspired by observations of imprinting, in 1969 the British psychoanalyst John Bowlby
applied ethological theory to the understanding of the relationship between an infant and its
parents. He argued that attachment behaviors of babies, such as smiling, babbling, grasping,
and crying, are built-in social signals that encourage the parents to approach, care for, and
interact with the baby. By keeping a parent near, these behaviors help ensure that the baby
will be fed, protected from danger, and provided with the stimulation and affection necessary
for healthy growth. The development of attachment in human infants is a lengthy process
involving changes in psychological structures that lead to a deep affectional tie between
parent and baby.
199

Version 3.0

[]

5.1.5 Comparison and Contrast


Comparison refers to looking for similarities;
Contrast refers to looking for differences
Key Features
Paragraph based on the comparison and contrast pattern may mention both similarities
and differences. However, they can also focus solely on one or the other.

Example 139
The source of Roman obsession with unity and cohesion may well have lain in the pattern of
Romes early development. Whereas Greece had grown from scores of scattered cities, Rome
grew from one single organism. While the Greek world had expanded along the
Mediterranean seas lanes, the Roman world was assembled by territorial conquest. Of course,
the contrast is not quite so stark: in Alexander the Great the Greeks had found the greatest
territorial conqueror of all time; and the Romans, once they moved outside Italy, did not fail
to learn the lessons of sea power. Yet the essential difference is undeniable. The Key to the
Greek world lay in its high-powered ships; the key to Roman power lay in its marching
legions. The Greeks were wedded to the sea; the Romans, to the land. The Greek was a sailor
at heart; the Roman, a landsman.
This paragraph
A. compares two topics
B. contrasts two topics
C. compares and contrasts two topics
What two topics are compared and/or contrasted?

List any similarities mentioned

List any differences mentioned

139

Adapted from Ancient Rome and Greece, TPO 7, Educational Testing Services.

200

[] Version 3.0
Exercise 1
Want to know how important nurses are to health care? Then consider these statistics. A 2002
nationwide study found that patient complications increased every time a nurse had to
increase the normal patient load by one single person. Mortality also rose by 7 per cent. In a
similar study, University of Utah researcher Susan Horn found that a nursing staff decrease
forced convalescent home nurses to spend less than fifteen minutes per day with their patients.
What was the result? The incidence of bed sores, falls, and infections quickly began to climb.
When more nurses were hired, time spent with patients increased to thirty or forty minutes
per day, and patient outcomes improved dramatically. As these statistics suggest, nurses are
the heart and soul of hospital and nursing-home care. Yet the nursing shortage continues to
grow as trained nurses suffer from burnout, and nursing schools turn away new applicants
due to a lack of adequate funding. The fewer nurses tending the sick the more complications
and deaths will continue to rise. The American public and its representatives need to take
heed and do something about the dangerous problem currently haunting U.S. health care, or
we will all suffer the consequences.
Primary Pattern:
A. Definition
B. Time order: process
C. Time order: dates and events
D. Cause and effect
E. Comparison and contrast
F. Classification
Exercise 2
The notion of the third-person effect was first expressed by researcher W. P. Davidson in
the journal Public Opinion Quarterly. According to Davidson, people generally underestimate
the effects of the media on themselves. They are far more concerned about the medias
impact on others. The assumption is that other people will be negatively affected or
influenced by pornography, music lyrics, and television violence. This worry about the
medias effect on others is often the basis for imposing censorship. In other words, those
favoring censorship usually think it would benefit others. They assume that they themselves
could not be so easily influenced.
Primary Pattern:
A. Definition
B. Time order: process
C. Time order: dates and events
D. Cause and effect
E. Comparison and contrast
F. Classification

201

Version 3.0

[]

5.2 Additional Practice


Exercise 1: The Bald Eagle 140
When the bald eagle became the national symbol of the United States in 1782, soon after the
country was born, it is estimated that there were as many as 75,000 nesting pairs in North
America. By the early 1960s, however, the number of nesting pairs had been reduced to only
around 450.
The demise of the bald eagle is generally attributed to the effects of the pesticide DDT
(dichlorodiphenyl-trichloroethane). This pesticide was used to kill insects harmful to
agriculture, thereby increasing agricultural production. One unintended negative effects of the
use of DDT was that, while it did get rid of the undesirable insects, it also made its way along
the food chain into fish, a favorite food source of the bald eagle.
The bald eagle is now protected by federal laws. It was originally protected by the Bald Eagle
Act of 1940 and later by the Endangered Species Act of 1973. However, it is not just the laws
directly related to endangered species that aided in the resurgence of the bald eagle; its
resurgence has also been widely attributed to the banning of DDT in 1972. Today there are
more than 5,000 pairs of bald eagles, a tenfold increase over the low point of 450, and the
bird was removed from the list of endangered species in July, 1999.
Question: An introductory sentence for a brief summary of the passage is provided below.
Complete the summary by selecting the THREE answer choices that express the most
important ideas in the passage. Some sentences do not belong in the summary because they
express ideas that are not presented in the passage or are minor ideas in the passage.
This passage discusses radical shifts in population that the bald eagle has undergone.

Answer Choices (choose 3 to complete the chart):


a. The numbers of bald eagles was greatly reduced by the 1960s, at least in part due to the
effects of a pesticide.
b. The pesticide DDT was successful in removing undesirable insects.
c. The bald eagle was named as the national symbol of the United States in the late 18th
century.
d. A certain pesticide had a negative effect on the number of bald eagles.
e. Two different pieces of legislation were enacted thirty-three years apart.
f. Legislation was specifically designed to protect the bald eagles as well as to outlaw the
pesticide DDT.
140

Adapted from Deborah Phillips, Longman Preparation Course for the TOEFL Test: iBT

202

[] Version 3.0
Exercise 2: Types of Seismic Waves 141
Seismic waves are waves that travel through Earth as a consequence of an earthquake. There
are two types of seismic waves. One is the body wave, which travels internally through the
Earth, and the other is the surface wave, which travels just under Earths exterior.
The initial tremors experienced during an earthquake can be attributed to body waves, which
are comprised of both P waves and S waves. The paths these two waves journey are
dependent on both the composition and density of Earth. P waves elicit alternate
compressions and dilations of the ground Meanwhile, S waves are busy displacing the Earth
in a transverse fashion.
Surface waves move with a low frequency, long duration, and large amplitude. They are
culpable for an earthquakes corresponding destructiveness. There are two types of surface
waves; The Rayleigh wave causes the ground to roll like ripples of water, and the Love wave
causes the horizontal sheering of the ground.
Question: An introductory sentence for a brief summary of the passage is provided below.
Complete the summary by selecting the THREE answer choices that express the most
important ideas in the passage. Some sentences do not belong in the summary because they
express ideas that are not presented in the passage or are minor ideas in the passage.
Earthquakes result in seismic waves

Answer Choices (choose 3 to complete the chart):


1. Surface waves are comprised of Rayleigh waves and Love waves that cause surface
displacement.
2. Seismic waves can be either body waves or surface waves.
3. P waves and S waves compress and dilate the ground repeatedly to cause tremors.
4. An earthquakes first tremors are caused by two types of body waves: P waves and S
waves.
5. There are four major types of seismic waves.
6. Fault lines mark areas that are predisposed to earthquake activity.

141

Adapted from Timothy Hall et al., How to master skills for the TOEFL iBT Reading advanced, Darakon Press

203

Version 3.0

[]

Exercise 3: Winslow Homer 142


Winslow Homer, one of the most prominent 19th century painters, was responsible for
raising watercolor to its position as an important medium in American art. Homer was a
master of watercolor, and his best watercolor paintings equal his larger oil paintings in both
structure and intensity. Through long practice, Homer understood and exploited the
requirements of watercolor, which he applied where most appropriateto the recording of
immediate experience. He had great powers of visual analysis and never looked at a scene
without seeing its underlying structure.
Some of Homers watercolors of the Adirondack woods, with their complicated weaving of
vertical tree trunks against a background of deep autumnal tones, are demonstrations of
masterful completeness. In one particular Adirondack painting. The Blue Boat (1892), all
elements come together with perfect unity: the deep blue of the boats hull, the green and
gold landscape, the alertness of the fishermen, the brilliant clouds and their reflections on the
water. Furthermore, its design unites the structural elements with the artists enjoyment of
marking and coloring the paperall are blended as though in a single moment of vision and
action.
Question: An introductory sentence for a brief summary of the passage is provided below.
Complete the summary by selecting the THREE answer choices that express the most
important ideas in the passage. Some sentences do not belong in the summary because they
express ideas that are not presented in the passage or are minor ideas in the passage.
The painter Winslow Homer made watercolor an important medium in American art.

Answer Choices (choose 3 to complete the chart):


a. Homer was a master of watercolor and used it to record immediate experience.
b. Homer is best known for his dramatic oil paintings of seascapes.
c. His understanding of structure is shown in watercolors of masterful completeness.
d. Winslow Homer was born in Massachusetts in 1836.
e. The Blue Boat is a watercolor painting of fishermen in a boat on the water.
f. Homers Adirondack watercolors combine structural elements and color in perfect
unity.

142

Adapted from Nancy Gallagher, Deltas key to the next generation TOEFL test: Advanced skill
practice for the iBT, Delta Publishing Company.

204

[] Version 3.0
Exercise 4: William Faulkner
Author William Faulkner is today recognized as one of Americas greatest writers on the basis of
a body of novels that so convincingly portray the culture of the South in the years following the Civil
War, with its citizens overcome by grief and defeat and trying to cling to old values while struggling to
take their place in a changing world. The acclaim that today is Faulkners, however, was slow in
coming.
Though Faulkner was praised by some critics and reviewers during the first part of his career, his
novels did not sell well and he was considered a fairly marginal author. For the first few decades of his
career, he made his living writing magazine articles and working as a screenwriter rather than as a
novelist. Throughout this period, he continued to write, though his novel, sometimes noted for the
stirring portrait that they presented of life in the post-Civil War South, were generally relegated to the
category of strictly regional writing and were not widely appreciated.
Beginning in 1946, Faulkners career took an unexpected and dramatic turn as Faulkner came to be
recognized as considerably more than a reginal writer. The Portable Faulkner was published in that
year by Viking Press; two years later he was elected to the prestigious National Academy of Arts and
Letters; he was awarded the Nobel Prize for literature in 1949. Over the next decade, his work was
recognized in various ways, including a National Book Award and two Pulitzer Prizes, and he became
a novelist in residence at the University of Virginia. His success led to a degree of affluence that
enabled him to take up the life of a southern gentleman, including horseback riding and fox hunting.
Ironically, he died as a result of an accident related to these gentlemenly pursuits, succumbing as a
result of injuries suffered during a fall from a horse.

Directions: Select the appropriate phrases from the answer choices, and match them to the phase of
William Faulkner s career to which they relate. TWO of the answer choices will not be used. This
question is worth 3 points.
Faulkner in the
first phase of his
career

Faulkner in the
second phase of
his career

Answer Choices (choose 6 to complete the chart):


a.
b.
c.
d.
e.
f.
g.
h.

Was considered one of Americas greatest writers


Received a small amount of critical acclaim
Died as a result of a horseback-riding incident
Received numerous awards and acclaim
Was considered merely a regional writer
Wrote novels about various American regions
Made his living as a novelist
Made his living with writing other than novels

205

Version 3.0

[]

Exercise 5: Aggression
Aggression behavior is any behavior that is intended to cause injury, pain, suffering, damage, or
destruction. While aggressive behavior is often thought of as purely physical, verbal attacks such as
screaming and shouting or belittling and humiliating comments aimed at causing harm and
suffering can also be a type of aggression. What is key to definition of aggression is that whenever
harm is inflicted, be it physical or verbal, it is intentional.
Question about th causes of aggression have long been of concern to both social and biological
scientists. Theories about the causes of aggression cover a broad spectrum, ranging from those with
biological or instinctive emphases to those that portray aggression as a learned behavior.
Numerous theories are based on the idea that aggression is an inherent and natural human instinct.
Aggression has been explained as an instinct that is directed externally toward others in a process
called displacement, and it has been noted that aggressive impulses that are not channeled toward a
specific person or group may be expressed indirectly through socially acceptable activities such as
sports and competition in a process called catharsis. Biological, or instinctive, theories of
aggression have also been put forth by ethologists, who study the behavior of animal in
their natural environments. A number of ethologists have, based upon their observations of animal,
supported the view that aggression is an innate instinct common to human.
Two different schools of thought exist among those who view aggression as instinct. One group
holds the view that aggression can build up spontaneously, with without outside provocation, and
violent behavior will thus result, perhaps as a result of little or no provocation. Another suggests
that aggression is indeed an instinctive response but that, rather than occurring spontaneously and
without provocation, it is a direct response to provocation from an outside source.
In contrast to instinct theories, social learning theories view aggression as a learned behavior. This
approach focuses on the effect that role models and reinforcement of behavior have on the
acquisition of aggressive behavior. Research has shown that aggressive behavior can be learned
through a combination of modeling and positive reinforcement of the aggressive behavior and that
children are influenced by the combined forces of observing aggressive behavior in parents, pears,
or fictional role models and of noting either positive reinforcement for the aggressive behavior or,
minimally, a lack of negative reinforcement for the behavior. While research has provided evidence
that the behavior of a live models is more influential than that of a fictional model, fictional models
of aggressive behavior such as those seen in movies and on television, do still have an impact on
behavior. On-screen deaths or acts of violent behavior in certain television programs or movies can
be counted in tens, or hundreds, or even thousands; while some have argued that this sort of
fictional violence does not in and of itself cause violence and may even have a beneficial cathartic
effect, studies have shown correlations between viewing of violence and incidences of aggressive
behavior in both childhood and adolescence. Studies have also shown that it is not just the modeling
of aggressive behavior in either its real life or fictional form that correlates with increased acts of
violence in youths; a critical factor in increasing aggressive behavior is the reinforcement of the
behavior. If the aggressive role model is rewarded rather than punished for violent behavior, that
behavior is more likely to be seen as positive and is thus more likely to be imitated.

206

[] Version 3.0
1. Directions: Select the appropriate phrases from the answer choices, and match them to the
theories to which they relate. TWO of the answer choices will not be used. This question is
worth 3 points.

Theories attributing aggression to


instinct

Theories attributing aggression to


learned behaviors

Answer Choices(chose 4 to complete the chart):


(1)Aggression occurs in response to rewards for aggressive behavior.
(2) Aggression occurs without outside provocation
(3) Aggression occurs in order to provoke confrontations.
(4) Aggression occurs in response to observed behavior.
(5) Aggression occurs in response to negative reinforcement of aggressive behavior.
(6) Aggression occurs as a natural response to provocation.
2.
This passage discusses causes of aggression.

Answer Choices (chose 4 to complete the chart):


(1) Aggression may be learned behavior that occurs in order to model aggression from others.
(2) Aggression may be instinctive behavior that occurs without provocation.
(3) Aggression may be learned behavior that occurs in response to observed behavior.
(4) Aggression may be learned behavior that occurs in order to provoke others.
(5) Aggression may be learned behavior that occurs in response to rewards for aggressive behavior.
(6) Aggression may be instinctive behavior that occurs in response to provocation.

207

Version 3.0

[]

Approaching Themes
6.1 Natural and Physical Science
6.1.1 Desert Formation
The deserts, which already occupy approximately a fourth of the Earths land surface, have in
recent decades been increasing at an alarming pace. The expansion of desert like conditions
into areas where they did not previously exist is called desertification. It has been estimated
that an additional one-fourth of the Earths land surface is threatened by this process.
Desertification is accomplished primarily through the loss of stabilizing natural vegetation
and the subsequent accelerated erosion of the soil by wind and water. In some cases the loose
soil is blown completely away, leaving a stony surface. In other cases, the finer particles may
be removed, while the sand-sized particles are accumulated to form mobile hills or ridges of
sand.
Even in the areas that retain a soil cover, the reduction of vegetation typically results in the
loss of the soils ability to absorb substantial quantities of water. The impact of raindrops on
the loose soil tends to transfer fine clay particles into the tiniest soil spaces, sealing them and
producing a surface that allows very little water penetration. Water absorption is greatly
reduced; consequently runoff is increased, resulting in accelerated erosion rates. The gradual
drying of the soil caused by its diminished ability to absorb water results in the further loss of
vegetation, so that a cycle of progressive surface deterioration is established.
In some regions, the increase in desert areas is occurring largely as the result of a trend
toward drier climatic conditions. Continued gradual global warming has produced an increase
in aridity for some areas over the past few thousand years. The process may be accelerated in
subsequent decades if global warming resulting from air pollution seriously increases.
There is little doubt, however, that desertification in most areas results primarily from human
activities rather than natural processes. The semiarid lands bordering the deserts exist in a
delicate ecological balance and are limited in their potential to adjust to increased
environmental pressures. Expanding populations are subjecting the land to increasing
pressures to provide them with food and fuel. In wet periods, the land may be able to respond
to these stresses. During the dry periods that are common phenomena along the desert
margins, though, the pressure on the land is often far in excess of its diminished capacity, and
desertification results.
Four specific activities have been identified as major contributors to the desertification
processes: over cultivation, overgrazing, firewood gathering, and over irrigation. The
cultivation of crops has expanded into progressively drier regions as population densities
have grown. These regions are especially likely to have periods of severe dryness, so that
crop failures are common. Since the raising of most crops necessitates the prior removal of
208

[] Version 3.0
the natural vegetation, crop failures leave extensive tracts of land devoid of a plant cover and
susceptible to wind and water erosion.
The raising of livestock is a major economic activity in semiarid lands, where grasses are
generally the dominant type of natural vegetation. The consequences of an excessive number
of livestock grazing in an area are the reduction of the vegetation cover and the trampling and
pulverization of the soil. This is usually followed by the drying of the soil and accelerated
erosion.
Firewood is the chief fuel used for cooking and heating in many countries. The increased
pressures of expanding populations have led to the removal of woody plants so that many
cities and towns are surrounded by large areas completely lacking in trees and shrubs. The
increasing use of dried animal waste as a substitute fuel has also hurt the soil because this
valuable soil conditioner and source of plant nutrients is no longer being returned to the land.
The final major human cause of desertification is soil salinization resulting from
overirrigation. Excess water from irrigation sinks down into the water table. If no drainage
system exists, the water table rises, bringing dissolved salts to the surface. The water
evaporates and the salts are left behind, creating a white crustal layer that prevents air and
water from reaching the underlying soil.
The extreme seriousness of desertification results from the vast areas of land and the
tremendous numbers of people affected, as well as from the great difficulty of reversing or
even slowing the process. Once the soil has been removed by erosion, only the passage of
centuries or millennia will enable new soil to form. In areas where considerable soil still
remains, though, a rigorously enforced program of land protection and cover-crop planting
may make it possible to reverse the present deterioration of the surface.
[Passage End]
---------------------------------------------------------------------------------------------------------------Paragraph 1: The deserts, which already occupy approximately a fourth of the Earths land
surface, have in recent decades been increasing at an alarming pace. The expansion of desert
like conditions into areas where they did not previously exist is called desertification. It has
been estimated that an additional one-fourth of the Earths land surface is threatened by this
process.
1. The word threatened in the passage is closest in meaning to
A. Restricted
B. Endangered
C. Prevented
D. Rejected

209

Version 3.0

[]

Paragraph 3: Even in the areas that retain a soil cover, the reduction of vegetation typically
results in the loss of the soils ability to absorb substantial quantities of water. The impact of
raindrops on the loose soil tends to transfer fine clay particles into the tiniest soil spaces,
sealing them and producing a surface that allows very little water penetration. Water
absorption is greatly reduced; consequently runoff is increased, resulting in accelerated
erosion rates. The gradual drying of the soil caused by its diminished ability to absorb water
results in the further loss of vegetation, so that a cycle of progressive surface deterioration is
established.
2. According to paragraph 3, the loss of natural vegetation has which of the following
consequences for soil?
A. Increased stony content
B. Reduced water absorption
C. Increased numbers of spaces in the soil
D. Reduced water runoff
Paragraph 5: There is little doubt, however, that desertification in most areas results
primarily from human activities rather than natural processes. The semiarid lands bordering
the deserts exist in a delicate ecological balance and are limited in their potential to adjust to
increased environmental pressures. Expanding populations are subjecting the land to
increasing pressures to provide them with food and fuel. In wet periods, the land may be able
to respond to these stresses. During the dry periods that are common phenomena along the
desert margins, though, the pressure on the land is often far in excess of its diminished
capacity, and desertification results.
3. The word delicate in the passage is closest in meaning to
A. Fragile
B. Predictable
C. Complex
D. Valuable
4. According to paragraph 5, in dry periods, border areas have difficulty
A. Adjusting to stresses created by settlement
B. Retaining their fertility after desertification
C. Providing water for irrigating crops
D. Attracting populations in search of food and fuel

210

[] Version 3.0
Paragraph 6: Four specific activities have been identified as major contributors to the
desertification processes: over cultivation, overgrazing, firewood gathering, and over
irrigation. The cultivation of crops has expanded into progressively drier regions as
population densities have grown. These regions are especially likely to have periods of severe
dryness, so that crop failures are common. Since the raising of most crops necessitates the
prior removal of the natural vegetation, crop failures leave extensive tracts of land devoid of a
plant cover and susceptible to wind and water erosion.
5. The word progressively in the passage is closest inmeaning to
A. Openly
B. Impressively
C. Objectively
D. Increasingly
6. According to paragraph 6, which of the following is often associated with raising crops?
A. Lack of proper irrigation techniques
B. Failure to plant crops suited to the particular area
C. Removal of the original vegetation
D. Excessive use of dried animal waste
7. The phrase devoid of in the passage is closest in meaning to
A. Consisting of
B. Hidden by
C. Except for
D. Lacking in
Paragraph 9: The final major human cause of desertification is soil salinization resulting
from overirrigation. Excess water from irrigation sinks down into the water table. If no
drainage system exists, the water table rises, bringing dissolved salts to the surface. The water
evaporates and the salts are left behind, creating a white crustal layer that prevents air and
water from reaching the underlying soil.
8. According to paragraph 9, the grounds absorption of excess water is a factor in
desertification because it can
A. Interfere with the irrigation of land
B. Limit the evaporation of water
C. Require more absorption of air by the soil
D. Bring salts to the surface

211

Version 3.0

[]

9. All of the following are mentioned in the passage as contributing to desertification


EXCEPT
A. Soil erosion
B. Global warming
C. Insufficient irrigation
D. The rising of livestock
Paragraph 10: The extreme seriousness of desertification results from the vast areas of land
and the tremendous numbers of people affected, as well as from the great difficulty of
reversing or even slowing the process. Once the soil has been removed by erosion, only the
passage of centuries or millennia will enable new soil to form. In areas where considerable
soil still remains, though, a rigorously enforced program of land protection and cover-crop
planting may make it possible to reverse the present deterioration of the surface.
10. Which of the sentences below best expresses the essential information in the highlighted
sentence in the passage? Incorrect choices change the meaning in important ways or leave
out essential information.
A. Desertification is a significant problem because it is so hard to reverse and affects
large areas of land and great numbers of people.
B. Slowing down the process of desertification is difficult because of population growth
that has spread over large areas of land.
C. The spread of deserts is considered a very serious problem that can be solved only if
large numbers of people in various countries are involved in the effort.
D. Desertification is extremely hard to reverse unless the population is reduced in the
vast areas affected.
11. It can be inferred from the passage that the author most likely believes which of the
following about the future of desertification?
A. Governments will act quickly to control further desertification.
B. The factors influencing desertification occur in cycles and will change in the future.
C. Desertification will continue to increase.
D. Desertification will soon occur in all areas of the world.
Paragraph 7: The raising of livestock is a major economic activity in semiarid lands,
where grasses are generally the dominant type of natural vegetation. The consequences of
an excessive number of livestock grazing in an area are the reduction of the vegetation cover
and the trampling and pulverization of the soil. This is usually followed by the drying of
the soil and accelerated erosion.

212

[] Version 3.0
12. Look at the four squares [] that indicate where the following sentence can be added to
the passage.
This economic reliance on livestock in certain regions makes large tracts of land
susceptible to overgrazing.
Where would the sentence best fit?
13. Directions: An introductory sentence for a brief summary of the passage is provided below.
Complete the summary by selecting the THREE answer choices that express the most
important ideas in the passage. Some answer choices do not belong in the summary
because they express ideas that are not presented in the passage or are minor ideas in the
passage.
Many factors have contributed to the great increase in desertification in recent
decades.

Answer Choices
a. Growing human populations and the agricultural demands that come with such growth
have upset the ecological balance in some areas and led to the spread of deserts.
b. As periods of severe dryness have become more common, failures of a number of
different crops have increased.
c. Excessive numbers of cattle and the need for firewood for fuel have reduced grasses
and trees, leaving the land unprotected and vulnerable.
d. Extensive irrigation with poor drainage brings salt to the surface of the soil, a process
that reduces water and air absorption.
e. Animal dung enriches the soil by providing nutrients for plant growth.
f. Grasses are generally the dominant type of natural vegetation in semiarid lands.

213

Version 3.0

[]

6.1.2 Geology and Landscape


Most people consider the landscape to be unchanging, but Earth is a dynamic body, and its
surface is continually altering-slowly on the human time scale, but relatively rapidly when
compared to the great age of Earth (about 4,500 billion years). There are two principal
influences that shape the terrain: constructive processes such as uplift, which create new
landscape features, and destructive forces such as erosion, which gradually wear away
exposed landforms.
Hills and mountains are often regarded as the epitome of permanence, successfully resisting
the destructive forces of nature, but in fact they tend to be relatively short-lived in geological
terms. As a general rule, the higher a mountain is, the more recently it was formed; for
example, the high mountains of the Himalayas are only about 50 million years old. Lower
mountains tend to be older, and are often the eroded relics of much higher mountain chains.
About 400 million years ago, when the present-day continents of North America and Europe
were joined, the Caledonian mountain chain was the same size as the modern Himalayas.
Today, however, the relics of the Caledonian progeny (mountain-building period) exist as the
comparatively low mountains of Greenland, the northern Appalachians in the United States,
the Scottish Highlands, and the Norwegian coastal plateau.
The Earth's crust is thought to be divided into huge, movable segments, called plates, which
float on a soft plastic layer of rock. Some mountains were formed as a result of these plates
crashing into each other and forcing up the rock at the plate margins. In this process,
sedimentary rocks that originally formed on the seabed may be folded upwards to altitudes of
more than 26,000 feet. Other mountains may be raised by earthquakes, which fracture the
Earth's crust and can displace enough rock to produce block mountains. A third type of
mountain may be formed as a result of volcanic activity which occurs in regions of active
fold mountain belts, such as in the Cascade Range of western North America. The Cascades
are made up of lavas and volcanic materials. Many of the peaks are extinct volcanoes.
Whatever the reason for mountain formation, as soon as land rises above sea level it is
subjected to destructive forces. The exposed rocks are attacked by the various weather
processes and gradually broken down into fragments, which are then carried away and later
deposited as sediments. Thus, any landscape represents only a temporary stage in the
continuous battle between the forces of uplift and those of erosion.
The weather, in its many forms, is the main agent of erosion. Rain washes away loose soil
and penetrates cracks in the rocks. Carbon dioxide in the air reacts with the rainwater,
forming a weak acid (carbonic acid) that may chemically attack the rocks. The rain seeps
underground and the water may reappear later as springs. These springs are the sources of
streams and rivers, which cut through the rocks and carry away debris from the mountains to
the lowlands.

214

[] Version 3.0
Under very cold conditions, rocks can be shattered by ice and frost. Glaciers may form in
permanently cold areas, and these slowly moving masses of ice cut out valleys, carrying with
them huge quantities of eroded rock debris. In dry areas the wind is the principal agent of
erosion. It carries fine particles of sand, which bombard exposed rock surfaces, thereby
wearing them into yet more sand. Even living things contribute to the formation of
landscapes. Tree roots force their way into cracks in rocks and, in so doing, speed their
splitting. In contrast, the roots of grasses and other small plants may help to hold loose soil
fragments together, thereby helping to prevent erosion by the wind.
[Passage End]
---------------------------------------------------------------------------------------------------------------Paragraph 1 Most people consider the landscape to be unchanging, but Earth is a dynamic
body, and its surface is continually altering-slowly on the human time scale, but relatively
rapidly when compared to the great age of Earth (about 4,500 billion years). There are two
principal influences that shape the terrain: constructive processes such as uplift, which create
new landscape features, and destructive forces such as erosion, which gradually wear away
exposed landforms.
2. According to paragraph 1, which of the following statements is true of changes in Earth's
landscape?
A. They occur more often by uplift than by erosion
B. They occur only at special times.
C. They occur less frequently now than they once did.
D. They occur quickly in geological terms.
3. The word relatively in the passage is closest in meaning to
A. Unusually
B. Comparatively
C. Occasionally
D. Naturally
Paragraph 2: Hills and mountains are often regarded as the epitome of permanence,
successfully resisting the destructive forces of nature, but in fact they tend to be relatively
short-lived in geological terms. As a general rule, the higher a mountain is, the more recently
it was formed; for example, the high mountains of the Himalayas are only about 50 million
years old. Lower mountains tend to be older, and are often the eroded relics of much higher
mountain chains. About 400 million years ago, when the present-day continents of North
America and Europe were joined, the Caledonian mountain chain was the same size as the
modern Himalayas. Today, however, the relics of the Caledonian progeny (mountainbuilding period) exist as the comparatively low mountains of Greenland, the northern
Appalachians in the United States, the Scottish Highlands, and the Norwegian coastal plateau.
215

Version 3.0

[]

4. Which of the following can be inferred from paragraph 2 about the mountains of the
Himalayas?
A. Their current height is not an indication of their age.
B. At present, they are much higher than the mountains of the Caledonian range.
C. They were a uniform height about 400 million years ago.
D. They are not as high as the Caledonian mountains were 400 million years ago.
5. The word relics in the passage IS closest in meaning to
A. Resemblances
B. Regions
C. Remains
D. Restorations
Paragraph 3: The Earth's crust is thought to be divided into huge, movable segments, called
plates, which float on a soft plastic layer of rock. Some mountains were formed as a result of
these plates crashing into each other and forcing up the rock at the plate margins. In this
process, sedimentary rocks that originally formed on the seabed may be folded upwards to
altitudes of more than 26,000 feet. Other mountains may be raised by earthquakes, which
fracture the Earth's crust and can displace enough rock to produce block mountains. A third
type of mountain may be formed as a result of volcanic activity which occurs in regions of
active fold mountain belts, such as in the Cascade Range of western North America. The
Cascades are made up of lavas and volcanic materials. Many of the peaks are extinct
volcanoes.
6. According to paragraph 3, one cause of mountain formation is the
A. effect of climatic change on sea level
B. slowing down of volcanic activity
C. force of Earth's crustal plates hitting each other
D. replacement of sedimentary rock with volcanic rock
Paragraph 5: The weather, in its many forms, is the main agent of erosion. Rain washes away
loose soil and penetrates cracks in the rocks. Carbon dioxide in the air reacts with the
rainwater, forming a weak acid (carbonic acid) that may chemically attack the rocks. The rain
seeps underground and the water may reappear later as springs. These springs are the sources
of streams and rivers, which cut through the rocks and carry away debris from the mountains
to the lowlands.

216

[] Version 3.0
7. Why does the author mention Carbon dioxide in the passage?
A. To explain the origin of a chemical that can erode rocks
B. To contrast carbon dioxide with carbonic acid
C. To give an example of how rainwater penetrates soil
D. To argue for the desirability of preventing erosion
7. The word seeps in the passage is closest in meaning to
A. Dries gradually
B. Flows slowly
C. Freezes quickly
D. Warms slightly
Paragraph 6: Under very cold conditions, rocks can be shattered by ice and frost. Glaciers
may form in permanently cold areas, and these slowly moving masses of ice cut out valleys,
carrying with them huge quantities of eroded rock debris. In dry areas the wind is the
principal agent of erosion. It carries fine particles of sand, which bombard exposed rock
surfaces, thereby wearing them into yet more sand. Even living things contribute to the
formation of landscapes. Tree roots force their way into cracks in rocks and, in so doing,
speed their splitting. In contrast, the roots of grasses and other small plants may help to hold
loose soil fragments together, thereby helping to prevent erosion by the wind.
8. The word them in the passage refers to
A. Cold areas
B. Masses of ice
C. Valleys
D. Rock debris
Paragraph 2: Hills and mountains are often regarded as the epitome of permanence,
successfully resisting the destructive forces of nature, but in fact they tend to be relatively
short-lived in geological terms. As a general rule, the higher a mountain is, the more recently
it was formed; for example, the high mountains of the Himalayas are only about 50 million
years old. Lower mountains tend to be older, and are often the eroded relics of much higher
mountain chains. About 400 million years ago, when the present-day continents of North
America and Europe were joined, the Caledonian mountain chain was the same size as the
modern Himalayas. Today, however, the relics of the Caledonian progeny (mountainbuilding period) exist as the comparatively low mountains of Greenland, the northern
Appalachians in the United States, the Scottish Highlands, and the Norwegian coastal plateau.

217

Version 3.0

[]

9. Which of the sentences below best expresses the essential information in the highlighted
sentence in the passage? Incorrect choices change the meaning in important ways or leave
out essential information.
A. When they are relatively young, hills and mountains successfully resist the destructive
forces of nature.
B. Although they seem permanent, hills and mountains exist for a relatively short period
of geological time.
C. Hills and mountains successfully resist the destructive forces of nature, but only for a
short time.
D. Hills and mountains resist the destructive forces of nature better than other types of
landforms.
Paragraph 6: Under very cold conditions, rocks can be shattered by ice and frost. Glaciers
may form in permanently cold areas, and these slowly moving masses of ice cut out valleys,
carrying with them huge quantities of eroded rock debris. In dry areas the wind is the
principal agent of erosion. It carries fine particles of sand, which bombard exposed rock
surfaces, thereby wearing them into yet more sand. Even living things contribute to the
formation of landscapes. Tree roots force their way into cracks in rocks and, in so doing,
speed their splitting. In contrast, the roots of grasses and other small plants may help to hold
loose soil fragments together, thereby helping to prevent erosion by the wind.
10. According to paragraph 6, which of the following is both a cause and result of erosion?
A. Glacial activity
B. Rock debris
C. Tree roots
D. Sand
11. Look at the four squares that indicate where the following sentence could be added to
the passage.
Under different climatic conditions, another type of destructive force contributes to
erosion.
Where would the sentence best fit?
12. Directions: Three of the answer choices below are used in the passage to illustrate
constructive processes and two are used to illustrate destructive processes. Complete the
table by matching appropriate answer choices to the processes they are used to illustrate.

218

[] Version 3.0
CONSTRUCTIVE PROCESSES

DESTRUCTIVE PROCESSSES

Answer Choices
A. Collision of Earth's crustal plates
B. Separation of continents
C. Wind-driven sand
D. Formation of grass roots in soil
E. Earthquakes
F. Volcanic activity
G. Weather processes

219

Version 3.0

[]

6.1.3 Green Icebergs


Icebergs are massive blocks of ice, irregular in shape; they float with only about 12 percent of
their mass above the sea surface. They are formed by glacierslarge rivers of ice that begin
inland in the snows of Greenland, Antarctica, and Alaskaand move slowly toward the sea.
The forward movement, the melting at the base of the glacier where it meets the ocean, and
waves and tidal action cause blocks of ice to break off and float out to sea.
Icebergs are ordinarily blue to white, although they sometimes appear dark or opaque because
they carry gravel and bits of rock. They may change color with changing light conditions and
cloud cover, glowing pink or gold in the morning or evening light, but this color change is
generally related to the low angle of the Sun above the horizon. However, travelers to
Antarctica have repeatedly reported seeing green icebergs in the Weddell Sea and, more
commonly, close to the Amery Ice Shelf in East Antarctica.
One explanation for green icebergs attributes their color to an optical illus ion when blue
ice is illuminated by a near-horizon red Sun, but green icebergs stand out am ong white an d
blue icebergs under a great variety of light conditions. Another suggestion is that the color
might be related to ice with high levels of metallic compounds, including copper and iron.
Recent expeditions have taken ice samples from green icebergs and ice coresvertical,
cylindrical ice samples reaching down to great depthsfrom the glacial ice shelves along the
Antarctic continent. Analyses of these cores and samples provide a different solution to the
problem.
The ice shelf cores, with a total length of 215 meters (705 feet), were long enough to
penetrate through glacial icewhich is formed from the compaction of snow and contains air
bubblesand to continue into the clear, bubble-free ice formed from seawater that freezes
onto the bottom of the glacial ice. The properties of this clear sea ice were very similar to the
ice from the green iceberg. The scientists concluded that green icebergs form when a twolayer block of shelf ice breaks away and capsizes (turns upside down), exposing the bubblefree shelf ice that was formed from seawater.
A green iceberg that stranded just west of the Amery Ice Shelf showed two distinct layers:
bubbly blue-white ice and bubble-free green ice separated by a one meter-long ice layer
containing sediments. The green ice portion was textured by seawater erosion. Where cracks
were present, the color was light green because of light scattering; where no cracks were
present, the color was dark green. No air bubbles were present in the green ice, suggesting
that the ice was not formed from the compression of snow but instead from the freezing of
seawater. Large concentrations of single-celled organisms with green pigments (coloring
substances) occur along the edges of the ice shelves in this region, and the seawater is rich in
their decomposing organic material. The green iceberg did not contain large amounts of
particles from these organisms, but the ice had accumulated dissolved organic matter from
the seawater. It appears that unlike salt, dissolved organic substances are not excluded from

220

[] Version 3.0
the ice in the freezing process. Analysis shows that the dissolved organic material absorbs
enough blue wavelengths from solar light to make the ice appear green.
Chemical evidence shows that platelets (minute flat portions) of ice form in the water and
then accrete and stick to the bottom of the ice shelf to form slush (partially melted snow).
The slush is compacted by an unknown mechanism, and solid, bubble free ice is formed from
water high in soluble organic substances. When an iceberg separates from the ice shelf and
capsizes, the green ice is exposed.
The Amery Ice Shelf appears to be uniquely suited to the production of green icebergs. Once
detached from the ice shelf, these bergs drift in the currents and wind systems surrounding
Antarctica and can be found scattered among Antarcticas less colorful icebergs.
[Passage End]
---------------------------------------------------------------------------------------------------------------Paragraph 1: Icebergs are massive blocks of ice, irregular in shape; they float with only
about 12 percent of their mass above the sea surface. They are formed by glacierslarge
rivers of ice that begin inland in the snows of Greenland, Antarctica, and Alaskaand move
slowly toward the sea. The forward movement, the melting at the base of the glacier where it
meets the ocean, and waves and tidal action cause blocks of ice to break off and float out to
sea.
1. According to paragraph 1, all of the following are true of icebergs EXCEPT:
A. They do not have a regular shape.
B. They are formed where glaciers meet the ocean.
C. Most of their mass is above the sea surface.
D. Waves and tides cause them to break off glaciers.
Paragraph 2: Icebergs are ordinarily blue to white, although they sometimes appear dark or
opaque because they carry gravel and bits of rock. They may change color with changing
light conditions and cloud cover, glowing pink or gold in the morning or evening light, but
this color change is generally related to the low angle of the Sun above the horizon. However,
travelers to Antarctica have repeatedly reported seeing green icebergs in the Weddell Sea and,
more commonly, close to the Amery Ice Shelf in East Antarctica.
2. According to paragraph 2, what causes icebergs to sometimes appear dark or opaque?
A. A heavy cloud cover
B. The presence of gravel or bits of rock
C. The low angle of the Sun above the horizon
D. The presence of large cracks in their surface
221

Version 3.0

[]

Paragraph 3: One explanation for green icebergs attributes their color to an optical illus ion
when blue ice is illuminated by a near-horizon red Sun, but green icebergs stand out am ong
white an d blue icebergs under a great variety of light conditions. Another suggestion is that
the color might be related to ice with high levels of metallic compounds, including copper
and iron. Recent expeditions have taken ice samples from green icebergs and ice cores
vertical, cylindrical ice samples reaching down to great depthsfrom the glacial ice shelves
along the Antarctic continent. Analyses of these cores and samples provide a different
solution to the problem.
3. Which of the sentences below best expresses the essential information in the highlighted
sentence in the passage? Incorrect choices change the meaning in important ways or leave
out essential information.
A. One explanation notes that green icebergs stand out among other icebergs under a
great variety of light conditions, but this is attributed to an optical illusion.
B. One explanation for the color of green icebergs attributes their color to an optical
illusion that occurs when the light from a near-horizon red Sun shines on a blue
iceberg.
C. One explanation for green icebergs attributes their color to a great variety of light
conditions, but green icebergs stand out best among other icebergs when illuminated
by a near-horizon red Sun.
D. One explanation attributes the color of green icebergs to an optical illusion under
special light conditions, but green icebergs appear distinct from other icebergs under a
great variety of light conditions.
Paragraph 4: The ice shelf cores, with a total length of 215 meters (705 feet), were long
enough to penetrate through glacial icewhich is formed from the compaction of snow and
contains air bubblesand to continue into the clear, bubble-free ice formed from seawater
that freezes onto the bottom of the glacial ice. The properties of this clear sea ice were very
similar to the ice from the green iceberg. The scientists concluded that green icebergs form
when a two-layer block of shelf ice breaks away and capsizes (turns upside down), exposing
the bubble-free shelf ice that was formed from seawater.
4. The word penetrate in the passage is closest in meaning to
A. collect
B. pierce
C. melt
D. endure

222

[] Version 3.0
5. According to paragraph 4, how is glacial ice formed?
A. By the compaction of snow
B. By the freezing of seawater on the bottom of ice shelves
C. By breaking away from the ice shelf
D. By the capsizing of a two-layer block of shelf ice
6. According to paragraph 4, ice shelf cores helped scientists explain the formation of green
icebergs by showing that
A. the ice at the bottom of green icebergs is bubble-free ice formed from frozen seawater
B. bubble-free ice is found at the top of the ice shelf
C. glacial ice is lighter and floats better than sea ice
D. the clear sea ice at the bottom of the ice shelf is similar to ice from a green iceberg
Paragraph 5: A green iceberg that stranded just west of the Amery Ice Shelf showed two
distinct layers: bubbly blue-white ice and bubble-free green ice separated by a one meter-long
ice layer containing sediments. The green ice portion was textured by seawater erosion.
Where cracks were present, the color was light green because of light scattering; where no
cracks were present, the color was dark green. No air bubbles were present in the green ice,
suggesting that the ice was not formed from the compression of snow but instead from the
freezing of seawater. Large concentrations of single-celled organisms with green pigments
(coloring substances) occur along the edges of the ice shelves in this region, and the seawater
is rich in their decomposing organic material. The green iceberg did not contain large
amounts of particles from these organisms, but the ice had accumulated dissolved organic
matter from the seawater. It appears that unlike salt, dissolved organic substances are not
excluded from the ice in the freezing process. Analysis shows that the dissolved organic
material absorbs enough blue wavelengths from solar light to make the ice appear green.
7. Why does the author mention that The green ice portion was textured by seawater
erosion?
A. To explain why cracks in the iceberg appeared light green instead of dark green
B. To suggest that green ice is more easily eroded by seawater than white ice is
C. To support the idea that the green ice had been the bottom layer before capsizing
D. To explain how the air bubbles had been removed from the green ice
8. The word accumulated in the passage is closest in meaning to
A. collected
B. frozen
C. released
D. covered
223

Version 3.0

[]

9. The word excluded in the passage is closest in meaning to


A. kept out
B. compressed
C. damaged
D. gathered together
Paragraph 6: Chemical evidence shows that platelets (minute flat portions) of ice form in
the water and then accrete and stick to the bottom of the ice shelf to form slush (partially
melted snow). The slush is compacted by an unknown mechanism, and solid, bubble free ice
is formed from water high in soluble organic substances. When an iceberg separates from the
ice shelf and capsizes, the green ice is exposed.
10. The word accrete in the passage is closest inmeaning to
A. advance
B. transfer
C. flatten out
D. come together
11. Which of the following is NOT explained in the passage?
A. Why blocks of ice break off where glaciers meet the ocean
B. Why blocks of shelf ice sometimes capsize after breaking off
C. Why green icebergs are commonly produced in some parts of Antarctica
D. Why green icebergs contain large amounts of dissolved organic pigments
Paragraph 7: The Amery Ice Shelf appears to be uniquely suited to the production of green
icebergs. Once detached from the ice shelf, these bergs drift in the currents and wind systems
surrounding Antarctica and can be found scattered among Antarcticas less colorful icebergs.
12. The passage supports which of the following statements about the Amery Ice Shelf?
A. The Amery Ice Shelf produces only green icebergs.
B. The Amery Ice Shelf produces green icebergs because its ice contains high levels of
metallic compounds such as copper and iron.
C. The Amery Ice Shelf produces green icebergs because the seawater is rich in a
particular kind of soluble organic material.
D. No green icebergs are found far from the Amery Ice Shelf.

224

[] Version 3.0
Icebergs are ordinarily blue to white, although they sometimes appear dark or opaque because
they carry gravel and bits of rock. They may change color with changing light conditions and
cloud cover, glowing pink or gold in the morning or evening light, but this color change is
generally related to the low angle of the Sun above the horizon. However, travelers to
Antarctica have repeatedly reported seeing green icebergs in the Weddell Sea and, more
commonly, close to the Amery Ice Shelf in East Antarctica.
One explanation for green icebergs attributes their color to an optical illus ion when blue
ice is illuminated by a near-horizon red Sun, but green icebergs stand out am ong white an d
blue icebergs under a great variety of light conditions. Another suggestion is that the color
might be related to ice with high levels of metallic compounds, including copper and iron.
Recent expeditions have taken ice samples from green icebergs and ice coresvertical,
cylindrical ice samples reaching down to great depthsfrom the glacial ice shelves along the
Antarctic continent. Analyses of these cores and samples provide a different solution to the
problem.
13. Look at the four squares that indicate where the following sentence could be added to the
passage.
Scientists have differed as to whether icebergs appear green as a result of light
conditions or because of something in the ice itself.
Where would the sentence best fit?
14. Directions: An introductory sentence for a brief summary of the passage is provided
below. Complete the summary by selecting the THREE answer choices that express the
most important ideas in the passage. Some sentences do not belong in the summary
because they express ideas that are not presented in the passage or are minor ideas in the
passage.
Several suggestions, ranging from light conditions to the presence of metallic
compounds, have been offered to explain why some icebergs appear green.

225

Version 3.0

[]

Answer Choices
a. Ice cores were used to determine that green icebergs were formed from the
compaction of metallic compounds, including copper and iron.
b. All ice shelves can produce green icebergs, but the Amery Ice Shelf is especially well
suited to do so.
c. Green icebergs form when a two layer block of ice breaks away from a glacier and
capsizes, exposing the bottom sea ice to view.
d. Ice cores and samples revealed that both ice shelves and green icebergs contain a
layer of bubbly glacial ice and a layer of bubble-free sea ice.
e. Green icebergs are white until they come into contact with seawater containing
platelets and soluble organic green pigments.
f. In a green iceberg, the sea ice contains large concentrations of organic matter from the
seawater.

226

[] Version 3.0

6.2 Biological Sciences


6.2.1 The origins of Cetaceans
It should be obvious that cetaceanswhales, porpoises, and dolphins-are mammals. They
breathe through lungs, not through gills, and give birth to live young. Their streamlined
bodies, the absence of hind legs, and the presence of a fluke1 and blowhole2 cannot disguise
their affinities with land dwelling mammals. However, unlike the cases of sea otters and
pinnipeds (seals, sea lions, and walruses, whose limbs are functional both on land and at sea),
it is not easy to envision what the first whales looked like. Extinct but already fully marine
cetaceans are known from the fossil record. How was the gap between a walking mammal
and a swimming whale bridged? Missing until recently were fossils clearly intermediate, or
transitional, between land mammals and cetaceans.
Very exciting discoveries have finally allowed scientists to reconstruct the most likely
origins of cetaceans. In 1979, a team looking for fossils in northern Pakistan found what
proved to be the oldest fossil whale. The fossil was officially named Pakicetus in honor of the
country where the discovery was made. Pakicetus was found embedded in rocks formed from
river deposits that were 52 million years old. The river that formed these deposits was
actually not far from an ancient ocean known as the Tethys Sea.
The fossil consists of a complete skull of an archaeocyte, an extinct group of ancestors of
modern cetaceans. Although limited to a skull, the Pakicetus fossil provides precious details
on the origins of cetaceans. The skull is cetacean-like but its jawbones lack the enlarged
space that is filled with fat or oil and used for receiving underwater sound in modern whales.
Pak icetus probably detected sound through the ear op ening as in land mammals. The skull
also lacks a blowhole, another cetacean adaptation for diving. Other features, however, show
experts that Pakicetus is a transitional form between a group of extinct flesh-eating mammals,
the mesonychids, and cetaceans. It has been suggested that Pakicetus fed on fish in shallow
water and was not yet adapted for life in the open ocean. It probably bred and gave birth on
land.
Another major discovery was made in Egypt in 1989. Several skeletons of another early
whale, Basilosaurus, were found in sediments left by the Tethys Sea and now exposed in the
Sahara desert. This whale lived around 40 million years ago, 12 million years after Pakicetus.
Many incomplete skeletons were found but they included, for the first time in an archaeocyte,
a complete hind leg that features a foot with three tiny toes. Suc h legs would have been far
too small to have supported the 50-foot-long Basilosaurus on land. Basilosaurus was
undoubtedly a fully marine whale with possibly nonfunctional, or vestigial, hind legs.
An even more exciting find was reported in 1994, also from Pakistan. The now extinct whale
Ambulocetus natans ("the walking whale that swam") lived in the Tethys Sea 49 million
years ago. It lived around 3 million years after Pakicetus but 9 million before Basilosaurus.
The fossil luckily includes a good portion of the hind legs. The legs were strong and ended in
227

Version 3.0

[]

long feet very much like those of a modern pinniped. The legs were certainly functional both
on land and at sea. The whale retained a tail and lacked a fluke, the major means of
locomotion in modern cetaceans. The structure of the backbone shows, however, that
Ambulocetus swam like modern whales by moving the rear portion of its body up and down,
even though a fluke was missing. The large hind legs were used for propulsion in water. On
land, where it probably bred and gave birth, Ambulocetus may have moved around very much
like a modern sea lion. It was undoubtedly a whale that linked life on land with life at sea.
[Passage End]
---------------------------------------------------------------------------------------------------------------1. In paragraph 1, what does the author say about the presence of a blowhole in cetaceans?
A. It clearly indicates that cetaceans are mammals.
B. It cannot conceal the fact that cetaceans are mammals.
C. It is the main difference between cetaceans and land-dwelling mammals.
D. It cannot yield clues about the origins of cetaceans.
2. Which of the following can be inferred from paragraph 1 about early sea otters?
A. It is not difficult to imagine what they looked like
B. There were great numbers of them.
C. They lived in the sea only.
D. They did not leave many fossil remains.
3. The word precious in the passage is closest inmeaning to
A. Exact
B. Scarce
C. Valuable
D. Initial
4. Pakicetus and modern cetaceans have similar
A. Hearing structures
B. Adaptations for diving
C. Skull shapes
D. Breeding locations
5. The word it in the passage refers to
A. Pakicetus
B. Fish
C. Life
D. Ocean
228

[] Version 3.0
6. The word exposed in the passage is closest inmeaning to
A. Explained
B. Visible
C. Identified
D. Located
7. The hind leg of Basilosaurus was a significant find because it showed that Basilosaurus
A. Lived later than Ambulocetus natans
B. Lived at the same time as Pakicetus
C. Was able to swim well
D. Could not have walked on land
8. It can be inferred that Basilosaurus bred and gave birth in which of the following locations
A. On land
B. Both on land and at sea
C. In shallow water
D. In a marine environment
9. Why does the author use the word luckily in mentioning that the Ambulocetus natans
fossil included hind legs?
A. Fossil legs of early whales are a rare find.
B. The legs provided important information about the evolution of cetaceans.
C. The discovery allowed scientists to reconstruct a complete skeleton of the whale.
D. Until that time, only the front legs of early whales had been discovered.
10. Which of the sentences below best expresses the essential information in the highlighted
sentence in the passage? Incorrect choices change the meaning in important ways or leave
out essential information.
A. Even though Ambulocetus swam by moving its body up and down, it did not have a
backbone.
B. The backbone of Ambulocetus, which allowed it to swim, provides evidence of its
missing fluke.
C. Although Ambulocetus had no fluke, its backbone structure shows that it swam like
modern whales.
D. By moving the rear parts of their bodies up and down, modern whales swim in a
different way from the way Ambulocetus swam.

229

Version 3.0

[]

11. The word propulsion in the passage is closest in meaning to


A. Staying afloat
B. Changing direction
C. Decreasing weight
D. Moving forward
12. Look at the four squares [] that indicate where the following sentence can be added to
the passage.
This is a question that has puzzled scientists for ages.
Where would the sentence best fit?
13. Directions: An introductory sentence for a brief summary of the passage is provided
below. Complete the summary by selecting the THREE answer choices that express the
most important ideas in the passage. Some answer choices do not belong in the summary
because they express ideas that are not presented in the passage or are minor ideas in the
passage.
This passage discusses fossils that help to explain the likely origins of
cetaceanswhales, porpoises, and dolphins.

Answer Choices
a. Recent discoveries of fossils have helped to show the link between land mammals and
cetaceans.
b. The discovery of Ambulocetus natans provided evidence for a whale that lived both
on land and at sea.
c. The skeleton of Basilosaurusw as found in what had been the Tethys Sea, an area rich
in fossil evidence.
d. Pakicetus is the oldest fossil whale yet to be found.
e. Fossils thought to be transitional forms between walking mammals and swimming
whales were found.
f. Ambulocetus hind legs were used for propulsion in the water.

230

[] Version 3.0
6.2.2 Swimming Machines
Tunas, mackerels, and billfishes (marlins, sailfishes, and swordfish) swim continuously.
Feeding, courtship, reproduction, and even rest are carried out while in constant motion. As
a result, practically every aspect of the body form and function of these swimming machines
is adapted to enhance their ability to swim.
Many of the adaptations of these fishes serve to reduce water resistance (drag). Interestingly
enough, several of these hydrodynamic adaptations resemble features designed to improve
the aerodynamics of high-speed aircraft. Though human engineers are new to the game, tunas
and their relatives evolved their high-tech designs long ago.
Tunas, mackerels, and billfishes have made streamlining into an art form. Their bodies are
sleek and compact. The body shapes of tunas, in fact, are nearly ideal from an engineering
point of view. Most species lack scales over most of the body, making it smooth and slippery.
The eyes lie flush with the body and do not protrude at all. They are also covered with a slick,
transparent lid that reduces drag. The fins are stiff, smooth, and narrow, qualities that also
help cut drag. When not in use, the fins are tucked into special grooves or depressions so that
they lie flush with the body and do not break up its smooth contours. Airplanes retract their
landing gear while in flight for the same reason.
Tunas, mackerels, and billfishes have even more sophisticated adaptations than these to
improve their hydrodynamics. The long bill of marlins, sailfishes, and swordfish probably
helps them slip through the water. Many supersonic aircraft have a similar needle at the nose.
Most tunas and billfishes have a series of keels and finlets near the tail. Although most of
their scales have been lost, tunas and mackerels retain a patch of coarse scales near the head
called the corselet. The keels, finlets, and corselet help direct the flow of water over the body
surface in such as way as to reduce resistance (see the figure). Again, supersonic jets have
similar features.
Because they are always swimming, tunas simply have to open their mouths and water is
forced in and over their gills. Accordingly, they have lost most of the muscles that other
fishes use to suck in water and push it past the gills. In fact, tunas must swim to breathe. They
must also keep swimming to keep from sinking, since most have largely or completely lost
the swim bladder, the gas-filled sac that helps most other fish remain buoyant.
One potential problem is that opening the mouth to breathe detracts from the streamlining of
these fishes and tends to slow them down. Some species of tuna have specialized grooves in
their tongue. It is thought that these grooves help to channel water through the mouth and out
the gill slits, thereby reducing water resistance.
There are adaptations that increase the amount of forward thrust as well as those that reduce
drag. Again, these fishes are the envy of engineers. Their high, narrow tails with swept-back
231

Version 3.0

[]

tips are almost perfectly adapted to provide propulsion with the least possible effort. Perhaps
most important of all to these and other fast swimmers is their ability to sense and make use
of swirls and eddies (circular currents) in the water. They can glide past eddies that would
slow them down and then gain extra thrust by pushing off the eddies. Scientists and
engineers are beginning to study this ability of fishes in the hope of designing more efficient
propulsion systems for ships.
The muscles of these fishes and the mechanism that maintains a warm body temperature are
also highly efficient. A bluefin tuna in water of 7C (45F) can maintain a core temperature
of over 25C (77"F). This warm body temperature may help not only the muscles to work
better, but also the brain and the eyes. The billfishes have gone one step further. They have
evolved special heaters of modified muscle tissue that warm the eyes and brain,
maintaining peak performance of these critical organs.

[Passage End]
---------------------------------------------------------------------------------------------------------------Paragraph 1: Tunas, mackerels, and billfishes (marlins, sailfishes, and swordfish) swim
continuously. Feeding, courtship, reproduction, and even rest are carried out while in
constant motion. As a result, practically every aspect of the body form and function of these
swimming machines is adapted to enhance their ability to swim.
1. The word enhance in the passage is closest in meaning to
A. Use
B. Improve
C. Counteract
D. Balance
232

[] Version 3.0
Paragraph 3: Tunas, mackerels, and billfishes have made streamlining into an art form. Their
bodies are sleek and compact. The body shapes of tunas, in fact, are nearly ideal from an
engineering point of view. Most species lack scales over most of the body, making it smooth
and slippery. The eyes lie flush with the body and do not protrude at all. They are also
covered with a slick, transparent lid that reduces drag. The fins are stiff, smooth, and narrow,
qualities that also help cut drag. When not in use, the fins are tucked into special grooves or
depressions so that they lie flush with the body and do not break up its smooth contours.
Airplanes retract their landing gear while in flight for the same reason.
2. The word they in the passage refers to
A. Qualities
B. Fins
C. Grooves
D. Depressions
3. Why does the author mention that Airplanes retract their landing gear while in flight?
A. To show that air resistance and water resistance work differently from each other
B. To argue that some fishes are better designed than airplanes are
C. To provide evidence that airplane engine have studied the design of fish bodies
D. To demonstrate a similarity in design between certain fishes and airplanes
Paragraph 4: Tunas, mackerels, and billfishes have even more sophisticated adaptations
than these to improve their hydrodynamics. The long bill of marlins, sailfishes, and swordfish
probably helps them slip through the water. Many supersonic aircraft have a similar needle at
the nose.
4. The word sophisticated in the passage is closest in meaning to
A. Complex
B. Amazing
C. Creative
D. Practical
5. According to paragraph4, the long bills of marlins, sailfish, and swordfish probably help
these fishes by
A. Increasing their ability to defend themselves
B. Allowing them to change direction easily
C. Increasing their ability to detect odors
D. Reducing water resistance as they swim

233

Version 3.0

[]

Paragraph 6: Because they are always swimming, tunas simply have to open their mouths
and water is forced in and over their gills. Accordingly, they have lost most of the muscles
that other fishes use to suck in water and push it past the gills. In fact, tunas must swim to
breathe. They must also keep swimming to keep from sinking, since most have largely or
completely lost the swim bladder, the gas-filled sac that helps most other fish remain buoyant.
6. According to the passage, which of the following is one of the reasons that tunas are in
constant motion?
A. They lack a swim bladder.
B. They need to suck in more water than other fishes do.
C. They have large muscles for breathing.
D. They cannot open their mouths unless they are in motion.
Paragraph 7: One potential problem is that opening the mouth to breathe detracts from the
streamlining of these fishes and tends to slow them down. Some species of tuna have
specialized grooves in their tongue. It is thought that these grooves help to channel water
through the mouth and out the gill slits, thereby reducing water resistance.
7. Which of the sentences below best expresses the essential information in the highlighted
sentence in the passage? Incorrect answer choices change the meaning in important ways
or leave out essential information,
A. These fishes often have a problem opening their mouths while swimming.
B. The streamlining of these fishes prevents them from slowing down.
C. The streamlining of these fishes tends to slow down their breathing.
D. Opening the mouth to breathe can reduce the speed of these fishes.
8. The word channel in the passage is closest inmeaning to
A. Reduce
B. Remove
C. Direct
D. Provide
Paragraph 8: There are adaptations that increase the amount of forward thrust as well as
those that reduce drag. Again, these fishes are the envy of engineers. Their high, narrow tails
with swept-back tips are almost perfectly adapted to provide propulsion with the least
possible effort. Perhaps most important of all to these and other fast swimmers is their ability
to sense and make use of swirls and eddies (circular currents) in the water. They can glide
past eddies that would slow them down and then gain extra thrust by pushing off the eddies.
Scientists and engineers are beginning to study this ability of fishes in the hope of designing
more efficient propulsion systems for ships.
234

[] Version 3.0
9. According to the passage, one of the adaptations of fast-swimming fishes that might be
used to improve the performance of ships is these fishes ability to
A. Swim directly through eddies
B. Make efficient use of water currents
C. Cover great distances without stopping
D. Gain speed by forcing water past their gills
Paragraph 9: The muscles of these fishes and the mechanism that maintains a warm body
temperature are also highly efficient. A bluefin tuna in water of 7C (45F) can maintain a
core temperature of over 25C (77F). This warm body temperature may help not only the
muscles to work better, but also the brain and the eyes. The billfishes have gone one step
further. They have evolved special heaters of modified muscle tissue that warm the eyes
and brain, maintaining peak performance of these critical organs.
10. According to paragraph 9, which of the following is true of bluefin tunas?
A. Their eyes and brain are more efficient than those of any other fish.
B. Their body temperature can change greatly depending on the water temperature.
C. They can swim in waters that are much colder than their own bodies.
D. They have special muscle tissue that warms their eyes and brain.
Paragraph 5: Most tunas and billfishes have a series of keels and finlets near the tail.
Although most of their scales have been lost, tunas and mackerels retain a patch of coarse
scales near the head called the corselet. The keels, finlets, and corselet help direct the flow of
water over the body surface in such as way as to reduce resistance (see the figure). Again,
supersonic jets have similar features.
Paragraph 6: Because they are always swimming, tunas simply have to open their mouths
and water is forced in and over their gills. Accordingly, they have lost most of the muscles
that other fishes use to suck in water and push it past the gills. In fact, tunas must swim to
breathe. They must also keep swimming to keep from sinking, since most have largely or
completely lost the swim bladder, the gas-filled sac that helps most other fish remain buoyant.
11. Look at the four squares [l that indicate where the following sentence can be added to the
passage.
Consequently, tunas do not need to suck in water.
Where would the sentence best fit?

235

Version 3.0

[]

12. Directions: Complete the table below by indicating which features of fishes are associated
in the passage with reducing water resistance and which are associated with increasing
thrust.
REDUCING WATER RESISTANCE

INCREASING THRUST

Features of Fishes
a. The absence of scales from most of the body
b. The ability to take advantage of eddies
c. The ability to feed and reproduce while swimming
d. Eyes that do not protrude
e. Fins those are stiff, narrow, and smooth
f. The habit of swimming with the mouth open
g. A high, narrow tail with swept-back tips

236

[] Version 3.0
6.2.3 Opportunists and Competitors
Growth, reproduction, and daily metabolism all require an organism to expend energy. The
expenditure of energy is essentially a process of budgeting, just as finances are budgeted. If
all of ones money is spent on clothes, there may be none left to buy food or go to the movies.
Similarly, a plant or animal cannot squander all its energy on growing a big body if none
would be left over for reproduction, for this is the surest way to extinction.
All organisms, therefore, allocate energy to growth, reproduction, maintenance, and storage.
No choice is involved; this allocation comes as part of the genetic package from the parents.
Maintenance for a given body design of an organism is relatively constant. Storage is
important, but ultimately that energy will be used for maintenance, reproduction, or growth.
Therefore the principal differences in energy allocation are likely to be between growth and
reproduction.
Almost all of an organisms energy can be diverted to reproduction, with very little allocated
to building the body. Organisms at this extreme are opportunists. At the other extreme are
competitors, almost all of whose resources are invested in building a huge body, with a
bare minimum allocated to reproduction.
Dandelions are good examples of opportunists. Their seed heads raised just high enough
above the ground to catch the wind, the plants are no bigger than they need be, their stems are
hollow, and all the rigidity comes from their water content. Thus, a minimum investment has
been made in the body that becomes a platform for seed dispersal. These very short-lived
plants reproduce prolifically; that is to say they provide a constant rain of seed in the
neighborhood of parent plants. A new plant will spring up wherever a seed falls on a suitable
soil surface, but because they do not build big bodies, they cannot compete with other plants
for space, water, or sunlight. These plants are termed opportunists because they rely on their
seeds falling into settings where competing plants have been removed by natural processes,
such as along an eroding riverbank, on landslips, or where a tree falls and creates a gap in the
forest canopy.
Opportunists must constantly invade new areas to compensate for being displaced by more
competitive species. Human landscapes of lawns, fields, or flowerbeds provide settings with
bare soil and a lack of competitors that are perfect habitats for colonization by opportunists.
Hence, many of the strongly opportunistic plants are the common weeds of fields and gardens.
Because each individual is short-lived, the population of an opportunist species is likely to be
adversely affected by drought, bad winters, or floods. If their population is tracked through
time, it will be seen to be particularly unstablesoaring and plummeting in irregular cycles.
The opposite of an opportunist is a competitor. These organisms tend to have big bodies, are
long- lived, and spend relatively little effort each year on reproduction. An oak tree is a good
example of a competitor. A massive oak claims its ground for 200 years or more, out
237

Version 3.0

[]

competing all other would-be canopy trees by casting a dense shade and drawing up any free
water in the soil. The leaves of an oak tree taste foul because they are rich in tannins, a
chemical that renders them distasteful or indigestible to many organisms. The tannins are part
of the defense mechanism that is essential to longevity. Although oaks produce thousands of
acorns, the investment in a crop of acorns is small compared with the energy spent on
building leaves, trunk, and roots. Once an oak tree becomes established, it is likely to survive
minor cycles of drought and even fire. A population of oaks is likely to be relatively stable
through time, and its survival is likely to depend more on its ability to withstand the pressures
of competition or predation than on its ability to take advantage of chance events. It should be
noted, however, that the pure opportunist or pure competitor is rare in nature, as most species
fall between the extremes of a continuum, exhibiting a blend of some opportunistic and some
competitive characteristics.
[Passage End]
----------------------------------------------------------------------------------------------------------------

Paragraph 1: Growth, reproduction, and daily metabolism all require an organism to expend
energy. The expenditure of energy is essentially a process of budgeting, just as finances are
budgeted. If all of ones money is spent on clothes, there may be none left to buy food or go
to the movies. Similarly, a plant or animal cannot squander all its energy on growing a big
body if none would be left over for reproduction, for this is the surest way to extinction.
1. The word squander in the passage is closest inmeaning to
A. Extend
B. Transform
C. Activate
D. Waste
2. The word none in the passage refers to
A. Food
B. Plant or animal
C. Energy
D. Big body
3. In paragraph 1, the author explains the concept of energy expenditure by
A. Identifying types of organisms that became extinct
B. Comparing the scientific concept to a familiar human experience
C. Arguing that most organisms conserve rather than expend energy
D. Describing the processes of growth, reproduction, and metabolism
238

[] Version 3.0
Paragraph 3Almost all of an organisms energy can be diverted to reproduction, with very
little allocated to building the body. Organisms at this extreme are opportunists. At the
other extreme are competitors, almost all of whose resources are invested in building a
huge body, with a bare minimum allocated to reproduction.
4. According to the passage, the classification of organisms as opportunists or
competitors is determined by
A. How the genetic information of an organism is stored and maintained
B. The way in which the organism invests its energy resources
C. Whether the climate in which the organism lives is mild or extreme
D. The variety of natural resources the organism consumes in its environment
Paragraph 4Dandelions are good examples of opportunists. Their seed heads raised just
high enough above the ground to catch the wind, the plants are no bigger than they need be,
their stems are hollow, and all the rigidity comes from their water content. Thus, a minimum
investment has been made in the body that becomes a platform for seed dispersal. These very
short-lived plants reproduce prolifically; that is to say they provide a constant rain of seed in
the neighborhood of parent plants. A new plant will spring up wherever a seed falls on a
suitable soil surface, but because they do not build big bodies, they cannot compete with
other plants for space, water, or sunlight. These plants are termed opportunists because they
rely on their seeds falling into settings where competing plants have been removed by
natural processes, such as along an eroding riverbank, on landslips, or where a tree falls and
creates a gap in the forest canopy.
5. The word dispersal in the passage is closest in meaning to
A. Development
B. Growth
C. Distribution
D. Protection
6. Which of the sentences below best expresses the essential information in the highlighted
sentence in the passage?
A. Because their seeds grow in places where competing plants are no longer present,
dandelions are classified as opportunists.
B. Dandelions are called opportunists because they contribute to the natural processes of
erosion and the creation of gaps in the forest canopy.
C. The term opportunists apply to plants whose seeds fall in places where they can
compete with the seeds of other plants.
D. The term opportunists apply to plants whose falling seeds are removed by natural
processes.
239

Version 3.0

[]

Paragraph 7The opposite of an opportunist is a competitor. These organisms tend to have


big bodies, are long- lived, and spend relatively little effort each year on reproduction. An
oak tree is a good example of a competitor. A massive oak claims its ground for 200 years or
more, out competing all other would-be canopy trees by casting a dense shade and drawing
up any free water in the soil. The leaves of an oak tree taste foul because they are rich in
tannins, a chemical that renders them distasteful or indigestible to many organisms. The
tannins are part of the defense mechanism that is essential to longevity. Although oaks
produce thousands of acorns, the investment in a crop of acorns is small compared with the
energy spent on building leaves, trunk, and roots. Once an oak tree becomes established, it is
likely to survive minor cycles of drought and even fire. A population of oaks is likely to be
relatively stable through time, and its survival is likely to depend more on its ability to
withstand the pressures of competition or predation than on its ability to take advantage of
chance events. It should be noted, however, that the pure opportunist or pure competitor is
rare in nature, as most species fall between the extremes of a continuum, exhibiting a blend of
some opportunistic and some competitive characteristics.
7. The word massive in the passage is closest in meaning to
A. Huge
B. Ancient
C. Common
D. Successful
8. All of the following are mentioned in paragraph 7 as contributing to the longevity of an
oak tree EXCEPT
A. The capacity to create shade
B. Leaves containing tannin
C. The ability to withstand mild droughts and fire
D. The large number of acorns the tree produces
9. According to the passage, oak trees are considered competitors because
A. They grow in areas free of opportunists
B. They spend more energy on their leaves, trunks and roots than on their acorns
C. Their population tends to increase or decrease in irregular cycles
D. Unlike other organisms, they do not need much water or sunlight
10. In paragraph 7, the author suggests that most species of organisms
A. Are primarily opportunists
B. Are primarily competitors
C. Begin as opportunists and evolve into competitors
D. Have some characteristics of opportunists and some of competitors
240

[] Version 3.0
Paragraph 5 Opportunists must constantly invade new areas to compensate for being
displaced by more competitive species. Human landscapes of lawns, fields, or flowerbeds
provide settings with bare soil and a lack of competitors that are perfect habitats for
colonization by opportunists. Hence, many of the strongly opportunistic plants are the
common weeds of fields and gardens. Because each individual is short-lived, the
population of an opportunist species is likely to be adversely affected by drought, bad winters,
or floods. If their population is tracked through time, it will be seen to be particularly
unstablesoaring and plummeting in irregular cycles.
11. Look at the four squares that indicate where the following sentence could be added to
the passage.
Such episodic events will cause a population of dandelions, for example, to vary
widely.
Where would the sentence best fit?
12. Directions: Complete the table by matching the phrases below Directions: Select the
appropriate phrases from the answer choices and match them to the type of organism to
which they relate. TWO of the answer choices will NOT be used.
Opportunists

Competitors

Answer Choices
a. Vary frequently the amount of energy they spend in body maintenance
b. Have mechanisms for protecting themselves from predation
c. Succeed in locations where other organisms have been removed
d. Have relatively short life spans
e. Invest energy in the growth of large, strong structures
f. Have populations that are unstable in response to climate conditions
g. Can rarely find suitable soil for reproduction
h. Produce individuals that can withstand changes in the environmental conditions
i. Reproduce in large numbers

241

Version 3.0

[]

6.3 Arts
6.3.1 Applied Arts and Fine Arts
Although we now tend to refer to the various crafts according to the materials used to
construct themclay, glass, wood, fiber, and metalit was once common to think of crafts
in terms of function, which led to their being known as the "applied arts." Approaching crafts
from the point of view of function, we can divide them into simple categories: containers,
shelters and supports. There is no way around the fact that containers, shelters, and supports
must be functional. The applied arts are thus bound by the laws of physics, which pertain to
both the materials used in their making and the substances and things to be contained,
supported, and sheltered. These laws are universal in their application, regardless of cultural
beliefs, geography, or climate. If a pot has no bottom or has large openings in its sides, it
could hardly be considered a container in any traditional sense. Since the laws of physics, not
some arbitrary decision, have determined the general form of applied-art objects, they follow
basic patterns, so much so that functional forms can vary only within certain limits. Buildings
without roofs, for example, are unusual because they depart from the norm. However, not all
functional objects are exactly alike; that is why we recognize a Shang Dynasty vase as being
different from an Inca vase. What varies is not the basic form but the incidental details that do
not obstruct the objects primary function.
Sensitivity to physical laws is thus an important consideration for the maker of applied-art
objects. It is often taken for granted that this is also true for the maker of fine-art objects. This
assumption misses a significant difference between the two disciplines. Fine-art objects are
not constrained by the laws of physics in the same way that applied-art objects are. Because
their primary purpose is not functional, they are only limited in terms of the materials used to
make them. Sculptures must, for example, be stable, which requires an understanding of the
properties of mass, weight distribution, and stress. Paintings must have rigid stretchers so that
the canvas will be taut, and the paint must not deteriorate, crack, or discolor. These are
problems that must be overcome by the artist because they tend to intrude upon his or her
conception of the work. For example, in the early Italian Renaissance, bronze statues of
horses with a raised foreleg usually had a cannonball under that hoof. This was done because
the cannonball was needed to support the weight of the leg. In other words, the demands of
the laws of physics, not the sculptors aesthetic intentions, placed the ball there. That this
device was a necessary structural compromise is clear from the fact that the cannonball
quickly disappeared when sculptors learned how to strengthen the internal structure of a
statue with iron braces (iron being much stronger than bronze).
Even though the fine arts in the twentieth century often treat materials in new ways, the basic
difference in attitude of artists in relation to their materials in the fine arts and the applied arts
remains relatively constant. It would therefore not be too great an exaggeration to say that
practitioners of the fine arts work to overcome the limitations of their materials, whereas
those engaged in the applied arts work in concert with their materials.
242

[] Version 3.0
Paragraph 1: Although we now tend to refer to the various crafts according to the materials
used to construct themclay, glass, wood, fiber, and metalit was once common to think of
crafts in terms of function, which led to their being known as the "applied arts." Approaching
crafts from the point of view of function, we can divide them into simple categories:
containers, shelters and supports. There is no way around the fact that containers, shelters,
and supports must be functional. The applied arts are thus bound by the laws of physics,
which pertain to both the materials used in their making and the substances and things to be
contained, supported, and sheltered. These laws are universal in their application, regardless
of cultural beliefs, geography, or climate. If a pot has no bottom or has large openings in its
sides, it could hardly be considered a container in any traditional sense. Since the laws of
physics, not some arbitrary decision, have determined the general form of applied-art objects,
they follow basic patterns, so much so that functional forms can vary only within certain
limits. Buildings without roofs, for example, are unusual because they depart from the norm.
However, not all functional objects are exactly alike; that is why we recognize a Shang
Dynasty vase as being different from an Inca vase. What varies is not the basic form but the
incidental details that do not obstruct the objects primary function.
[Passage End]
---------------------------------------------------------------------------------------------------------------1. The word they in the passage refers to
A. Applied-art objects
B. The laws of physics
C. Containers
D. The sides of pots
2. Which of the following best expresses the essential information in the highlighted
sentence? Incorrect answer choices change the meaning in important ways or leave out
essential information.
A. Functional applied-art objects cannot vary much from the basic patterns determined
by the laws of physics.
B. The function of applied-art objects is determined by basic patterns in the laws of
physics.
C. Since functional applied-art objects vary only within certain limits, arbitrary decisions
cannot have determined their general form.
D. The general form of applied-art objects is limited by some arbitrary decision that is
not determined by the laws of physics.

243

Version 3.0

[]

3. According to paragraph 2, sculptors in the Italian Renaissance stopped using cannonballs


in bronze statues of horses because
A. They began using a material that made the statues weigh less
B. They found a way to strengthen the statues internally
C. The aesthetic tastes of the public had changed over time
D. The cannonballs added too much weight to the statues
4. Why does the author discuss the bronze statues of horses created by artists in the early
Italian Renaissance?
A. To provide an example of a problem related to the laws of physics that a fine artist
must overcome
B. To argue that fine artists are unconcerned with the laws of physics
C. To contrast the relative sophistication of modern artists in solving problems related to
the laws of physics
D. To note an exceptional piece of art constructed without the aid of technology
5. An introductory sentence for a brief summary of the passage is provided below. Complete
the summary by selecting the THREE answer choices that express the most important
ideas in the passage. Some sentences do not belong in the summary because they express
ideas that are not presented in the passage or are minor ideas in the passage.
This passage discusses fundamental differences between applied-art objects and fineart objects.

Answer Choices
a. The fine arts are only affected by the laws of physics because of the limitations of the
materials that are used.
b. Applied-art objects are bound by the laws of physics in two ways: by the materials used
to make them, and the function they are to serve.
c. Crafts are known as "applied arts" because it used to be common to think of them in
terms of their function.
d. In the fine arts, artists must work to overcome the limitations of their materials, but in
the applied arts, artists work in concert with their materials.
e. Making fine-art objects stable requires an understanding of the properties of mass,
weight, distribution, and stress.
f. In the twentieth century, artists working in the fine arts often treat materials in new
ways whereas applied arts specialists continue to think of crafts in terms of function.
244

[] Version 3.0
6. Directions: Complete the table below to summarize information about the two types of art
discussed in the passage. Match the appropriate statements to the types of art with which
they are associated.
TYPES OF ART STATEMENTS
The Applied Arts Select 3

The Fine Arts Select 2

Statements
a. An objects purpose is primarily aesthetic.
b. Objects serve a functional purpose.
c. The incidental details of objects do not vary.
d. Artists work to overcome the limitations of their materials.
e. The basic form of objects varies little across cultures.
f. Artists work in concert with their materials.
g. An objects place of origin is difficult to determine.

245

Version 3.0

[]

6.3.2 Loie Fuller


The United States dancer Loie Fuller (18621928) found theatrical dance in the late
nineteenth century artistically unfulfilling. She considered herself an artist rather than a mere
entertainer, and she, in turn, attracted the notice of other artists.
Fuller devised a type of dance that focused on the shifting play of lights and colors on the
voluminous skirts or draperies she wore, which she kept in constant motion principally
through movements of her arms, sometimes extended with wands concealed under her
costumes. She rejected the technical virtuosity of movement in ballet, the most prestigious
form of theatrical dance at that time, perhaps because her formal dance training was minimal.
Although her early theatrical career had included stints as an actress, she was not primarily
interested in storytelling or expressing emotions through dance; the drama of her dancing
emanated from her visual effects.
Although she discovered and introduced her art in the United States, she achieved her
greatest glory in Paris, where she was engaged by the Folies Bergre in 1892 and soon
became La Loie, the darling of Parisian audiences. Many of her dances represented
elements or natural objectsFire, the Lily, the Butterfly, and so onand thus accorded well
with the fashionable Art Nouveau style, which emphasized nature imagery and fluid, sinuous
lines. Her dancing also attracted the attention of French poets and painters of the period, for it
appealed to their liking for mystery, their belief in art for arts sake, a nineteenth-century idea
that art is valuable in itself rather than because it may have some moral or educational benefit,
and their efforts to synthesize form and content.
Fuller had scientific leanings and constantly experimented with electrical lighting (which was
then in its infancy), colored gels, slide projections, and other aspects of stage technology. She
invented and patented special arrangements of mirrors and concocted chemical dyes for her
draperies. Her interest in color and light paralleled the research of several artists of the period,
notably the painter Seurat, famed for his Pointillist technique of creating a sense of shapes
and light on canvas by applying extremely small dots of color rather than by painting lines.
One of Fullers major inventions was underlighting, in which she stood on a pane of frosted
glass illuminated from underneath. This was particularly effective in her Fire Dance (1895),
performed to the music of Richard Wagners Ride of the Valkyries. The dance caught the
eye of artist Henri de Toulouse-Lautrec, who depicted it in a lithograph.
As her technological expertise grew more sophisticated, so did the other aspects of her dances.
Although she gave little thought to music in her earliest dances, she later used scores by
Gluck, Beethoven, Schubert, Chopin, and Wagner, eventually graduating to Stravinsky, Faur,
Debussy, and Mussorgsky, composers who were then considered progressive. She began to
address more ambitious themes in her dances such as The Sea, in which her dancers invisibly
agitated a huge expanse of silk, played upon by colored lights. Always open to scientific
and technological innovations; she befriended the scientists Marie and Pierre Curie upon their
246

[] Version 3.0
discovery of radium and created a Radium Dance, which simulated the phosphorescence of
that element. She both appeared in films then in an early stage of developmentand
made them herself; the hero of her fairy-tale film Le Lys de la Vie (1919) was played
by Ren Clair, later a leading French film director.
At the Paris Exposition in 1900, she had her own theater, where, in addition to her own
dances, she presented pantomimes by the Japanese actress Sada Yocco. She assembled an allfemale company at this time and established a school around 1908, but neither survived her.
Although she is remembered today chiefly for her innovations in stage lighting, her activities
also touched Isadora Duncan and Ruth St. Denis, two other United States dancers who were
experimenting with new types of dance. She sponsored Duncans first appearance in Europe.
Her theater at the Paris Exposition was visited by St. Denis, who found new ideas about
stagecraft in Fullers work and fresh sources for her art in Sada Yoccos plays. In
1924 St. Denis paid tribute to Fuller with the duet Valse la Loie.
[Passage End]
---------------------------------------------------------------------------------------------------------------Paragraph 1: The United States dancer Loie Fuller (18621928) found theatrical dance in
the late nineteenth century artistically unfulfilling. She considered herself an artist rather than
a mere entertainer, and she, in turn, attracted the notice of other artists.
1. What can be inferred from paragraph 1 about theatrical dance in the late nineteenth
century?
A. It influenced many artists outside of the field of dance.
B. It was very similar to theatrical dance of the early nineteenth century.
C. It was more a form of entertainment than a form of serious art.
D. It was a relatively new art form in the United States.
Paragraph 2: Fuller devised a type of dance that focused on the shifting play of lights and
colors on the voluminous skirts or draperies she wore, which she kept in constant motion
principally through movements of her arms, sometimes extended with wands concealed under
her costumes. She rejected the technical virtuosity of movement in ballet, the most
prestigious form of theatrical dance at that time, perhaps because her formal dance training
was minimal. Although her early theatrical career had included stints as an actress, she was
not primarily interested in storytelling or expressing emotions through dance; the drama of
her dancing emanated from her visual effects.

247

Version 3.0

[]

2. According to paragraph 2, all of the following are characteristic of Fullers type of dance
EXCEPT
A. experimentation using color
B. large and full costumes
C. continuous movement of her costumes
D. technical virtuosity of movement
3. The word prestigious in the passage is closest in meaning to
A. highly regarded
B. financially rewarding
C. demanding
D. serious
4. Which of the sentences below best expresses the essential information in the highlighted
sentence in the passage? Incorrect choices change the meaning in important ways or leave
out essential information.
A. Fuller was more interested in dances visual impact than in its narrative or emotional
possibilities.
B. Fuller used visual effects to dramatize the stories and emotions expressed in her work.
C. Fuller believed that the drama of her dancing sprang from her emotional style of
storytelling.
D. Fullers focus on the visual effects of dance resulted from her early theatrical training
as an actress.
Paragraph 3: Although she discovered and introduced her art in the United States, she
achieved her greatest glory in Paris, where she was engaged by the Folies Bergre in 1892
and soon became La Loie, the darling of Parisian audiences. Many of her dances
represented elements or natural objectsFire, the Lily, the Butterfly, and so onand thus
accorded well with the fashionable Art Nouveau style, which emphasized nature imagery and
fluid, sinuous lines. Her dancing also attracted the attention of French poets and painters of
the period, for it appealed to their liking for mystery, their belief in art for arts sake, a
nineteenth-century idea that art is valuable in itself rather than because it may have some
moral or educational benefit, and their efforts to synthesize form and content.
5. The word engaged in the passage is closest in meaning to
A. noticed
B. praised
C. hired
D. attracted
248

[] Version 3.0
6. The word synthesize in the passage is closest in meaning to
A. improve
B. define
C. simplify
D. integrate
7. According to paragraph 3, why was Fullers work well received in Paris?
A. Parisian audiences were particularly interested in artists and artistic movements from
the United States.
B. Influential poets tried to interest dancers in Fullers work when she arrived in Paris.
C. Fullers work at this time borrowed directly from French artists working in other
media.
D. Fullers dances were in harmony with the artistic values already present in Paris.
Paragraph 4: Fuller had scientific leanings and constantly experimented with electrical
lighting (which was then in its infancy), colored gels, slide projections, and other aspects of
stage technology. She invented and patented special arrangements of mirrors and concocted
chemical dyes for her draperies. Her interest in color and light paralleled the research of
several artists of the period, notably the painter Seurat, famed for his Pointillist technique of
creating a sense of shapes and light on canvas by applying extremely small dots of color
rather than by painting lines. One of Fullers major inventions was underlighting, in which
she stood on a pane of frosted glass illuminated from underneath. This was particularly
effective in her Fire Dance (1895), performed to the music of Richard Wagners Ride of the
Valkyries. The dance caught the
eye of artist Henri de Toulouse-Lautrec, who depicted it in a lithograph.
8. According to paragraph 4, Fullers Fire Dance was notable in part for its
A. use of colored gels to illuminate glass
B. use of dyes and paints to create an image of fire
C. technique of lighting the dancer from beneath
D. draperies with small dots resembling the Pointillist technique of Seurat
Paragraph 5: As her technological expertise grew more sophisticated, so did the other
aspects of her dances. Although she gave little thought to music in her earliest dances, she
later used scores by Gluck, Beethoven, Schubert, Chopin, and Wagner, eventually graduating
to Stravinsky, Faur, Debussy, and Mussorgsky, composers who were then considered
progressive. She began to address more ambitious themes in her dances such as The Sea, in
which her dancers invisibly agitated a huge expanse of silk, played upon by colored lights.
Always open to scientific and technological innovations; she befriended the scientists Marie
and Pierre Curie upon their discovery of radium and created a Radium Dance, which
249

Version 3.0

[]

simulated the phosphorescence of that element. She both appeared in films then in an early
stage of developmentand made them herself; the hero of her fairy-tale film Le Lys de la Vie
(1919) was played by Ren Clair, later a leading French film director.
9. Why does the author mention Fullers The Sea?
A. To point out a dance of Fullers in which music did not play an important role
B. To explain why Fuller sometimes used music by progressive composers
C. To illustrate a particular way in which Fuller developed as an artist
D. To illustrate how Fullers interest in science was reflected in her work
10. The word agitated in the passage is closest in meaning to
A. emerged from beneath
B. created movement in
C. arranged themselves in
D. pretended to be
Paragraph 6: At the Paris Exposition in 1900, she had her own theater, where, in addition to
her own dances, she presented pantomimes by the Japanese actress Sada Yocco. She
assembled an all-female company at this time and established a school around 1908, but
neither survived her. Although she is remembered today chiefly for her innovations in stage
lighting, her activities also touched Isadora Duncan and Ruth St. Denis, two other United
States dancers who were experimenting with new types of dance. She sponsored Duncans
first appearance in Europe. Her theater at the Paris Exposition was visited by St. Denis, who
found new ideas about stagecraft in Fullers work and fresh sources for her art in Sada
Yoccos plays. In 1924 St. Denis paid tribute to Fuller with the duet Valse la Loie.
11. According to paragraph 6, what was true of Fullers theater at the Paris Exposition?
A. It presented some works that were not by Fuller.
B. It featured performances by prominent male as well as female dancers.
C. It became a famous school that is still named in honor of Fuller.
D. It continued to operate as a theater after Fuller died.
12. The passage mentions which of the following as a dance of Fullers that was set to music?
A. Fire Dance
B. Radium Dance
C. Le Lys de la Vie
D. Valse la Loie

250

[] Version 3.0
As her technological expertise grew more sophisticated, so did the other aspects of her dances.
Although she gave little thought to music in her earliest dances, she later used scores by
Gluck, Beethoven, Schubert, Chopin, and Wagner, eventually graduating to Stravinsky, Faur,
Debussy, and Mussorgsky, composers who were then considered progressive. She began to
address more ambitious themes in her dances such as The Sea, in which her dancers invisibly
agitated a huge expanse of silk, played upon by colored lights. Always open to scientific
and technological innovations; she befriended the scientists Marie and Pierre Curie upon their
discovery of radium and created a Radium Dance, which simulated the phosphorescence of
that element. She both appeared in films then in an early stage of developmentand
made them herself; the hero of her fairy-tale film Le Lys de la Vie (1919) was played
by Ren Clair, later a leading French film director.
13. Look at the four squares that indicate where the following sentence could be added to
the passage.
For all her originality in dance, her interests expanded beyond it into newly emerging
artistic media.
Where would the sentence best fit?
14. Directions: An introductory sentence for a brief summary of the passage is provided below.
Complete the summary by selecting the THREE answer choices that express the most
important ideas in the passage. Some sentences do not belong in the summary because
they express ideas that are not presented in the passage or are minor ideas in the passage.
Loie Fuller was an important and innovative dancer.

Answer Choices
A. Fuller believed that audiences in the late nineteenth century had lost interest in most
theatrical dance.
B. Fuller transformed dance in part by creating dance interpretations of works by poets
and painters.
C. Fullers work influenced a number of other dancers who were interested in
experimental dance.
D. Fuller introduced many technical innovations to the staging of theatrical dance.
E. Fuller continued to develop throughout her career, creating more complex works and
exploring new artistic media.
F. By the 1920s, Fullers theater at the Paris Exhibition had become the world center for
innovative dance.
251

Version 3.0

[]

6.3.3 Lascaux Cave Paintings


In Southwest France in the 1940s, playing children discovered Lascaux Grotto, a series of
narrow cave chambers that contain huge prehistoric paintings of animals. Many of these
beasts are as large as 16 feet (almost 5 meters). Some follow each other in solemn parades,
but others swirl about, sideways and upside down. The animals are bulls, wild horses,
reindeer, bison, and mammoths outlined with charcoal and painted mostly in reds, yellow,
and browns. Scientific analysis reveals that the colors were derived from ocher and other iron
oxides ground into a fine powder. Methods of applying color varied: some colors were
brushed or smeared on rock surfaces and others were blown or sprayed. It is possible that
tubes made from animal bones were used for spraying because hollow bones, some stained
with pigment, have been found nearby.
One of the most puzzling aspects of the paintings is their location. Other rock paintingsfor
example, those of Bushmen in South Africaare either located near cave entrances or
completely in the open. Cave paintings in France and Spain, however, are in recesses and
caverns far removed from original cave entrances. This means that artists were forced to
work in cramped spaces and without sources of natural light. It also implies that whoever
made them did not want them to be easily found. Since cave dwellers normally lived close to
entrances, there must have been some reason why so many generations of Lascaux cave
dwellers hid their art.
Scholars offer three related but different opinions about the mysterious origin and
significance of these paintings. One opinion is that the paintings were a record of seasonal
migrations made by herds. Because some paintings were made directly over others,
obliterating them, it is probable that a paintings value ended with the migration it pictured.
Unfortunately, this explanation fails to explain the hidden locations, unless the migrations
were celebrated with secret ceremonies.
Another opinion is that the paintings were directly related to hunting and were an essential
part of a special preparation ceremony. This opinion holds that the pictures and whatever
ceremony they accompanied were an ancient method of psychologically motivating hunters.
It is conceivable that before going hunting the hunters would draw or study pictures of
animals and imagine a successful hunt. Considerable support exists for this opinion because
several animals in the pictures are wounded by arrows and spears. This opinion also attempts
to solve the overpainting by explaining that an animals picture had no further use after the
hunt.
A third opinion takes psychological motivation much further into the realm of tribal
ceremonies and mystery: the belief that certain animals assumed mythical significance as
ancient ancestors or protectors of a given tribe or clan. Two types of images substantiate this
theory: the strange, indecipherable geometric shapes that appear near some animals, and the
few drawings of men. Wherever men appear they are crudely drawn and their bodies are
elongated and rigid. Some men are in a prone position and some have bird or animal heads.
252

[] Version 3.0
Advocates for this opinion point to reports from people who have experienced a trance state,
a highly suggestive state of low consciousness between waking and sleeping. Uniformly,
these people experienced weightlessness and the sensation that their bodies were being
stretched lengthwise. Advocates also point to people who believe that the forces of nature are
inhabited by spirits, particularly shamans* who believe that an animals spirit and energy is
transferred to them while in a trance. One Lascaux narrative picture, which shows a man with
a birdlike head and a wounded animal, would seem to lend credence to this third opinion, but
there is still much that remains unexplained. For example, where is the proof that the man in
the picture is a shaman? He could as easily be a hunter wearing a head mask. Many tribal
hunters, including some Native Americans, camouflaged themselves by wearing animal
heads and hides.
Perhaps so much time has passed that there will never be satisfactory answers to the cave
images, but their mystique only adds to their importance. Certainly a great art exists, and by
its existence reveals that ancient human beings were not without intelligence, skill, and
sensitivity.
[Passage End]
---------------------------------------------------------------------------------------------------------------Paragraph 1: In Southwest France in the 1940s, playing children discovered Lascaux Grotto,
a series of narrow cave chambers that contain huge prehistoric paintings of animals. Many of
these beasts are as large as 16 feet (almost 5 meters). Some follow each other in solemn
parades, but others swirl about, sideways and upside down. The animals are bulls, wild
horses, reindeer, bison, and mammoths outlined with charcoal and painted mostly in reds,
yellow, and browns. Scientific analysis reveals that the colors were derived from ocher and
other iron oxides ground into a fine powder. Methods of applying color varied: some colors
were brushed or smeared on rock surfaces and others were blown or sprayed. It is possible
that tubes made from animal bones were used for spraying because hollow bones, some
stained with pigment, have been found nearby.
1. The word others in the passage refers to
A. Chambers
B. Paintings
C. Beasts
D. Parades
2. The word Methods in the passage is closest in meaning to
A. Ways
B. Shades
C. Stages
D. Rules
253

Version 3.0

[]

3. What are the bones found in the Lascaux caves believed to indicate?
A. Wild animals sometimes lived in the cave chambers.
B. Artists painted pictures on both walls and bones.
C. Artists ground them into a fine powder to make paint.
D. Artists developed special techniques for painting the walls.
Paragraph 2 One of the most puzzling aspects of the paintings is their location. Other rock
paintingsfor example, those of Bushmen in South Africaare either located near cave
entrances or completely in the open. Cave paintings in France and Spain, however, are in
recesses and caverns far removed from original cave entrances. This means that artists were
forced to work in cramped spaces and without sources of natural light. It also implies that
whoever made them did not want them to be easily found. Since cave dwellers normally lived
close to entrances, there must have been some reason why so many generations of Lascaux
cave dwellers hid their art.
4. Why does the author mention Bushmen in South Africa in paragraph 2?
A. To suggest that ancient artists from all over the world painted animals on rocks
B. To contrast the location of their rock paintings to those found at Lascaux
C. To support the claim that early artists worked in cramped spaces
D. To give an example of other artists who painted in hidden locations
5. What can be inferred from paragraph 2 about cave painters in France and Spain?
A. They also painted rocks outside caves.
B. They did not live close to the cave entrances.
C. They developed their own sources of light to use while painting.
D. Their painting practices did not last for many years.
Paragraph 3Scholars offer three related but different opinions about the mysterious origin
and significance of these paintings. One opinion is that the paintings were a record of
seasonal migrations made by herds. Because some paintings were made directly over others,
obliterating them, it is probable that a paintings value ended with the migration it pictured.
Unfortunately, this explanation fails to explain the hidden locations, unless the migrations
were celebrated with secret ceremonies.
6. Why does the author mention secret ceremonies?
A. To present a common opinion held by many scholars
B. To suggest a similarity between two opinions held by scholars
C. To suggest a possible explanation for a weakness in an opinion expressed in passage
D. To give evidence that contradicts a major opinion expressed in the passage
254

[] Version 3.0
Paragraph 4: Another opinion is that the paintings were directly related to hunting and were
an essential part of a special preparation ceremony. This opinion holds that the pictures and
whatever ceremony they accompanied were an ancient method of psychologically motivating
hunters. It is conceivable that before going hunting the hunters would draw or study pictures
of animals and imagine a successful hunt. Considerable support exists for this opinion
because several animals in the pictures are wounded by arrows and spears. This opinion also
attempts to solve the overpainting by explaining that an animals picture had no further use
after the hunt.
7. The word accompanied in the passage is closest in meaning to
A. Represented
B. Developed into
C. Were associated with
D. Came after
8. According to paragraph 4, why do some scholars believe that the paintings were related to
hunting?
A. Because some tools used for painting were also used for hunting
B. Because cave inhabitants were known to prefer animal food rather than plant food
C. Because some of the animals are shown wounded by weapons
D. Because many hunters were also typically painters
Paragraph 5: A third opinion takes psychological motivation much further into the realm of
tribal ceremonies and mystery: the belief that certain animals assumed mythical significance
as ancient ancestors or protectors of a given tribe or clan. Two types of images substantiate
this theory: the strange, indecipherable geometric shapes that appear near some animals, and
the few drawings of men. Wherever men appear they are crudely drawn and their bodies are
elongated and rigid. Some men are in a prone position and some have bird or animal heads.
Advocates for this opinion point to reports from people who have experienced a trance state,
a highly suggestive state of low consciousness between waking and sleeping. Uniformly,
these people experienced weightlessness and the sensation that their bodies were being
stretched lengthwise. Advocates also point to people who believe that the forces of nature are
inhabited by spirits, particularly shamans who believe that an animals spirit and energy is
transferred to them while in a trance. One Lascaux narrative picture, which shows a man with
a birdlike head and a wounded animal, would seem to lend credence to this third opinion, but
there is still much that remains unexplained. For example, where is the proof that the man in
the picture is a shaman? He could as easily be a hunter wearing a head mask. Many tribal
hunters, including some Native Americans, camouflaged themselves by wearing animal
heads and hides.

255

Version 3.0

[]

9. According to paragraph 5, why do some scholars refer to a trance state to help understand
the cave paintings?
A. To explain the state of consciousness the artists were in when they painted their
pictures
B. To demonstrate the mythical significance of the strange geometric shapes
C. To indicate that trance states were often associated with activities that took place
inside caves
D. To give a possible reason for the strange appearance of the men painted on the cave
walls
10. According to paragraph 5, if the man pictured with the birdlike head is not a shaman, he
may have worn the head mask
A. to look like an animal while a hunt took place
B. to frighten off other hunters competing for food
C. to prove that he is not a shaman
D. to resist forces of nature thought to be present in animals
Paragraph 6: Perhaps so much time has passed that there will never be satisfactory answers
to the cave images, but their mystique only adds to their importance. Certainly a great art
exists, and by its existence reveals that ancient human beings were not without intelligence,
skill, and sensitivity.
11. According to paragraph 6, why might the puzzling questions about the paintings never be
answered?
A. Keeping the paintings a mystery will increase their importance.
B. The artists hid their tools with great intelligence and skill.
C. Too many years have gone by since the images were painted.
D. Answering the questions is not very important to scholars.
Paragraph 2: One of the most puzzling aspects of the paintings is their location. Other rock
paintingsfor example, those of Bushmen in South Africaare either located near cave
entrances or completely in the open. Cave paintings in France and Spain, however, are in
recesses and caverns far removed from original cave entrances. This means that artists
were forced to work in cramped spaces and without sources of natural light. It also implies
that whoever made them did not want them to be easily found. Since cave dwellers
normally lived close to entrances; there must have been some reason why so many
generations of Lascaux cave dwellers hid their art.

256

[] Version 3.0
12. Look at the four squares that indicate where the following sentence could be added to
the passage.
This made it easy for the artists to paint and display them for the rest of the cave
dwellers.
Where would the sentence best fit?
13. Directions: An introductory sentence for a brief summary of the passage is provided
below. Complete the summary by selecting the THREE answer choices that express the
most important ideas in the passage. Some sentences do not belong in the summary
because they express ideas that are not presented in the passage or are minor ideas in the
passage.
Scholars have wondered about the meaning of the subjects, location, and over
painting of Lascaux cave images.

Answer Choices
a. The paintings may have recorded information about animal migrations, and may only
have been useful for one migration at a time.
b. The human figures represented in the paintings appear to be less carefully shaped than
those of animals.
c. It is possible that the animals in the paintings were of mythical significance to the
tribe, and the paintings reflected an important spiritual practice.
d. Unlike painters of the recently discovered paintings, other Lascaux cave painters
usually painted on rocks near cave entrances or in open spaces outside the caves.
e. Some scholars believe that the paintings motivated hunters by allowing them to
picture a successful hunt.
f. Scientific analysis suggests that paintings were sprayed onto the rock walls with tubes
made from animal bones.

257

Version 3.0

6.4

[]

History and Humanities

6.4.1 Early cinema


The cinema did not emerge as a form of mass consumption until its technology evolved from
the initial peepshow format to the point where images were projected on a screen in a
darkened theater. In the peepshow format, a film was viewed through a small opening in a
machine that was created for that purpose. Thomas Edisons peepshow device, the
Kinetoscope, was introduced to the public in 1894. It was designed for use in Kinetoscope
parlors, or arcades, which contained only a few individual machines and permitted only one
customer to view a short, 50-foot film at any one time. The first Kinetoscope parlors
contained five machines. For the price of 25 cents (or 5 cents per machine), customers moved
from machine to machine to watch five different films (or, in the case of famous prizefights,
successive rounds of a single fight).
These Kinetoscope arcades were modeled on phonograph parlors, which had proven
successful for Edison several years earlier. In the phonograph parlors, customers listened to
recordings through individual ear tubes, moving from one machine to the next to hear
different recorded speeches or pieces of music. The Kinetoscope parlors functioned in a
similar way. Edison was more interested in the sale of Kinetoscopes (for roughly $1,000
apiece) to these parlors than in the films that would be run in them (which cost approximately
$10 to $15 each). He refused to develop projection technology, reasoning that if he made and
sold projectors, then exhibitors would purchase only one machine-a projector-from him
instead of several.
Exhibitors, however, wanted to maximize their profits, which they could do more readily
by projecting a handful of films to hundreds of customers at a time (rather than one at a time)
and by charging 25 to 50 cents admission. About a year after the opening of the first
Kinetoscope parlor in 1894, showmen such as Louis and Auguste Lumiere, Thomas Armat
and Charles Francis Jenkins, and Orville and Woodville Latham (with the assistance of
Edisons former assistant, William Dickson) perfected projection devices. These early
projection devices were used in vaudeville theaters, legitimate theaters, local town halls,
makeshift storefront theaters, fairgrounds, and amusement parks to show films to a mass
audience.
With the advent of projection in 1895-1896, motion pictures became the ultimate form of
mass consumption. Previously, large audiences had viewed spectacles at the theater, where
vaudeville, popular dramas, musical and minstrel shows, classical plays, lectures, and slideand-lantern shows had been presented to several hundred spectators at a time. But the movies
differed significantly from these other forms of entertainment, which depended on either live
performance or (in the case of the slide-and-lantern shows) the active involvement of a
master of ceremonies who assembled the final program.

258

[] Version 3.0
Although early exhibitors regularly accompanied movies with live acts, the substance of the
movies themselves is mass-produced, prerecorded material that can easily be reproduced by
theaters with little or no active participation by the exhibitor. Even though early exhibitors
shaped their film programs by mixing films and other entertainments together in whichever
way they thought would be most attractive to audiences or by accompanying them with
lectures, their creative control remained limited. What audiences came to see was the
technological marvel of the movies; the lifelike reproduction of the commonplace motion of
trains, of waves striking the shore, and of people walking in the street; and the magic made
possible by trick photography and the manipulation of the camera.
With the advent of projection, the viewers relationship with the image was no longer private,
as it had been with earlier peepshow devices such as the Kinetoscope and the Mutoscope,
which was a similar machine that reproduced motion by means of successive images on
individual photographic cards instead of on strips of celluloid. It suddenly became public-an
experience that the viewer shared with dozens, scores, and even hundreds of others. At the
same time, the image that the spectator looked at expanded from the minuscule peepshow
dimensions of 1 or 2 inches (in height) to the life-size proportions of 6 or 9 feet.
[Passage End]
----------------------------------------------------------------------------------------------------------------

Paragraph 1: The cinema did not emerge as a form of mass consumption until its technology
evolved from the initial peepshow format to the point where images were projected on a
screen in a darkened theater. In the peepshow format, a film was viewed through a small
opening in a machine that was created for that purpose. Thomas Edisons peepshow device,
the Kinetoscope, was introduced to the public in 1894. It was designed for use in Kinetoscope
parlors, or arcades, which contained only a few individual machines and permitted only one
customer to view a short, 50-foot film at any one time. The first Kinetoscope parlors
contained five machines. For the price of 25 cents (or 5 cents per machine), customers moved
from machine to machine to watch five different films (or, in the case of famous prizefights,
successive rounds of a single fight).
1. According to paragraph 1, all of the following were true of viewing films in Kinetoscope
parlors EXCEPT:
A. One individual at a time viewed a film.
B. Customers could view one film after another.
C. Prizefights were the most popular subjects for films.
D. Each film was short.
Paragraph 2: These Kinetoscope arcades were modeled on phonograph parlors, which had
proven successful for Edison several years earlier. In the phonograph parlors, customers
listened to recordings through individual ear tubes, moving from one machine to the next to
259

Version 3.0

[]

hear different recorded speeches or pieces of music. The Kinetoscope parlors functioned in a
similar way. Edison was more interested in the sale of Kinetoscopes (for roughly $1,000
apiece) to these parlors than in the films that would be run in them (which cost approximately
$10 to $15 each). He refused to develop projection technology, reasoning that if he made and
sold projectors, then exhibitors would purchase only one machine-a projector-from him
instead of several.
2. The author discusses phonograph parlors in paragraph 2 in order to
A. Explain Edisons financial success
B. Describe the model used to design Kinetoscope parlors
C. Contrast their popularity to that of Kinetoscope parlors
D. Illustrate how much more technologically advanced Kinetoscope parlors were
3. Which of the sentences below best expresses the essential information in the highlighted
sentence from the passage? Incorrect answer choices change the meaning in important
ways or leave out essential information.
A. Edison was more interested in developing a variety of machines than in developing a
technology based on only one.
B. Edison refused to work on projection technology because he did not think exhibitors
would replace their projectors with newer machines.
C. Edison did not want to develop projection technology because it limited the number
of machines he could sell.
D. Edison would not develop projection technology unless exhibitors agreed to purchase
more than one projector from him.
Exhibitors, however, wanted to maximize their profits, which they could do more readily by
projecting a handful of films to hundreds of customers at a time (rather than one at a time)
and by charging 25 to 50 cents admission. About a year after the opening of the first
Kinetoscope parlor in 1894, showmen such as Louis and Auguste Lumiere, Thomas Armat
and Charles Francis Jenkins, and Orville and Woodville Latham (with the assistance of
Edisons former assistant, William Dickson) perfected projection devices. These early
projection devices were used in vaudeville theaters, legitimate theaters, local town halls,
makeshift storefront theaters, fairgrounds, and amusement parks to show films to a mass
audience.
4. The word readily in the passage is closest in meaning to
A. Frequently
B. Easily
C. Intelligently
D. Obviously
260

[] Version 3.0
5. The word assistance in the passage is closest in meaning to
A. Criticism
B. Leadership
C. Help
D. Approval
Paragraph 4: With the advent of projection in 1895-1896, motion pictures became the
ultimate form of mass consumption. Previously, large audiences had viewed spectacles at the
theater, where vaudeville, popular dramas, musical and minstrel shows, classical plays,
lectures, and slide-and-lantern shows had been presented to several hundred spectators at a
time. But the movies differed significantly from these other forms of entertainment, which
depended on either live performance or (in the case of the slide-and-lantern shows) the active
involvement of a master of ceremonies who assembled the final program.
6. According to paragraph 4, how did the early movies differ from previous spectacles that
were presented to large audiences?
A. They were a more expensive form of entertainment.
B. They were viewed by larger audiences.
C. They were more educational.
D. They did not require live entertainers.
Paragraph 5: Although early exhibitors regularly accompanied movies with live acts, the
substance of the movies themselves is mass-produced, prerecorded material that can easily be
reproduced by theaters with little or no active participation by the exhibitor. Even though
early exhibitors shaped their film programs by mixing films and other entertainments together
in whichever way they thought would be most attractive to audiences or by accompanying
them with lectures, their creative control remained limited. What audiences came to see was
the technological marvel of the movies; the lifelike reproduction of the commonplace motion
of trains, of waves striking the shore, and of people walking in the street; and the magic made
possible by trick photography and the manipulation of the camera.
7. According to paragraph 5, what role did early exhibitors play in the presentation of movies
in theaters?
A. They decided how to combine various components of the film program.
B. They advised film-makers on appropriate movie content.
C. They often took part in the live-action performances.
D. They produced and prerecorded the material that was shown in the theaters.

261

Version 3.0

[]

Paragraph 6: With the advent of projection, the viewers relationship with the image was no
longer private, as it had been with earlier peepshow devices such as the Kinetoscope and the
Mutoscope, which was a similar machine that reproduced motion by means of successive
images on individual photographic cards instead of on strips of celluloid. It suddenly became
public-an experience that the viewer shared with dozens, scores, and even hundreds of others.
At the same time, the image that the spectator looked at expanded from the minuscule
peepshow dimensions of 1 or 2 inches (in height) to the life-size proportions of 6 or 9 feet.
8. Which of the following is mentioned in paragraph 6 as one of the ways the Mutoscope
differed from the Kinetoscope?
A. Sound and motion were simultaneously produced in the Mutoscope.
B. More than one person could view the images at the same time with the Mutoscope.
C. The Mutoscope was a less sophisticated earlier prototype of the Kinetoscope.
D. A different type of material was used to produce the images used in the Mutocope.
9. The word it in the passage refers to
A. The advent of projection
B. The viewers relationship with the image
C. A similar machine
D. Celluloid
10. According to paragraph 6, the images seen by viewers in the earlier peepshows, compared
to the images projected on the screen, were relatively
A. Small in size
B. Inexpensive to create
C. Unfocused
D. Limited in subject matter
11. The word expanded in the passage is closest in meaning to
A. Was enlarged
B. Was improved
C. Was varied
D. Was rejected

262

[] Version 3.0
Paragraph 3: Exhibitors, however, wanted to maximize their profits, which they could do
more readily by projecting a handful of films to hundreds of customers at a time (rather than
one at a time) and by charging 25 to 50 cents admission. About a year after the opening of
the first Kinetoscope parlor in 1894, showmen such as Louis and Auguste Lumiere, Thomas
Armat and Charles Francis Jenkins, and Orville and Woodville Latham (with the assistance
of Edisons former assistant, William Dickson) perfected projection devices. These early
projection devices were used in vaudeville theaters, legitimate theaters, local town halls,
makeshift storefront theaters, fairgrounds, and amusement parks to show films to a mass
audience.
12. Look at the four squares [] that indicate where the following sentence can be added to
the passage.
When this widespread use of projection technology began to hurt his Kinetoscope
business, Edison acquired a projector developed by Ar mat and introduced it as
Edisons latest marvel, the Vitascope."
Where would the sentence best fit?
13. Directions: An introductory sentence for a brief summary of the passage is provided
below. Complete the summary by selecting the THREE answer choices that express the
most important ideas in the passage. Some answer choices do not belong in the summary
because they express ideas that are not presented in the passage or are minor ideas in the
passage.
The technology for modern cinema evolved at the end of the nineteenth century.

Answer Choices
a. Kinetoscope parlors for viewing films were modeled on phonograph parlors.
b. Thomas Edisons design of the Kinetoscope inspired the development of large screen
projection.
c. Early cinema allowed individuals to use special machines to view films privately.
d. Slide-and-lantern shows had been presented to audiences of hundreds of spectators.
e. The development of projection technology made it possible to project images on a
large screen.
f. Once film images could be projected, the cinema became form of mass consumption.

263

Version 3.0

[]

6.4.2 Artisans and Industrialization


Before 1815 manufacturing in the United States had been done in homes or shops by skilled
artisans. As master craft workers, they imparted the knowledge of their trades to apprentices
and journeymen. In addition, women often worked in their homes part-time, making finished
articles from raw material supplied by merchant capitalists. After 1815 this older form of
manufacturing began to give way to factories with machinery tended by unskilled or
semiskilled laborers. Cheap transportation networks, the rise of cities, and the availability of
capital and credit all stimulated the shift to factory production.
The creation of a labor force that was accustomed to working in factories did not occur easily.
Before the rise of the factory, artisans had worked within the home. Apprentices were
considered part of the family, and masters were responsible not only for teaching their
apprentices a trade but also for providing them some education and for supervising their
moral behavior. Journeymen knew that if they perfected their skill, they could become
respected master artisans with their own shops. Also, skilled artisans did not work by the
clock, at a steady pace, but rather in bursts of intense labor alternating with more leisurely
time.
The factory changed that. Goods produced by factories were not as finished or elegant as
those done by hand, and pride in craftsmanship gave way to the pressure to increase rates of
productivity. The new methods of doing business involved a new and stricter sense of time.
Factory life necessitated a more regimented schedule, where work began at the sound of a
bell and workers kept machines going at a constant pace. At the same time, workers were
required to discard old habits, for industrialism demanded a worker who was alert,
dependable, and self-disciplined. Absenteeism and lateness hurt productivity and, since work
was specialized, disrupted the regular factory routine. Industrialization not only produced a
fundamental change in the way work was organized; it transformed the very nature of work.
The first generation to experience these changes did not adopt the new attitudes easily. The
factory clock became the symbol of the new work rules. One mill worker who finally quit
complained revealingly about obedience to the ding-dong of the bell-just as though we are
so many living machines. With the loss of personal freedom also came the loss of standing
in the community. Unlike artisan workshops in which apprentices worked closely with the
masters supervising them, factories sharply separated workers from management. Few
workers rose through the ranks to supervisory positions, and even fewer could achieve the
artisans dream of setting up ones own business. Even well-paid workers sensed their decline
in status.
In this newly emerging economic order, workers sometimes organized to protect their rights
and traditional ways of life. Craft workers such as carpenters, printers, and tailors formed
unions, and in 1834 individual unions came together in the National Trades Union. The labor
movement gathered some momentum in the decade before the Panic of 1837, but in the
depression that followed, labors strength collapsed. During hard times, few workers were
264

[] Version 3.0
willing to strike or engage in collective action. And skilled craft workers, who spearheaded
the union movement, did not feel a particularly strong bond with semiskilled factory workers
and unskilled laborers. More than a decade of agitation did finally bring a workday shortened
to 10 hours to most industries by the 1850s, and the courts also recognized workers right to
strike, but these gains had little immediate impact.
Workers were united in resenting the industrial system and their loss of status, but they were
divided by ethnic and racial antagonisms, gender, conflicting religious perspectives,
occupational differences, political party loyalties, and disagreements over tactics. For them,
the factory and industrialism were not agents of opportunity but reminders of their loss of
independence and a measure of control over their lives. As United States society became
more specialized and differentiated, greater extremes of wealth began to appear. And as the
new markets created fortunes for the few, the factory system lowered the wages of workers
by dividing labor into smaller, less skilled tasks.
[Passage End]
---------------------------------------------------------------------------------------------------------------Paragraph 1: Before 1815 manufacturing in the United States had been done in homes or
shops by skilled artisans. As master craft workers, they imparted the knowledge of their
trades to apprentices and journeymen. In addition, women often worked in their homes parttime, making finished articles from raw material supplied by merchant capitalists. After 1815
this older form of manufacturing began to give way to factories with machinery tended by
unskilled or semiskilled laborers. Cheap transportation networks, the rise of cities, and the
availability of capital and credit all stimulated the shift to factory production.
1. Which of the following can be inferred from the passage about articles manufactured
before 1815?
A. They were primarily produced by women.
B. They were generally produced in shops rather than in homes.
C. They were produced with more concern for quality than for speed of production.
D. They were produced mostly in large cities with extensive transportation networks.
Paragraph 2: The creation of a labor force that was accustomed to working in factories did
not occur easily. Before the rise of the factory, artisans had worked within the home.
Apprentices were considered part of the family, and masters were responsible not only for
teaching their apprentices a trade but also for providing them some education and for
supervising their moral behavior. Journeymen knew that if they perfected their skill, they
could become respected master artisans with their own shops. Also, skilled artisans did not
work by the clock, at a steady pace, but rather in bursts of intense labor alternating with more
leisurely time.

265

Version 3.0

[]

2. Which of the sentences below best expresses the essential information in the highlighted
sentence in the passage? Incorrect answer choices change the meaning in important ways
or leave out essential information.
A. Masters demanded moral behavior from apprentices but often treated them
irresponsibly.
B. The responsibilities of the master to the apprentice went beyond the teaching of a
trade.
C. Masters preferred to maintain the trade within the family by supervising and
educating the younger family members.
D. Masters who trained members of their own family as apprentices demanded
excellence from them.
Paragraph 3: The factory changed that. Goods produced by factories were not as finished or
elegant as those done by hand, and pride in craftsmanship gave way to the pressure to
increase rates of productivity. The new methods of doing business involved a new and stricter
sense of time. Factory life necessitated a more regimented schedule, where work began at the
sound of a bell and workers kept machines going at a constant pace. At the same time,
workers were required to discard old habits, for industrialism demanded a worker who was
alert, dependable, and self-disciplined. Absenteeism and lateness hurt productivity and, since
work was specialized, disrupted the regular factory routine. Industrialization not only
produced a fundamental change in the way work was organized; it transformed the very
nature of work.
3. The word disrupted in the passage is closest inmeaning to
A. Prolonged
B. Established
C. Followed
D. Upset
Paragraph 4: The first generation to experience these changes did not adopt the new
attitudes easily. The factory clock became the symbol of the new work rules. One mill worker
who finally quit complained revealingly about obedience to the ding-dong of the bell-just as
though we are so many living machines. With the loss of personal freedom also came the
loss of standing in the community. Unlike artisan workshops in which apprentices worked
closely with the masters supervising them, factories sharply separated workers from
management. Few workers rose through the ranks to supervisory positions, and even fewer
could achieve the artisans dream of setting up ones own business. Even well-paid workers
sensed their decline in status.

266

[] Version 3.0
4. In paragraph 4, the author includes the quotation from a mill worker in order to
A. Support the idea that it was difficult for workers to adjust to working in factories
B. To show that workers sometimes quit because of the loud noise made by factory
machinery
C. Argue that clocks did not have a useful function in factories
D. Emphasize that factories were most successful when workers revealed their
complaints
5. All of the following are mentioned in paragraph 4 as consequences of the new system for
workers EXCEPT a loss of
A. Freedom
B. Status in the community
C. Opportunities for advancement
D. Contact among workers who were not managers
Paragraph 5: In this newly emerging economic order, workers sometimes organized to
protect their rights and traditional ways of life. Craft workers such as carpenters, printers, and
tailors formed unions, and in 1834 individual unions came together in the National Trades
Union. The labor movement gathered some momentum in the decade before the Panic of
1837, but in the depression that followed, labors strength collapsed. During hard times, few
workers were willing to strike or engage in collective action. And skilled craft workers, who
spearheaded the union movement, did not feel a particularly strong bond with semiskilled
factory workers and unskilled laborers. More than a decade of agitation did finally bring a
workday shortened to 10 hours to most industries by the 1850s, and the courts also
recognized workers right to strike, but these gains had little immediate impact.
6. The phrase gathered some momentum in the passage is closest in meaning to
A. Made progress
B. Became active
C. Caused changes
D. Combined forces
7. The word spearheaded in the passage is closest in meaning to
A. Led
B. Accepted
C. Changed
D. Resisted

267

Version 3.0

[]

8. Which of the following statements about the labor movement of the 1800s is supported
by paragraph 5?
A. It was most successful during times of economic crisis.
B. Its primary purpose was to benefit unskilled laborers.
C. It was slow to improve conditions for workers.
D. It helped workers of all skill levels form a strong bond with each other.
Paragraph 6: Workers were united in resenting the industrial system and their loss of status,
but they were divided by ethnic and racial antagonisms, gender, conflicting religious
perspectives, occupational differences, political party loyalties, and disagreements over
tactics. For them, the factory and industrialism were not agents of opportunity but reminders
of their loss of independence and a measure of control over their lives. As United States
society became more specialized and differentiated, greater extremes of wealth began to
appear. And as the new markets created fortunes for the few, the factory system lowered the
wages of workers by dividing labor into smaller, less skilled tasks.
9. The author identifies political party loyalties, and disagreements over tactics as two of
several factors that
A. Encouraged workers to demand higher wages
B. Created divisions among workers
C. Caused work to become more specialized
D. Increased workers resentment of the industrial system
10. The word them in the passage refers to
A. Workers
B. Political patty loyalties
C. Disagreements over tactics
D. Agents of opportunity
Before 1815 manufacturing in the United States had been done in homes or shops by skilled
artisans. As master craft workers, they imparted the knowledge of their trades to
apprentices and journeymen. In addition, women often worked in their homes part-time,
making finished articles from raw material supplied by merchant capitalists. After 181 5
this older form of manufacturing began to give way to factories with machinery tended by
unskilled or semiskilled laborers. Cheap transportation networks, the rise of cities, and the
availability of capital and credit all stimulated the shift to factory production.

268

[] Version 3.0
11. Look at the four squares that indicate where the following sentence can be added to the
passage.
This new form of manufacturing depended on the movement of goods to distant
locations and a centralized source of laborers.
Where would the sentence best fit?
12. Directions: Complete the table below by indicating which of the ans wer choices describe
characteristics of the period before 1815 and which describe characteristics of the 181 5-1
850 period.
Before 1815

1815-1850

Answer choices
a. A united, highly successful labor movement took shape.
b. Workers took pride in their workmanship.
c. The income gap between the rich and the poor increased greatly.
d. Transportation networks began to decline.
e. Emphasis was placed on following schedules.
f. Workers went through an extensive period of training.
g. Few workers expected to own their own businesses.

269

Version 3.0

[]

6.4.3 Nineteenth-Century Politics in the United States


The development of the modern presidency in the United States began with Andrew Jackson
who swept to power in 1829 at the head of the Democratic Party and served until 1837.
During his administration, he immeasurably enlarged the power of the presidency. The
President is the direct representative of the American people, he lectured the Senate when it
opposed him. He was elected by the people, and is responsible to them. With this
declaration, Jackson redefined the character of the presidential office and its relationship to
the people.
During Jacksons second term, his opponents had gradually come together to form the Whig
party. Whigs and Democrats held different attitudes toward the changes brought about by the
market, banks, and commerce. The Democrats tended to view society as a continuing conflict
between the people-farmers, planters, and workers-and a set of greedy aristocrats. This
"paper money aristocracy" of bankers and investors manipulated the banking system for their
own profit, Democrats claimed, and sapped the nations virtue by encouraging speculation
and the desire for sudden, unearned wealth. The Democrats wanted the rewards of the market
without sacrificing the features of a simple agrarian republic. They wanted the wealth that the
market offered without the competitive, changing society; the complex dealing; the
dominance of urban centers; and the loss of independence that came with it.
Whigs, on the other hand, were more comfortable with the market. For them, commerce and
economic development were agents of civilization. Nor did the Whigs envision any conflict
in society between farmers and workers on the one hand and businesspeople and bankers on
the other. Economic growth would benefit everyone by raising national income and
expanding opportunity. The governments responsibility was to provide a well-regulated
economy that guaranteed opportunity for citizens of ability.
Whigs and Democrats differed not only in their attitudes toward the market but also about
how active the central government should be in peoples lives. Despite Andrew Jacksons
inclination to be a strong President, Democrats as a rule believed in limited government.
Governments role in the economy was to promote competition by destroying monopolies
and special privileges. In keeping with this philosophy of limited government, Democrats
also rejected the idea that moral beliefs were the proper sphere of government action.
Religion and politics, they believed, should be kept clearly separate, and they generally
opposed humanitarian legislation.
The Whigs, in contrast, viewed government power positively. They believed that it should be
used to protect individual rights and public liberty, and that it had a special role where
individual effort was ineffective. By regulating the economy and competition, the
government could ensure equal opportunity. Indeed, for Whigs the concept of government
promoting the general welfare went beyond the economy. In particular, Whigs in the northern
sections of the United States also believed that government power should be used to foster
270

[] Version 3.0
the moral welfare of the country. They were much more likely to favor social-reform
legislation and aid to education.
In some ways the social makeup of the two parties was similar. To be competitive in winning
votes, Whigs and Democrats both had to have significant support among farmers, the largest
group in society, and workers. Neither party could win an election by appealing exclusively
to the rich or the poor. The Whigs, however, enjoyed disproportionate strength among the
business and commercial classes. Whigs appealed to planters who needed credit to finance
their cotton and rice trade in the world market, to farmers who were eager to sell their
surpluses, and to workers who wished to improve themselves. Democrats attracted farmers
isolated from the market or uncomfortable with it, workers alienated from the emerging
industrial system, and rising entrepreneurs who wanted to break monopolies and open the
economy to newcomers like themselves. The Whigs were strongest in the towns, cities, and
those rural areas that were fully integrated into the market economy, whereas Democrats
dominated areas of semi-subsistence farming that were more isolated and languishing
economically.
[Passage End]
---------------------------------------------------------------------------------------------------------------Paragraph 1: The development of the modern presidency in the United States began with
Andrew Jackson who swept to power in 1829 at the head of the Democratic Party and served
until 1837. During his administration, he immeasurably enlarged the power of the presidency.
The President is the direct representative of the American people, he lectured the Senate
when it opposed him. He was elected by the people, and is responsible to them. With this
declaration, Jackson redefined the character of the presidential office and its relationship to
the people.
1. The word immeasurably in the passage is closest inmeaning to
A. Frequently
B. Greatly
C. Rapidly
D. Reportedly
2. According to paragraph 1, the presidency of Andrew Jackson was especially significant
for which of the following reasons?
A. The President granted a portion of his power to the Senate.
B. The President began to address the Senate on a regular basis.
C. It was the beginning of the modern presidency in the United States.
D. It was the first time that the Senate had been known to oppose the President.

271

Version 3.0

[]

Paragraph 2: During Jacksons second term, his opponents had gradually come together to
form the Whig party. Whigs and Democrats held different attitudes toward the changes
brought about by the market, banks, and commerce. The Democrats tended to view society as
a continuing conflict between the people-farmers, planters, and workers-and a set of greedy
aristocrats. This "paper money aristocracy" of bankers and investors manipulated the banking
system for their own profit, Democrats claimed, and sapped the nations virtue by
encouraging speculation and the desire for sudden, unearned wealth. The Democrats wanted
the rewards of the market without sacrificing the features of a simple agrarian republic. They
wanted the wealth that the market offered without the competitive, changing society; the
complex dealing; the dominance of urban centers; and the loss of independence that came
with it.
3. The author mentions bankers and investors in the passage as an example of which of the
following?
A. The Democratic Partys main source of support
B. The people that Democrats claimed were unfairly becoming rich
C. The people most interested in a return to a simple agrarian republic
D. One of the groups in favor of Andrew Jacksons presidency
Paragraph 3: Whigs, on the other hand, were more comfortable with the market. For them,
commerce and economic development were agents of civilization. Nor did the Whigs
envision any conflict in society between farmers and workers on the one hand and
businesspeople and bankers on the other. Economic growth would benefit everyone by
raising national income and expanding opportunity. The governments responsibility was to
provide a well-regulated economy that guaranteed opportunity for citizens of ability.
4. According to paragraph 3, Whigs believed that commerce and economic development
would have which of the following effects on society?
A. They would promote the advancement of society as a whole.
B. They would cause disagreements between Whigs and Democrats
C. They would supply new positions for Whig Party members.
D. They would prevent conflict between farmers and workers.
5. According to paragraph 3, which of the following describes the Whig Partys view of the
role of government?
A. To regulate the continuing conflict between farmers and businesspeople
B. To restrict the changes brought about by the market
C. To maintain an economy that allowed all capable citizens to benefit
D. To reduce the emphasis on economic development

272

[] Version 3.0
Paragraph 4: Whigs and Democrats differed not only in their attitudes toward the market but
also about how active the central government should be in peoples lives. Despite Andrew
Jacksons inclination to be a strong President, Democrats as a rule believed in limited
government. Governments role in the economy was to promote competition by destroying
monopolies and special privileges. In keeping with this philosophy of limited government,
Democrats also rejected the idea that moral beliefs were the proper sphere of government
action. Religion and politics, they believed, should be kept clearly separate, and they
generally opposed humanitarian legislation.
6. The word inclination in the passage is closest in meaning to
A. Argument
B. Tendency
C. Example
D. Warning
7. According to paragraph 4, a Democrat would be most likely to support government action
in which of the following areas?
A. Creating a state religion
B. Supporting humanitarian legislation
C. Destroying monopolies
D. Recommending particular moral beliefs
Paragraph 5: The Whigs, in contrast, viewed government power positively. They believed
that it should be used to protect individual rights and public liberty, and that it had a special
role where individual effort was ineffective. By regulating the economy and competition, the
government could ensure equal opportunity. Indeed, for Whigs the concept of government
promoting the general welfare went beyond the economy. In particular, Whigs in the northern
sections of the United States also believed that government power should be used to foster
the moral welfare of the country. They were much more likely to favor social-reform
legislation and aid to education.
8. The word concept in the passage is closest in meaning to
A. Power
B. Reality
C. Difficulty
D. Idea

273

Version 3.0

[]

9. Which of the following can be inferred from paragraph 5 about variations in political
beliefs within the Whig Party?
A. They were focused on issues of public liberty.
B. They caused some members to leave the Whig party.
C. They were unimportant to most Whigs.
D. They reflected regional interests.
Paragraph 6: In some ways the social makeup of the two parties was similar. To be
competitive in winning votes, Whigs and Democrats both had to have significant support
among farmers, the largest group in society, and workers. Neither party could win an election
by appealing exclusively to the rich or the poor. The Whigs, however, enjoyed
disproportionate strength among the business and commercial classes. Whigs appealed to
planters who needed credit to finance their cotton and rice trade in the world market, to
farmers who were eager to sell their surpluses, and to workers who wished to improve
themselves. Democrats attracted farmers isolated from the market or uncomfortable with it,
workers alienated from the emerging industrial system, and rising entrepreneurs who wanted
to break monopolies and open the economy to newcomers like themselves. The Whigs were
strongest in the towns, cities, and those rural areas that were fully integrated into the market
economy, whereas Democrats dominated areas of semi-subsistence farming that were more
isolated and languishing economically.
10. According to paragraph 6, the Democrats were supported by all of the following groups
EXCEPT
A. workers unhappy with the new industrial system
B. planters involved in international trade
C. rising entrepreneurs
D. individuals seeking to open the economy to newcomers
11. Which of the sentences below best expresses the essential information in the highlighted
sentence in the passage? Incorrect choices change the meaning in important ways or leave
out essential information.
A. Whigs were able to attract support only in the wealthiest parts of the economy
because Democrats dominated in other areas.
B. Whig and Democratic areas of influence were naturally split between urban and rural
areas, respectively.
C. The semisubsistence farming areas dominated by Democrats became increasingly
isolated by the Whigs control of the market economy.
D. The Democrats power was greatest in poorer areas while the Whigs were strongest in
those areas where the market was already fully operating.

274

[] Version 3.0
Paragraph 2: During Jacksons second term, his opponents had gradually come together to
form the Whig party. Whigs and Democrats held different attitudes toward the changes
brought about by the market, banks, and commerce. The Democrats tended to view society
as a continuing conflict between "the people-farmers, planters, and workers-and a set of
greedy aristocrats. This "paper money aristocracy" of bankers and investors manipulated
the banking system for their own profit, Democrats claimed, and sapped the nations virtue
by encouraging speculation and the desire for sudden, unearned wealth. The Democrats
wanted the rewards of the market without sacrificing the features of a simple agrarian
republic. They wanted the wealth that the market offered without the competitive, changing
society; the complex dealing; the dominance of urban centers; and the loss of independence
that came with it.
12. Look at the four squares II that indicate where the following sentence can be added to the
passage.
This new party argued against the policies of Jackson and his party in a number of
important areas, beginning with the economy.
Where would the sentence best fit?
13. Directions: An introductory sentence for a brief summary of the passage is provided below.
Complete the summary by selecting the THREE answer choices that express the most
important ideas in the passage. Some answer choices do not belong in the summary
because they express ideas that are not presented in the passage or are minor ideas in the
passage.
The political system of the United States in the mid-nineteenth century was strongly
influenced by the social and economic circumstances of the time.

Answer Choices
a. The Democratic and Whig Parties developed in response to the needs of competing
economic and political constituencies.
b. During Andrew Jacksons two terms as President, he served as leader of both the
Democratic and Whig Parties.
c. The Democratic Party primarily represented the interests of the market, banks, and
commerce.
d. In contrast to the Democrats, the Whigs favored government aid for education.
e. A fundamental difference between Whigs and Democrats involved the importance of
the market in society.
f. The role of government in the lives of the people was an important political
distinction between the two parties.
275

Version 3.0

6.5

[]

Social Sciences

6.5.1 Aggression
When one animal attacks another, it engages in the most obvious example of aggressive
behavior. Psychologists have adopted several approaches to understanding aggressive
behavior in people.
The Biological Approach. Numerous biological structures and chemicals appear to be
involved in aggression. One is the hypothalamus, a region of the brain. In response to certain
stimuli, many animals show instinctive aggressive reactions. The hypothalamus appears to be
involved in this inborn reaction pattern: electrical stimulation of part of the hypothalamus
triggers stereotypical aggressive behaviors in many animals. In people, however, whose
brains are more complex, other brain structures apparently moderate possible instincts.
An offshoot of the biological approach called sociobiology suggests that aggression is natural
and even desirable for people. Sociobiology views much social behavior, including
aggressive behavior, as genetically determined. Consider Darwins theory of evolution.
Darwin held that many more individuals are produced than can find food and survive into
adulthood. A struggle for survival follows. Those individuals who possess characteristics that
provide them with an advantage in the struggle for existence are more likely to survive and
contribute their genes to the next generation. In many species, such characteristics include
aggressiveness. Because aggressive individuals are more likely to survive and reproduce,
whatever genes are linked to aggressive behavior are more likely to be transmitted to
subsequent generations.
The sociobiology view has been attacked on numerous grounds. One is that peoples capacity
to outwit other species, not their aggressiveness, appears to be the dominant factor in human
survival. Another is that there is too much variation among people to believe that they are
dominated by, or at the mercy of, aggressive impulses.
The Psychodynamic Approach. Theorists adopting the psychodynamic approach hold that
inner conflicts are crucial for understanding human behavior, including aggression. Sigmund
Freud, for example, believed that aggressive impulses are inevitable reactions to the
frustrations of daily life. Children normally desire to vent aggressive impulses on other
people, including their parents, because even the most attentive parents cannot gratify all of
their demands immediately. Yet children, also fearing their parents punishment and the
loss of parental love, come to repress most aggressive impulses. The Freudian perspective,
in a sense: sees us as steam engines. By holding in rather than venting steam, we set
the stage for future explosions. Pent-up aggressive impulses demand outlets. They may be
expressed toward parents in indirect ways such as destroying furniture, or they may be
expressed toward strangers later in life.

276

[] Version 3.0
According to psychodynamic theory, the best ways to prevent harmful aggression may be to
encourage less harmful aggression. In the steam-engine analogy, verbal aggression may vent
some of the aggressive steam. So might cheer on ones favorite sports team. Psychoanalysts,
therapists adopting a psychodynamic approach, refer to the venting of aggressive impulses as
"catharsis." Catharsis is theorized to be a safety valve. But research findings on the usefulness
of catharsis are mixed. Some studies suggest that catharsis leads to reductions in tension and
a lowered likelihood of future aggression. Other studies, however, suggest that letting some
steam escape actually encourages more aggression later on.
The Cognitive Approach. Cognitive psychologists assert that our behavior is influenced by
our values, by the ways in which we interpret our situations and by choice. For example,
people who believe that aggression is necessary and justified-as during wartime-are likely to
act aggressively, whereas people who believe that a particular war or act of aggression is
unjust, or who think that aggression is never justified, are less likely to behave aggressively.
One cognitive theory suggests that aggravating and painful events trigger unpleasant feelings.
These feelings, in turn, can lead to aggressive action, but not automatically. Cognitive factors
intervene. People decide whether they will act aggressively or not on the basis of factors such
as their experiences with aggression and their interpretation of other peoples motives.
Supporting evidence comes from research showing that aggressive people often distort other
peoples motives. For example, they assume that other people mean them harm when they do
not.
[Passage End]
---------------------------------------------------------------------------------------------------------------Paragraph 2: The Biological Approach. Numerous biological structures and chemicals
appear to be involved in aggression. One is the hypothalamus, a region of the brain. In
response to certain stimuli, many animals show instinctive aggressive reactions. The
hypothalamus appears to be involved in this inborn reaction pattern: electrical stimulation of
part of the hypothalamus triggers stereotypical aggressive behaviors in many animals. In
people, however, whose brains are more complex, other brain structures apparently moderate
possible instincts.
1. According to paragraph 2, what evidence indicates that aggression in animals is related to
the hypothalamus?
A. Some aggressive animal species have a highly developed hypothalamus.
B. Artificial stimulation of the hypothalamus results in aggression in animals.
C. Animals behaving aggressively show increased activity in the hypothalamus.
D. Animals who lack a hypothalamus display few aggressive tendencies.

277

Version 3.0

[]

Paragraph 3: An offshoot of the biological approach called sociobiology suggests that


aggression is natural and even desirable for people. Sociobiology views much social behavior,
including aggressive behavior, as genetically determined. Consider Darwins theory of
evolution. Darwin held that many more individuals are produced than can find food and
survive into adulthood. A struggle for survival follows. Those individuals who possess
characteristics that provide them with an advantage in the struggle for existence are more
likely to survive and contribute their genes to the next generation. In many species, such
characteristics include aggressiveness. Because aggressive individuals are more likely to
survive and reproduce, whatever genes are linked to aggressive behavior are more likely to be
transmitted to subsequent generations.
2. According to Darwins theory of evolution, members of a species are forced to struggle for
survival because
A. Not all individuals are skilled in finding food
B. Individuals try to defend their young against attackers
C. Many more individuals are born than can survive until the age of reproduction
D. Individuals with certain genes are more likely to reach adulthood

Paragraph 5: The Psychodynamic Approach. Theorists adopting the psychodynamic


approach hold that inner conflicts are crucial for understanding human behavior, including
aggression. Sigmund Freud, for example, believed that aggressive impulses are inevitable
reactions to the frustrations of daily life. Children normally desire to vent aggressive impulses
on other people, including their parents, because even the most attentive parents cannot
gratify all of their demands immediately. Yet children, also fearing their parents punishment
and the loss of parental love, come to repress most aggressive impulses. The Freudian
perspective, in a sense: sees us as steam engines. By holding in rather than venting steam,
we set the stage for future explosions. Pent-up aggressive impulses demand outlets. They may
be expressed toward parents in indirect ways such as destroying furniture, or they may be
expressed toward strangers later in life.
3. The word inevitable in the passage is closest in meaning to
A. Unavoidable
B. Regrettable
C. Controllable
D. Unsuitable

278

[] Version 3.0
4. The word gratify in the passage is closest in meaning to
A. Identify
B. Modify
C. Satisfy
D. Simplify
5. The word they in the passage refers to
A. Future explosions
B. Pent-up aggressive impulses
C. Outlets
D. Indirect ways
6. According to paragraph 5, Freud believed that children experience conflict between a
desire to vent aggression on their parents and
A. A frustration that their parents do not give them everything they want
B. A fear that their parents will punish them and stop loving them
C. A desire to take care of their parents
D. A desire to vent aggression on other family members
7. Freud describes people as steam engines in order to make the point that people
A. Deliberately build up their aggression to make themselves stronger
B. Usually release aggression in explosive ways
C. Must vent their aggression to prevent it from building up
D. Typically lose their aggression if they do not express it
Paragraph 7: The Cognitive Approach. Cognitive psychologists assert that our behavior is
influenced by our values, by the ways in which we interpret our situations and by choice. For
example, people who believe that aggression is necessary and justified-as during wartime-are
likely to act aggressively, whereas people who believe that a particular war or act of
aggression is unjust, or who think that aggression is never justified, are less likely to behave
aggressively.
Paragraph 8: One cognitive theory suggests that aggravating and painful events trigger
unpleasant feelings. These feelings, in turn, can lead to aggressive action, but not
automatically. Cognitive factors intervene. People decide whether they will act aggressively
or not on the basis of factors such as their experiences with aggression and their interpretation
of other peoples motives. Supporting evidence comes from research showing that aggressive
people often distort other peoples motives. For example, they assume that other people mean
them harm when they do not.
279

Version 3.0

[]

8. Which of the sentences below best expresses the essential information in the highlighted
sentence in the passage? Incorrect answer choices change the meaning in important ways
or leave out essential information.
A. People who believe that they are fighting a just war act aggressively while those who
believe that they are fighting an unjust war do not.
B. People who believe that aggression is necessary and justified are more likely to act
aggressively than those who believe differently.
C. People who normally do not believe that aggression is necessary and justified may act
aggressively during wartime.
D. People who believe that aggression is necessary and justified do not necessarily act
aggressively during wartime.
9. According to the cognitive approach described in paragraphs 7 and 8, all of the following
may influence the decision whether to act aggressively EXCEPT a persons
A. Moral values
B. Previous experiences with aggression
C. Instinct to avoid aggression
D. Beliefs about other peoples intentions
10. The word distort in the passage is closest in meaning to
A. Mistrust
B. Misinterpret
C. Criticize
D. Resent
The Psychodynamic Approach. Theorists adopting the psychodynamic approach hold that
inner conflicts are crucial for understanding human behavior, including aggression. Sigmund
Freud, for example, believed that aggressive impulses are inevitable reactions to the
frustrations of daily life. Children normally desire to vent aggressive impulses on other
people, including their parents, because even the most attentive parents cannot gratify all of
their demands immediately. Yet children, also fearing their parents punishment and the
loss of parental love, come to repress most aggressive impulses. The Freudian perspective,
in a sense: sees us as steam engines. By holding in rather than venting steam, we set
the stage for future explosions. Pent-up aggressive impulses demand outlets. They may be
expressed toward parents in indirect ways such as destroying furniture, or they may be
expressed toward strangers later in life.

280

[] Version 3.0
11. Look at the four squares [] that indicate where the following sentence can be added to the
passage.
According to Freud, however, impulses that have been repressed continue to exist
and demand expression.
Where would the sentence best fit?
12. Directions: Complete the table below by matching five of the six answer choices with the
approach to aggression that they exemplify.
Approach to Understanding Aggression Associated Claims
Biological approach

Psychodynamic approach

Cognitive approach

Answer choices
a. Aggressive impulses toward people are sometimes expressed in indirect ways.
b. Aggressiveness is often useful for individuals in the struggle for survival.
c. Aggressive behavior may involve a misunderstanding of other peoples intentions.
d. The need to express aggressive impulses declines with age.
e. Acting aggressively is the result of a choice influenced by a persons values and
beliefs.
f. Repressing aggressive impulses can result in aggressive behavior.

281

Version 3.0

[]

6.5.2 The Expression of Emotions


Joy and sadness are experienced by people in all cultures around the world, but how can we
tell when other people are happy or despondent? It turns out that the expression of many
emotions may be universal. Smiling is apparently a universal sign of friendliness and
approval. Baring the teeth in a hostile way, as noted by Charles Darwin in the nineteenth
century, may be a universal sign of anger. As the originator of the theory of evolution,
Darwin believed that the universal recognition of facial expressions would have survival
value. For example, facial expressions could signal the approach of enemies (or friends) in
the absence of language.
Most investigators concur that certain facial expressions suggest the same emotions in all
people. Moreover, people in diverse cultures recognize the emotions manifested by the facial
expressions. In classic research Paul Ekman took photographs of people exhibiting the
emotions of anger, disgust, fear, happiness, and sadness. He then asked people around the
world to indicate what emotions were being depicted in them. Those queried ranged from
European college students to members of the Fore, a tribe that dwells in the New Guinea
highlands. All groups, including the Fore, who had almost no contact with Western culture,
agreed on the portrayed emotions. The Fore also displayed familiar facial expressions when
asked how they would respond if they were the characters in stories that called for basic
emotional responses. Ekman and his colleagues more recently obtained similar results in a
study of ten cultures in which participants were permitted to report that multiple emotions
were shown by facial expressions. The participants generally agreed on which two emotions
were being shown and which emotion was more intense.
Psychological researchers generally recognize that facial expressions reflect emotional states.
In fact, various emotional states give rise to certain patterns of electrical activity in the facial
muscles and in the brain. The facial-feedback hypothesis argues, however, that the causal
relationship between emotions and facial expressions can also work in the opposite direction.
According to this hypothesis, signals from the facial muscles ("feedback) are sent back to
emotion centers of the brain, and so a person's facial expression can influence that person's
emotional state. Consider Darwin's words: "The free expression by outward signs of an
emotion intensifies it. On the other hand, the repression, as far as possible, of all outward
signs softens our emotions." Can smiling give rise to feelings of good will, for example, and
frowning to anger?
Psychological research has given rise to some interesting findings concerning the facialfeedback hypothesis. Causing participants in experiments to smile, for example, leads them to
report more positive feelings and to rate cartoons (humorous drawings of people or situations)
as being more humorous. When they are caused to frown, they rate cartoons as being more
aggressive.
What are the possible links between facial expressions and emotion? One link is arousal,
which is the level of activity or preparedness for activity in an organism. Intense contraction
282

[] Version 3.0
of facial muscles, such as those used in signifying fear, heightens arousal. Self-perception of
heightened arousal then leads to heightened emotional activity. Other links may involve
changes in brain temperature and the release of neurotransmitters (substances that transmit
nerve impulses.) The contraction of facial muscles both influences the internal emotional
state and reflects it. Ekman has found that the so-called Duchenne smile, which is
characterized by ''crows feet" wrinkles around the eyes and a subtle drop in the eye cover
fold so that the skin above the eye moves down slightly toward the eyeball, can lead to
pleasant feelings.
Ekmans observation may be relevant to the British expression keep a stiff upper lip as a
recommendation for handling stress. It might be that a stiff lip suppresses emotional
response -- as long as the lip is not quivering with fear or tension. But when the emotion that
leads to stiffening the lip is more intense, and involves strong muscle tension, facial feedback
may heighten emotional response.
[Passage End]
---------------------------------------------------------------------------------------------------------------Paragraph 1Joy and sadness are experienced by people in all cultures around the world, but
how can we tell when other people are happy or despondent? It turns out that the expression
of many emotions may be universal. Smiling is apparently a universal sign of friendliness and
approval. Baring the teeth in a hostile way, as noted by Charles Darwin in the nineteenth
century, may be a universal sign of anger. As the originator of the theory of evolution,
Darwin believed that the universal recognition of facial expressions would have survival
value. For example, facial expressions could signal the approach of enemies (or friends) in
the absence of language.
1. The word despondent in the passage is closest in meaning to
A. Curious
B. Unhappy
C. Thoughtful
D. Uncertain
2. The author mentions "Baring the teeth in a hostile way" in order to
A. Differentiate one possible meaning of a particular facial expression from other
meanings of it
B. Support Darwin's theory of evolution
C. Provide an example of a facial expression whose meaning is widely understood
D. Contrast a facial expression that is easily understood with other facial expressions

283

Version 3.0

[]

Paragraph 2: Most investigators concur that certain facial expressions suggest the same
emotions in all people. Moreover, people in diverse cultures recognize the emotions
manifested by the facial expressions. In classic research Paul Ekman took photographs of
people exhibiting the emotions of anger, disgust, fear, happiness, and sadness. He then asked
people around the world to indicate what emotions were being depicted in them. Those
queried ranged from European college students to members of the Fore, a tribe that dwells in
the New Guinea highlands. All groups, including the Fore, who had almost no contact with
Western culture, agreed on the portrayed emotions. The Fore also displayed familiar facial
expressions when asked how they would respond if they were the characters in stories that
called for basic emotional responses. Ekman and his colleagues more recently obtained
similar results in a study of ten cultures in which participants were permitted to report that
multiple emotions were shown by facial expressions. The participants generally agreed on
which two emotions were being shown and which emotion was more intense.
3. The word concur in the passage is closest in meaning to
A. Estimate
B. Agree
C. Expect
D. Understand
4. The word them in the passage refers to
A. Emotions
B. People
C. Photographs
D. Cultures
5. According to paragraph 2, which of the following was true of the Fore people of New
Guinea?
A. They did not want to be shown photographs.
B. They were famous for their story-telling skills.
C. They knew very little about Western culture.
D. They did not encourage the expression of emotions.
6. Which of the sentences below best expresses the essential information in the highlighted
sentence in the passage? Incorrect choices change the meaning in important ways or leave
out essential information.

284

[] Version 3.0
A. The Fore's facial expressions indicated their unwillingness to pretend to be story
characters.
B. The Fore was asked to display familiar facial expressions when they told their stories.
C. The Fore exhibited the same relationship of facial expressions and basic emotions that
is seen in Western culture when they acted out stories.
D. The Fore was familiar with the facial expressions and basic emotions of characters in
stories.
Paragraph 3: Psychological researchers generally recognize that facial expressions reflect
emotional states. In fact, various emotional states give rise to certain patterns of electrical
activity in the facial muscles and in the brain. The facial-feedback hypothesis argues,
however, that the causal relationship between emotions and facial expressions can also work
in the opposite direction. According to this hypothesis, signals from the facial muscles
("feedback) are sent back to emotion centers of the brain, and so a person's facial expression
can influence that person's emotional state. Consider Darwin's words: The free expression
by outward signs of an emotion intensifies it. On the other hand, the repression, as far as
possible, of all outward signs softens our emotions. Can smiling give rise to feelings of good
will, for example, and frowning to anger?
7. According to the passage, what did Darwin believe would happen to human emotions that
were not expressed?
A. They would become less intense.
B. They would last longer than usual.
C. They would cause problems later.
D. They would become more negative
Paragraph 4: Psychological research has given rise to some interesting findings concerning
the facial-feedback hypothesis. Causing participants in experiments to smile, for example,
leads them to report more positive feelings and to rate cartoons (humorous drawings of
people or situations) as being more humorous. When they are caused to frown, they rate
cartoons as being more aggressive.
8. According to the passage, research involving which of the following supported the facialfeedback hypothesis?
A. The reactions of people in experiments to cartoons
B. The tendency of people in experiments to cooperate
C. The release of neurotransmitters by people during experiments
D. The long-term effects of repressing emotions

285

Version 3.0

[]

9. The word rate in the passage is closest in meaning to


A. Judge
B. Reject
C. Draw
D. Want
Paragraph 6: Ekmans observation may be relevant to the British expression keep a stiff
upper lip as a recommendation for handling stress. It might be that a stiff lip suppresses
emotional response -- as long as the lip is not quivering with fear or tension. But when the
emotion that leads to stiffening the lip is more intense, and involves strong muscle tension,
facial feedback may heighten emotional response.
10. The word relevant in the passage is closest inmeaning to
A. Contradictory
B. Confusing
C. Dependent
D. Applicable
11. According to the passage, stiffening the upper lip may have which of the following effects?
A. It first suppresses stress, and then intensifies it.
B. It may cause fear and tension in those who see it.
C. It can damage the lip muscles.
D. It may either heighten or reduce emotional response.
Paragraph 2: Most investigators concur that certain facial expressions suggest the same
emotions in all people. Moreover, people in diverse cultures recognize the emotions
manifested by the facial expressions. In classic research Paul Ekman took photographs of
people exhibiting the emotions of anger, disgust, fear, happiness, and sadness. He then
asked people around the world to indicate what emotions were being depicted in them. Those
queried ranged from European college students to members of the Fore, a tribe that dwells in
the New Guinea highlands. All groups, including the Fore, who had almost no contact with
Western culture, agreed on the portrayed emotions. The Fore also displayed familiar facial
expressions when asked how they would respond if they were the characters in stories that
called for basic emotional responses. Ekman and his colleagues more recently obtained
similar results in a study of ten cultures in which participants were permitted to report that
multiple emotions were shown by facial expressions. The participants generally agreed on
which two emotions were being shown and which emotion was more intense.

286

[] Version 3.0
12. Look at the four squares that indicate where the following sentence could be added to
the passage.
This universality in the recognition of emotions was demonstrated by using rather
simple methods.
Where would the sentence best fit?
13. Directions: An introductory sentence for a brief summary of the passage is provided
below. Complete the summary by selecting the THREE answer choices that express the
most important ideas in the passage. Some sentences do not belong in the summary
because they express ideas that are not presented in the passage or are minor ideas in the
passage.
Psychological research seems to confirm that people associate particular facial
expressions with the same emotions across cultures.

Answer Choices
a. Artificially producing the Duchenne smile can cause a person to have pleasant
feelings.
b. Facial expressions and emotional states interact with each other through a variety of
feedback mechanisms.
c. People commonly believe that they can control their facial expressions so that their
true emotions remain hidden.
d. Aperson's facial expression may reflect the person's emotional state.
e. Ekman argued that the ability to accurately recognize the emotional content of facial
expressions was valuable for human beings.
f. Facial expressions that occur as a result of an individual's emotional state may
themselves feedback information that influences the person's emotions.

287

Version 3.0

[]

Answer Key & Explanation


Chapter 1
Page 16-20
1. C
particular A (Natural) B (Final) C
(Specific) D (Complex)
2. C
climax communities
the long-lived community at the end of succession is called a climax
community A D B
communities 1-500 climax 500 climax C
do not change climax
3. A

A
4. B

climax communities
5. C

C
6. B
climax communities
climax
climax climax B
7. C
redwood forest redwood
mid climax
redwood redwood
A B D

288

[] Version 3.0
8. B
guarantee A increaseB ensureC favor
D complicate
9. A

A C
10. C
pale
A increase proportionallyB differs
C lose significanceD is common
11. D
. patchiness
. Patchiness

12. D
adjacent A foreignB stableC fluidD
neighboring
13. B
human A/B/C natural events and processes
B/C
B C
14. B, C, E

AThe changes
BA high degree
CThe level
DEcologists
EDisagreements
FThe resilience

289

Version 3.0

[]

Chapter 2
Page 23
Filling in the blanks with one of the words listed below

derived
annihilation
imminent
advocate
innovative
components
dearth
facilitate
stimulant
repercussions

1. Knowing that the arrival of the president was (imminent), officials were scurrying around
at a frantic pace trying to make sure that everything was in order and ready for his arrival.
2. Changes in the regulations governing the countrys banks had enormous and devastating
(repercussions) on taxpayers.
3. The fear of civil war in Iraq persists because the Sunni and Shia sects of Islam feel so
much hostility toward each other that their goal is not cooperation but (annihilation).
4. Many people still believe that playing Mozart for infants will improve their IQ; however,
there is a(n) (dearth) of hard evidence proving such claims.
5. Throughout his Supreme Court career, Chief Justice Earl Warren was a strong (advocate)
for the rights of the individual.
6. The journalist who was supposed to (facilitate) the debate between the two candidates
failed miserably at the job.
7. People think that alcohol a(n) (stimulant),but actually it has the opposite effect.
8. Opium is a drug (derived) from poppies.
9. The directions suggested that not all of the (components) for the hot tub had been included
in the box.
10. Steve Jobs, the head of the Apple, was from the beginning, a(n) (innovative) thinker who
was always one step ahead of his competitors.
290

[] Version 3.0

Page 26-38 (Even)


1. D (region)
2. D (noticeably)
3. A (progressive)
4. B (linked)
5. B (heavy)
6. B (areas)
7. A (unusual)
8. C (a dominance)
9. A (honor or observe an occasion)
10. D (waste)
11. C (advanced skill)
12. D (memories)
13. B (giving)
14. B (serious)
15. D (characterized by a negative outlook)
16. A (refuses to accept)
17. C (complained)
18. B (nobility)
19. B (brave)
20. C (illegal)
21. B (economical)
22. D (thought)
23. C (horribly)
24. B (people who sleep excessively)
25. C (lowered)
26. A (color)
27. D (create)
28. B (revelation)
29. A (tiny)
30. D (supported)
291

Version 3.0

[]

Page 27

By meeting the legitimate demands of its citizens for freedom and openness, Chinas
leaders can help their country grow into a modern, prosperous, and confident nation.

Page 29

The acronym BRICS has come into widespread use across the world.

Page 31

As a result of evolution, humans should have no more value than any other creature.

Page 33

Everyone, regardless of age or background, can come up with new ideas and excellent
insights.

Page 35

Youve got to admit, he does have a way with words.

Students may also apply for different US schools simultaneously, but here, they only get
one shot.

Page 39

You might be skilled in a particular field but that doesnt necessarily make you good at
other field.

292

[] Version 3.0

Page 39
1. The letter E
2. The word Wrong
3. The wise man tells them to switch camels.
4. There are six Fs in the sentence.
A person of average intelligence finds three of them.
If you spotted four, youre above average.
If you got five, you can turn your nose at most anybody.
If you caught six, you are a genius.
5. They are all palindromes. A palindrome is a word, phrase or number that reads exactly
the same backwards as forwards. The name "palindrome" comes from the Greek
palindromos meaning running back again.

Page 40-48 (Even)


1. A (cover)
2. B (inclination)
3. C (achieve)
4. A (difficult)
5. A (continuing)
6. D (immediately)
7. A (nearby)
8. C (makes easier)
9. A (powerful)
10. A (emergency of many varieties)
11. A (carefully)
12. C (basic)
13. B (strike)
14. D (incentive)
15. B (inactive)
16. D (without a definite pattern)
17. A (distracted from)
18. C (motionless)
19. D (measurable)
20. B (travelling)
293

Version 3.0

[]

Page 41

They tend to express themselves only after they know all the opinions.
1. TWENTY NINE.
2. Herb. (American English only) Job. Nice. Polish. Differently.
3. That that is, is; that that is not, is not. Is not that it? It is.
4. Herein. The words it contains are: he, her, here, ere, re, rein, I, and
in.

Page 43

The weak and unfit species die out, and to be replaced by a stronger, healthier, more
impressive species.

Page 45

Jim records his workout so that he can refer to it and think about how he should improve
his workout the next day

Page 47
Google
Today, the company announced an update to Google Maps that it says will offer "an even
better way to find places nearby

Page 49
10
While many wealthy Chinese like to be driven, were guessing that at least 10 will prefer
to do the driving themselves

Page 50-51
Address: articleassumebillcomplexes
depressedfairsfineminuteplane
schoolscreenedspringweather
294

[] Version 3.0

Chapter 3
Page 56-57
1. C (He had been told)
2. A (Having been told)
3. B (Lost)
4. B (Taking all the possibilities into consideration)
5. D (The thief being caught)
6. C (being opened and closed)
7. C (Being caught)
8. C (Being examined)
9. B (His not being allowed)

Page 59
1. A (to has come)
2. B (whose)
3. A (during which time)
4. C (where)
5. C (in which)
6. B (which)
7. D (where)
8. D (whose real name)
9. B (As)
10. D (with whom)
11. D (which)
12. C (/ ; where)

Page 61

Demographic and epidemiological transitions now combine with nutritional and


behavioral transitions to create ominous new trends.

295

Version 3.0

[]

Page 63

Many people fail to become successful in life simply because they associate themselves
with the wrong people.

Page 65

He is accustomed to this sort of pessimism and readily acknowledges how complicated
the brain is.

Page 67
5

Iraq government announced that it will send up to 10000 students per year to colleges in
the US, UK, Canada and Australia over the next 5 years to help reconstruction efforts.

Page 69

With limited time and attention, we have to be careful what we look for and what we pay
attention to.

Page 71

The peak phase is characterized by short, fast workouts that simulate racing.
1. 4 seasons in a year.
2. 30 days in September
3. 32 teeth in a mouth.
4. 26 letters in the English alphabet
5. 50 stars on the American flag.
6. 206 bones in the human body.
7. 88 keys on a piano.
8. 52 cards in a pack of playing cards.
9. 11 players in a football team.
10. 44 Presidents of America (Obama is number 44)
296

[] Version 3.0

Page 75
1. what
2. who
3. that
4. that
5. when
6. that/which
7. since
8. that
9. where
10. before
11. who; which
12. which
13. that
14. while; where
15. where; who
16. (1) who (2) until (3) where (4) that (5) when (6) Though

Page 77

Through this growth you are capable of reaching new levels of passion and intimacy.

Page 79

By no means will this method produce satisfactory results.

Page 83

The experimental results are coincident with the analytic results of dynamic model.

297

Version 3.0

[]

Page 85

In place of praying, you read.

Page 100-104 (Even Numbers)


Illustrate: C (The deformed shapes of trees at high altitudes show that wind velocity, which
increase with altitude, can cause serious hardship for trees.)
Negative: C (Most people are quite ignorant about snails)
Explain: B (But detractors argue that the terrace may have been formed by geological
activity rather than by the presence of water)
Give Reasons: C (Edison did not want to develop projection technology because it limited
the number of machines he could sell.)
Show Results: C (The Southerners felt motivated by the win at Fort Sumter.)
Compare and Contrast: C (Although Ambulocetus had no fluke; its backbone structure
shows that it swam like modern whales.)
Limit: D (Experiments revealed that caged starlings orientation was accurate unless the
weather was overcast.)
Emphasize: D (The United States suffers more than any other country due to the strength of
its tornadoes.)

Page 103

These proposals have nothing to do with grandiose schemes for reinventing market
capitalism.

Page 105

Using these tips you can set up an effective budget for yourself and your household.

298

[] Version 3.0

Page 106
1. D (The climate did not change dramatically from season to season)
2. A (Using tones that are clumped together)
3. D (an economic depression)

Page 107

Love is a telephone which is not always associated with happiness.

Page 108
1. C (a common language)
2. C (Instincts to avoid aggression)

Page 109

For some teachers, half of their rating is based on how well their students do.

Page 111

Sin may lead to depression, but all depression does not come from sin.
Brain Food: Stringier, Strapping, cleansers, wrappings, trappings

Page 112-120
Exercise 1:
1. B. Deciduous and evergreen oaks are found in cold and hot Asia and the Americas.
2. A. There are many reasons for the war in which more people died than any other war.
3. D. If the war had not been with the same race, the pain would have been much less.
4. C. It is impossible to hold a different opinion.

299

Version 3.0

[]

Exercise 2:
1. A (development of synthetic colors foe textiles during the nineteenth century)
2. C (Plants)
3. A (It was possible to cultivate madder plants in almost every location)
4. A (without exception)
5. C (the yellow dye. that was used in the tapestries had faded)
6. C (replaced)
7. D (The chemical dyes held up belter after washing)
8. A (Previously developed dyes did not work on new types of fibers.)

Exercise 3:
1. A (Principal)
2. B (Cut)
3. A (Unique)
4. B (requires that paper be cut with a burin)
5. B (Produce)
6. D (They allow multiple copies to be produced from one original)
7. A (Prints could be made at low cost)
8. D (require a printing press)

Exercise 4:
1. B (Rococo art was less serious than Baroque art)
2. D (Rococo was a style that kept up with what as fashionable by being entertaining and
not educational)
3. C (Rococo became one of the most important art styles of its time even though it was
very criticized by many)
4. A (Although critics considered Rococo an old-fashioned style at first, they soon
recognized its important place in art history.)
Exercise 5:
1. B (Housing problems caused by industrialization)
2. D (Draw workers to the railway car industry)
3. B (Become active in town politics)
4. A (He personally showed tourists around the town)
5. B (His conflicts with the inhabitants of the town of Pullman)
6. C (Keep)
7. D (Complaints)
300

[] Version 3.0

Chapter 4
Page 130-134 (Even numbers)
Exercise 1: B (height change during puberty)
Exercise 2: The effect of televised violence on young people/in current research
Exercise 3: Research in the rain forest
Exercise 4: C (Reasons behind Benedict Arnolds treason)
Exercise 5: A (Polygamy, past and present)
Exercise 6: B (Tricks used by Psychics)

Page 133

Many people, including college students of all ages, spend little time in pursuit of
physical fitness.

Page 135

Their discussions were impractical and did not hit the mark.

Page 138-140 (Even numbers)


Exercise 1: B (In his pursuit of wealth, James Fisk never let law or morality stand in his
way)
Exercise 2: C (The ancient Romans made luxury and socializing a part of bathing)
Exercise 3: B (Birds turn up frequently in superstitions as sings of both good and evil)
Exercise 4: A (Despite a common topic, French and American cookbooks differ in the way
they give directions)

Page 139

His furious activity put us all to shame.
301

Version 3.0

[]

Page 141

And in this time of high unemployment, more people are inclined to do just that.

Page 143

Poverty will rise as they sink into informal work or move back to the land.

Page 144-146 (Even numbers)


Exercise 1: C (Although for a while now, the media has celebrated the Mozart Effect as if
it were a proven fact, it turns out that there is little hard evidence it exists)
Exercise 2: B (The mountains challenging and awe-inspiring height is one reason why so
many people have been obsessed by the desire to climb it died trying.)
Exercise 3: B (Horse racing is a dangerous and deadly sport for the horses, no matter how
eager and spirited the animals might look at the starting gate.)
Exercise 4: C (Not surprisingly, parents of school-age children were the biggest winners
from the passage of the Buckley amendment, which gave them access to their childrens
records and evaluations.)

Page 147

People inflict pain on others in the selfish pursuit of their happiness or satisfaction.

Page 148-150 (Even numbers)


Exercise 1: Sentence 1
Exercise 2: Sentence 2
Exercise 3: Sentence 2
Exercise 4: Sentence 1
Exercise 5: Sentence 1
302

[] Version 3.0

Page 149

A doctrine of the American way of life is that all persons are entitled to an education.

Page 151
, .
If you live on a mountain, you live off the mountain; if you live by the water, you live off
the water.

Page 152
Exercise 1: Sentence 3
Exercise 2: Sentence 7

Page 153

The police has promised to clamp down on criminal activities.

Page 154
Exercise 1: Sentence 5
Exercise 2: Sentence 8
Exercise 3: Sentence ?

Page 155

I know that we will get through this sadness, just as each of us must go through the loss.

Page 158
Exercise 1: D (In the same poll)
Exercise 2: B (All 50 states have laws)
Exercise 3: C (Not all scientists)
303

Version 3.0

[]

Page 164-168 (Even numbers)


Exercise 1: D (the development of plants that produce seeds)
Exercise 2: C (The separation of myths from rites)
Exercise 3: B (Society)
Exercise 4: D (The particles carried in water from melting glaciers)
Exercise 5: B (Giant Planets);
Exercise 6: B (substances)
Exercise 7: B (Aviculturists)
Exercise 8: B (baskets)
Exercise 9: C (African American artists)

Page 165

The days when you could just act out of your noble instincts, are over.

Page 167

As a public utility, the street lamp of Public administration is laden with social
responsibility.

Page 172-176 (Even numbers)


Exercise 1: 4
Exercise 2: 4
Exercise 3: 4
Exercise 4: 2
Exercise 5: 2
Exercise 6: 1
Exercise 7: 2
Exercise 8: 4
Exercise 9: 2

Page 173
,
Their basic fear is of cheap foreign labor producing goods that will compete with
American products.
304

[] Version 3.0

Page 177
,.
Social stability may be affected as rumors spread out.

Page 178-180 (Even numbers)


Exercise 1: C (They are found in temperate zones)
Exercise 2: C (It has the capacity to store large amounts of water)
Exercise 3: C (They developed their own sources of light to use while painting)
Exercise 4: A (It is not difficult to imagine what they looked like)

Page 179
.
Yet there are also problems associated with the use of fiscal policy.

Page 182-186 (Even numbers)


Exercise 1: B (to persuade)
Exercies 2: C (Provide an example of a facial expression whose meaning is widely
understood)
Exercise 3: B (to argue that philosophical perspectives must be considered in literary
criticism)
Exercise 4: A (to illustrate a general principle about the stability of system by using an
everyday example)
Exercise 5: D (To illustrate the variety of coverings among angiosperm seeds)

Page 183

The government took this position at the expense of an increasing deficit and national debt.

Page 188
BBAAD

Page 189
CCBBAA
305

Version 3.0

[]

Chapter 5
Page 190
Exercise 2:
Process: The stage of development of Moth
Transitional Clues: Within a few days; During this stage; In the pupa stage; At this
point; In the final stage

Page 194
Example:
1. What larger group is divided into smaller subgroups or categories?
Symbiotic Relationship
2. How many subgroups are mentioned? 3
3. Name and describe each subgroup
Parasitism, commensalism, and mutualism

Page 196
Exercise 1:
1. What term is defined in the paragraph?
Litigation
2. In your own words, what is the definition of that term?
An adversarial kind of communication in which a dispute is settled by a judge, who
decides who is lying
3. Describe the example that the author uses to illustrate the definition
Litigation is usually conducted by lawyers who call witnesses and question them to
reveal information.

Exercise 2:
1. What term is defined in the paragraph?
Phishing
2. In your own words, what is the definition of that term?
The act of tricking people into revealing information like passwords and credit card
numbers
3. Describe the example that the author uses to illustrate the definition
The mid-1990 example
306

[] Version 3.0

Page 198
Exercise 2: A
Page 200
1. This paragraph: B. contrasts two topics
2. What two topics are compared and/or contrasted? Ancient Rome and Greece
3. List any similarities mentioned
Both Rome and Greece expanded in the basis of Sea and Land power
4. List any differences mentioned
Rome (Sea conqueror)

Greece (Land conqueror)

Grow from one single organism

Grow from scattered cities

Assembled by territorial conquest

Expanded along Mediterranean seas lanes

Marching legions

High powered ships

Page 201
Exercise 1: D (Cause and Effect)

Exercise 2: A (Definition)

Page 202-207
Exercise 1: ACF
Exercise 2: ABD
Exercise 3: ACF
Exercise 4: 258; 147
Exercise 5: 2614; 2356

307

Version 3.0

[]

Chapter 6
Desert Formation
1. B
2. B
3. A
4. A
5. D
6. C
7. D
8. D
9. B
10. A
11. C
12. B
13. acd
Geology and Landscape
1. D
2. B
3. B
4. C
5. C
6. A
7. B
8. B
9. B
10. D
11. A
12. Constructive Processes: aef; Destructive Processes: cg

308

[] Version 3.0
Green Icebergs
1. C
2. B
3. C
4. D
5. A
6. D
7. C
8. A
9. A
10. D
11. B
12. C
13. B
14. cdf

The origins of Cetaceans


1. B
2. A
3. C
4. C
5. A
6. B
7. D
8. D
9. B
10. C
11. D
12. B
13. abe

309

Version 3.0

[]

Swimming Machines
1. B
2. B
3. D
4. A
5. D
6. A
7. D
8. C
9. B
10. C
11. B
12. Reducing Water resistance: ade; Increasing thrust: bg
Opportunists and Competitors
1. D
2. C
3. B
4. B
5. C
6. A
7. A
8. D
9. B
10. D
11. C
12. Opportunists: cdfi; Competitors: beh
Applied Arts and Fine Arts
1. A
2. A
3. B
4. A
5. bdf
6. The Applied Arts: bef; The Fine Arts: ad
310

[] Version 3.0
Loie Fuller
1. C
2. D
3. A
4. A
5. C
6. D
7. D
8. C
9. C
10. B
11. A
12. A
13. D
14. CDE
Lascaux Cave Paintings
1. C
2. A
3. D
4. B
5. C
6. C
7. C
8. C
9. D
10. A
11. C
12. A
13. ACE

311

Version 3.0

[]

Early Cinema
1. B
2. B
3. C
4. B
5. C
6. D
7. A
8. D
9. B
10. A
11. A
12. D
13. CEF
Artisans and Industrialization
1. C
2. B
3. D
4. A
5. D
6. A
7. A
8. C
9. B
10. A
11. D
12. Before 1815: bf; 1815-1850: ceg

312

[] Version 3.0
Nineteenth-Century Politics in the United States
1. B
2. C
3. B
4. A
5. C
6. B
7. C
8. D
9. D
10. B
11. D
12. A
13. AEF
Aggression
1. C
2. C
3. A
4. C
5. B
6. B
7. C
8. B
9. C
10. B
11. B
12.
Biological approach: B
Psychodynamic approach: AF
Cognitive approach: CE

313

Version 3.0

[]

The expression of Emotions


1. B
2. C
3. B
4. C
5. C
6. C
7. A
8. A
9. A
10. D
11. D
12. C
13. BDF

314

[] Version 3.0

315

Version 3.0

[]

316

[] Version 3.0

317

Version 3.0

[]

318

[] Version 3.0

319

Version 3.0

[]

320

You might also like